Está en la página 1de 488

Fsica Universitaria

Problemas de Fsica

Manuel R. Ortega Girn


Rafael Lpez Luque

Departamento de Fsica Aplicada.


Universidad de Crdoba.

Fsica Universitaria
Problemas de Fsica
Primera edicin: julio 2009

Copyright. Reservados todos los derechos.


Ninguna parte de este libro puede ser reproducida
por cualquier medio, incluidas las fotocopias,
sin el permiso por escrito del autor.

Copyright:

Manuel R. Ortega Girn


Rafael Lpez Luque

Editor:

Manuel R. Ortega Girn


CL Santa Cruz, 10
14.012 Crdoba. Espaa.
Tfnos.: +34 957 280051 (particular)
+34 957 218483 (departamento)
Fax:
+34 957 218483
e-mail: mr.ortega@uco.es
http://www.uco.es/users/mr.ortega

Impresin:

Reprografa Don Folio


14.013 Crdoba. Espaa.

I.S.B.N.
Depsito legal:

ii

Fsica Universitaria

Prlogo
Este libro completa nuestras obras Lecciones de Fsica y Fsica Universitaria que
vienen teniendo una amplia y buena acogida, durante ms de dos dcadas, en diversas
Universidades Espaolas.
Problemas de Fsica, as como el conjunto de la obra en la que se integra, es un libro
concebido como apoyo a la enseanza de la Fsica en los estudios universitarios, tanto
de carcter tcnico como cientfico, presentando un nivel apropiado para la Fsica que
se imparte en los Primeros Ciclos de nuestras Facultades y Escuelas Tcnicas.
Desde la ms remota antigedad, la enseanza se ha enfrentado con dos problemas
bsicos: decidir qu conocimientos se deben transmitir (contenidos) y acertar con
cmo puede hacerse esa transmisin (forma).
En el aspecto de contenidos, la mayor parte del contenido de este libro procede de
nuestra experiencia personal y de los exmenes propuestos a los alumnos a quienes
hemos impartido la asignatura, y corresponde a los descriptores oficiales
correspondientes a los Fundamentos Fsicos de la Ingeniera.
En el aspecto formal, durante la preparacin de este libro hemos pretendido la
consecucin de dos objetivos principales que entendemos que deben orientar la
docencia de las asignaturas de Fsica de Primer Ciclo de los estudios universitarios:
familiarizar al alumno con el conjunto de los conceptos y leyes bsicas que constituyen
la esencia de la Fsica y desarrollar en el estudiante la habilidad para manejar esas
ideas y para aplicarlas a situaciones concretas
En Problemas de Fsica hacemos un uso intensivo de figuras y esquemas para facilitar
la comprensin de los problemas, su tratamiento y soluciones. Adems, hay un aspecto
que conviene destacar: en muchos de los problemas, las figuras representan en gran
medida la solucin del mismo, lo que realza la importancia de las figuras y esquemas
en el planteamiento resolucin de los problemas.
Crdoba, julio 2009

Fsica Universitaria

iii

iv

Fsica Universitaria

A Estela y Olga

Desde la infancia he sido criado en el estudio de las


letras y, como quiera que me aseguraban que por medio
de stas se poda adquirir un conocimiento claro y
seguro de todo aquello que es til para la vida, yo tena
un vivsimo deseo de aprenderlas. Pero cuando acab el
curso de los estudios, al finalizar los cules es costumbre
ser admitido en la jerarqua de los doctos, cambi
enteramente de opinin. Por que me encontraba turbado
y confuso entre tantas dudas y errores que me pareca no
haber obtenido otro provecho, al procurar instruirme,
que el descubrir cada vez mejor mi ignorancia.
REN DESCARTES (1596-1650)
El Discurso del Mtodo.

Fsica Universitaria

vi

Fsica Universitaria

Fsica Universitaria
Problemas de Fsica

Fsica Universitaria

vii

Materias
Los cdigos de materias se corresponden con el ndice de Materias de la obra Fsica
Universitaria, del mismo autor.

M01. lgebra vectorial.


M02. Vectores deslizantes.
M03. Anlisis vectorial.
M04. Cinemtica de la partcula.
M05. Cinemtica del slido rgido.
M06. Principios de la Mecnica Clsica. La ley de la inercia.
M07. Segunda y tercera leyes de Newton. Conservacin de la cantidad de movimiento.
M08. Las fuerzas de la Naturaleza.
M09. Sistemas de referencia en rotacin.
M10. Trabajo y energa.
M11. Conservacin de la energa.
M12. Momento angular. Fuerzas centrales.
M13. Movimiento armnico simple.
M15. Superposicin de movimientos armnicos simples.
M16. Geometra de masas.
M17. Sistemas de partcilas.
M18. Sistemas de masa variable. El problema de 2-cuerpos.
M19. Colisiones.
M20. Esttica del slido rgido.
M21. Dinmica del slido rgido.
M22. Trabajo y energa en el movimiento general del sl. rg.
M24. Dinmica impulsiva del slido rgido.
M25. La ley de la Gravitacin Universal.
M27. Elementos de elasticidad.
M29. Esttica de los fluidos.
M31. Cinemtica de los fluidos.
M32. Dinmica de los fluidos ideales.
M33. Dinmica de los fluidos reales.
M34. Flujo viscoso.
M35. Ondas mecnicas.
T00. Termodinmica.
E01. Campo elctrico.
E02. Capacidad elctrica.
E03. Corriente continua.
E04. Campo magntico
E05. Induccin magntica.
E06. Corriente alterna.

viii

Fsica Universitaria

Fsica Universitaria: Problemas de Fsica

Vectores. M01.1

1. Consideremos el vector A y la direccin definida por el vector B. Descompongamos el vector A en dos: uno
paralelo y otro perpendicular a la direccin del vector B. Demostrar que los vectores componentes de A son
(AB/$)eB y (Bu(AuB)/$2.

El vector A tiene como componentes los vectores A1 y A2, tal como se indica en la figura.
El mdulo de la componente del vector A en la direccin del vector B es la proyeccin de A
sobre B, de modo que lo obtenemos multiplicando escalarmente A por el versor en la
direccin de B; esto es,

A1  A eB  A

B
B

De modo que el vector A1 viene expresado por

A2

B
A1  A eB
B

A1

En cuanto a la componente A2, de la definicin del producto vectorial se sigue la expresin


del mdulo de A2; esto es,

A q B  AB sen R  B A sen R
 BA2

A2 

AqB
B

Puesto que la direccin del producto vectorial AuB es normal al plano del papel y entrante, la
del producto Bu(AuB) ser la del vector A2, de modo que este vector vendr dado por

A 2  eB q

A q B

-1-

B q A q B

B2

Fsica Universitaria: Problemas de Fsica

Vectores. M01.2

2. Hallar el vector que representa la superficie del tringulo determinado por los vectores
A  5i 8 j 9k y B  6i  j 5k concurrentes en un punto dado.

El vector que define la superficie del tringulo formado


S
por los dos vectores viene dado por la mitad de su
producto vectorial. En consecuencia, el vector S es un
vector normal (perpendicular al plano) determinado por
los vectores A y B, y su mdulo vale 12 A B sen R , siendo
 el ngulo que forman entre s los vectores dados, y su
sentido viene determinado por la regla de la mano
derecha.
Analticamente, tenemos
5 6
49 24.5
1
1 1

S  A q B  8 q1  29  14.5


2
2 9 5 2 53 26.5

Y su mdulo (superficie del tringulo) es


S  24.52 14.52 26.52  38.9

-2-

B
A

Fsica Universitaria: Problemas de Fsica

Vectores deslizantes. M02.1

1. El mdulo de la resultante de un sistema de vectores es R = 6, el invariante escalar del sistema es M$R = 30


y las ecuaciones del eje central del sistema son 2x = y = 2z. Hallar: a) el momento mnimo; b) la resultante;
c) el momento respecto al origen; d) el momento con respecto al punto (2, 1, 0).

El eje central del sistema de vectores pasa por el origen de coordenadas (0,0,0) y sus
ecuaciones pueden expresarse en la forma:
x y z
2x y 2z o
,
1 2 1
Eje
R
central por lo que su versor director es

1
1
e
2
6
1

a) El momento mnimo es igual a la proyeccin sobre el eje
central del momento en cualquier punto del espacio; esto es,

M0

M mn

M <R
R

30
6

5 o M mn

M mn e

1
5
2
6
1

b) La direccin de la resultante es la del eje central: esto es,


1
1
6

R Re
2
6 2
6
1
1

c) Dado que el origen de coordenadas pertenece al eje central, ser
1
5 6
2
6
1
d) Aplicamos la frmula de cambio de momentos:
M0

MP

JJJG
M O  PO u R

M mn

1 2
1
5 6

2  1 u 6 2

6
1
1 0

-3-

1
1
5 6 6 6
2 
2
6
6
1


3

1
6

22

13

Fsica Universitaria: Problemas de Fsica

Vectores deslizantes. M02.2

2. Un sistema de vectores deslizantes es tal que en el origen el momento resultante es nulo y en los puntos
A(1,0,0) y B(0,1,0) los momentos son MA = aj + k y MB = i + b j - k, respectivamente. Determinar: a) Los
valores de a y b en las expresiones de los momentos. b) La resultante del sistema. c) El eje central. d) Si
estuvisemos describiendo con este ejercicio el movimiento de un slido rgido, escriba de nuevo el
enunciado del problema.

a) y b) Sea R = (li + mj + nk) la resultante del sistema. Relacionamos los momentos en A y B


con el momento en O:

M A

M
B

0
JJJG

M O  AO u R o a
1

1
JJJG

BO
uR o b
MO 
1

1 l

0 um
0 n

0 l

1 u m
0 n

0
a n
a

b
n o m 1

m

o l
n
n 1
m

0
0
o
b


n
l 1
l

1
0
1
1
1

de modo que
0
1
1



M A 1
MB 0
R 1
1
1
1



c) Puesto que el momento en el origen es nulo, el eje central pasa por el origen de
coordenadas y tiene la direccin de la resultante R, de modo que viene dado por las
ecuaciones:
x
y
z
o x y z
1 1 1
d) Un slido rgido tiene un movimiento tal que, en un instante dado, las velocidades de tres
de sus puntos... ... a) ... b) La rotacin resultante. c) El eje instantneo de rotacin y
deslizamiento.

-4-

Fsica Universitaria: Problemas de Fsica

Vectores deslizantes. M02.3

3. Dado el sistema de vectores deslizantes de la figura, determinar: a) Los invariantes del

sistema. b) El eje central. c) El momento respecto al eje Oy. d) Un sistema equivalente al


1
anterior formado por dos vectores tales que la recta de accin de uno de ellos sea el eje v2
1
Oy.

v1
1
y

a) Los invariantes del sistema son:


V1 =(0 1 0)

V2 =(0 0 1)
MO

P1 =(0,0,1)

P2 =(1,0,0)

JJJG
JJJG
OP1 u V1  OP 2 u V2

jk

0 0 1 0

0 u 1  0 u 0
1 0 0 1

z
1

1
0

EC

v1

v2
y

0 1

R M O 1 < 1 1 R < M O 1
x
1 0

b) Obtenemos la ecuacin del eje central determinando el vector de posicin de un punto E
que pertenece a dicho eje:
JJJG
OE

R u MO
R2

0 1
1
1 u 1
2
1 0

1
1
1
2
1

1/ 2

1/ 2
1/ 2

y la ecuacin del eje es:


x  xE
Rx

y  yE
Ry

z  zE
Rz

x  1/ 2
0

y  1/ 2
1

z  1/ 2

x 1/ 2

z 1  y

c) El momento respecto a un eje es la proyeccin sobre el eje del momento respecto a un


punto cualquiera de ese eje.
1 0

M yy M O j 1< 1 1 M yy  j
0 0

d) Sea A el vector cuya recta de accin es el eje Oy. Dado que R=A+B, el nuevo sistema ser:
A (0 O 0)

B R  A (0 1-O 1)

PA

(0, 0, 0)

PB

( x, y , z )

El momento en cualquier punto debe ser el mismo para los dos sistemas
MO

x 0 y  (1  O ) z 1



x
y u 1  O
1
z 1 (1  O ) x 0



1
O 1
A j (0, 0, 0)

x 1

B k (1, 1, 0)
y 1

JJJG
JJJG
OP A u A  OP B u B

y  (1  O ) z

 x 1
(1  O ) x 0

-5-

Fsica Universitaria: Problemas de Fsica

Vectores deslizantes. M02.4

4. Sean dos sistemas de vectores deslizantes definidos por sus torsores {R;M} respectivos:

0 0

1 2

2
,
4
P
1,
0,
0
1
,
3

T1 
T

1
P2 0,1, 0

2


1
2

1 3

a) Reducir cada uno de los sistemas al origen de coordenadas. b) Obtener la resultante y el momento resultante
del sistema total. c) Determinar el eje central del sistema total. d) Obtener el torsor resultante.

El torsor de un sistema de vectores deslizantes queda definido por su resultante R y su


momento resultante M con respecto a un punto P del eje central del sistema (reduccin
cannica).
a) La resultante Ri de cada sistema es invariante; el
momento resultante Mi cambia al pasar a otro punto de
{T1}
reduccin. Reducimos los sistemas al origen de
R1
coordenadas:
{T2}


M1
JJJG
1
2 1 1 2

R2
R1  2 M1,O  M1,P1 OP1 q R1  4 0q2  3
P1
P2
1
2 0 1 4


M2
R
0

M
O
JJJG

0 0 0 1
R 2  1 M 2,O  M 2,P2 OP2 q R 2  3 1q1  3

1
3 0 1 3
b) La resultante R y el momento resultante MO del sistema total de vectores en el origen de
coordenadas es la suma de las resultantes y momentos resultantes de cada uno de los sistemas:
1 0 1
2 1 3

R  R1 R 2  2 1  3
M O  M1,O M 2,O  3 3  6
R  14

1 1 2
4 3 7
c) Determinamos un punto P del eje central del sistema total y la ecuacin de su eje central:
1 3
9
JJG R q M
14 x  9 14 y 1 14 z 3
1 1
O


 3q6  1
OP 
2

1
3
2
R
14 2 7 14 3


d) Proyectamos el momento MO sobre la resultante R para obtener el momento mnimo:
3 1
1
1
M 0 < R 1
R
35
35 1 5
M mn  M mn 
M mn 
6<3 
3  3
R
R
14 7 2
14
14 14 2 2 2
De modo que el torsor resultante es
1 1
5
T 
3 , 3
2 2 2

9
1
3
P ,  , 
14 14 14

-6-

Fsica Universitaria: Problemas de Fsica

Anlisis vectorial. M03.1

1. Sea el campo vectorial: A  12x 2 z 3y 2


i 6xyj 4x3k . a) Demostrar que es conservativo.
b) Calcular su circulacin entre los puntos (0,0,0) y (2,-1,3).

a) Calculamos el rotacional del campo vectorial


0
0
s sx 12 x 2 z 3 y 2



 12 x 2 12 x 2  0  0
6 xy
q A  s syq

6 y  6 y 0
s sz


4 x3

Puesto que el campo vectorial es irrotacional, podemos asegurar que es conservativo.


b) Calculamos la circulacin pedida:
(2,1,3)

A<dr  12 x 2 z 3 y 2
dx
2

(0,0,0)
2

1

y 0
z 0

1

6 xy dy 4 x 3 dz 

x 2
z 0

 0 dx 12 y dy 32 dz  0 6 y 2
0

z
(x,y,z)
(0,0,0)
y

-7-

1
0

x 2
y 1

32 z 0  6 96  102

Fsica Universitaria: Problemas de Fsica

Anlisis vectorial. M03.2

2. Consideremos el campo vectorial A  y i z j x k . a) Es conservativo? Si lo fuese, determnese su


funcin potencial. b) Calcular la circulacin del campo vectorial entre los puntos (2,0,0) y (0,2,2S) a lo largo
de la curva definida por sus ecuaciones paramtricas x = 2 cos T , y = 2 sen T , z = 4 T.

a) La condicin necesaria y suficiente para que un campo vectorial sea conservativo es que
sea irrotacional:
s / sx y 1


rot A  q A  s / syq z  1 v 0

s / sz x 1
de modo que es rotacional y, por ende, no es conservativo.
b) El valor del campo en los puntos de la curva es A  2sen R i 4 j 2 cos R k
El vector desplazamiento infinitesimal (dr) sobre la curva dada es:
dx  2sen R dR
x  2 cos R

y  2sen R l

l dr  2senR i 2 cos R j 4 k
dR
dy  2 cos R dR

z
4
R
d
z
4
d
R



De modo que:
A dr  2sen R i 4 j 2 cos R k
2senR i 2 cos R j 4 k
 4sen 2 R 8R cos R 8cos R

Los ngulos T correspondientes a los puntos inicial y final sobre la cuva son:
P1 (2, 0, 0) :
z1  0
l 4R1  0
l R1  0
P2 (0, 2, 2Q ) : z2  2Q l 4R2  2Q l R2  Q
2
La circulacin ser 1:

1,C

A dr 

4sen 2 R 8R cos R 8cos R


dR 
Q

2
2
2
 2 R sen R cos R 8 R sen R cos R 8 sen R 
0
0
0
 Q 4Q  8 8  3Q

Hemos tenido en cuenta:

sen

R dR 

R  sen R cos R 1
 2 R  14 sen 2R
2

R cos R dR  R sen R cos R


-8-

cos R dR  sen R

Fsica Universitaria: Problemas de Fsica

Anlisis vectorial. M03.3

3. Sea el campo vectorial A  2 xy z 2


i 2 yz x 2
j 2 zx y 2
k . a) Averiguar si este campo es
conservativo y, si lo fuese, determinar la funcin potencial correspondiente. b) En cualquier caso, calcular la
circulacin de este campo vectorial entre los puntos (0,0,0) y (1,2,3) a lo largo de la recta que los une.

a) La condicin necesaria y suficiente para que un campo vectorial sea conservativo es que
sea irrotacional; i.e. que su rotacional sea nulo:
s / sx 2 xy z 2 2 y  2 y

q A  rot A  s / syq2 yz x 2  2 z  2 z  0

s / sz 2 zx y 2 2 x  2 x
Puesto que el campo es irrotacional, es conservativo.
Calculamos la funcin potencial asociada al campo vectorial:
( x, y, z )

G x, y , z


(0,0,0)

A<dr 

2 xy z 2
dx

0
y 0
z 0

 0 dx x 2 d y
0

2 yz x 2
dy

2 zx y 2
dz

x x
z 0

2 zx y 2
dz 
x x
y y

 x 2 y y 2 z z 2 x G0

De modo que

G( x, y, z )  x 2 y y 2 z z 2 x G0

b) Calculamos la circulacin entre los dos puntos dados como


la diferencia de valores que toma la funcin potencial en esos
dos puntos:

1,2,3

0,0,0

A d r  G 1, 2,3
 G 0, 0, 0
 23
 0
 23

(x,y,z)
(0,0,0)
x

-9-

Fsica Universitaria: Problemas de Fsica

Anlisis vectorial. M03.4

4. Sea el campo vectorial A = (x + yz) i +( y + xz) j + (z + xy) k.. a) Demostrar que es un campo de potencial.
b) Obtener su funcin potencial. c) Calcular la circulacin del campo vectorial entre los puntos de
coordenadas (3,0,0) y (0,3,0) a lo largo del arco de circunferencia determinado por esos dos puntos y que
tiene su centro en el origen de coordenadas. d) Calcular la divergencia del campo vectorial en el origen de
coordenadas. e) Utilizando el teorema de Gauss, calcular el flujo del campo vectorial a travs de una esfera
de radio 3 unidades centrada en (0,0,0).

a) Calculamos el rotacional del campo:


s / sx x yz x  x

q A  s / sy q y xz  y  y  0

s / sz z xy z  z

por tanto, por ser irrotacional, es conservativo.


b) Para obtener la funcin potencial calculamos la circulacin del campo entre el origen de
coordenadas (0,0,0) y un punto genrico (x,y,z) a lo largo de tres tramos rectilneos en las
direcciones de los ejes coordenados respectivos.
( x, y, z )

G  G0 

(0,0,0)

A dr 

( x yz ) dx

y0

z0

De este modo que


1
G  ( x 2 y 2 z 2 ) xyz G0
2
c) Calculamos la circulacin entre los dos puntos dados como la diferencia de valores que
toma la funcin en esos dos puntos:

0,3,0

3,0,0

A d r  G 0,3, 0
 G 3, 0, 0
 0

d) La divergencia vale
A 

sAx sAy sAz




 1 1 1  3
sx
sy
sz

en todos los puntos del espacio.


e) Teorema de Gauss:
'
v A dS  ( A) dV  3 dV  3 V
S

y por tratarse de una superficie esfrica de 3 unidades de radio, ser:


4
'  3 q Q R 3  4Q R 3  4Q33  339.3 (unid. de flujo)
3

- 10 -

Fsica Universitaria: Problemas de Fsica

Anlisis vectorial. M03.5

5. Consideremos el campo vectorial A  2 x yz


i 2 y xz
j 2 z xy
k . a) Es conservativo? Si lo
fuese, determnese su funcin potencial. b) Calcular la circulacin del campo vectorial entre los puntos
(2,0,0) y (0,2,2S) a lo largo de la a lo largo de la recta que los une.

a) La condicin necesaria y suficiente para que un campo vectorial sea conservativo es que
sea irrotacional:



s / sx 2 x yz 0

rot A  q A  s / sy q2 y xz  0  0

s / sz 2 z xy 0
de modo que es irrotacional y, por ende, es conservativo.
Calculamos la funcin potencial:
( x, y, z )

G x, y , z


(0,0,0)

A<dr  2 x yz
dx 2 y xz
dy 2 z xy
dz 
0
y 0
z 0

x x
z 0

x x
y y

 2 xdx 2 ydy 2 z xy
dz  x 2 y 2 z 2 xyz G0
0

De modo que G( x, y, z )  x 2 y 2 z 2 xyz G0


b) Calculamos la circulacin entre los dos puntos dados como la diferencia de valores que
toma la funcin en esos dos puntos:

0,2,2 Q

2,0,0

A d r  G 0, 2, 2Q
 G 2, 0, 0
 4 4Q 2
 4
 4Q 2

Otro mtodo
Por ser conservativo, la circulacin entre dos puntos es independiente del camino que
sigamos. Para mayor simplicidad, calculamos la circulacin entre los puntos (2,0,0) y (0,2,2S)
a lo largo de la lnea quebrada que se indica en la figura:
(0,2,2 Q )

(2,0,0)

2
y 0
z 0

2Q

 2 xdx 2 ydy
2

2Q

A<dr  2 x yz
dx 2 y xz
dy

x0
z 0
0

2Q

2 zdz  x 2 y 2 z 2

x0
y2

 4 4 4Q 2  4Q 2

(0,2,2)

x=0
y=2
x=0
z=0

y=0
z=0

(2,0,0)

- 11 -

2 z xy
dz 

Fsica Universitaria: Problemas de Fsica

Anlisis vectorial. M03.6

6. En un sistema de coordenadas cartesianas, el campo


vectorial A es perpendicular al eje z y dirigido hacia dicho
eje en todo punto del espacio. El mdulo del vector A en
un punto cualquiera es inversamente proporcional al cubo
de la distancia del punto al eje z. a) Expresar el campo
vectorial A en coordenadas cartesianas. b) Demostrar que
tiene funcin potencial. c) Determinar la funcin potencial.
d) Describir la forma de las superficies equipotenciales.

er A
P(x,y,z)

er

y
P(x,y,0)

a) Puesto que el campo tiene simetra cilndrica,


podemos reducirlo a un campo vectorial plano, contenido en el plano xy o plano z = 0, tal
como se indica en la figura. La distancia del punto genrico P(x,y,0) al eje z y el versor e r en
la direccin radial vienen dados por
JJG
xi y j
E 2  x2 y 2 l E  x2 y 2
OP  x i y j  Ee r l e r 
x2 y2

k
k JJG
k
kx
ky

j
i
j
x

y



A( x, y )   3 e r   4 OP  
i

2
2
2 2
2 2
2
2 2
E
E
x y

x y

x y

b) Puesto que se trata de un campo vectorial central, podemos afirmar que es irrotacional y
que, consecuentemente, tendr funcin. A pesar de ello, comprobaremos que es irrotacional:

s / sx Ax ( x, y )
 0
0
q A  s / syq Ay ( x, y ) 

s / sz 0 sA sx
 sA sy

y
x

sAy
2ky x 2 y 2
2 x 4kxy x 2 y 2

ky
4kxy







3

2
2 2
2
2 4
2
2 4
2

x
s
x y

x y

x y

x y2

Ya que

2kx x 2 y 2
2 y 4kxy x 2 y 2

sAx
4kxy
kx






2
4
4
3

2
2
2
2
2
2
2
sy

x y

x y

x y

x y2

Por consiguiente, el campo es irrotacional y existe una funcin potencial asociada al mismo.
c) Puesto que el campo vectorial se anula en los puntos
infinitamente alejados del eje, tomaremos el nivel de
potencial nulo en el infinito y realizamos la integracin a
z
lo largo de una recta radial:
E

G E
 A<dr  
d

k
k
k
e r <dr   3 dE  2
E3
E
2
E
d

De modo que

G ( x, y , z ) 

k
2 x y 2

e
y

P(x,y,0)

d) El potencial ser el mismo en todos los puntos equidistantes del eje z; por consiguiente, las
superficies equipotenciales consisten en superficies cilndricas que tienen al eje z como eje
de revolucin.

- 12 -

Fsica Universitaria: Problemas de Fsica

Cinemtica de la partcula. M04.1

1. El maquinista de un tren expreso que circula con una velocidad v1 observa a una distancia d el furgn de
cola de un tren de mercancas que marcha por delante del expreso, sobre la misma va y en el mismo
sentido, con una velocidad v2, menor que la del expreso. El maquinista del expreso aplica inmediatamente
los frenos, producindose una desaceleracin constante a, mientras que el tren de mercancas contina su
marcha a velocidad constante. Determinar el menor valor de la desaceleracin para que pueda evitarse la
colisin.

Escribimos las ecuaciones horarias o temporales del movimiento de cada uno de los dos
mviles intervinientes, conforme a la notacin indicada en el esquema adjunto:
1

x1 v1t  at 2
expreso
2

expreso
mercancas x2 d  v2 t
d

En un diagrama espacio-tiempo (x,t), estas funciones estn representadas por una parbola y
una recta, respectivamente, como se indica en
la figura. Resolviendo el sistema de las dos
x
movimiento
ecuaciones, determinamos si en algn instante
uniforme del
coinciden en el mismo lugar la mquina del
mercancas
expreso con el furgn de cola del mercancas;
esto es,
1
v1t  at 2 d  v2 t o
2
desaceleracion
2
crtica
at  2 v1  v2 t  2d 0
a

v1

v2

mercancas
x

movimiento
del expreso

Esta ecuacin de segundo grado tiene como


soluciones o races

v1  v2 r v1  v2

 2ad

a
Esto es, una, dos o ninguna solucin. dependiendo del valor del discriminante
2
 v1  v2  2ad :
Si  >0 : v1  v2 ! 2ad

o a

Si  =0 : v1  v2

o a

2ad

Si  <0 : v1  v2  2ad
2

a!

v1  v2

, dos soluciones reales distintas. Hay colisin.

2d

v1  v2

, dos soluciones reales iguales. Hay contacto.

2d

v1  v2
2d

, no hay solucin real. No hay colisin.

Otro mtodo: Movimiento relativo. Describimos el movimiento del expreso en el referencial del mercancas, de
modo que su velocidad es v12 v1  v2 . Para evitar la colisin, la velocidad relativa deber anularse antes de
que el expreso recorra la distancia d que le separa del mercancas: i.e.,
2
vrel

v122  2ad

0 o a

- 13 -

v122
2d

v1  v2
2d

Fsica Universitaria: Problemas de Fsica

Cinemtica de la partcula. M04.2

2. Despus de parar el motor de una canoa, sta tiene una aceleracin en sentido opuesto a su velocidad y
directamente proporcional al cuadrado de sta. a) Expresar la velocidad de la canoa en funcin del tiempo.
b) dem la distancia recorrida al cabo de un tiempo t. c) dem la velocidad de la canoa despus de haber
recorrido una distancia x. d) Supongamos que cuando se para el motor la velocidad de la canoa era de
20 m/s y que 15 s despus dicha velocidad se haya reducido a la mitad. Determinar el valor de la constante
de proporcionalidad que aparece en la definicin de la aceleracin.

De acuerdo con el enunciado, la aceleracin


a v
a0 v
0
viene dada en funcin de la velocidad mediante
la expresin a  kv 2 , por lo que se trata de un
t
t0
movimiento rectilneo variado general; i.e., no se
x
trata de un movimiento rectilneo uniformemente
acelerado, ya que la aceleracin no es constante
a) A partir de la definicin de la aceleracin y mediante integracin obtenemos la velocidad
en funcin del tiempo:

dv
dt

kv 2

dv
v0 v2
v

t
1
k dt o 
0
v v0

kt o

1
v

1
 kt
v0

b) Del mismo modo, a partir de la definicin de la velocidad y mediante integracin,


obtenemos la posicin o distancia recorrida en funcin del tiempo:

dx
dt

dt
v o
dx

1
v

1
 kt o
v0

1 du
k u

1
ln u
k

1 1
ln  kt
k v0
0

dx

dt

1
 kt
v0

1
 kt
1 v0
ln
1
k
v0

 kt
u
v0

du kdt

1
ln 1  kv0 t
k

c) De nuevo, a partir de la definicin de la aceleracin y mediante integracin obtenemos la


velocidad en funcin del espacio recorrido:
v dv
x
v
dv d x
dv
a
v
 kv 2 o
 k dx o ln
 kt o v v0 e  kt
0
v0 v
v0
dx dt
dx
d) A partir de la expresin de la velocidad en funcin del tiempo, obtenida en el primer
apartado, despejamos la constante de proporcionalidad k y determinamos su valor:
1  1
1 1
1

v
v0
1 1
1
10
20
20
 kt o k
0.003 m -1
v v0
t
15
15
300

- 14 -

Fsica Universitaria: Problemas de Fsica

Cinemtica de la partcula. M04.3

3. La velocidad de un vehculo quitanieves es inversamente proporcional al tiempo transcurrido desde que


comenz a nevar. Transcurrido un cierto tiempo, t0, a partir del instante en que empez a nevar, el vehculo
se pone en marcha y recorre 2 km en la primera hora y 1 km en la segunda. a) Determinar la ecuacin del
movimiento del vehculo, i.e., x(t). b) Calcular el valor de t0 y el de la constante de proporcionalidad.
c) Qu distancia recorrer el vehculo durante la tercera hora de funcionamiento?

a) Aplicamos la definicin de velocidad e integramos para obtener la distancia al origen (x) en


funcin del tiempo:

dx
dt

k
t

dx

t0

dt
t

k ln

t
t0

b) Sustituimos en esta expresin los datos que nos proporciona el enunciado, expresando las
distancias en kilmetros (km) y los tiempos en horas (h):

t0  1

2
t1 t0  1 o x1 2 o k ln t

0
o (y)

t t  2 o x 3 o k ln t0  2 3
2
2 0
t0
Desarrollamos la ecuacin anterior
3

t0  1
t0

2 ln

? t0

1 r 1  4
2

2
3

t  1 t0  2
t0  2
o 0

o
t0
t0 t0
t03  3t02  3t0  1 t03  4t02  4t0 o t02  t0  1 0
3ln

t0  1
t0
t0  2
ln
t0
ln

1 r 5
2

t0  1

t0 t 0  2

1  5
0.618 h

1  5  0
2

Ahora determinamos el valor de la constante k:


x

k ln

t
t0

o k

x
t
ln
t0

x1
t0  1
ln
t0

2
1.618
ln
0.618

2.078 km o

2.078ln

t
0.618

c) Utilizamos la expresin anterior para determinar la posicin de la mquina quitanieves en


el instante t3 = t0+3 y el recorrido durante la tercera hora de funcionamiento:
3.618
x3 2.078ln
3.672 km o x3 x3  x2 0.672 km
0.618

t=

t0

t0+1

t0+2

x=

2 km

3 km

2 km

1 km

- 15 -

Fsica Universitaria: Problemas de Fsica

Cinemtica de la partcula. M04.4

4. Un transbordador navega en lnea recta con una velocidad constante v0 = 8 m/s durante 60 s. A continuacin,
detiene sus motores; entonces, su velocidad viene dada en funcin del tiempo por la expresin

v0 t12 / t 2 , siendo t1 = 60 s. Cul es el desplazamiento del transbordador en el intervalo 0  t  f ?

El espacio recorrido con velocidad constante hasta el instante t1 = 60 s es


x1 v0 t1 8 u 60 480 m
A partir de ese instante, la velocidad va disminuyendo, por lo que obtendremos el recorrido
mediante una integracin:
v

v0 t12
t2

dx
dt

o dx

v0 t12
dt o
t2
f

xf  x1

v0 t12 

1
t t1

xf

x1

dx

1 1
v0 t12 
t1 f

v0 t12

t1

dt
t2

v0 t1

de modo que el desplazamiento total en el intervalo 0  t  f es


xf x1  v0 t1 v0 t1  v0 t1 2v0 t1 960 m

8 m/s
960 m

480 m

x
v

60 s

- 16 -

120 s

Fsica Universitaria: Problemas de Fsica

Cinemtica de la partcula. M04.5

5. El bloque de la figura est unido al extremo un hilo inextensible que pasa por una polea B. Para acercar el
bloque masa hacia s, un operario hace descender el extremo A del hilo con una velocidad constante de
1 m/s. Calcular la velocidad y la aceleracin que tendr la masa cuando pase por el punto C, indicado en la
figura, situado a 8 m del operario.

Consideramos el sistema de ejes de la figura y establecemos


la relacin existente entre la distancia l y la distancia x:
l2 = x2 + 62
y la derivamos respecto al tiempo
dl
dx
dl
dx
2l
2x
l
x
dt
dt
dt
dt
As, cuando x = 8 m y teniendo en cuenta que dl/dt es la
velocidad de decrecimiento de la longitud l, que coincide
con la velocidad con que desciende el extremo A del hilo

6m

l
x

(i.e., dl/dt = - 1 m/s), se obtiene


dx
dt

l dl
x dt

82  6 2
5
1  m/s = - 1.25 m/s
8
4

Derivamos de nuevo
2

d2l
dl
l 2
dt
dt
y teniendo en cuenta que

d2l
d t2

d2 x
dx

x 2
dt
dt

0 y que para x = 8 m es

dx
dt

5
 m/s , despus de despejar se
4

obtiene
d2 x
d t2

2
2
1 d l d x


x d t d t

1 2 25
1 
8
16

- 17 -

9
m/s 2
128

 0.0703 m/s 2

Fsica Universitaria: Problemas de Fsica

Cinemtica de la partcula. M04.6

6. Si el cuerpo A de la figura se mueve hacia la izquierda con una celeridad de


6 m/s, determinar la celeridad del cuerpo B. Adems, si la celeridad del cuerpo A
disminuye a razn de 1 m/s2, determinar la aceleracin del cuerpo B.

Establecemos la condicin de que la longitud de la cuerda permanece constante, adoptando el


convenio de signos que se indica en la figura,
4 xA  2 xB cte.
y la derivamos con respecto al tiempo
4 xA  2 xB 0 o xB 2 xA

o vB

2vA

2 u 6

12 m/s

de modo que el cuerpo B se mueve hacia la izquierda (al contrario de lo indicado en la figura).
Derivamos de nuevo con respecto al tiempo para obtener las aceleraciones:
4 
xA  2 
xB

0 o 
xB

2 
xA

o aB

2aA

2 u (1)

2 m/s 2

de modo que el cuerpo B presenta una aceleracin en sentido contrario a su velocidad, por lo
que sta disminuye.
En la figura adjunta se indican los sentidos reales de las velocidades y aceleraciones.

vA, aA

vB, aB

B
O
xA

- 18 -

xB

Fsica Universitaria: Problemas de Fsica

Cinemtica de la partcula. M04.7

7. La deslizadera A se mueve hacia la derecha, por la gua rectilnea horizontal, con una velocidad vA
constante. La deslizadera A est unida al bloque B mediante un hilo inextensible que pasa por una polea en
C. Calcular velocidad y aceleracin del bloque B en funcin de la distancia x que se indica en la figura.

Sea L la longitud del hilo. Escribimos la condicin


geomtrica de ligadura:
L

x h s
2

y la derivamos con respecto al tiempo:


2 xx
x
 s 0 o s vB 
vA
2
2
2
2 x h
x  h2
Para obtener la aceleracin tangencial del bloque B
debemos derivar de nuevo con respecto al tiempo:

x x 2  h 2  x
at


s 

x h
2

C
s

T
O

xx
x h
2

vA

x 2 x  x ( x 2  h 2 )
vA
( x 2  h 2 )3 / 2

La aceleracin normal del bloque B ser:


an

vB2
R

x 2 vA2
x2  h2 R

- 19 -

h 2 vA2
( x  h 2 )3 / 2
2

Fsica Universitaria: Problemas de Fsica

Cinemtica de la partcula. M04.8

8. Sobre un terreno horizontal, lanzamos una pelota, verticalmente hacia arriba, con una velocidad inicial de
10 m/s. El viento ejerce sobre la pelota una fuerza horizontal igual a la quinta parte de su peso. a) Calcular
la altura mxima que alcanza la pelota y su velocidad (mdulo y direccin) en ese instante. b) Determinar la
distancia entre el impacto en el suelo y el punto de lanzamiento, as como la velocidad de la pelota (mdulo
y direccin) en ese instante.

y
6

Se trata de la composicin de dos movimientos uniformemente acelerados en direcciones perpendiculares


entre s, cuyas aceleraciones son

v(A)

ax 0.2 g

a y  g
Mediante dos integraciones sucesivas, obtenemos

g/5

3
2

v0
g

1
1

B
2

5

v(B)

1 2

0.1gt 2
x 2 ax t
x
o
v0  gt
y v t  1 gt 2
0

2
a) En el punto ms alto de la trayectoria ser:
v0
v y (A) 0 o tA
g

vx

v y

ax t

0.2 gt

La altura mxima alcanzada y la velocidad en ese instante sern:


v02 1 v02
 g
g 2 g2

yA
v(A)

vx (A)

v02
2g
v
0.2 g 0
g

102
2 u 9.8
0.2v0

5.10 m
0.2 u 10

2 m/s

b) Cuando la pelota regresa al suelo, ser:

y (B)

v0 tB 

1 2
gtB
2

0 o tB

2v0
g

El alcance y las componentes de la velocidad en ese instante sern:


xB

0.1g

4v02
g2

0.4 g

v02
g

0.4

102
9.8

4.08 m

2v0

0.4v0 0.4 u 10 4 m/s


vx (B) 0.2 gtB 0.2 g g

2v
v (B) v  gt
v0  g 0 v0 10 m/s
0
B
y
g
El mdulo y direccin de dicha velocidad son:
10
v(B)
42  102
116 10.77 m/s
arg tg 2.5
T arctg
4

- 20 -

68

Fsica Universitaria: Problemas de Fsica

Cinemtica de la partcula. M04.9

9. Un muchacho que est situado a 4 m de una pared vertical lanza


contra ella una pelota segn indica la figura. La pelota sale de su
mano a 2 m por encima del suelo con una velocidad inicial v = (10i +
10j) m/s. Cuando la pelota choca en la pared, se invierte la
componente horizontal de su velocidad mientras que permanece sin
variar su componente vertical. A qu distancia de la pared caer la
pelota al suelo?

v0

Tomamos un sistema coordenado de referencia con origen en


el punto de lanzamiento de la pelota, como se indica en la
figura. Podemos simplificar la resolucin de problema observando que la pared acta como
un espejo, de modo que consideraremos la trayectoria virtual que se indica en la figura
inferior. Escribimos las ecuaciones paramtricas del movimiento de la pelota y, a partir de
ellas, eliminando el tiempo, obtenemos la ecuacin de la trayectoria:
x

x v0 x t o t v
0x

? y

y v t  1 gt 2 o y v0 y x  g x 2
0y
2

v0 x
2v02x
La pelota toca el suelo cuando y = -2 m, de modo que
2
x

x  0.049 x 2

10
9.8 2
x
x
10
2 u 102

o 0.049 x 2  x  2

1 r 1  8 u 0.049
2 u 0.049

1 r 1.1798
0.098

0 o

22.24 m

(negativo)

lo que representa una distancia a la pared de


D 22.24  4.00 18.24 m

x  0.049 x 2

v0
x

- 21 -

Trayectoria
virtual

x  0.049 x 2

Fsica Universitaria: Problemas de Fsica

Cinemtica de la partcula. M04.10

10. En un cierto instante la celeridad de una partcula es de 20 m/s y el mdulo de su aceleracin es 3 m/s2. En
ese instante, los vectores velocidad y aceleracin forman entre s un ngulo de 30. Determinar la curvatura
y el radio de curvatura de la trayectoria de la partcula en ese instante.

Componentes intrnsecas de la aceleracin:


at a cos T
o
an a sen T
De la relacin existente entre la aceleracin centrpeta o
normal y el radio de curvatura, se sigue:
at e t  an e n

an

v2

o U

v2
an

v2
a sen T

Sustituyendo los valores dados en el enunciado:


v2
202
267 m
a sen T 3sen 30
La curvatura se define como la inversa del radio de curvatura:
1 a sen T 3sen 30
0.00375 m -1
N
U
v2
202

- 22 -

v
at

a
an

C

Fsica Universitaria: Problemas de Fsica

Cinemtica de la partcula. M04.11

11. Una partcula, que se mueve con aceleracin constante a = 2 i + 3 j + k (S.I.), pasa por el origen de
coordenadas en el instante inicial (t = 0) con una velocidad v = - 3 i - 2 j (S.I.). a) Escribir las expresiones de
la velocidad y las ecuaciones de la trayectoria en funcin del tiempo. b) Determinar el instante en que la
velocidad es mnima y el valor de sta. c) Dgase que tipo de trayectoria sigue la partcula (circular,
rectilnea, elptica, u otra).

Puesto que la aceleracin es constante, las expresiones de la velocidad y del vector de


posicin sern:

v 0  at

r0  v 0 t  12 at 2

a) Sustituyendo en estas expresiones las condiciones propuestas, obtenemos:


v

3 2

2  3 t
0 1

3  2t

2  3t
t

3
2
1 2
2 t  2 3 t
0
1

3  t 2

2
2  1.5t
0.5t 2

y las ecuaciones paramtricas de la trayectoria son


x

3  t 2

2  1.5t 2

0.5t 2

b) La celeridad o mdulo de la velocidad viene dado por


v2

(3  2t ) 2  (2  3t ) 2  t 2

14t 2  24t  13

de modo que derivando con respecto al tiempo e igualando a cero (condicin de mximo o de
mnimo), tenemos:
d(v 2 )
24 6
28t  24 0 o t
s
dt
28 7
y la celeridad en ese instante es
36
6
504  1008  637 133
v 2 14  24  13
2.71 (m/s) 2 o v 1.65 m/s
49
7
49
49
c) Con carcter general, cualquier movimiento en el que la aceleracin sea constante presenta
una trayectoria parablica. El paradigma de tales movimientos es el movimiento de un
proyectil en el campo gravitatorio.

- 23 -

Fsica Universitaria: Problemas de Fsica

Cinemtica de la partcula. M04.12

12. Una partcula se mueve en el plano xy con aceleracin constante. Para t = 0, la partcula se encuentra en la
posicin r0 = 4 i + 3 j m y se mueve con velocidad v0. Para t = 2 s, la partcula se ha desplazado a la
posicin r2 =10 i 2 j m y su velocidad ha cambiado a v2=5 i 6 j m/s. Determinar: a) La velocidad v0.
b) La aceleracin de la partcula. c) La velocidad de la partcula en funcin del tiempo. d) La ecuacin de la
trayectoria. e) Las aceleraciones normal y tangencial y el radio de curvatura para t = 2 s.

Puesto que la aceleracin es constante, las expresiones de la velocidad y del vector de


posicin sern:

v 0  at

r0  v 0 t  12 at 2

Sustituyendo en estas expresiones las condiciones propuestas, obtenemos:


5 v0 x
ax
[1] v0 x  2ax 5


6 v0 y  2 a y o [2] v  2a
6
0y
y

0 0
0


10 4
v0 x
ax
2
[3] 2v0 x  2ax 6

2
2 3  2 v0 y  2 a y o [4] 2v  2a
5
0y
y

0 0
0
0



de modo que disponemos de cuatro ecuaciones con cuatro incgnitas. Resolvindolas,
tenemos:
[1] v0 x  2ax 5
v0 x 1 m/s
o

2
[3]
2
v
2
a
6

ax 2 m/s
0x
x

Los resultados pedidos son:

v0

1

1 m/s a
0

2
3.5 m/s
0

[2] v0 y  2a y 6
v0 y 1 m/s
o

2
ax 3.5 m/s
[4] 2v0 y  2a y 5

4.03 m/s

1  2t

1  3.5t m/s r
0

4  t  t2

2
3  t  1.75t m

e) En el instante t = 2 s, sern
at

av v

v v

an

av
2 v
v

a  at

2 5
5
155
1
31 1

<
3.5
6
6
at 3.97 m/s 2



v

61 186
61
61
0
0
0 0

2
155
33

1
1

an 0.70 m/s 2
3.5  61 186 61 27.5
0
0
0

v2
an

61
= 86.6 m
0.70

- 24 -

Fsica Universitaria: Problemas de Fsica

Cinemtica de la partcula. M04.13

13. Las ecuaciones temporales del movimiento de una partcula son: x = 4 cos t, y = 4 sen t, z = 3t. Hllense, de
forma genrica, para cualquier instante: a) La velocidad y aceleracin. Determinar, tambin, los mdulos de
la velocidad y de la aceleracin. b) Si la trayectoria es plana o no. c) El radio de curvatura de la trayectoria.

a) Obtenemos la velocidad y la aceleracin de la partcula por derivacin


r

4 cos t

4sen t o v
3t

dr
dt

4sen t

4 cos t o a
3

dv
dt

4 cos t

4sen t
0

Sus mdulos son:


v

16sen 2 t  16 cos 2 t  9

16 cos t  16sen t
2

16  9
16

c) La curvatura y el radio de curvatura de la trayectoria se obtienen a partir de la velocidad y


de la aceleracin utilizando la expresin:

vua

v3

Calculamos el producto vectorial y su modulo:


vua

4sen t 4 cos t

4 cos t u 4sen t

3 0
vua

12sen t


12
cos
t

16sen 2 t  16 cos 2 t

122  162

12sen t

12 cos t
16

20

El radio de curvatura es constante y vale:


v3
53 125
6.25
v u a 20 20
b) En el triedro mvil, intrnseco o de Frenet, calculamos el versor binormal:

12sen t 0.6sen t
1

eb
12 cos t 0.6 cos t
20

16 0.8
Este versor no es constante, ya que cambia su direccin en el transcurso del tiempo (i.e., de un
punto a otro de la trayectoria), por lo que sta no es plana, sino alabeada. En concreto, se
trata de una trayectoria helicoidal uniforme, de radio R = 4, cuyo eje es el eje z, de paso
constante h, tal que
v
3
h 2S z 2S
6S 18.85
1
Z

vua
vua

- 25 -

Fsica Universitaria: Problemas de Fsica

Cinemtica de la partcula. M04.14

14. Una partcula se mueve en el plano de tal forma que las componentes cartesianas de su velocidad vienen
3
dadas en funcin del tiempo por las expresiones: vx 4t  4t , v y 4t (SI). En el instante inicial
t0 = 0 s, el mvil se encontraba en la posicin x0 = 1 m, y0 = 2 m. Calcular: a) Las componentes de la
aceleracin en cualquier instante. b) Las coordenadas x e y del mvil en funcin del tiempo.

a) Obtenemos las componentes de la aceleracin derivando las de la velocidad:


dv x
d

ax
4t 3  4t 12t 2  4

dt
dt
o
dv y
4t
d
a
4t 4
y

dt
dt
b) Obtenemos las coordenadas de posicin integrando las componentes de la velocidad:
vx

v y

4t 3  4t

dx

3
x dx
vx dt 4t  4t
dx 4t 3  4t dt
1
o
o y

d
y
v
dy
dy 4tdt
4t
2
y dt
x  1 t 4  2t 2
x t 4  2t 2  1
o

2
2
y  2 2t
y 2t  2

- 26 -

4t  4t dt
4tdt
t

Fsica Universitaria: Problemas de Fsica

Cinemtica de la partcula. M04.15

15. El movimiento de una partcula viene dado por el vector


r

(1  t 2 ) i  (1  t  3t 2 ) j  (t  2t 2 ) k .

a) Demostrar que dicho movimiento es plano, con el origen del vector r en dicho plano. b) Hallar un vector
normal al plano del movimiento.

a) Determinamos la velocidad y la aceleracin por derivacin:


r

1 t2
2t
2


2
1  t  3t o v 1  6t o a 6
1  4t
4
t  2t 2

El versor binormal a la trayectoria en un punto


genrico del mismo viene dado por
eb

vua
vua

1
2
1
2 3

1

1
3
1
3

1

eb

trayectoria

et
v

en

plano
osculador

cte.

ya que
2t 2 4  24t  6  24t 2
1




vua 2 3

u


t
t
t
1
6
6
2
8
8
2
2



1
1  4t 4 12t  2  12t 2
1




Como el versor binormal es perpendicular al plano osculador, definido por los vectores
velocidad y aceleracin en cada punto de la trayectoria, ste tambin permanecer constante,
por lo que la trayectoria descrita por el mvil es plana, por estar contenida en dicho plano.
Para demostrar que el origen de coordenadas est contenido en el plano de la trayectoria
(plano osculador) , es suficiente demostrar que el vector de posicin r es perpendicular al
versor binormal en todos los puntos de la trayectoria; i.e., que el vector de posicin r est
contenido en el plano osculador. En efecto,
vua

r < eb

1 t2
1

3
2
1  t  3t < 3 1
1
t  2t 2

3
1  t 2  1  t  3t 2  t  2t 2
3

b) El versor binormal determinado en el apartado anterior es perpendicular al plano de la


trayectoria

- 27 -

Fsica Universitaria: Problemas de Fsica

Cinemtica de la partcula. M04.16

16. El vector posicin de un mvil puntual viene dado en funcin del tiempo por la expresin

r  4cos10t i 5sen10t j 3cos10t k en la que todos los valores estn expresados en unidades del
sistema internacional. a) Hallar la velocidad y aceleracin del mvil en cualquier instante. b) Demostrar que
la trayectoria es plana y determinar el versor normal a dicho plano. c) Demostrar que el origen de
coordenadas est contenido en el plano de la trayectoria. d) Hallar las aceleraciones tangencial y normal y el
radio de curvatura en un punto genrico de la trayectoria. e) De acuerdo con los resultados anteriores,
indquese que tipo de movimiento tiene el mvil.

a) Determinamos la velocidad y la aceleracin por derivacin:


r

4 cos10t

5 sen10t m o v
3 cos10t

40 sen10t

m
50 cos10t s
30 sen10t

dr
dt

o a

dv
dt

400 cos10t

m
500 sen10t s 2
300 cos10t

b) El versor binormal a la trayectoria en un punto genrico


del mismo viene dado por

z
trayectoria

eb

et
15000
3 0.6
vua
1
1

v
eb
0
0
0
cte.
plano
r
v u a 25000 20 000 5 4 0.8
en
osculador

y como dicho vector es perpendicular al plano osculador,


a
y
definido por los vectores velocidad y aceleracin en cada
x
punto de la trayectoria, ste tambin permanecer constante,
por lo que la trayectoria descrita por el mvil es plana, ya que est contenida en dicho plano.
c) Para demostrar que el origen de coordenadas est contenido en el plano de la trayectoria, es
suficiente demostrar que el vector de posicin r es perpendicular al versor binormal en todos
los puntos de la trayectoria; en efecto,

4 cos10t 0.6

r < e b 5sen10t < 0 2.4 cos10t  2.4 cos10t 0


3cos10t 0.8

d) Las componentes intrnsecas de la aceleracin y el radio de curvatura se obtienen a partir


de las expresiones:

at

v <a
v

0
50

an

vua
v

25000
50

500 m/s 2

v2
an

502
500

2500
500

5m

e) El mvil recorre una trayectoria circular de 5 m de radio con una celeridad constante de
50 m/s.
v 2  1600 sen 2 10t 2500 cos 2 10t 900 sen 2 10t  2500 sen 2 10t cos 2 10t
 2 500 m/s

a 2  160 000 cos 2 10t 250 000 sen 2 10t 90 000 cos 2 10t  250 000 m/s

2 2

l v  50 m/s

l a  500 m/s 2

40sen10t 400cos10t
v < a  50 cos10t <500sen10t  16 000  25000 9 000
sen10t cos10t  0
30sen10t 300cos10t
15000
40sen10t 400 cos10t
15000cos 2 10t  15000sen 2 10t

v q a  50 cos10t q500sen10t  12 000sen10t cos10t  12 000sen10t cos10t  0


30sen10t 300 cos10t

2
2
20000
20000sen 10t 20000 cos 10t

v q a  150002 200002  25000 m 2 / s3

- 28 -

Fsica Universitaria: Problemas de Fsica

Cinemtica de la partcula. M04.17

17. Dadas las ecuaciones paramtricas (temporales) del movimiento de una partcula: x = 2t, y = t2, z = t3/3,
determinar: a) Las componentes intrnsecas de su aceleracin en el instante t = 1; b) el radio de curvatura de
la trayectoria en dicho instante.

Calculamos la velocidad y la aceleracin de la partcula en un instante genrico t:


2t

r t2 o v
t3 / 3

En el instante t=1 ser:


r

1
1/ 3

2

2t o a
t2

2

2 o v
1

0
v

2
o


a
2t

4  4t 2  t 4
4  4t 2

2 1 t2

0
v 3

2 o a 2 2

a) Determinamos el versor tangente a la trayectoria en ese instante: e t

v
v

2
1
2 .
3
1

Calculamos el mdulo de la aceleracin tangencial en el instante t = 1:


0 2
1 1
at a<e t 2 < 2
0  4  2 2
2 3 1 3

Ahora podemos determinar las componentes intrnsecas (tangencial y normal) de la
aceleracin en el instante t = 1:
2 4 / 3
2

2
4 / 3 o an a  at
3

1 2 / 3
b) El radio de curvatura se determina a partir de
aceleracin normal, ya que an v 2 U , con
at

at e t

an

0 4 / 3

2  4 / 3
2 2 / 3

los mdulos

2
4 1 4
3

De modo que

v2
an

9
2

- 29 -

4.5

4 / 3
2

2
2
/
3

1
4/3 3 2


de la velocidad y de la

Fsica Universitaria: Problemas de Fsica

Cinemtica de la partcula. M04.18

18. Una lancha motora, que navega ro arriba, se encontr con una balsa arrastrada por la corriente. Una hora
despus de este encuentro, el motor de la lancha se averi. La reparacin dur 30 min; durante este tiempo
la lancha fue arrastrada por la corriente. Reparado el motor, la lancha naveg ro abajo con la misma
velocidad (respecto del ro) que antes de la avera, y alcanz a la balsa a una distancia de 7.5 km del punto
de su primer encuentro. Determinar la velocidad de la corriente del ro, considerndola constante.

La resolucin del problema es muy simple si lo planteamos en un referencial (el del ro) en el
que la balsa se encuentra en reposo. En ese referencial, la lancha tambin est en reposo
durante los 30 min que dura la reparacin de la avera y su velocidad (en mdulo, no en
direccin) es la misma cuando navega ro arriba que cuando lo hace ro abajo.
En consecuencia, cuando la lancha navega ro abajo, despus de la reparacin, emplear de
nuevo 1 h en alcanzar a la balsa.
As, el tiempo total que habr transcurrido desde el primer encuentro y el reencuentro con la
balsa ser de 1 h + 30 min + 1 h =2.5 h. Durante ese tiempo, la balsa, arrastrada por la
corriente, ha recorrido una distancia (respecto a tierra) de 7.5 km. De este modo, la velocidad
de la balsa (respecto a tierra), y tambin la velocidad de la corriente, ser:
7.5 km
km
v
3
2.5 h
h

balsa v0 (2.5 h)
v0 (0.5 h)

v+v0 (1 h)

lancha

corriente

v-v0 (1 h)

7.5 km

0 km

Tambin podemos resolver el problema en el sistema de referencia de tierra. En este referencial, la balsa se
desplaza con velocidad constante v0 (la misma que lleva la corriente del ro). Sea v la velocidad de la lancha con
respecto al ro. La lancha motora lleva una velocidad (vv0) durante 1 h (cuando remonta el ro), una velocidad
v0 durante 0.5 h (durante la avera, arrastrada por la corriente) y una velocidad +(vv0) durante un cierto tiempo t
(cuado desciende por el ro, hasta reencontrar la balsa). Las posiciones de la balsa y de la lancha en este
referencial sern:

balsa

x1

v0 (1.50  t )

lancha

x2

1.00 (v  v0 )  0.50 v0  (v  v0 )t

de modo que igualando esta dos expresiones (instante de reencuentro) obtenemos

v0 (1.5  t )

1.00 (v  v0 )  0.5 v0  (v  v0 )t o 0

v  vt o t

1h

Durante las 2.50 h, la balsa se ha desplazado 7.5 km, arrastrada por la corriente, de modo que su velocidad, que
ser la de la corriente, es

v0

7.5 km
2.5 h

- 30 -

km
h

Fsica Universitaria: Problemas de Fsica

Cinemtica de la partcula. M04.19

19. Una pequea embarcacin es arrastrada por una corriente de 3 km/h dirigida hacia el Este. Quiere ir a un
lugar situado al Nordeste de su posicin actual y su velocidad de mquinas es de 15 km/h (relativa).
Determinar el rumbo que debe seguir la embarcacin y su velocidad efectiva (absoluta).

va

Norte

NE

vba
vb
45

vba sen T

La velocidad efectiva o absoluta (vb) de la embarcacin


es igual a la suma vectorial de su velocidad relativa a la
corriente de agua (vba) y a la velocidad de sta (va); esto
es,
v b v ba  v a
tal como se indica en el diagrama vectorial adjunto.

o vb cos 45 vba cos T  va

x
n vb sen 45 vba sen T
y, puesto que sen 45 =cos 45, se sigue
va
o vba sen T  cos T va o sen T  cos T
vba

Este

vba cos T  va

3
15

0.2

Elevando al cuadrado ambos miembros de la ecuacin anterior y reduciendo trminos,


tenemos
sen 2 T  cos 2 T  2sen T cos T

0.22

o sen 2T

1  0.22

de donde se sigue:
2T

73.74
o T

180 73.74 106.26

y la velocidad absoluta de la embarcacin es


sen T
sen 53.1
vb vba
15 u
sen 45
sen 45

- 31 -

36.9

53.1

16.97 km/h

0.96

Fsica Universitaria: Problemas de Fsica

Cinemtica del slido rgido. M05.1

1. En un instante determinado, las velocidades de tres de los puntos de un slido rgido, de coordenadas
A(0,0,0), B(1,1,0) y C(0,1,1), son vA(6,-2,6) vB(4,0,5) y vC(5,-2,a). Determinar: a) El valor de a para que el
movimiento sea posible. b) La velocidad angular del slido en dicho instante. c) Las ecuaciones del eje
instantneo de rotacin y deslizamiento. d) La velocidad de deslizamiento.

a) Condicin cinemtica de rigidez:


6 0 5 0
JJJG
JJJG

v A <AC  v C < AC l 2<1  2<1 l 4  2 a l a  6
6 1 a 1
b) Relacionamos las velocidades de los puntos A, B y C:

JJJG
4 6 l 1
v B  v A   q AB l 0  2  mq1 l
5 6 n 0

5 6 l 0
JJJG

v C  v A   q AC l 2  2  mq1
6 6 2 1

2 n


n  2

2  n l

1 l  m

m  l  1

1 m  2


m  1

l 0  l l

0 l
l  0

0

de modo que   1 , con X = 5 .
2

c) Determinamos las coordenadas de un punto del EIRD, sabiendo que vO = vA:


0 6
10
 q v O 1 1

q


1
2

12

5 2 6 5 6
X2


de modo que las ecuaciones del EIRD son

x  2
5 x 10 5 y 12 5 z 6


l

0
1
2

10 y  5 z  30

d) La velocidad de deslizamiento o mnima es la proyeccin de la velocidad de cualquier


punto del slido sobre la direccin de Z. Su mdulo es
6 0

1 10
2 5
vdesl  v A < 
2<1 
X
5 6 2
5
y su expresin vectorial:
v desl  vdesl

0 0
 2 5

1  2
X
5 2 4

- 32 -

Fsica Universitaria: Problemas de Fsica

Cinemtica del slido rgido. M05.2

2. Tres puntos de un slido rgido tienen en un instante dado las siguientes velocidades:
A = (0, 0, 0)
vA = a i -2 j+ a k
B = (1, 1, 0)
vB = b i +5 k
C = (0, 1, 1)
vC = 5 i +c j+ 6 k
Determinar: a) Las componentes a, b, c de las tres velocidades. b) La velocidad angular del slido. c) Ecuacin
del eje instantneo de rotacin y deslizamiento.

a) Relacionamos las velocidades de los puntos A, B y C:


b  a  n

JJJG
b a l 1
b  a n

v B  v A   q AB l 0  2  mq1 l 2  n l n  2

5 a n 0
5  a l  m

5  a  l  m

5  a  m  n
5 a l 0
5  a m  2
JJJG

v C  v A   q AC l c  2  mq1 l c 2  l l c 2  l

6 a 2 1
6  a l

6  a  l
de modo que disponemos de 6 ecuaciones con 6 incgnitas (a, b, c, l, m, n), que una vez
resuelto da:
a6
b4
c  2
l0
m 1 n  2
b) Las velocidades de los tres puntos y la velocidad angular del slido en ese instante son
6




4
5
0

v A  2 , v B  0 , v C  2 ,
  1 , con X = 5

6
5
6
2
c) El ERID tiene la direccin del vector velocidad angular y pasa por un punto E tal que
0 6
10
JJJG  q v
1 1
O
 1q2  12
OE 
X2
5 2 6 5 6


de modo que las ecuaciones del EIRD son

x  2
5 x 10 5 y 12 5 z 6


l

0
1
2

2 y  z  6

- 33 -

Fsica Universitaria: Problemas de Fsica

Cinemtica del slido rgido. M05.3

3. Sobre un disco hay identificados tres puntos A, B y C cuyas coordenadas respecto a un sistema de referencia
son A(0,0,0), B(5,3,0) y C(6,2,0) en cm. Sabiendo que se trata de un movimiento plano y que en un cierto
instante es (vA)x = 4 cm/s, (vB)y = -3 cm/s y (vC)x = 16 cm/s, determinar completamente las velocidades de
los tres puntos y la velocidad angular del disco en ese instante.

B(5,3,0

vA

vB
C(6,2,0)
vC

A(0,0,0)
I (CIR)

Puesto que el movimiento es plano y los tres puntos estn


contenidos inicialmente en el plano z = 0, con componentes
de velocidad contenidas en dicho plano, sern nulas las
componentes de velocidad de cada uno de los tres puntos en
la direccin del eje z. Por el mismo motivo, el vector
velocidad angular ser perpendicular al plano del
movimiento, por lo que tan solo tendr componente sobre el
eje z. As, podemos escribir:

4

v A  a


b
v B  3
0


16
v C  c

0

  0

Relacionamos entre s las velocidades de los tres puntos:


vB  vA

vC  vA

b 4
JJJG

 u AB o 3  a
0 0

c 4
JJJG

 u AC o 9  a
0 0

0 5
b4

0
u
3
o

3  a
Z 0
0

c4
0 6


0 u 2 o 9  a
0
Z 0

3Z
b  3Z 4

5Z o a  5Z 3

2Z
c  2Z 4

6Z o a  6Z 9

Resolviendo este sistema de cuatro ecuaciones con cuatro incgnitas resulta: a = 27cm/s, b =
22 cm/s, c = -9 cm/s y  = - 6 rad/s, de modo que tenemos:
4




22
16
0
v A  27 cm/s
v B  3 cm/s
v C  9 cm/s
  0 cm/s
0
0
0
6




Determinacin de la posicin del C.I.R.:
0 4
162 4.5
JJG JJG  q v
1 1

A

q

24  2 / 3 cm
OI  AI 
0
27

X
36 6 0 36 0 0

- 34 -

Fsica Universitaria: Problemas de Fsica

Cinemtica del slido rgido. M05.4

4. El rotor de un generador elctrico est girando a 200 r.p.m. cuando el motor se apaga. Debido a efectos de

friccin, la aceleracin angular del rotor, en rad/s2, despus de que se apaga el motor viene dada por la
expresin D = 0.01Z, donde Z es la velocidad angular en rad/s. Cuntas revoluciones gira el rotor hasta
que se detiene?

Datos:

B  k X

con k  0.01 s -1 y X0 

200
10
10
r.p.s. = r.p.s.  q 2Q =20.94 rad/s
60
3
3

A partir de la relacin dada entre la aceleracin y velocidad angulares, escribimos la ecuacin


diferencial del movimiento:
dX
dX dR
dX
 k X l
X
 k X l d X  k d R
dt
dt dR
dR
cuya integracin nos conduce a

X0

d X  k d R l X  X0  k R l X  X0  k R
0

Cuando el rotor se detenga ser Z=0, de modo que el ngulo girado por el rotor hasta ese
instante vendr dado por
10
r.p.s. 1000
X
X0  k R  0 R  0  3 -1 =
 333.33 rev.
k
0.01s
3
Otro mtodo (ms largo, pero ms completo)
Procedemos a una primera integracin para determinar la velocidad angular en funcin del
tiempo:
X dX
t
dX
X
 k X l
 kdt l ln
 kt l X  X0 ekt
0
X0 X
dt
X0

Este resultado nos indica que se necesita un tiempo infinito (?) para que se detenga el rotor.
Una nueva integracin nos permite obtener el ngulo girado en funcin del tiempo:
dR
 X0 ekt
dt

t
X
X
dR  X0 ekt dt l R   0 ekt  0 1 ekt
0
k
k
0

A partir de esta ltima expresin determinamos el ngulo girado cuando transcurra un tiempo
suficientemente largo para poder considerar que el rotor ya se ha detenido.
tld R

10
r.p.s. 1000
X0
 3 -1 =
 333.33 rev.
k
0.01s
3

- 35 -

Fsica Universitaria: Problemas de Fsica

Cinemtica del slido rgido. M05.5

5. Los extremos de una barra de longitud L deslizan sobre dos guas rectas
perpendiculares entre s, sin perder contacto con las mismas, de modo que el
extremo B de la barra posee una velocidad vB constante, alejndose del punto
de unin de las dos guas. Determinar: a) La trayectoria descrita por el punto
medio de la barra. b) La velocidad y la aceleracin del otro extremo de la barra
en funcin del ngulo  indicado en la figura.

L sen


M
L

vA


B

vB

a) El movimiento del punto medio de la barra viene


descrito por el vector
JJJJG L
JJJG L sen R
OM  cos R l OM   cte.
2 0
2

I


L cos

de modo que su trayectoria es circular, con centro


en O y radio L/2.
O
b) Se trata de un movimiento plano en el que se
determina fcilmente la posicin del CIR (centro instantneo de rotacin) trazando las
perpendiculares a las guas en los extremos de la barra. Tenemos

vB

X

v
v
L cos R
X A  B l

IA IB
L sen R
IA

vA 
vB 
vB  vB tg R

L cos R
IB

vB

La aceleracin del extremo A la calculamos derivando con respecto al tiempo la expresin de


su velocidad:
aA 

dvA
v dR
v
vB2
d
d tg R
 vB tg R
 vB
 B2
 B2 X 
dt
dt
dt
cos R dt cos R
L cos3 R
vB2
= aA 
L cos3 R

- 36 -

Fsica Universitaria: Problemas de Fsica

Cinemtica del slido rgido. M05.6

6. Para que vire un tractor que se mueve con una velocidad v0 = 18 km/h, el tractorista frena una de las orugas
de modo que el eje de la rueda motriz de sta comienza a avanzar con velocidad v1 = 14 km/h. La distancia
entre las orugas es D = 1.5 m. a) Determinar el radio de la trayectoria que describe el centro del tractor.
b) Cunto tarda el tractor en dar media vuelta?

a) Designamos por R el radio que buscamos y por Z la velocidad angular asociada al


movimiento circular del tractor.
Puesto que conocemos las velocidades de dos puntos
del tractor, A y B, y stas son paralelas, el CIR se
encuentra en un punto I de la perpendicular comn a
ambas velocidades en A y B. Por consiguiente,
R
v0
podemos escribir:
v1

D
I
v0  X ( R )
A
B
D
v0 R 2
2

u l



D
v1 R  D
2
v1  X ( R  )
2

de modo que
v v1 D 18 14 1.5

R  0
6m
v0  v1 2 18 14 2

b) La velocidad del tractor ser


v v1 18 14
v 0

 16 km/h = 4.44 m/s
2
2
y describe un arco cuya longitud es
s  Q R  6Q  18.85 m

por lo que emplear:

s 18.85
t 
 4.2 s
v
4.44

- 37 -

Fsica Universitaria: Problemas de Fsica

Cinemtica del slido rgido. M05.7

7. El extremo superior de la varilla AB desliza a lo largo de una gua vertical (vide figura), en tanto que la
varilla no pierde contacto en C con el apoyo. a) Determinar el valor del ngulo  al que corresponde una
velocidad horizontal para el extremo libre, B, de la varilla. b) En ese instante, calcular las velocidades de los
puntos B y C en funcin de la velocidad del punto A.

Mtodo grfico.
a) Puesto que conocemos las direcciones de las velocidades de los puntos A, B y C
pertenecientes a la varilla, trazando las perpendiculares a stas en los correspondientes puntos,
determinamos el CIR (Centro Instantneo de Rotacin) correspondiente al movimiento plano
de la varilla, ubicado en el punto I de interseccin de las tres perI
A
pendiculares, tal como se indica en la figura.


vA
Una vez que hemos ajustado el dibujo a las condiciones
a
D
C
impuestas por el enunciado del problema, podemos determinar el
ngulo 0 a partir de simples consideraciones geomtricas.
vC
Considerando los tringulos ADC, ACI y AIB,
l
sucesivamente, encontramos la siguiente relacin entre a, l y 0:
a  AC cos R0  AI cos R0
cos R0  AI cos 2 R0 
 l cos R0
cos 2 R0  l cos3 R0
vB

a
De modo que cos R0 
l

1/3

b) El Teorema de las Velocidades Proyectadas (condicin cinemtica de rigidez) nos permite


determinar fcilmente los mdulos de las velocidades de los puntos B y C:
vB cos R0  vA sen R0 l vB  vA tg R0
vC  vA sen R0
Mtodo analtico (vectorial):

JJJG

Determinamos analticamente las velocidades de los puntos C y B y tenemos en cuenta que v C q AC  0 y


que vB es horizontal.

JJJG 0 0 a X a tg R0
v C  v A  q AC  vA 0 qa tg R0  X a  vA

0 X 0 0

0
JJJG X a tg R0 a

Xa
2
v C q AC  X a  vA qa tg R0 
0
 0 l vA  X a 1 tg R0

cos 2 R0

0 0
2
2
2


X a tg R0  X a avA

JJJG 0 0 l cos R0 Xl sen R0 vB


vB  Xl sen R0
I
v B  v A  q AB  vA 0 ql sen R0  Xl cos R0  vA  0 l


vA  Xl cos R0
0
0 X 0

gualando las expresiones de vA, tenemos

a
 l cos R0
cos 2 R0

- 38 -

a
 cos3 R0
l

Fsica Universitaria: Problemas de Fsica

Cinemtica del slido rgido. M05.8

8. La varilla AC que se muestra en la figura tiene un movimiento plano tal que

su extremo A desliza a lo largo de un eje horizontal, en tanto que la varilla


pasa por un pasador fijo y orientable (B) situado a una distancia fija h del eje
horizontal. Supongamos que el extremo A de la varilla se mueve con
velocidad constante vA de izquierda a derecha. a) Expresar la velocidad
angular (Z) de la varilla en funcin del ngulo T que se indica en la figura.
b) Calcular la velocidad y aceleracin del punto de la varilla que se
encuentra en B en funcin de dicho nguloT, expresando sus componentes
en la base vectorial indicada en la figura.

B
h

v
X  A cos 2 R
vA
vB
h
X

l

IA IB
vB  X IB  vA sen R

x
A

a1) Mtodo grfico. Determinamos la posicin de CIR, tal como se indica


en la figura. Entonces

IA  AB  h / cos R  h

cos R
cos R
cos 2 R

h
h sen R
tg R 
IB  AB tg R 
cos R
cos 2 R

y
C

vB

T
O

x
A

vA

a2) Mtodo analtico. Determinamos la velocidad de B a partir de la de


A:

vB sen R vA 0 h tg R vA  X h
JJJG
v sen R  vA  Xh

v B  v A q AB l vB cos R  0 0 q h  Xh tg R l B

vB cos R  Xh tg R

0 X 0 0
0

vB sen R  vA  X h
v  Xh
v
l tg R  A
l X  A cos 2 R y vB  vA sen R

vB cos R  X h tg R
X h tg R
h
JJG
JJG
 q AB  q ( q AB)
b) Determinamos la aceleracin de B a partir de la de A: a B  a A 

0

v
2XvA
  0 con X  A (2sen R cos R )X  
sen R cos R
h
h
X
2sen R cos R
1
JJG 0 h tg R

 q AB  0 q h  hX tg R  XvA 2sen 2 R

X 0
0



JJG 0 h tg R
1
q AB  0 q h  hX tg R
0
X 0
sen R cos R
0

JJG

1
 tg R

 q ( q AB)  0 q(hX ) tg R  hX 2 1  XvA  cos 2 R

X
0
0
0

R cos R
3sen R cos R
3sen

vA2
2
2
2
2
4

= a B  XvA 2sen R  cos R  2sen R cos R  cos R

h
0
0

- 39 -

Fsica Universitaria: Problemas de Fsica

Cinemtica del slido rgido. M05.9

9. La varilla AC que se muestra en la figura tiene un movimiento plano tal que su


extremo A desliza a lo largo de una circunferencia, en tanto que pasa por un
pasador fijo y orientable (B). Supongamos que el extremo A de la varilla se
mueve con celeridad constante. a) Expresar la velocidad angular (Z) de la varilla
en funcin del ngulo T que se indica en la figura. b) Calcular la velocidad y
aceleracin del punto de la varilla que se encuentra en B en funcin de dicho
nguloT, expresando sus componentes en la base vectorial indicada en la figura.

y
R

T
x

Trazamos las normales a las velocidades en A y B, para determinar la posicin del CIR
(punto I). El tringulo ABI es rectngulo, por lo que podemos afirmar que el CIR est situado
sobre la circunferencia que sirve de gua al extremo A de la varilla.
a) Una vez localizado el CIR, podemos escribir:
y
IA  2 R
v
v

X A  B

IB=2r sen R
IA IB

CIR
I
de donde se sigue
Z

R
A

vB

2T

T
B

vA
IB
 cte.
vB 
vA  vA sen R
2R
IA
b) Expresamos la velocidad del punto B de la varilla en
funcin de sus componentes:
sen R cos R
vB cos R

v B  vB sen R  vA sen 2 R

0
0

X

vA

Para determinar la aceleracin de B tendremos en cuenta que


JJJG
JJJG
a B  a A  q AB  q ( q AB)
y teniendo en cuenta que
cos 2R
v2

a A  A sen 2R
R 0

cos R
JJJG

AB  2 R cos R sen R
0



0
vA
0
2 R 1

  0

resulta
2

2 cos 2R

0 0 cos R vA2 2 cos 2R  cos R


vA2
vA vA

2 R cos R 0q 0qsen R 
aB 
2sen 2R
2sen 2R  sen R cos R
2 R 0 2 R 2 R

1 1 0 2 R
0

que tambin podemos escribir en la forma


aB 

cos 2R 1 3
3vA2

sen 2R

4 R
0

- 40 -

Fsica Universitaria: Problemas de Fsica

Cinemtica del slido rgido. M05.10

10. La barra BC de la figura est articulada con la manivela AB y


desliza por el interior de la gua pivotante D como se indica en la
figura. En el instante representado el punto D de la barra BC se
mueve a razn de 375 mm/s. Determinar la velocidad angular de
la manivela en ese instante.

125mm

375 mm/s

C
100mm

300mm

El punto B recorre una trayectoria circular alrededor con centro en A, por lo que su velocidad
ser perpendicular al vector AB; por tanto
 sen K

sen K  75 / 125  0.6

0.6

v B  vB cos K
l v B  vB 0.8
con
0
0
cos K  100 / 125  0.8

La velocidad del punto D est dirigida en cada instante en la direccin de la barra BD, tal
como se indica en la figura, de modo que ser:
vD  375 mm/s
 cos R
363

mm
v D  vD sen R
con sen R  75 / 752 300 2  0.242 l v D  90

0
0 s
cos R  300 / 752 300 2  0.970

Aplicando la condicin cinemtica de rigidez (teorema de las velocidades proyectadas) para


los puntos B y D de la barra BC, se sigue que
0.6 300 363 300
JJJG
JJJG

v B < DB  v D < DB l vB 0.8 < 75  90 < 75

0 0 0 0
vB 0.6
q 300
0.8q 75  364
q 300
90q 75
de donde resulta
0.6 299

mm

mm
vB  483
l v B  483 0.8 = 386
s
0 0 s

y
B

125mm

375 mm/s

D
C

100mm

300mm

- 41 -

Fsica Universitaria: Problemas de Fsica

Cinemtica del slido rgido. M05.11

11. La manivela CB oscila en torno a C describiendo un arco limitado y haciendo


que la manivela OA oscile en torno a O. Cuando la biela AB pasa por la
posicin representada, en que CB est horizontal y OA vertical, la velocidad
angular de CB es 4 rad/s en sentido antihorario. Hallar las velocidades
angulares de OA y AB en ese instante. Datos: a = 100 mm, b = 225 mm, c =
75 mm y d = 50 mm.

A
B

O
b

manivela BC (rotacin pura en C): Determinamos la velocidad del extremo B,


vB  XBC c  4q 75  300 mm/s
biela AB (rototraslatorio): Determinamos el CIR
grficamente, como se indica en la figura. Entonces,
escribimos
v
v
XAB  A  B
l
ad bc
vA
300
XAB 

l
100  50 225  75
v
XAB  A  2 rad/s z l vA  100 mm/s
50

A
a

B cC
CIR

d
b

manivela OA (rotacin pura en O): A partir de la velocidad del extremo A determinamos la


velocidad angular de la manivela,
v
100
vA  XOA a l XOA  A 
 1 rad/s z
a 100
Con notacin vectorial

0 c 0
v B  v C  CB q CB  0 q 0  cXCB con XCB  4 rad/s
X 0 0
CB
= v B  cXCB j  75q 4 j  300 j mm/s

JJG 0 0 (b  c) (a  d )XAB

v A  v B  AB q BA  cXCB 0 q a  d  cXCB  (b  c)XAB

0 X 0

AB

JJG 0 0 aXOA

v A  v O  OA q OA  0 qa  0

XOA 0 0

ad
100  50

XOA  a XAB  100 (2)  1 rad/s


(
a

d
)
X


a
X

AB
OA

cXCB  (b  c)XAB  0 X  c X  75 q 4  2 rad/s

AB b  c CB 225  75
= XAB  2 rad/s z
vA  100 mm/s
JJG

- 42 -

Fsica Universitaria: Problemas de Fsica

Cinemtica del slido rgido. M05.12

12. Una barra de 2 m de longitud est doblaba en escuadra y dotada en


sus extremos de correderas articuladas. En el instante representado
en la figura, la corredera A se mueve hacia la derecha con una
velocidad de 5.7 m/s. Hallar la velocidad angular de la escuadra y la
velocidad de la corredera B.

1m
1m

90
B

55

40

En primer lugar, por consideraciones puramente geomtricas, determinamos los ngulos


significativos, tal como se indica en la figura.
I

(CIR)

Z
40
2.17 m
1.91 m

90

vB

1m

30

1m

60

2m

45
55

10

B
40

vA

La velocidad de la corredera B se determina inmediatamente a partir de la condicin


cinemtica de rigidez (teorema de las velocidades proyectadas) aplicada a la barra doblada.
En efecto,
cos10
0.98
vA cos10  vB cos 30 l vB 
vA 
q5.7  6.48 m/s
cos 30
0.87
Puesto que las velocidades de los extremos de la barra tienen las direcciones de las
respectivas guas, determinamos la posicin del CIR (punto I) como punto interseccin de las
normales a dichas velocidades.
En el tringulo ABI, aplicando el teorema de los senos, tenemos:
AI
AB
sen60
0.87
AB 
2  1.91 m

l AI 
sen60 sen40
sen40
0.64
Ahora podemos determinar la velocidad angular de la barra en el instante indicado en la
figura:
v
5.7
vA  X IA l X  A 
 2.99 rad/s
IA 1.91

- 43 -

Fsica Universitaria: Problemas de Fsica

Cinemtica del slido rgido. M05.13

13. Una varilla AB est apoyada sobre un cilindro de radio R = 1 cm de modo que puede deslizar a lo largo de
una gua tangente a dicho cilindro, como se indica en la figura. La longitud de la varilla es cuatro veces el
radio del cilindro. En el instante en que el centro C de la varilla se apoya en el cilindro, la velocidad del
punto A es 10 cm/s. Calcular, en dicho instante, las velocidades de los puntos B y C y la velocidad angular
de la varilla.

R R
 0.5 l R  53.13
A partir del tringulo +OAC : tg 
2 2 R
Aplicamos la condicin cinemtica de rigidez (teorema de las velocidades proyectadas) a los
puntos A, B y C:
vB  vA  10 cm/s
vA cos R  vB cos R  vC l
vC  10 cos 53.13  6 cm/s
Mtodo geomtrico
B
Determinamos el CIR (I) como se indica en la figura. A
partir del CIR, determinamos las velocidades de los
T
puntos de la varilla:
I
2R
T
vA  X IA
vB  X IB
vC  X IC
T Z
C
IA  IB  AC/senR  2 / sen 53.13  2.5 cm
R
T
Con
IC  AC / tg R  4 / tg 53.13  1.5 cm
y

x
2R
O
X  v / IA  10 / 2.5  4 rad/s
A

T
A
De modo que vA  vB  10 cm/s

T
vC  4q1.5  6 cm/s

Mtodo analtico
A partir de las velocidades de A y C calculamos la velocidad angular:
vC cos R vA 0 2 R cos R vA  2X R sen R
JJJG

v C  v A  q AC l vC sen R  0 0 q 2 R sen R  2X R cos R

0
0
0
0 X

vC  vA / cos R  2X R tg R  10 / cos 53.13 2q 4q1q tg 53.13  6 cm/s

l
X  vC tg R  6 tg 53.13  4 cm/s

2R
2q1
Calculamos la velocidad de B a partir de la de A:
JJJG vA 0 4 R cos R vA  4X R sen R
v B  v A  q AB  0 0 q 4 R sen R  4X R cos R

0 X

0
0

10  4q 4q1qsen 53.13 2.8

vB  2.82 9.62  10 cm/s


cm

v B  4q 4q1q cos 53.13  9.6


l
9.6

s

tg G 
 3.43 l G  73.74 180  253.74

0
0

2.8

- 44 -

Fsica Universitaria: Problemas de Fsica

Cinemtica del slido rgido. M05.14

14. Un cilindro de radio R rueda sin deslizar sobre una superficie plana y horizontal, con una velocidad angular

Z constante. Determinar: a) el eje instantneo de rotacin; b) la velocidad y la aceleracin de los puntos del
eje del cilindro; c) dem de un punto cualquiera del cilindro de coordenadas (x,y,0); d) dem de los puntos
del cilindro que instantneamente estn en contacto con el plano.

Se trata de un movimiento plano, por lo que analizamos el movimiento en el plano xy


indicado en la figura, con  = -k.
a) El eje instantneo de rotacin coincide con la
y
generatriz del cilindro que en cada instante est en
contacto con la superficie sobre la que rueda sin
P(x,y)
deslizar (puntos de velocidad nula).
R aP
b) Calculamos la velocidad del punto O a partiendo
vP
x del punto I (CIR):
O


aI

vO  X IO  X R l v O  X R i

vO

La aceleracin del punto O ser nula, por ser


constante (en mdulo y direccin) la velocidad de
dicho punto; esto es,
aO 0

c) Consideremos un punto P cualquiera del cilindro, de coordenadas (x,y,0). Calculamos su


velocidad y aceleracin partiendo del punto O:

JJG X R 0 x X R y

v P  v O  q OP  0 0 q y  X x

0 X 0
0

2
x
JJG 0 X y X 2 x
JJG

a P  a O B q OP  q  q OP  0 qX x  X y  X y

X 0 0
0

de modo que la aceleracin del punto P est dirigida hacia el eje del cilindro 1.
d) Las coordenadas del punto I son (0,-R,0), de modo que sustituyendo en las expresiones
anteriores tenemos
X R  R

v I  X 0  0
a I  X 2 R  X 2 R j

0
0

JJG

JJG

En general, por ser  ? OP en el movimiento plano, el vector  q  q OP

tiene la misma direccin y

JJG
JJG
JJG
JJG
JJG
2
2
sentido opuesto al OP , ya que  q  q OP   <OP  X OP  X OP

- 45 -

Fsica Universitaria: Problemas de Fsica

Cinemtica del slido rgido. M05.15


y

15. Un disco de radio R rueda sin deslizar, sobre una superficie plana y horizontal,
con una velocidad angular Z, constante en direccin y sentido y aceleracin D
constante. Determinar: a) la velocidad y la aceleracin del dentro del disco.
b) dem de un punto P del cilindro diametralmente opuesto al de contacto del
disco con la superficie plana. c) dem del punto I del disco que instantneamente
est en contacto con la superficie plana.

,D

Se trata de un movimiento rototraslatorio plano consistente en una rotacin  alrededor del


eje del disco y una traslacin horizontal de dicho eje. Por ser un movimiento plano, basta con
analizar el movimiento en el plano z = 0, con  = -k y D = -Dk. Puesto que rueda sin
deslizar, punto del disco que instantneamente est en
y
contacto con el plano es el centro instantneo de
vP
rotacin (CIR).
P
AP
a) Calculamos la velocidad del punto O a partir del
punto I (CIR)
O vO
JJG 0 0 X R
x
aO

v O  v I  q IO  0 q R  0
X 0 0
aI
X R B R
,D
I
dv
d
a O  0  0  0
dt
dt 0 0

b) Para el punto P tenemos
JJG X R 0 0 X R X R 2X R
 0
0
v P  v O  q OP  0 0 q R 

0 X 0
0
0

JJG B R 0 0 0 X R 2B2R
d JJG

a P  aO
q OP  q  q OP  0 0 q R 0 q 0  X R
dt
0 B 0 X 0 0

c) Aunque la velocidad del punto I del disco es instantneamente nula, su aceleracin no es


nula. La calculamos a partir de la aceleracin del punto O:
JJG
d JJG
a I  aO
q OI  q q OI 
dt
B R 0 0 0 0 0 0 0 X R 0


0 0 qR 0 q 0 qR  0 0 q 0  X 2 R

0 B 0 X X 0 0 X 0 0

- 46 -

Fsica Universitaria: Problemas de Fsica

Cinemtica del slido rgido. M05.16

16. Un disco de radio r est girando alrededor de su eje de simetra con velocidad angular  y aceleracin
angular . Simultneamente, el disco est girando, con velocidad angular constante , alrededor de un eje
fijo en el espacio que est contenido en el plano del disco y es tangente al permetro de ste en un punto Q.
a) Determinar la velocidad y aceleracin del punto P del permetro del disco diametralmente opuesto al
punto Q de tangencia. b) dem de un punto genrico de la periferia del disco.

El disco est sometido a dos rotaciones simultneas: una


rotacin intrnseca  alrededor de su eje de simetra de
revolucin y una rotacin de precesin . Elegido un
referencial como el indicado en la figura, podemos escribir:
X

0
X
  0
  0 l  res     0
0
8
8
B
JJG 0
JJG 0

  0
CP  r
QP  2r
0
0
0

z

, 
Q

P
y

La velocidad del punto P del disco se obtiene como la superposicin o suma de las
correspondientes a cada una de las dos rotaciones; i.e.,
JJG
JJG X 0 0 0 28r
v P   q CP qQP  0 q r 0 q2r  0
0 0 8 0 X r
Determinamos la aceleracin de P a partir de la aceleracin del punto C (centro del disco);
i.e.,
JJG
JJG
d
a P  aC res q CP  res q  res q CP
dt
La aceleracin del punto C es la aceleracin centrpeta asociada a una trayectoria circular de
radio r con velocidad angular  constante:
0

aC  82 r
0

Calculamos la derivada temporal de la velocidad angular resultante teniendo en cuenta que 


precesa con velocidad angular , de modo que
B 0 X B
d res d d
d res JJG B 0 0



  q 0  0 0 q 0  X8 l
q CP  X8q r  0
dt
dt
dt
dt
0 8 0 0
0 0 Br

Calculamos el ltimo termino:


0


JJG

X 0 X 8r
 res q  res q CP   res q 0 q r  0 q 0  X 2 r 82 r

8 0 8 X r
0

Finalmente, tenemos

0
0


0 0
a P  82 r 0 X 2 r 82 r   X 2 282
r
0 Br



Br

- 47 -

Fsica Universitaria: Problemas de Fsica

Cinemtica del slido rgido. M05.17

Para un punto genrico


El disco est sometido a dos rotaciones simultneas: una
rotacin intrnseca  alrededor de su eje de simetra de
revolucin y una rotacin de precesin . Elegido un
referencial como el indicado en la figura, podemos escribir:
X

0
X
  0
  0 l  res     0
0
8
8
B

0
JJJG 0
JJJG

  0
CG  r cos R
QG  r (1 cos R )
0
r sen R
r sen R

z


G
, 


La velocidad del punto G del disco se obtiene como la superposicin o suma de las
correspondientes a cada una de las dos rotaciones; i.e.,
8r (1 cos R )
0
JJJG
JJJG X 0 0

v P   q CG q QG  0 q r cos R 0 qr (1 cos R )  X r sen R


0 r sen R 8 r sen R X r cos R
Determinamos la aceleracin de P a partir de la aceleracin del punto C (centro del disco);
i.e.,
JJG
JJG
d
a P  aC res q CP  res q  res q CP
dt
La aceleracin del punto C es la aceleracin centrpeta asociada a una trayectoria circular de
radio r con velocidad angular  constante:
0

aC  82 r
0

Calculamos la derivada temporal de la velocidad angular resultante teniendo en cuenta que 


precesa con velocidad angular , de modo que
B 0 X B
d res d d



  q 0  0 0 q 0  X8 l
dt
dt
dt
0 8 0 0
d res JJJG B 0 X8r sen R
qCG  X8qr cos R  Br sen R
dt
0 r sen R Br cos R
Calculamos el ltimo trmino:
X 0 X 8r cos R X8r sen R
JJJG

 res q  res q CG   res q 0 qr cos R  0 qX r sen R   82 X 2


r cos R

8 r sen R 8 X r cos R
X 2 r sen R

Finalmente, tenemos
X8r sen R

0
0 2 X8r sen R

a P  8 r Br sen R  82 X 2
r cos R  Br sen R 82 r 1 cos R
 X 2 r cos R



0 Br cos R
Br cos R  X 2 r sen R

X 2 r sen R

- 48 -

Fsica Universitaria: Problemas de Fsica

Cinemtica del slido rgido. M05.18

17. La hlice de un avin gira a razn de 6000 r.p.m., en tanto que el avin tiene una

velocidad horizontal, en lnea recta, de 360 km/h. Determinar: a) El tipo de movimiento que realiza un punto de la hlice distante 1 m del eje de la misma; b) la
velocidad y aceleracin de dicho punto.


y
x

a) Se trata de un movimiento helicoidal cuyo eje es el eje de rotacin de la


hlice (eje x, en la figura), que constituye el EIRD (Eje Instantneo de Rotacin y
Deslizamiento).
b) La velocidad y la aceleracin del punto P se calculan a
partir de la velocidad y aceleracin del punto O (en el eje
z
de la hlice), de modo que
JJG

v P  v O  q OP

vP
JJG

d JJG

OP
(
OP)
a

a

q


q

q

P
O
O

dt


y con
v

JJG 0

X
vo

OP  0
v O  0
  0

x
0
0
l

aO  0

d
0
dt

De modo que

v X 0 v 0 v

v P  0 0 q0  0 Xl  Xl

0 0 l 0 0 0

JJG X 0 0

a P   q ( q OP)  0 qXl  0

0 0 X l

Y sustituyendo valores, con


2Q
1000
X  6000
 200Q  628 rad/s
v  360
 100 m/s
60
3600
resulta:

100

vP  636 m/s
v P  628 m/s

m/s 2
0

a

394384

- 49 -

Fsica Universitaria: Problemas de Fsica

Cinemtica del slido rgido. M05.19

18. Sobre un plano horizontal rueda sin deslizar un cono recto de seccin circular, de generatriz l y semingulo
en el vrtice . Sea  la velocidad angular constante de rotacin del cono alrededor del eje vertical indicado
en la figura. Determinar: a) la velocidad angular  intrnseca de rotacin del cono alrededor de su eje de
simetra; b) el punto del cono cuya velocidad (con respecto al plano fijo) es mxima, as como la velocidad
y aceleracin de dicho punto.

Se trata de un slido rgido sometido a dos rotaciones simultneas: una rotacin intrnseca 
alrededor del eje de revolucin del cono, al tiempo que este eje presenta una rotacin 
alrededor del eje z indicado en la figura.
a) La generatriz OM del cono que en un instante dado est en contacto con el plano horizontal
constituye el EIR (eje instantneo de rotacin) del cono. La velocidad del punto M
perteneciente al EIR ser instantneamente nula, de modo que,
l
l
8
vM  8l  X R  0 l X  8 
8
R
l sen R
sen R
b) El punto P es el que presentar una velocidad
P
z
mxima, por ser el ms distante del EIR. Su

velocidad y aceleracin se calculan fcilmente a
l
C
partir de la velocidad y aceleracin del punto O


(vrtice del cono):
R
JJG

O
v P  v O  R q OP
l
M y
R
JJG
d R JJG
x

a P  aO
q OP  R q  R q OP
dt
con
0 0 0
JJG 0

OP  l cos 2R
 R     X cos R 0  X cos R

l sen 2R
X sen R 8 0
0 0 8X cos R

d R
d
d d

 q  0 qX cos R  0
  


dt
dt
dt
dt
8 X sen R 0

de modo que
2

JJG 0 0 Xl cos R sen 2R 28l cos R


0
0
v P   R q OP  X cos R ql cos 2R 


0 l sen 2R

0
0

JJG 8X cos R 0 0 Xl cos R sen 2R


d R JJG
 ...
0
q OP  R q  R q OP  0 ql cos 2R X cos R q
aP 

dt
0 l sen 2R 0
0

2
2
... 
8Xl cos R sen 2R
 8 l 2 cos R

2
2
cotg R
8Xl cos R cos 2R  X l cos R sen 2R

Otro mtodo....

- 50 -

Fsica Universitaria: Problemas de Fsica






l

x
x

Cinemtica del slido rgido. M05.20

Movimiento absoluto = Movimiento relativo +


movimiento de arrastre.
Descomponemos todos los vectores en la base
vectorial ijk asociada con el referencia absoluto o
fijo xyz, con
0

 rel    X cos R
 arr    0

X sen R
8
ijk
ijk

Dado que ambas velocidades angulares son constantes en mdulo y que hemos hecho
coincidir los orgenes O y O de los referenciales absoluto (xyz) y relativo (xyz), podemos
utilizar las relaciones siguientes:
0
cos R sen R  sen R cos 2R

JJJG 0

Xl sen R

v rel   rel q OaP  X cos R ql cos 2R  Xl


0
 ...  0

0
X sen R ijk l sen 2R ijk

0 ijk
ijk
0
8l cos 2R
JJG 0

v arr   arr q OP  0 ql cos 2R 


0

8 ijk l sen 2R ijk


0
ijk
28l cos 2 R
Xl sen R 8l cos 2R
8l 8l cos 2R

0
0
0
= v abs  v rel v arr 



0
0
0

ijk
ijk
ijk

0
0
0
Xl sen R

a rel   rel q v rel  X cos R q 0  X 2l sen 2 R 


8 2l

X sen R
0
2
sen
cos
cotg
X
l
R
R
8
l
R



ijk
ijk
ijk
ijk

8l cos 2R

 8 2l cos 2R
0
aarr   arr q v arr  0 q

8
0
0

ijk
ijk
ijk
0

0
0
Xl sen R

aCor  2 arr q v rel  2 0 q 0  28Xl sen R  28 2l

0
8 ijk 0 ijk
0 ijk
ijk
= aabs  a rel a arr a Cor

0
0

2
2
2
2

 8 l  8 l cos 2R  28 l  8 l 1 cos 2R  8 l 2 cos 2 R

2
cotg R ijk
cotg R

8 l cotg R
ijk
ijk

- 51 -

Fsica Universitaria: Problemas de Fsica

Cinemtica del slido rgido. M05.21

19. Una moneda, de 1.5 cm de radio, rueda inclinada manteniendo un ngulo de 60


respecto al plano horizontal. En su movimiento, el punto de contacto con el plano
horizontal describe sobre ste una circunferencia, de 0.75 cm de radio, cada tercio de
segundo. Determinar las velocidades y aceleraciones del centro de la moneda A y del
punto B de la periferia, en el instante en el que se encuentra en una posicin
diametralmente opuesta al punto de contacto con el plano horizontal.

60

La velocidad angular : vale 3 r.p.s.; o sea : = 3u2S = 6S rad/s


a) Dado que r = R cos60, el punto A permanece estacionario en la vertical del centro O de la
trayectoria circular descrita por el punto de contacto C; i.e.,
z
B
30
vA  0
aA  0
:

ZR

60

C
r

aB

Adems, el punto C de la moneda que en cada instante est en


contacto con plano horizontal se encuentra instantneamente en
reposo, ya que nos dicen que la moneda rueda; esto es vC = 0.
En consecuencia, todos los puntos del dimetro CAB se
encuentran instantneamente en reposo; i.e., dicho dimetro
coincide con el eje instantneo de rotacin (EIR). En
consecuencia, el punto B tambin se encuentra instantneamente en reposo; i.e.,
vB  0

b) Determinamos la velocidad angular intrnseca de la moneda (Z) a partir de la condicin de


rodadura:
r
0.75
8
vC  r8  RX  0 l X  8 
8   3 rad/s
1.50
2
R

0
0

X  X sen 60  3 3 / 2 rad/s
8  0 rad/s
XR  X 8  3 3 / 2 rad/s

9 / 2
X cos 60 3 / 2
6

donde ZR es la velocidad angular total o resultante de la moneda.


Determinamos la aceleracin del punto B a partir de la del punto A (nula):
JJJG
d  R JJJG
aB  aA
q AB  R q ( R q AB)
dt
0 0 9 3Q 2

d R
  q R  0 q 3 3Q / 2  0 rad/s 2
dt
6Q 9Q / 2 0


-9 3Q 2 0
0
0




d  R JJJG
aB 
q AB  0 q 3 / 4  81Q 2 / 4  200 cm/s 2




dt
2
0 3 3 / 4 27 3Q / 4 115
115
a B  2002 1152  231 cm/s 2
tg R 
l R  30 con la horizontal.
200

- 52 -

Fsica Universitaria: Problemas de Fsica

Cinemtica del slido rgido. M05.22

20. La cubierta del rodamiento a rodillos representado en la figura est fija, mientras

que el rbol interior gira con velocidad constante Z1 en sentido horario. Los
rodillos ruedan sin resbalar por las pistas. Determinar: a) La velocidad angular Z2
de un rodillo, indicando su sentido. b) La velocidad vB y la aceleracin aB del punto
B del eje de un rodillo. c) La aceleracin aA del punto A de la periferia del rodillo.

RR

Z1
6R

Se trata de un movimiento plano en el que el CIR del rodillo se encuentra en el punto A, que
se encuentra instantneamente en reposo por no resbalar el rodillo respecto de la cubierta fija.
Por la misma razn, la velocidad del punto C del rodillo tiene la misma velocidad que la del
punto C perteneciente al rbol interior; esto es,
vC  6 RX1

a) Determinamos la velocidad angular Z2 del rodillo a partir del conocimiento de la posicin


del CIR (punto A) y de la velocidad del punto C:
v
6 RX1
vC  X2 AC l X2  C 
 3X1
2R
AC
en el sentido antihorario, tal como se indica en la figura.
b) La velocidad del punto B es la mitad de la del
A (CIR)
punto C, tal como se deduce de la construccin grfica
de la figura; i.e.,
R
Z2
vB  RX2  3RX1
vB

B
aB

El punto B est describiendo una trayectoria circular


de radio 7R con celeridad constante. En consecuencia,
presenta una aceleracin centrpeta (dirigida hacia el
centro de dicha trayectoria) cuyo mdulo es

vC

Z1

3RX1
9 2
v

 RX1
7R
7R
7
c) Determinamos la aceleracin del punto A o CIR a partir de la del punto B mediante la
expresin:
JJG
JJG
a A  a B  2 q BA  2 q  2 q BA con  2  0
aB 

2
B

de modo que
0
0

0
0

0 0 0
0

9
9
72

a A   RX12 0 q 0 q R   RX12 RX22   RX12 9 RX12   RX12


7
7
7

7
X
X
0
0

0 2 2 0
0
0

- 53 -

Fsica Universitaria: Problemas de Fsica

Cinemtica del slido rgido. M05.23

21. Un disco de radio r1 = 10 cm rueda sin deslizar sobre otro disco de radio
r2 = 20 cm, a velocidad angular 1 = 3 rad/s. Al conjunto de ambos discos se
les dota de una nueva rotacin 2 = 6 rad/s alrededor del eje del disco grande,
en el mismo sentido que 1. En el instante en que los dos discos estn tal
como aparecen en la figura, determinar: a) Las velocidades de los puntos A y
B. b) El centro instantneo de rotacin del disco pequeo. c) La velocidad del
punto C.

C
A
B

a) En la rodadura del disco pequeo sobre el grande, al punto A le corresponde una velocidad
nula (CIR).
En la rotacin del conjunto de los dos discos, al punto A le corresponde una velocidad
0 0.20 0


OA

q

v

0q 0  1.20 m/s
y
A
2
6 0 0

2
O I

A 1

x
B

El disco pequeo est sometido a una rotacin


resultante
  1  2  3k 6k  9k rad/s

Calculamos la velocidad del punto B a partir de la del


punto A:
0 0 0.20 0



v B  v A  q AB  1.20 0q 0  3.00 m/s

0 9 0 0
b) El CIR del disco pequeo se encuentra en el punto I, tal que


v
1.20
 0.13 m = 13.3 cm
vA  X IA IA  A 
X
9
o sea, a una distancia de 6.6 cm del punto O.
c) Calculamos la velocidad del punto C a partir de la del punto A:
0 0 0.10 0.90
JJJG

v C  v A  q AC  1.20 0q0.10  2.10 m/s vC  2.28 m/s


0 9 0 0

- 54 -

Fsica Universitaria: Problemas de Fsica

Cinemtica del slido rgido. M05.24

22. En el dispositivo que se muestra en la figura, el brazo tiene una


longitud l y est girando alrededor de un eje fijo que pasa por
O. En un instante dado, su velocidad angular es Zb y su
aceleracin angular Db (sentido antihorario). El otro extremo del
brazo arrastra un pin, de radio R, que rueda sin resbalar por
el interior de una corona fija. a) Determinar la velocidad
angular (Zp) y la aceleracin angular (Dp) del pin en ese
instante. b) Determinar la velocidad y la aceleracin de punto A
del pin en ese instante. c) dem del punto B.

brazo

corona

Z bD b

x
O

pin

a) Determinamos la velocidad y aceleracin del punto C del brazo que sern tambin las del
punto C del pin:
JJG 0 l 0
v C  v O  b q OC  0 q0  Xb l
X 0 0
b

2
JJG
JJG 0 l 0 0 Xb l

a C  a O  b q OC  b q ( b q OC)  0 q0 0 qXb l  Bb l

B b 0 Xb 0 0
La velocidad del punto B del pin es nula (rodadura, CIR del pin), de modo que podemos
escribir:
v
Xl
l
l
l
vC  Xp BC l Xp  C  b l Xp   Xb (horario) Bp  X p   X b   Bb
R
R
R
R
BC
b) Determinamos la velocidad y la aceleracin del punto A del pin a partir del
conocimiento de la velocidad del punto B (CIR) y de la aceleracin del punto C:

JJG 0 2 R 0 0

v A  v B  p q BA  0 q 0  2 RXp  2Xp l  2 v C

Xp 0 0 0
JJG
JJG
a A  a C  p q CA  p q ( p q CA ) 
2
2

X 2 l  l
Xb l 0 R 0 0 R b R

 Bb l 0 q 0 0 q 0 q 0  2Bb l

l l l
0
0  R Bb 0  R Xb  R Xb 0

c) La velocidad del punto B del pin es nula (CIR) y su aceleracin la determinamos a partir
de la del punto C (como en el apartado anterior):
JJG
JJG
vB  0
a B  aC  p q CB  p q ( p q CB) 
Xb2l 0 R 0 0 R Xb2 l 2 R l

 Bb l 0 q 0 0 q 0 q 0 
0
l l

0  R Bb 0  R Xb  R Xb 0

- 55 -

Fsica Universitaria: Problemas de Fsica

Cinemtica del slido rgido. M05.25

23. Dos discos, de 15 cm de radio, ruedan sin deslizar sobre una


superficie plana. Una barra de longitud igual al dimetro de los
discos est unida por sus extremos a puntos de la periferia de ambos
discos, mediante articulaciones. En el instante que se representa en la
figura, el disco de la izquierda rueda a razn de 2 rad/s en el sentido
de giro horario: a) Hllese la posicin del centro instantneo de
rotacin de la barra. b) Determnese el sentido de la rotacin y el valor de la velocidad angular del otro
disco. c) dem de la barra.

Datos: R =15 cm; l = 30 cm; 1 = 2 rad/s.


R
De la figura, I = 45 y sen R   0.5 l R  30
l
a) Se trata de un movimiento plano en el
que intervienen tres slidos rgidos.
I 

Los CIR de los discos se encuentran en
B 
los puntos I1 e I2 indicados en la figura. A
vB
partir de ellos podemos determinar las
l=2R R
direcciones de los puntos A y B de los
A

discos respectivos, tal como se indican en
O1 1
O2 2
I
la figura que sern perpendiculares a las
vA
I
rectas I1A e I1B, respectivamente. Dichas
D
I1
I2
velocidades sern tambin las que poseen
los extremos A y B de la barra.
Conocidas las velocidades (vA y vB) de dos puntos de la barra, el CIR de la misma se
encuentra en la interseccin de las perpendiculares trazadas a dichas velocidades en los puntos
A y B. Por consiguiente, el CIR de la barra se encuentra en el punto I indicado en la figura y
la rotacin () de la barra tiene la direccin indicada (antihoraria). Determinamos las
distancias de I a los puntos O2 y B:
3
q1  R 3  25.98 cm
2
BI  O 2 I  O 2 B  R 3  R  10.98 cm

O 2 I  l cos R
tg G  2 R

b) Calculamos la velocidad del punto A:


vA  X1 I 2 A  X1 R 2  2q15q 2  42.43 cm/s

El teorema de las velocidades proyectadas nos permite calcular la velocidad del punto B:
cos R 90 G

cos 75
 42.43q
 12.68 cm/s
cos R
cos 30
Ahora, partiendo del CIR de la barra (I) , calculamos la velocidad angular de sta:
12.68
v
 1.15 rad/s (antihoraria)
vB  8 IB l 8  B 
IB 10.98
c) Finalmente, partiendo del CIR del segundo disco (I2) , calculamos su velocidad angular:
v
v
12.68
vB  X2 I 2 B l X2  B  B 
 0.42 rad/s (horaria)
30
I 2 B 2R
vA cos R 90 G
 vB cos R l vB  vA

- 56 -

Fsica Universitaria: Problemas de Fsica

Mecnica Clsica. Ley de la Inercia. M06.1

1. Da solar y da sidreo.- El da solar corresponde al intervalo de tiempo que emplea el Sol en pasar dos
veces sucesivas por un mismo meridiano terrestre. El da solar medio es de 86 400 segundos (= 24 horas). El
da sidreo corresponde al intervalo de tiempo que emplea la Tierra en completar una revolucin alrededor
de su eje polar. Calcular la duracin del da sidreo.

El da solar es el intervalo de tiempo transcurrido


entre dos pasos del Sol por el meridiano del lugar.
El da sidreo o sideral 1 corresponde al intervalo
de tiempo que emplea la Tierra en completar una
revolucin alrededor de su eje polar.
Es fcil establecer la relacin que existe entre un
da solar medio y un da sidreo.
Consideremos la Tierra ubicada inicialmente en
T, en el instante en que culminan el Sol y una
estrella mucho ms lejana que pueda considerarse
como una referencia fija. El meridiano local es m
y el punto Sur es S. Un observador terrestre que
estuviera mirando al Sur vera al Sol y a la estrella
alineados y culminando (vieta izquierda).
A medida que transcurre el tiempo, la Tierra se
traslada de T a T', a la vez que rota. La estrella
lejana culmina de nuevo en T', mientras, que para
que culmine el Sol, la Tierra deber rotar an el
ngulo . Se dice que el Sol "retrasa" respecto a la
estrella (vieta central). Finalmente, el Sol
culmina por segunda vez en T" y se dice que ha
transcurrido un da solar.
El da solar medio se compone de un da sidreo ms una fraccin de da correspondiente a la
la rotacin de ngulo  anteriormente descrita:
86400 s
s
1 d solar  1 d sidreo  l  
 236.6
 3 min 56.55 s
365.2422 d solar
d solar

Este resultado tambin puede obtenerse teniendo presente que durante un ao, que dura
365.2422 das solares, el Sol pasa por el meridiano una vez menos que el punto Aries. As
durante este intervalo ha habido un da sidreo ms que das medios, de modo que
1 ao  365.2422 d solar  366.2422 d sidereo
365.2422
d solar  0.997 269 6 d solar q86 400  86 164 s
366.2422
de modo que el da sidreo es   86400  86164  236 s  3 min 56 s ms corto que el da
solar.
1 d sidereo 

1
En todo rigor, se llama da sidreo al tiempo transcurrido entre dos pasos consecutivos del punto Aries por el
meridiano del lugar. Excepto por la retrogradacin coincide con el da sideral del que como mucho se diferencia
en 0.01 s.

- 57 -

Fsica Universitaria: Problemas de Fsica

Mecnica Clsica. Ley de la Inercia. M06.2

2. La cinta transportadora de viajeros de un aeropuerto tiene una longitud de 100 m y avanza con una
velocidad de 1.2 m/s. Una persona se mueve sobre la cinta con una velocidad relativa a ella de 1.5 m/s.
Determinar el tiempo que estar la persona sobre la cinta: a) cuando camina en direccin del movimiento de
la cinta y b) cuando camina en sentido opuesto.

a) Cuando la persona camina en la misma direccin en


que se mueve la cinta, su velocidad absoluta ser
vP  vPC vC  1.5 1.2  2.7 m/s

Y emplear un tiempo t en recorrer la longitud L, de


modo que
L 100
t 
 37 s
vP 2.7

vPC

Y emplear un tiempo t en recorrer la longitud L, de


modo que
L 100
t 
 333 s  5 min 30 s
vP
0.3

- 58 -

vC

b) Cuando la persona camina en la direccin contraria al


del movimiento de la cinta, su velocidad absoluta ser
vP  vPC  vC  1.5 1.2  0.3 m/s

vP

vP
vPC
L

vC

Fsica Universitaria: Problemas de Fsica

Mecnica Clsica. Ley de la Inercia. M06.3

3. Una persona sube por una escalera mecnica, que se encuentra parada, en 8.2 s. Cuando la escalera est en
funcionamiento, puede subir a la persona en 5.0 s. Cunto tiempo empleara la persona en subir caminando
por la escalera en movimiento?

Sea L la longitud que cubre la escalera.


Cuando la escalera est parada, la persona emplea un tiempo t1 en cubrir la longitud L, modo
que su velocidad, tanto absoluta (vP) como relativa a la escalera (vPE), es
L
L  vPE t1 l vPE 
vP
t1
Cuando la escalera est en movimiento y la persona en
reposo respecto de ella, se emplea un tiempo t2 en cubrir
esa misma distancia, de modo que
L
L  vE t2 l vE 
t2

vE

Cuando la escalera est en movimiento y la persona camina


sobre ella, su velociadad absoluta ser vP  vPE vE y emplear un tiempo t en cubrir la
distancia L, resultando
L
L  vPE vE
t l vPE vE 
t
As, de las tres expresiones anteriores, resulta:
L L L
1 1 1

l
 (media harmnica)
t
t1 t2
t t1 t2
Sustituyendo valores
t

t1t2
8.2q5.0 41.0


 3.1 s
t1 t2 8.2 5.0 13.2

- 59 -

Fsica Universitaria: Problemas de Fsica

Segunda y tercera leyes de Newton. M07.1

1. Dos cuerpos A y B, que pesan 500 N cada uno, se mantienen en


equilibrio sobre sendas superficies, perfectamente lisas y perpendiculares
entre s, mediante un cable ligero y flexible que los une, segn se indica
en la figura. Determinar el valor del ngulo T correspondiente a la
posicin de equilibrio, as como las reacciones de las superficies sobre los
cuerpos y la tensin del cable en dicha posicin.

A
30

60

Cuando el sistema est en equilibrio, la resultante de las fuerzas que actan sobre cada cuerpo
ser nula. Esto es, descomponiendo en las direcciones horizontal y vertical, tenemos:

(1) T cos T N A sen30


Cuerpo A
(2) T sen T  N A cos 30 P
(3) T cos T N B cos 30
Cuerpo B
(4) N B sen 30 P  T sen T
Considerando el equilibrio del sistema en
su conjunto [o sumando m.a.m. las
ecuaciones (1)+(3) y (2)+(4)], se obtiene

60

NA

NB

T
60

A
P

30

P
N A sen30 N B cos 30
A+B
N A cos 30  N B sen 30 2 P
Sistema de dos ecuaciones con dos incgnitas que nos permite calcular directamente las
reacciones NA y NB:

0  cos 30
2 P cos 30 866 N
NA
2 P sen 30
sen 30  cos 30

2
2
'
sen 30  cos 30 1
cos 30 sen 30
sen 30 0
N
2 P sen 30 500 N
B cos 30 2 P

De las ecuaciones (2) y (1), al dividirlas m.a.m., se sigue el valor del ngulo :
T sen T

P  N A cos 30 500  750

T cos T

N A sen 30 433

250
tg T

250
433

0.577 o T

y la tensin del cable ser:


T cos T

N A sen 30 433 o T

- 60 -

433
cos(30 )

500 N

30

Fsica Universitaria: Problemas de Fsica

Segunda y tercera leyes de Newton. M07.2

2. Una masa m colocada sobre una superficie lisa horizontal est unida a una masa M mediante una cuerda
ligera que pasa por un agujero practicado en la superficie. La masa m se mueve describiendo una trayectoria
circular de radio r con una celeridad v. Determinar el valor de la masa M para que ese movimiento se
mantenga.

La masa m est en movimiento, describiendo una trayectoria circular con celeridad constante,
bajo la accin de una fuerza centrpeta proporcionada por la tensin de la cuerda.
Escribimos las ecuaciones del movimiento para cada una
de los dos masas; i.e.,
m

T ma

T  Mg

v2
m
r
0

De modo que

M
2

v
r

Mg o

v2
m
rg

Mg

- 61 -

Fsica Universitaria: Problemas de Fsica

Segunda y tercera leyes de Newton. M07.3

3. Un hilo flexible y uniforme, de longitud l, est colgado en una pared vertical pasando sobre un clavo fijo y
liso. Aunque el hilo se encuentra inicialmente en equilibrio, se le separa ligeramente de dicha posicin para
que comience a deslizar sobre el clavo. a) Describir el movimiento del hilo, determinando su aceleracin.
b) Calcular la velocidad que adquiere el hilo, en el instante en que abandona al clavo.

a) Cuando desequilibramos el hilo, tirando ligeramente de uno de sus


extremos, comenzar a moverse en esa direccin, aumentando
continuamente su velocidad. Supongamos que, en un instante dado,
una longitud x de hilo cuelga del lado derecho, como se ilustra en la
figura. Los pesos de las dos porciones de hilo sern xg y (l-x)g
respectivamente, siendo  la densidad lineal del hilo. La ecuacin del
movimiento del hilo ser
O x g  O (l  x) g O l 
x o lx (2 x  l ) g

l-x
x

(l-x)g

g
xg
(2 x  l )
l
de modo que la aceleracin que adquiere el hilo va aumentando a
medida que aumenta x, i.e., su aceleracin no es constante.
b) Puesto que la aceleracin no es constante, deberemos proceder a calcular la velocidad por
integracin.
dv d v d x
dv g
a
v
(2 x  l )
dt dx dt
dx l
v
l
g l
v2 g 2
1
x  lx
v
d
v
(2
x

l
)
d
x
o
gl
0

l/2
/
2
l
l
2
l
4
? v 2 12 gl o v 12 2 gl
? a

Otro mtodo
Puesto que no hay friccin, podemos calcular la velocidad a partir
de la Conservacin de la Energa:
l
l 1
mg
mg  mv 2 o v 2 12 gl
4
2 2
? v 12 2 gl

Ep=0
L/4
G

L/2
G
v

- 62 -

Fsica Universitaria: Problemas de Fsica

Las fuerzas de la Naturaleza. M08.1

1. El bloque A de la figura pesa 15 kg y el bloque B pesa 5 kg. El coeficiente de rozamiento entre todas las superficies en contacto vale 0.20. En cada uno de los casos que
se muestran en la figura, calcular la magnitud de la fuerza F necesaria para arrastrar el
bloque A hacia la derecha con velocidad constante.

B
F

En cada uno de los tres casos, la fuerza necesaria para arrastrar el bloque A,
con velocidad constante, ser igual a la suma de las fuerzas de rozamiento
que se oponen al movimiento del bloque A y del B (en el tercer caso).

NA
B

fA

a)
F  f A  N N A  N PA PB
 0.20 15 5
 4 kg  39.2 N
NB
B

fAB
fA

NA

b)

F  f A f AB  N N A N N B  N PA PB
N PB 
 0.20 15 5
0.20q5  4 1  5 kg  49.0 N
NB
B

fAB

fBA
F

NA

fA

c)

F  f A f AB f BA  N N A N N B N N B  N PA PB
N PB N PB 
 0.20 15 5
0.20q5 0.20q5  4 1 1  6 kg  58.8 N

- 63 -

Fsica Universitaria: Problemas de Fsica

Las fuerzas de la Naturaleza. M08.2

2. Un bloque de masa m1 est situado sobre otro de masa m2 que a su vez se apoya

m1
sobre una superficie horizontal lisa. Siendo los coeficientes de rozamiento
esttico y cintico entre los dos bloques Ps y Pk, determinar: a) Fuerza mxima
F
m2
F que puede aplicarse al bloque de masa m2 para que el de arriba no deslice. b)
Si F es la mitad de este valor mximo, determinar la aceleracin de cada bloque
y la fuerza de rozamiento que acta entre ellos. c) Si F es el doble del valor
mximo determinado en a), calcular la aceleracin de cada bloque. Datos: m1 = 2 kg; m2 = 4 kg; s = 0.3; k
= 0.2.

m1
m2

f
F

La fuerza de rozamiento esttico entre los dos bloque


presenta un valor mximo dado por
sm1g = 0.3u2u9.8 = 5.88 N.
El rozamiento cintico presenta un valor constante igual a
km1g = 0.2u2u9.8 = 3.92 N

a) Puesto que el bloque de arriba no desliza, ambos bloques tendrn una aceleracin comn
a. Aplicamos las ecuaciones del movimiento al bloque superior (valor mximo del
rozamiento esttico) y al conjunto de los dos:
a a  Ns g  0.3q9.8  2.94 m/s 2
f a  m1a a  Ns m1 g

F a  (m1 m2 )a a
F a  Ns (m1 m2 ) g  0.3q 6q9.8  17.64 N

b) Ahora, la aceleracin del bloque inferior es menor que la crtica y el bloque superior no
deslizar sobre el inferior, por lo que ambos bloques tendrn la misma aceleracin a.
Procedemos como antes, con F=F/2, comenzando por el conjunto de los dos bloques para
determinar la aceleracin y luego calculamos el valor del rozamiento esttico:

F aa
17.64 / 2
a aa 
F aa  (m1 m2 )a aa

 1.47 m/s 2

(m1 m2 )
6
l

f aa  m1a aa  Ns m1 g
f aa  2q1.47  2.94 N
c) Ahora, la aceleracin del bloque inferior supera el valor crtico, por lo que el bloque
superior deslizar hacia atrs sobre el bloque inferior, existiendo rozamiento cintico.
Escribimos las ecuaciones del movimiento para cada uno de los bloques, por separado, con
F aaa  2 F a :
a1aaa Nk g  0.2q9.8  1.96 m/s 2

f aaa  m1a1aaa Nk m1 g

l
F aaa  Nk m1 g 2q17.64  0.2q 2q9.8
a2aaa

 7.84 m/s 2
F aaa  Nk m1 g  m2 a2aaa

m2
4

- 64 -

Fsica Universitaria: Problemas de Fsica

Las fuerzas de la Naturaleza. M08.3

3. Dos bloques, de masas m1 = 4 kg y m2 = 8 kg, estn unidos mediante

m2

una varilla rgida y ligera y resbalan por un plano inclinado 30, como
se muestra en la figura. El coeficiente de rozamiento cintico entre el
plano y cada uno de los bloques es 1=0.20 y 2=0.30, respectivamente.
Calcular la aceleracin del sistema y la tensin en la varilla, indicando
si es tensora o compresora.

m1
30

Comenzamos valorando la fuerza de rozamiento cintico que acta sobre cada bloque

f1  N1 N1  N1m1 g cos 30  0.2q 4q9.8cos 30  6.79 N

f  27.16 N
f 2  N2 N 2  N2 m2 g cos 30  0.3q8q9.8cos 30  20.37 N

N2
N1

f2
m2g

f1
m1g 30

Aplicamos la 2 ley de Newton al sistema constituido por los dos bloques:

m1 m2
g sen 30  f1 f 2
 m1 m2
a
12q9.8sen 30 27.16  58.80  27.16  31.64  12a l

a  2.64 m/s 2

Aplicamos la 2 ley de Newton al bloque pequeo:


m1 g sen 30  f1  R1  m1a
19.60  6.79  R1  10.56 l

N1

R1

4q9.8sen 30 6.79  R1  4q 2.64


R1  2.25 N

f1

De modo que la tensin en la varilla es tensora, como se


ilustra en la figura.
Aunque no es necesario, a efectos de comprobacin aplicamos
la 2 ley de Newton al bloque grande:
m2 g sen 30  f 2 R2  m2 a

m1g

R2

varilla

30
R1

8q9.8sen 30 20.37 R2  8q 2.64


39.20  20.37 R2  21.12 l

N2

R2  2.29 N

La discrepancia de valores se debe a los redondeos; la tensin


en la varilla bien puede ser igual a 2.27 N (tensora).

R2

f2
m2g

- 65 -

Fsica Universitaria: Problemas de Fsica

Las fuerzas de la Naturaleza. M08.4

4. Dos bloques, de masas m1 = 4 kg y m2 = 8 kg, descienden por un pla-

m2

no inclinado 30, sin perder contacto entre s, como se muestra en la


figura. El coeficiente de rozamiento cintico entre el plano y cada uno
de los bloques es 1=0.30 y 2=0.10, respectivamente. Calcular la
aceleracin del sistema y la fuerza que se ejercen los dos bloques
entre si.

m1
30

a) Comenzamos valorando la fuerza de rozamiento cintico que acta sobre cada bloque
f1  N1 N1  N1m1 g cos R  0.3q 4q9.8q cos 30  10.18 N
f  f1 f 2  16.97 N
f 2  N2 N 2  N2 m2 g cos R  0.1q8q9.8q cos 30  6.79 N

m2

+
m1

N2
f2

30

N1

R
R

m2g

f1

m1g

A continuacin aplicamos la 2 ley de Newton en la direccin del movimiento a cada bloque y


tenemos 2 ecuaciones con 2 incgnitas (a, R):

m1 g sen 30 R  f1  m1a

m2 g sen 30 R  f 2  m2 a
Sumando m.a.m. obtenemos:

m1 m2
g sen 30  f1 f 2
 m1 m2
a
a  g sen 30 

16.97
f
 9.8sen 30 
 3.49 m/s 2
12
m1 m2

R  m1 a  g sen 30
f1  4q 3.49  4.9
10.18  4.57 N

- 66 -

Fsica Universitaria: Problemas de Fsica

Las fuerzas de la Naturaleza. M08.5

5. Un bloque de masa m puede deslizar sobre una superficie plana inclinada con una pendiente del 12% (i.e.,

tg T = 0.12). Cuando el bloque se lanza plano arriba, recorre la mitad del espacio que cuando se lanza plano
abajo, con la misma velocidad inicial en ambos casos. Calcular el valor del coeficiente de rozamiento entre
el plano y el bloque.

Tanto cuando el bloque asciende como cuando desciende, sern:


N  mg cos R l f  N N  N mg cos R l W f  f s  N mgs cos R

siendo Wf el trabajo realizado por la fuerza de rozamiento durante un recorrido s sobre el


plano inclinado.
Cuando el bloque sube, su energa cintica inicial se convierte en energa potencial
gravitatoria y en energa disipada en el proceso de rozamiento hasta detenerse:
1 2
mv0  mgs1 sen R Nmgs1 cos R  mgs1 sen R N cos R

2
Cuando el bloque baja, su energa cintica inicial y su energa potencial gravitatoria se
convierten en energa disipada en el proceso de rozamiento hasta detenerse:
1 2
mv0  mgs2 sen R Nmgs2 cos R  mgs2  sen R N cos R

2
Puesto que la velocidad inicial es la misma en ambos casos, igualando las expresiones
anteriores, y operando, tenemos:
N cos R sen R s2
mgs1 sen R N cos R
 mgs2  sen R N cos R
l
 2
N cos R  sen R s1
de modo que
N cos R sen R  2N cos R  2sen R l 3sen R  N cos R l N  3 tg R  3q 0.12  0.36

N
N
v0

v0

s1

s2


mg

- 67 -

mg

Fsica Universitaria: Problemas de Fsica

Las fuerzas de la Naturaleza. M08.6

6. Tres cuerpos de masa m = 5 kg estn unidos entre s por dos


cuerdas que pueden soportar una tensin mxima T = 20 N.
F
2
3
1
Los cuerpos se encuentran sobre una superficie horizontal y los
coeficientes de rozamiento son: P1 = 0.3, P2 = 0.2, P3 = 0.1. Si
aplicamos al cuerpo 3 una fuerza F que aumentamos lentamente, Qu cuerda se rompe y con qu fuerza
mnima ocurrir?

Cuando la magnitud de la fuerza aplicada sea tal que estemos en condiciones de movimiento
inminente, las fuerzas de rozamiento tendrn el valor mximo posible, i.e.:
f1  N1m1 g  0.3q5  1.5 kg = 14.7 N
T1
T2
F
f 2  N2 m2 g  0.2q5  1.0 kg = 9.8 N
1
2
3
f3  N3 m3 g  0.1q5  0.5 kg = 4.9 N
f1
f3
f2
y las tensiones de las cuerdas sern:
T1 = f1 = 14.7 N
T2 = f1 + f2 = 24.5 N
Como T2 sera superior a la tensin de rotura (20 N), no habr movimiento de conjunto de las
tres masas, ya que la cuerda 2 se romper antes de que eso ocurra. As, las tres masas
permanecern en reposo y la mxima fuerza que se puede ejercer sin romper la cuerda 2 ser:
F = T2,mx + f3 = 20 + 4.9 = 24.9 N

- 68 -

Fsica Universitaria: Problemas de Fsica

Las fuerzas de la Naturaleza. M08.7

7. En el esquema de la figura calcular la fuerza F requerida para que el bloque


de 24 kg empiece a subir por el plano inclinado 10. El coeficiente de
rozamiento esttico de cada pareja de superficies es de 0.30.

20kg

24kg
10

La situacin de los bloques es de movimiento inminente por lo que


las tres fuerzas de rozamiento son las mximas en cada caso.
La tg 10 = 0.18 es menor que P = 0.3, por lo que el bloque superior no tiene posibilidad de
resbalar hacia abajo sobre el bloque inferior. En consecuencia, al tirar del bloque de 24 kg
hacia arriba, ser nula la reaccin N3 indicada en la figura y los diagramas de cuerpo libre
de cada uno de los bloques son los indicados.

N12
N3=0

P1

N1

N12

f21

N2
x

10

10

f12

f2
P2

Aplicando las ecuaciones cardinales de la esttica a cada uno de los bloques, descomponiendo
en las direcciones indicadas, tenemos:
Bloque superior:

Fx  0 N N12 cos10  N1 N12 sen10


l



F
0
N
cos10
N
N
sen10
P

y
12
12
1

P1
20

 19.29 kg = 189 N
N12 
cos10 N sen10 cos10 0.3 sen10
N1  N12 (N cos10  sen10 )  2.35 kg = 23.03 N
Bloque inferior:
Fy '  0 N 2  N12 P2 cos10 l N 2  19.29 24 cos10  42.92 kg = 421 N

Fx '  0 F  P2 sen10 N N12 N 2


 4.17 5.79 12.88  22.83 kg = 224 N

- 69 -

Fsica Universitaria: Problemas de Fsica

Las fuerzas de la Naturaleza. M08.8

8. Determnense los valores de los ngulos D y E, correspondientes a la posicin de


equilibrio, para el sistema representado en la figura.

D
2m

E
2m

Las cuerdas ideales (inextensibles y de masa


despreciable) transmiten ntegramente las tensiones a lo
mg
largo de ellas, por lo que tenemos el diagrama de fuerzas
D
E
actuantes sobre el bloque central, correspondiente al
equilibrio del sistema, que se indica en la figura.
2mg
Escribiendo la condicin de equilibrio, tenemos
m
2m
2m
l P cos C  2 P cos B
2 cos B  cos C
l

2 P sen B P sen C  2 P
2sen B sen C  2
y resolviendo este sistema de ecuaciones,...
2mg

2sen B 1 cos 2 C  2 l 2sen B 1 4 cos 2 B  2 l


1 4 cos 2 B  2(1 sen B) l 1 4 cos 2 B  4(1 sen B) 2
1 4(1 sen B)  4 4sen B  8sen B l
2

3 4sen 2 B  4 4sen B 2  8sen B l sen B 


= B  61.04

- 70 -

C  14.48

7
8

Fsica Universitaria: Problemas de Fsica

Las fuerzas de la Naturaleza. M08.9

9. a) Determinar la aceleracin mxima que puede alcanzar un automvil de 1000 kg, con traccin en las

cuatro ruedas cuando sube por una rampa de 30 respecto a la horizontal. (coeficiente de rozamiento P = 1).
b) Calcular la inclinacin mxima de la rampa por la que podr subir este vehculo.

a) El diagrama de fuerzas del cuerpo libre es


el que se indica en la primera figura, que
NB
puede simplificarse en la forma que se indica
G
NA
en la segunda figura. Apreciamos que la
f
P
B
aceleracin del automvil es una consecuencia
P
A
30
de las fuerzas de rozamiento entre las ruedas y
la calzada. Aplicamos las Ecuaciones Cardinales d la Dinmica:
f  P sen 30  ma

f  P sen 30 ma

N  P cos 30
N
P
cos
30
0


l
l P sen 30 ma b N P cos 30

f b N P cos 30
f b NN

La aceleracin mxima del automvil ser:


m
P
a b N cos 30  sen 30
 N cos 30  sen 30
g  1q 0.866  0.5
g  0.366 g  3.59 2
s
m
b) Procedemos de forma anloga, para un ngulo arbitrario:
f  P sen R  ma
f  P sen R ma

P sen R ma b N P cos R
N  P cos R  0
l N  P cos R
l

a b N cos R  sen R
g
f b N N
f b N P cos R
Solo podr subir si la aceleracin es positiva; i.e., si el valor mximo del rozamiento es igual
o mayor que la componente del peso en la direccin de la rampa:
N

a b N cos R  sen R
g  0 l N cos R  sen R  0 l N cos R  sen R

l tgR 

1
 1 l R  45
N

- 71 -

Fsica Universitaria: Problemas de Fsica

Las fuerzas de la Naturaleza. M08.10

10. Un bloque de masa m est apoyado sobre la cara inclinada y spera de una
cua de masa M. La cua puede moverse libremente sobre una superficie
horizontal lisa. Se aplica una fuerza F a la cua, de modo que el bloque
queda a punto de deslizar hacia arriba en el plano inclinado. Si el
coeficiente de rozamiento esttico entre la cua y el bloque esP,
determinar: a) la aceleracin del sistema, b) la fuerza F necesaria para
producir esta aceleracin, c) la reaccin normal de la superficie
horizontal sobre la cua.

45

En la figura adjunta se ha dibujado el diagrama de fuerzas correspondiente a la cua y al


bloque.
a) Aplicamos las ecuaciones cardinales de la dinmica al bloque, descomponiendo las fuerzas
en las direcciones x e y indicadas en la figura, teniendo en cuenta que f  N N en las
condiciones de movimiento relativo inminente:


N 2  mg  f 2  0 l N 2  N N 2  mg l N 2  mg

2
2
2
2
2
1 N

2
2
2
2
2 1 N
l
N
mg  ma
f
 ma l N
NN
 1 N
N


2
2
2
2
2
1 N
de modo que
y
1 N
N
a
g
f
x
1 N

a
N

mg

f
45

Mg

b) Aplicamos la 2 ley de Newton al sistema completo


(M+m), teniendo en cuanta que F es la nica fuerza
exterior al sistema que tiene componente en la direccin
del movimiento del sistema:
1 N
F  M m
a 
M m
g
1 N

c) Consideramos el sistema completo (M+m) y aplicamos la condicin de equilibrio en la


direccin vertical, considerando tan solo las fuerzas exteriores al sistema. Puesto que ni la
cua ni el bloque presentan aceleracin en la direccin vertical, ser:
R  Mg mg  M m
g

- 72 -

Fsica Universitaria: Problemas de Fsica

Las fuerzas de la Naturaleza. M08.11

11. Una bloque de masa m descansa sobre una cua y sta, a su vez, sobre
un plano horizontal sin rozamiento. Determinar la fuerza F que se
debe aplicar a la cua para que la masa m comience a ascender si entre
ella y la cua existe un rozamiento de coeficiente P.

m
M

En la figura, hemos representado las


fuerzas que actan sobre cada uno de los
S
dos cuerpo (el bloque y la cua) que
constituyen el sistema.
f
N
Aplicamos
las
ecuaciones
del
ma0
movimiento
al
bloque,
en
el
referencial
F
f
S
N2 N
S (solidario con la cua), en el que el
a0
mg
bloque permanecer en reposo hasta que
M
T
la aceleracin que adquiera la cua sea
Mg
suficientemente grande como para que
comience a ascender, por lo que la fuerza
de rozamiento que acta sobre el bloque tendr la direccin indicada (hacia abajo). Tenemos
2 mg sen R f  ma0 cos R  0
[1] mg sen R N N  ma0 cos R  0

/ N  mg cos R  ma0 sen R  0 l

[2] N  mg cos R  ma0 sen R  0

con f  N N
Aplicamos las ecuaciones del movimiento a la cua, en el referencial fijo S, de modo que
F  f cos R  N sen R  Ma0 l [3] F  N N cos R  N sen R  Ma0
de modo que disponemos de un sistema de tres ecuaciones con tres incgnitas (F, N y P).
Resolvemos el sistema de ec. [1] y [2]
[1]  N N ma0 cos R  mg sen R
l

[2] N  ma0 sen R  mg cos R


sen R N cos R
mg
N
a0 
g
cos R  N sen R
cos R  N sen R
De la ec. [3] despejamos F:
[3] F  Ma0 (sen R N cos R ) N
de modo que
sen R N cos R
F
( M m) g
cos R  N sen R

- 73 -

Fsica Universitaria: Problemas de Fsica

Las fuerzas de la Naturaleza. M08.12

12. Un nio coloca una bscula sobre una plataforma que puede deslizar sin friccin sobre un plano inclinado,
como se indica en la figura. El nio se sube en la bscula y lee la indicacin de su "peso" cuando la
plataforma desciende (aceleradamente) por el plano inclinado. Si el peso del nio en condiciones normales
es P = 40 kg y el ngulo de inclinacin del plano es 10, cul ser la indicacin de la bscula?

La aceleracin con la que desciende el sistema


bscula-nio a lo largo del plano inclinado es:
a  g sen R
y sus componentes en las direcciones horizontal
y vertical (ejes xy ) son:
ax  g sen R cos R
a y  g sen 2 R

N
S

g sen T cos T

a0
g sen 2 T

Sobre el nio actan las dos fuerzas que se


mg
indican en la figura: su peso (mg) y la reaccin
normal (N) ejercida por la plataforma de la bscula. La reaccin de esta ltima fuerza es la
que mide la bscula (peso aparente del nio).
Aplicamos la ecuacin del movimiento en la direccin vertical, bien sea en el referencial
inercial [S] o en el no-inercial [S] ligado al sistema acelerado:

l N  mg  ma y  mg sen 2 R

Sa

l N  mg mg sen 2 R  ma ay  0

l N  mg 1 sen 2 R
 mg cos 2 R

Sustituyendo los valores del enunciado, obtenemos:

N = 40q cos 2 10 = 38.79 kg

- 74 -

Fsica Universitaria: Problemas de Fsica

Las fuerzas de la Naturaleza. M08.13

13. El coeficiente de friccin (esttico y cintico) entre la caja A y la vagoneta

de la figura vale 0.6. Se pide: a) Aceleracin mnima de la vagoneta para


que la caja no caiga. b) Tiempo que tardar en desprenderse la caja si la
aceleracin de la vagoneta es la mitad de la mnima.

2m

a)
En la figura mostramos el diagrama de fuerzas que actan sobre la caja. Aplicando las
ecuaciones cardinales de la dinmica de la partcula, siendo f la fuerza de rozamiento en
condiciones de movimiento (de cada) inminente, tenemos:

l (1) N  ma

con f  N N
a

(2) f  mg  0

De modo que
N N  mg l Nma  mg l a 

g 9.8

 16.33 m/s 2
N 0.6

b) Ahora, con a = a/2 = 8.16 m/s2, y siendo ac la aceleracin de la


caja ser:
l (1) N  ma a
con f  N N

(2) f  mg  mac
mac  Nma a  mg l ac  Na a  g  0.6q8.16  9.8  4.90 m/s 2

donde el signo negativo indica que est dirigida hacia abajo.


Calculamos el tiempo de cada a partir de la ecuacin para el
movimiento rectilneo uniformemente acelerado,
1
s  at 2
2

2s
l t
a

2q 2
l t
 0.903 s
4.90

- 75 -

mg

ac
mg

Fsica Universitaria: Problemas de Fsica

Las fuerzas de la Naturaleza. M08.14


m1

14. a) )Qu fuerza horizontal constante debe aplicarse al sistema que se


muestra en la figura de modo que los cuerpos de masa m1 y m2 no se
muevan con respecto al M. b) Si la fuerza aplicada es la mitad de la
calculada en el apartado anterior, Cules sern las aceleraciones de
los bloques m1 y m2 con respecto del bloque M?

m2

a) Planteamos el problema en el sistema de


referencia ligado al bloque de masa M, ya que en
este sistema los bloques m1 y m2 se encuentran en
reposo.
En la figura, hemos representado el
m1g
F2 T N
F
diagrama de fuerzas que actan sobre cada uno de
2
a0
los tres bloques, incluidas las fuerzas de inercia
M
m2g
(F1 y F2) asociadas a la no-inercialidad del
referencial.
Escribimos las ecuaciones del movimiento para los dos bloques pequeos:
l T  m1a0  0
m
m2 g  m1a0  0 l a0  2 g
m m2 g  T  0
m1
Y ahora, escribiendo la ecuacin del movimiento para todo el sistema en su conjunto,
determinamos la fuerza requerida1 para que los bloques pequeos permanezcan en reposo
respecto del grande:
m
l F  M m1 m2
a0  2 M m1 m2
g
m1
S
F1

N1

b) Replanteamos el problema, de nuevo en el referencial no-inercial ligado al bloque grande,


ya que nos interesa el movimiento de los bloques pequeos relativo al grande:
m
1
1
F '  F l a '0  a0  2 g
2
2
2m1

Escribiendo las ecuaciones del movimiento para cada uno de los bloques pequeos, teniendo
en cuenta que a1 = a2 = a (cuerda inextensible), tenemos:

l T  m1a '0  m1a '


mm
1

l m1 m2
a '  m2 g  1 2 g  m2 g

m m2 g  T  m2 a '
2m1
2

De modo que la aceleracin que presentan los bloques pequeos con respecto del bloque
grande 2 es:
m2
a'
g
2 m1 m2

1
2

Obsrvese que, si m1 = m2 = m, ser ao = g y F = (M+2m)g.


Obsrvese que, si m1 = m2 = m, ser a = g/4.

- 76 -

Fsica Universitaria: Problemas de Fsica

Las fuerzas de la Naturaleza. M08.15

15. Un ferrocarril ligero subterrneo est compuesto por tres vagones, los extremos de 100 t (toneladas) cada
uno, y el central de 50 t. Al aplicar el freno se ejerce la misma fuerza de frenada en cada vagn. El
coeficiente de rozamiento con los carriles vale 0.1. a) Calcular la aceleracin de frenada mxima que
admiten sin que deslice ningn vagn y las fuerza que ejercen los vagones en lo topes. Qu vagn o
vagones seran los primeros en deslizar. b) La aceleracin de frenada mxima que admitirn sin que deslicen
todos los vagones.

direccin del
movimiento

m = 50 t

2m

T3

d T1

2m
T1

T3

f2
f3
f1
Empezamos calculando los valores mximos de las fuerzas de rozamiento:
f1,mx  N N1  0.1q100q103  10q103 kg = 98 kN
f 2,mx  N N 2  0.1q50q103  5q103 kg = 49 kN
f3,mx  N N 3  0.1q100q103  10q103 kg = 98 kN

a) Para que no deslice ningn vagn, la fuerza de frenado (interior al


sistema) no puede superar ninguno de los valores mximos de las
fuerzas de rozamiento. En consecuencia, las fuerzas de rozamiento
sern iguales, en los tres vagones, al menor valor mximo de ellas;
esto es f1  f 2  f3  f mx = f 2,mx . Escribimos la ecuacin del

ffreno
v
froz

movimiento para todo el sistema en su conjunto:


3 f mx  mtot amx

l amx  

3 f mx
3q 49q103

  0.588 m/s 2
mtot
250q103

y para los vagones primero y ltimo:


T1  f mx  m1amx

l T1   f mx  m1amx  49q103 100q103 q (0.588)  9.8 kN

T3  f mx  m3 amx

l T3  f mx m3 amx  49q103 100q103 q (0.588)  9.8 kN


direccin del
movimiento

2m

T3

d T1

m
T3

f3

2m

+
c

T1
f2

f1

de modo que el diagrama de fuerzas es el que se representa en la tercera figura. Si se


superasen los lmites de frenada, el primer vagn en deslizar sera el central.
b) Ahora, permitimos que deslice el vagn central, pero no los de los extremos. Bastar con
que la fuerza de frenado no supere el valor mximo de la mayor de las fuerzas de rozamiento.
En estas condiciones, los valores de las fuerzas de rozamientos sern los mximos posibles en
cada vagn. Escribimos la ecuacin del movimiento para todo el sistema en su conjunto:
4f
245q103
 f1,mx  f 2,mx  f3,mx  mtot amx l amx   1,mx  
  0.98 m/s 2
mtot
250q103

- 77 -

Fsica Universitaria: Problemas de Fsica

Sistemas de referencia en rotacin. M09.1

1. Un punto P en el plano Oyz gira alrededor del eje Oy con velocidad angular

Z1, y todo el sistema de referencia gira alrededor del eje Oz con velocidad
angular Z2. a) Hllese la velocidad y aceleracin del punto P en el instante
en que est en la posicin de la figura. Indquese cual es el movimiento
relativo y cual el de arrastre. b) Si el movimiento relativo se convirtiese en
movimiento de arrastre y viceversa, sera el mismo movimiento? dara los
mismos resultados de velocidad? y de aceleracin? Justifquese las
respuestas.

Z2

P
a
y

Z1
x

a) Velocidad angular de arrastre:  arr   2


Movimiento relativo:
JJG 0 0 aX1
v rel  1 q OP  X1 qb  0
0 a 0

JJG 0 aX1 0
a rel  1 q (1 q OP)= X1 q 0  0

0 0
aX12

Movimiento de arrastre:
JJG 0 0 bX2
v arr   2 q OP  0 qb  0
X a 0
2

Aceleracin de Coriolis:

JJG 0 bX2 0 2

a arr   2 q ( 2 q OP)  0 q 0  bX2

X2 0 0
acor

0 aX1 0

 2 2 q v rel  2 0 q 0  2aX1X2

X2 0 0

Movimiento absoluto:
aX1  bX2

v abs  v rel v arr 


0

aabs  a rel aarr acor

2
 2aX1X2  bX2

aX12

b) Velocidad angular de arrastre:  arr  1

Se permutan los papeles de las velocidades angulares Z1 y Z2 en el apartado anterior.


Se obtienen movimientos diferentes en los dos casos (trayectorias diferentes).
En el instante que se indica en la figura, la velocidad ser la misma en ambos casos,
intercambindose los papeles de las velocidades relativa y de arrastre.
Aunque tambin se intercambian las aceleraciones relativas y de arrastre, en el instante
que se indica en la figura, la aceleracin de Coriolis es diferente en cada caso, ya que
ahora es
0 bX2 0

acor  21 q v rel  2 X1 q 0  0


0 0 2bX X
1 2

- 78 -

Fsica Universitaria: Problemas de Fsica

Sistemas de referencia en rotacin. M09.2

2. La barra BC de la figura gira con velocidad angular Z constante alrededor de B,


mientras que el manguito A se desliza con una velocidad v tambin constante
respecto a la barra. Determinar velocidad y aceleracin absolutas del manguito
A cuando est a una distancia r del punto B.

Referencial fijo o absoluto, XYZ, con origen en el extremo B de la barra.


Referencial mvil o relativo, xyz, con el mismo origen, solidario con la barra, de modo que
en el instante indicado en la figura, coinciden las direcciones de los ejes correspondientes.
Este referencial est en rotacin con una
Y,y
velocidad angular (de arrastre) que es la de la
barra, de modo que:
Z
C
0
B

 arr    0
 arr    0
X,x
A
X
La velocidad y la aceleracin absolutas del manguito vienen dadas por las expresiones
v abs  v rel v arr  v rel v 0  qr
aabs  a rel aarr aCor  a rel a 0  qr  q ( qr ) 2 q v rel
con
v0  0

a0  0

JJJG r
r  BA  0
0

v

v rel  0

a rel  0

de modo que
v 0 r v

v abs  v rel  qr  0 0 q0  X r
0 X 0 0

0 v
2

0 v X r
a abs   q ( qr ) 2 q v rel  0 qX r 2 0 q0  2Xv

X 0
X 0 0

- 79 -

Fsica Universitaria: Problemas de Fsica

Sistemas de referencia en rotacin. M09.3

3. Un punto P se mueve sobre la superficie terrestre de modo que su latitud O

fijo
mvil

y su longitud M vienen dadas por las expresiones

t
t
(S.I.)
K  106 (S.I.)
3
6
Considerando la Tierra como una esfera de 6400 km de radio, determnense,
Q
Q
para M 
y K  : a) la velocidad relativa a la Tierra, b) la velocidad
3
6
absoluta y c) la aceleracin de Coriolis.

M  106

P
 r

a) Expresamos vectorialmente la posicin y la velocidad relativa del punto P




x R cos M cos K
RM sen M cos K  RK cos M sen K

r
d

r  y  R cos M sen K
vM 
 RM sen M cos K RK cos M cos K

dt
z R sen M

RM cos M

106
106
M 
m/s y K 
m/s
3
6
Q
Q
de modo que, para M   60 y K   30 , sern:
3
6
3
x
cos 60 cos 30

r  y  R cos 60 sen 30  1

sen 60 4 2 3
z
con R  6.4q106 m,

1
1

 3 sen 60 cos K  6 cos 60 sen 30 1.87


1
1
v M  10 R  3 sen 60 cos K 6 cos 60 cos 30  0.46 m/s

1.07
3 cos 60
6

b) Velocidad absoluta: v F  v M v o  qr

1
1 116.36
0
6

R 3
R
2
Q
6.4
q
10

3  201.52 m/s
q
 qr  0 q 1  X 3 

4 0 86400
4

X 4 2 3

0 0
1.87 116.36 118.23



v F  0.46 201.52  201.06 m/s
1.07 0 1.07

c) Aceleracin de Coriolis: aCor  2 v M q

a Cor

1.87 0 6.72

 2 0.46q 0  27.1q105 m/s 2

1.07 2Q 0

86400

- 80 -

Fsica Universitaria: Problemas de Fsica

Sistemas de referencia en rotacin. M09.4

4. En el hemisferio Norte, un automvil, que pesa 1000 kg, circula por una autopista con una velocidad de
144 km/h. En un instante dado, el automvil avanza en la direccin Sur-Norte en un lugar de 40( de latitud.
a) Determinar la velocidad y la aceleracin absoluta del automvil en ese instante, considerando tan slo el
movimiento de la Tierra como rotacin pura alrededor de su eje polar. b) Calcular el valor (mdulo y
direccin) de la fuerza de Coriolis en ese instante.

Consideramos los Referenciales XYZ (fijo o absoluto) y


xyz (ligado a tierra, mvil o relativo), tal como se indica
en la figura. Nos servimos de las expresiones
correspondientes al movimiento relativo, teniendo en
JJG
cuenta que r  oP  0 :

Z
vM


O

d
qr  q  qr
2 q v M 
dt

a(o)F

v F  v M v F (o)  qr  v M v F (o)
a F  a M a F (o)


Y

 a F (o) 2 q v M

Expresamos las velocidades y aceleraciones


intervinientes en la base vectorial asociada al
referencial mvil o relativo (ligado a tierra) y
efectuamos los clculos correspondientes:
v


0
v M  0
v F (o)  X R cos M

0
0
X cos M 0 X cos M 0 X 2 R cos M sen M

q 0 
qX R cos M 
a F (o)   q  q R
  q
0
0
0


X sen M R X sen M 0 X 2 R cos 2 M
aCor

X cos M v

0
0

q 0  2Xv sen M l FCor  maCor  2mXv sen M


 2 q v M  2
0

X sen M 0

0
0

O sea
v 0 v

v F  0 X R cos M  X R cos M

0 0 0

X 2 R cos M sen M
X 2 R cos M sen M
0


a F 
0
2Xv sen M  2Xv sen M

2
2
2
2

0
X R cos M
X R cos M

Sustituimos los valores numricos:

2Q
rad/s; R  6.3q106 m; M  40 ; m  1000 kg
86400
0.016 68
0
0

2
a F  0.003 75 m/s
FCor  3.75 N  0.385 Kg

0.019 88
0
0

v  144 km/h=40 m/s; X 


40

v F  355.8 m/s
0

- 81 -

Fsica Universitaria: Problemas de Fsica

Sistemas de referencia en rotacin. M09.5


z, z

5. El plano vertical representado en la figura gira alrededor del eje Oz con


velocidad angular constante de 120/S r.p.m. Un disco de 2 cm de radio,
contenido en dicho plano, rueda sin deslizar sobre la interseccin del mismo
plano con el plano xy, con una velocidad de traslacin de 4 m/s. Calcular la
velocidad y la aceleracin del punto P del disco diametralmente opuesto al de
contacto con el plano xy cuando el centro del disco est a 3m del eje z y O
T = S/2.

P
C
vC

y
Consideremos un sistema de referencia fijo (xyz) y otro
sistema de referencia mvil (xyz) cuyo plano yz es solidario al plano vertical representado
en la figura. En el instante considerado el plano mvil coincide con el plano yz, de modo que,
en ese instante, coinciden las bases vectoriales de ambos referenciales.
El movimiento relativo del disco es una rodadura pura sobre el eje Oy y el movimiento de
arrastre es una rotacin pura alrededor del eje Oz con velocidad angular
0

120 2Q
q
= 4 rad/s  arr  0 rad/s
Xarr 
Q 60
4

La rodadura del disco en el plano mvil nos relaciona la velocidad del centro del disco,
vC = 4 j m/s con la velocidad angular de rotacin Zrel del mismo; i.e.,
200

v
4

 200 rad/s l  rel  0 rad/s


vC  Xrel R l Xrel  C 
R 0.02
0
La velocidad absoluta del punto P (vP) ser la suma de su velocidad relativa (vrel) y su
velocidad de arrastre (varr):

JJG 0 200 0 0

CP
v
v



q
 4 0 q 0  8 m

rel
C
rel

0 0 0.02 0 s

12

v
v
v
8




m/s
P
rel
arr

0 0 12

JJG
0

v arr   arr qOP  0q 3  0 m

4 0.04 0

La aceleracin absoluta punto P (aP) la calculamos como la suma de la aceleracin relativa


(arel), la de arrastre (aarr) y la de Coriolis (aCor):
JJG
JJG
JJG 0
d
2
rel

qCP  rel q  rel q CP  Xrel CP  0 m 2


a rel  aC

d t
800 s
JJG
JJG 0 12 0
d
arr

q OP  arr q  arr qOP  0q 0  48 m 2


aarr 
d t
4 0 0 s

64
0 0 64

aCor  2 arr q v rel  2 0q8  0 m 2 l a P  a rel a arr a Cor  48 m 2

s
800 s
4 0 0

- 82 -

Fsica Universitaria: Problemas de Fsica

Trabajo y energa. M10.1

1. Un proyectil de 5 g de masa lleva una velocidad de 400 m/s en el instante en que impacta en el tronco de un
gran rbol en el que penetra una distancia de 4 cm hasta detenerse. Supongamos, para simplificar, que la
resistencia que presenta la madera al avance del proyectil sea constante. a) Calcular la fuerza que ejerci el
proyectil sobre el rbol y la resistencia de la madera a la penetracin. b) Calcular la potencia desarrollada
por esa fuerza Es constante? c) Estimar el tiempo empleado en el frenado del proyectil.

a) Puesto que la resistencia que opone la madera al avance del proyectil se supone constante,
ser constante la desaceleracin que este experimenta. Podemos calcular la aceleracin de
frenado a partir una bien conocida frmula del movimiento rectilneo uniformemente
acelerado:
v02
4002

 2q106 m/s 2
2x 2q 0.04
La fuerza que ejerce el proyectil sobre el rbol ser igual y opuesta a la fuerza de resistencia
que presenta la madera al avance del proyectil; esto es,
vf2  v02  2ax  0 l a 

F  ma  5q103 q 2q106  104  10 000 N


b) La potencia desarrollada por esa fuerza viene dada por P  Fv . Aunque F se supone
constante, la potencia va disminuyendo a medida que se ralentiza el proyectil. En el instante
inicial vale
P  F v  Fv  104 q 400  4q106 W  4 MW
c) Podemos calcular el tiempo pedido a partir de
v
v
400
l t 

 2q104  200 s
a
t
a
2q106
Mtodo de la energa:
a) Durante la penetracin en la madera, hasta quedar en reposo, el proyectil pierde toda su
energa cintica

1
5q103 q 4002
Ek  mv02 
 400 J
2
2
Como la fuerza resistente que se opone al avance del proyectil se supone constante, el
teorema del trabajo y la energa cintica nos permite escribir:
Ek
400
Wresist  Fresist x  Ek l Fresist 

 10 000 N
x
0.04

- 83 -

Fsica Universitaria: Problemas de Fsica

Trabajo y energa. M10.2

2. Un automvil que pesa 750 kg circula por una carretera a nivel con una velocidad 54 km/h cuando su motor
desarrolla una potencia de 10 CV. a) Cunto vale la suma de todas las resistencias (rozamiento, resistencia
del aire, ...) que actan sobre el automvil? b) Qu potencia deber desarrollar el motor del automvil para
subir a 54 km/h una cuesta del 10% de pendiente? c) Qu potencia ser necesaria para que el automvil
baje a 54 km/h una pendiente del 3%? d) Qu pendiente permitir que el automvil baje a una velocidad de
54 km/h sin que funcione el motor? (Nota: supngase que todas las fuerzas de resistencia permanecen
constantes). Datos: 1 CV = 736 W.

Datos: v  54 km/h  15 m/s,

P  10 CV  7360 W

a) Puesto que la velocidad permanece constante, la potencia


desarrollada por el motor se emplear en vencer todas las
resistencias que se oponen al movimiento del automvil; esto
es,
P 7360
P  Fv  f v l f  
 490.7 N  50 kg
v
15
b) Una pendiente del 10% representa un ngulo  tal
que tg R  0.1 l R  5.7 . Ahora, la potencia desarrollada
deber vencer tambin la componente del peso del automvil
en la direccin del movimiento que se opone al movimiento
del automvil; esto es,

f
F

N
F
f
mg

P  Fv  f mg sen R
v 
 490.7 750q9.8sen 5.7
q15  18331 W  24.9 CV
c) Una pendiente del 3% representa un ngulo  tal
que tg R  0.03 l R  1.7 . Ahora, la componente del peso
del automvil en la direccin del movimiento favorece el
movimiento del automvil; esto es,

P  Fv  f  mg sen R
v 

F
mg

 490.7  750q9.8sen1.7
q15  4 055 W  5.5 CV
d) Con el motor parado, i.e., P = 0, la resistencia estar compensada con la componente del
peso del automvil en la direccin del movimiento :
490.7
50
f


 0.067
f  mg sen R l sen R 
mg 750q9.8 750

lo que representa una pendiente de


R  3.83 l tg R  0.067  6.7%

- 84 -

Fsica Universitaria: Problemas de Fsica

Trabajo y energa. M10.3

3. Una escalera homognea, de masa m y longitud L, est apoyada sobre una pared
vertical lisa y sobre un suelo horizontal rugoso, formando un ngulo 0 con la
horizontal (vide figura). El coeficiente de rozamiento entre el suelo y el pie de la
escalera es . Calcular el trabajo que debemos realizar para llevar la escalera a la
posicin vertical, empujndola horizontalmente a una distancia D de su pie.

D
F


En la figura se muestra el sistema de fuerzas que acta


sobre la escalera en un instante genrico durante el
proceso reversible (sucesin de estados de equilibrio)
que nos lleva desde el estado inicial ( = 0) hasta el
final ( = 90).
Mtodo Conservacin de la Energa:
f P N (movimiento inminente)
o f PP

(equilibrio)
N P

R
B

D
F

De modo que la fuerza de rozamiento permanece


constante durante todo el proceso y el trabajo realizado
por ella es
W f  f L cos T 0  P P cos T 0


A

Aplicamos el teorema de la energa cintica, teniendo en cuenta que Ecintica=0, por tratarse
de un proceso reversible:
W Wcons  WF  W f 'Ecintica 0
Wcons  WF  W f

0 o  'Ep  WF  W f

0 o WF

'Ep  W f

Variacin de la energa potencial:


Ep  P 12 L  12 L sen R0
 12 PL 1 12 sen R0

? WF

'Ep  W f

1
2

PL 1  sen T 0  P PL cos T 0

1
2

PL 1  sen T 0  2 P cos T 0

Clculo directo del trabajo de la fuerza F:


Escribimos las ecuaciones cardinales de le Esttica, tomando momentos en A:

N P

o F PP  R o R F  PP

f  R F
o F

cos T
D
RL
1
1
sen T 2 PL cos T  FD sen T o R 2 P sen T  F L
Calculamos el trabajo, teniendo en cuenta que
D
x  L  D
cos R l dx   L  D
sen R dR  L 1 sen R dR

cos T
2sen T P
D
1
L

P

De modo que
WF  

90

R0

F dx 

90

R0

90

cos R
PL N
sen R dR  PL N sen R 12 cos R
dR 

R0
2sen R

 PL <N cos R 12 sen R >R  PL 12 N cos R0  12 sen R0


 12 PL 1 2N cos R0  sen R0

90
0

- 85 -

Fsica Universitaria: Problemas de Fsica

Trabajo y energa. M10.4

4. Un bloque de masa m se deja caer sobre una cinta transportadora que se mueve con
velocidad constante v. Determinar el tiempo y el espacio recorrido por el bloque
hasta que ste adquiera la velocidad de la cinta, siendo P el coeficiente de
rozamiento entre la cinta y el bloque.

Mtodo 1 Planteamos el problema en el referencial S.


La fuerza de rozamiento que acta sobre el bloque es la
que lo acelera hasta que adquiere la misma velocidad
que la cinta: esto es,
f  N N  Nmg  ma l a  N g  cte

S
f
v

de modo que se trata de un movimiento rectilneo


uniformemente acelerado, en el que la velocidad inicial

del bloque es nula. Por consiguiente:


vf  v0 at l t 
vf2  v02 2ax l x 

v
v

a Ng
v2
v2

2 a 2N g

Mtodo 2 Planteamos el problema en el referencial S que se mueve con velocidad


constante (inercial).
La fuerza de rozamiento sigue siendo la misma que antes,
f  N N  Nmg  ma l a  N g  cte

pero v0 = -v y vf = 0
vf  v0 at l 0  v at l t 

v
v

a Ng

vf2  v02 2ax a l 0  v 2 2ax a l x a  

v2
v2

2a
2N g

donde el signo negativo significa que el bloque se mueve hacia atrs con respecto de la
cinta.
Mtodo 3 El trabajo realizado por la nica fuerza horizontal que acta sobre el bloque
(fuerza de rozamiento ) es igual al incremento de su energa cintica:

W f  Ek

v2
1
l Nmgx  mv 2 l x 
2
2N g

- 86 -

Fsica Universitaria: Problemas de Fsica

Trabajo y energa. M10.5

5. Un ascensor desciende con una velocidad constante de 0.75m/s. Del techo del ascensor se desprende una de
las bombillas de 40 g, que case sobre el suelo del ascensor. La altura de la caja del ascensor es 2.2 m.
Calcular el trabajo realizado por la fuerza gravitatoria sobre la bombilla y la variacin de la energa cintica
de la misma, desde que se desprende hasta que se estrella en el suelo del ascensor: a) en el referencial ligado
a la caja del ascensor; b) en el referencial ligado al edificio. c) Explicar las diferencia existentes entre los
resultados de los aparatos anteriores.

Consideramos dos sistemas de referencia inerciales:


1. Referencial inercial S, ligado al edificio.
A
A
2. Referencial inercial S, ligado a la caja del
S
ascensor
A
h
Durante
su movimiento de cada libre, la bombilla tan
v0
s
solo est sometida a la fuerza gravitatoria, i.e., a su peso
B
s
v0
mg, que representa una fuerza constante. La aceleracin
de la bombilla ser la misma (g) en ambos referenciales
v0t
inerciales. El trabajo realizado por dicha fuerza ser
B
B
igual al producto de la misma por el desplazamiento que
experimenta la bombilla, que ser diferente en cada
referencial. La velocidades y los cambios de energa
cintica tambin sern diferentes en cada referencial. El tiempo de cada es el mismo en
ambos referenciales:
S

h

1 2
gtcada
2

l tcada 

2h
2q 2.2

 0.67 s
9.8
g

a) En el referencial S:

W a  WA aBa  mg A aBa  mgs a  mgh  0.040q9.8q 2.2  0.86 J


vBa 2a  vAa 2a 2 gh l Eka  mvBa 2a  mvAa 2a  m vBa 2a  vAa 2a
 mgh  0.86 J
1

de modo que W a  Eka (teorema de la energa cintica).


b) En el referencial S:
s  s a v0t  h v0tcaida  2.20 0.75q 0.67  2.20 0.50  2.70 m
W  WAB  mg AB  mgs  0.040q9.8q 2.7  1.06 J
vB2  vA2 2 gs l Ek  mvB2  mvA2  m vB2  vA2
 mgs  1.06 J
1

de modo que W  Ek (teorema de la energa cintica).


c) El trabajo realizado es mayor en el referencia S que en el referencial S; lo que est de
acuerdo con las correspondientes variaciones de la energa cintica.
La explicacin radica en que tanto la energa cintica como sus cambios, depende del
referencial en el que se mida. Podemos asegurar que el trabajo suplementario que se mide en
el referencial S coincide con la variacin suplementaria del energa cintica que se mide en
ese mismo referencial.

- 87 -

Fsica Universitaria: Problemas de Fsica

Conservacin de la energa. M11.1

1. Desde lo alto de una torre de 30 m de altura se lanza un cuerpo con una velocidad de 20 m/s y una
inclinacin de 45 (en elevacin) con respecto a la horizontal. a) Con qu velocidad (mdulo) llegar al
suelo? b) Se conseguir que llegue al suelo con ms velocidad si lanzamos el objeto con otro ngulo de
inclinacin? Con cul?

a) Conservacin de la energa:
1
1
mgH mv02  mvf2 l vf  v02 2 gH  202 2q9.8q30  31.43 m/s
2
2
b) En el planteamiento y resultado del apartado anterior no interviene el ngulo de
lanzamiento. Por consiguiente, no se conseguir ni ms ni menos velocidad para otros
ngulos.

v0=20 m/s
T0=45

H=30 m

Vf=31 m/s

- 88 -

Fsica Universitaria: Problemas de Fsica

Conservacin de la energa. M11.2

2. Una masa puntual m est unida a un hilo inextensible de masa despreciable y

longitud l. La masa se deja caer desde la posicin horizontal con velocidad


inicial nula, como se indica en la figura. Determinar, en funcin del ngulo ,
la velocidad de la masa, la aceleracin tangencial, la aceleracin normal y la
tensin de la cuerda. Particularizar los valores de las magnitudes anteriores
para  = 0 y  = 90.

Conservacin de la energa
1
0  mgh mv 2 l v  2 gh  2 gl sen R
2
La aceleracin centrpeta o normal se determina fcilmente por
tratarse de una trayectoria circular:

v2
an   2 g sen R
l
La aceleracin tangencial se obtiene a partir de la ecuacin del
movimiento tangencial:
mg cos R  mat l at  g cos R

h
m
mg

Tambin podemos obtenerla como la derivada de la celeridad (mdulo de la velocidad):


2 gl sen R
dv dv dR
dv v
d
cos R

X

 g cos R
2 gl
sen R 
2 gl
d t dR d t
dR l
dR
l
2 sen R
El mdulo de la aceleracin es
at 

a  at2 an2  g 2 cos 2 R 4 g 2 sen 2 R  g 1 3sen 2 R .

Para determinar la tensin de la cuerda, escribimos la componente radial de la ec. del


movimiento:
N  mg sen R  man

l N  m g sen R an

en la que sustituimos el valor de la aceleracin normal para obtener la tensin en funcin del
ngulo :
N  m( g sen R 2 g sen R )  3mg sen R

ngulo velocidad ac. tangencial ac. normal md. aceleracin tensin


0

90

2gl

2g

2g

3mg

- 89 -

Fsica Universitaria: Problemas de Fsica

Conservacin de la energa. M11.3

3. Una masa m est suspendida de un hilo de longitud A atado en A. Si se libera desde la


posicin indicada, a que distancia mnima h habr que situar un obstculo fijo B, para
que el hilo se enrolle en el obstculo.
A

A
60

Conservacin de la energa entre 1 y 2:


1
mgl cos 60  mgl mv22
2
v22  2 gl (1 cos 60 )  gl [1]

El radio de la trayectoria circular centrada en B es R = l h.


Conservacin de la energa entre 2 y 3:
1
1
mg (l  h) mv22  mg (l  h) mv32
2
2
v32  v22  4 g (l  h)  gl  4 g (l  h)  (4h  3l ) g [2]
Aplicamos la 2 ley de Newton en el punto ms alto de
la trayectoria circular, teniendo en cuenta que, en las
1
circunstancias crticas de mnima velocidad, la tensin
de la cuerda sera nula:
mg T3  macp  m

2
3

v
l v32  g (l  h) [3]
(l  h)

60
h
v3 3
T mg
B
l-h

Combinamos las expresiones [2] y [3]:


4
(4h  3l ) g  (l  h) g l 5h  4l l h  l
5

- 90 -

v2

Fsica Universitaria: Problemas de Fsica

Conservacin de la energa. M11.4

4. Un pequeo bloque de masa m desliza sin rozamiento por una gua en


forma de lazo como la indicada en la figura. El bloque parte del reposo
desde el punto P.
a) Determinar la altura mnima, h, desde la que debe partir el bloque para
alcanzar la parte superior del lazo sin separarse de la gua.
Ahora, supongamos que la altura h sea igual a 5R. b) Determinar la
reaccin de la gua sobre el bloque en el instante en el que ste alcanza la
parte superior del lazo. c) Calcular las componentes normal y tangencial de
la aceleracin del bloque en dicho instante.

mg
N

P
m
h

a) Conservacin de la energa:
1
mgh  mg (2 R) mv 2 l v 2  2 gh  4 gR
2
Ecuacin del movimiento en el punto ms alto (Q):

[1]

v2
l v 2  gR [2]
R
con N = 0 en las condiciones crticas de llegar hasta Q.
Combinando las expresiones [1] y [2], se sigue
5
2 g h4 g R  g R l h  R
2
b) Rescribimos las expresiones [1] y [2] con h=2h:
1
5
mg (2h)  mg (2 R) mv a 2 l v a 2  4 gh  4 gR  4 g R  4 gR  6 gR
2
2
va2
va2
l N m
 mg  6mg  mg  5mg
mg N  m
R
R
c) Las componentes intrnsecas de la aceleracin, en el punto Q, son:
x at  0 , ya que no hay componente de fuerza en la direccin tangencial
mg N  m

an  acp 

va2
 6 g , es la componente normal o centrpeta de la aceleracin.
R

- 91 -

Fsica Universitaria: Problemas de Fsica

Conservacin de la energa. M11.5

5. Una masa m, de pequeas dimensiones, est sujeta mediante un hilo ligero

l
y longitud l a un punto fijo O. La masa se abandona desde el punto A,
como se indica en la figura. Consideremos el instante en que el hilo forma
O
T
un ngulo T con la horizontal. a) Determinar la velocidad de la masa.
l
b) Calcular las componentes intrnsecas de la aceleracin de la masa. c)
m
Hallar la tensin del hilo. d) Particularizar los resultados anteriores para
T = 0; T = 30; T = 90; recogiendo los valores en una tabla y dibujando los vectores en un esquema para
cada caso.

A
h

an
T
mg

a) Como el sistema es conservativo, aplicamos el


Principio de Conservacin de la Energa para
determina la velocidad en funcin del ngulo:
1
0  mgh mv 2 l v  2 gh
2
con h =l sen, de modo que
v  2 gl sen R

at

b) La aceleracin centrpeta o normal asociado al


movimiento circular es:

v2
 2 g sen R
l
c) Para determinar la aceleracin tangencial y la tensin del hilo escribimos las ecuaciones del
movimiento:

T  mg sen R  man
T  mg sen R 2mg sen R  3mg sen R

mg cos R  mat
at  g cos R

d) Casos particulares:
an 

30

v  2 gl sen R

gl

T  3mg sen R

1.5 mg

3mg

an  2 g sen R

2g

at  g cos R

0.87g

- 92 -

90
v  2 gl

Fsica Universitaria: Problemas de Fsica

Conservacin de la energa. M11.6

6. Una cadena de acero de 3 m de longitud y 20 N/m de peso est estirada sobre una mesa horizontal de forma
que 2 m de la misma permanecen sobre la mesa y 1 m cuelga verticalmente desde el borde de la misma. En
estas condiciones, el peso del segmento colgante es justamente suficiente para arrastrar toda la cadena fuera
del borde. a) Cul es el coeficiente de rozamiento esttico entre la cadena y la mesa? Supongamos que, una
vez que la cadena est en movimiento, el coeficiente de rozamiento cintico sea tan pequeo que pueda
despreciarse. b) Calcular el trabajo realizado sobre la cadena por la fuerza de la gravedad desde que empieza
a deslizar (2 m de cadena sobre la mesa) hasta que toda la cadena abandona la mesa. a) Determinar la
velocidad de la cadena en el instante en que sta abandona la mesa.

l-x
m-mx
f

x
mx
mxg

El peso de la cadena es mg = 3 m u 20 N/m = 60 N.


a) En las condiciones del enunciado, el peso de la porcin que
cuelga ( mx0 = 1/3) est compensado justamente por la fuerza de
rozamiento sobre la otra porcin que permanece sobre la mesa:
mx0
1/ 3
mx0 g  f  N m  mx0
g l N 

 0.5
m  mx0 2 / 3

b) El trabajo realizado sobre la cadena por la fuerza de la gravedad es igual a la disminucin

de la energa potencial gravitatoria de la cadena:


m

8
8
W  E p  Ep,in  Ep,fin   g q 0.5  mg q1.5
 mg  60  80 J
3

6
6
c) La disminucin de la energa potencial de la cadena es igual, en todo instante, al incremento

de su energa cintica. Considerando los instantes indicados en el enunciado, ser:


Ek  Ep

1 2 8
8g
mv  mg l v 
 5.11 m/s
2
6
3

- 93 -

Fsica Universitaria: Problemas de Fsica

Conservacin de la energa. M11.7

7. Una partcula de masa m est situada en la cima de una semiesfera lisa, de radio R, que est apoyada por su
base sobre un plano horizontal. Cuando desplazamos ligeramente la partcula de su posicin de equilibrio
comienza a deslizar sobre la superficie de la esfera. La posicin de la partcula queda determinada en cada
instante por el ngulo T que forma el radio-vector correspondiente con la vertical. Determinar el valor del
ngulo para el cul la partcula se despega de la semiesfera.

Conservacin de la energa:
1
mgR  mgR cos R mv 2 l v 2  2 gR(1 cos R )
2
Componente radial de la ecuacin del movimiento:

v2
v2
l N  mg cos R  m
R
R
Sustituyendo en esta ecuacin el valor de la velocidad,
tenemos
N  mg cos R  2mg (1 cos R )  mg (3cos R  2)
mg cos R  N  m

N
R


mg

En el instante en que la partcula se despega de la semiesfera, la reaccin normal ser nula


(se rompe la ligadura), de modo que
2
3cos Rmax  2 l cos Rmax 
l Rmax  48.2
3

- 94 -

Fsica Universitaria: Problemas de Fsica

Conservacin de la energa. M11.8

8. Un carrito pesa 8 kg y se mueve sin friccin, con una velocidad de 1 m/s,

sobre unos rieles rectilneos y horizontales. Dejamos caer verticalmente un


v0
pequeo objeto de 2 kg de masa sobre el extremo delantero del carrito.
M
Inicialmente, la velocidad del objeto es nula; pero, como consecuencia de su
friccin con el carrito (coeficiente P = 0.1), termina quedando en reposo
sobre el carrito, con tal que la longitud l de ste sea suficientemente grande.
l
a) Calcular la velocidad final del sistema y el tiempo empleado en alcanzarla.
b) Determinar el valor mnimo de la longitud del carrito que permita que el objeto se detenga sobre l.

a) Sea vf la velocidad final del sistema cuando el objeto,


m
despus de deslizar sobre el carrito, queda en reposo sobre el
f
f
M
mismo. Puesto que no existen fuerzas externas al sistema
carrito objeto que tengan componentes en la direccin
v
horizontal, se conserva la cantidad de movimiento del sistema
en esa direccin. Esto es,
M
8
Mv0  ( M m)vf l vf 
v0  q1  0.8 m/s
l
M m
10
La nica fuerza que acta sobre el objeto (en la direccin de su movimiento) es la de friccin,
de modo que la ec. de su movimiento es:
f  maobj l Nmg  maobj l aobj  N g  0.1q9.8  0.98 m/s 2

de modo que el objeto posee un movimiento uniformemente acelerado, partiendo del reposo,
hasta que adquiere la velocidad final vf. Podemos escribir:
v
0.8
v  at l vf  aobjtf l tf  f 
 0.82 s
aobj 0.98
b) El trabajo realizado por la fuerza de rozamiento durante el recorrido x que realiza el objeto
sobre el carrito es igual a la prdida de energa cintica que experimenta el sistema durante
ese proceso. Esto es,
1
1
Mv02  ( M m)vf2  Nmgx
2
2
Despejamos el recorrido x d esta expresin y obtenemos

1
1
8q12 10q 0.82
Mv02  ( M m)vf2

4  3.2
2
2
x 2
 2

 0.41 m  41 cm
Nmgx
0.1q 2q9.8
1.96
de modo que la longitud del carrito deber ser mayor de 41 cm a fin de que el objeto no se
salga por la parte trasera del carrito.
(sigue)

- 95 -

Fsica Universitaria: Problemas de Fsica

Conservacin de la energa. M11.9

Otro mtodo
b) La nica fuerza que acta sobre el carrito en la direccin de su movimiento es la fuerza de
rozamiento. Calculamos la aceleracin del carrito:
m
2
f  Macar l  Nmg  Macar l acar  N g  0.1q q9.8  0.25 m/s 2
M
8
En el referencial del carrito, el objeto se mueve inicialmente con una velocidad v0 (esto es,
hacia la parte trasera del carrito) y va disminuyendo su velocidad (relativa) hasta que
finalmente queda en reposo en ese referencial.
La aceleracin del objeto con respecto al carrito (aceleracin relativa) es:

arel

aobj  acar

0.98  25 1.23 m/s 2

de modo que el objeto presenta un movimiento uniformemente retardado relativo al carrito.


Utilizando la bien conocida frmula de la cinemtica v 2  v02 2ax , durante el proceso de
frenado, obtenemos el espacio que recorre el objeto sobre el carrito:

0  v02  2arel x l x 

v02
12

 0.41 m  41 cm
2arel 2q1.23

de modo que la longitud del carrito deber ser mayor de 41 cm a fin de que el objeto no se
salga por la parte trasera.

- 96 -

Fsica Universitaria: Problemas de Fsica

Fuerzas centrales. M12.1

1. La velocidad de escape es aquella velocidad inicial que necesita cualquier proyectil para escapar de la
atraccin gravitatoria de un planeta u cualquier otro cuerpo de gran masa Calcular la velocidad de escape
desde la superficie terrestre?
Datos: Radio ecuatorial de la Tierra: 6371 km; Masa de la Tierra: 5.979u1024 kg; G = 6.672u10-11 (SI)

r
R

vesc

Conservacin de la energa para el lanzamiento del


proyectil con una velocidad inicial vescape para
conseguir que llegue a una hipottica distancia
infinita con velocidad nula:
1 2
Mm
mvescape  G
 0 0
2
R
De modo que

1 2
Mm
2GM
mvescape  G
l vescape 
R
R
2
Podemos expresar este resultado en funcin de g (intensidad del campo gravitatorio en la
superficie del planeta), ya que
.m
.
G 2  mg l g  G 2
R
R
de modo que
vescape 

2GM
 2 gR
R

Sustituyendo valores para la Tierra, tenemos


2GM
2q 6.672q1011 q5.979q1024

 11.2q103 m/s  11.2 km/s
R
6.371q106
La velocidad de escape en la superficie de la Tierra es de 40 320 km/h. A velocidades
inferiores, el proyectil se convertira en un satlite artificial en rbita elptica alrededor de la
Tierra.
Esta velocidad inicial es aplicable tan solo a proyectiles; esto es, objetos que carecen de un
medio de propulsin propio de modo que dependen de su impulso inicial para vencer la
atraccin gravitatoria.
vescape 

- 97 -

Fsica Universitaria: Problemas de Fsica

Fuerzas centrales. M12.2

2. rbita geoestacionaria. Supngase que se desea establecer en el espacio una base interplanetaria que se
mueva en una rbita circular en el plano ecuatorial de la Tierra y a una altura tal que permanezca siempre
sobre el mismo punto. a) Cul deber ser el radio de esa rbita? b) Qu velocidad tendr en satlite en
dicha rbita?

Para que la rbita sea geoestacionaria, el satlite deber girar en su


rbita con la misma velocidad angular con que rota la Tierra
m
R
alrededor de su eje:
F
2Q
2Q
X

 7.29q105 rad/s
M
T
86164

siendo el periodo el correspondiente a un da sideral ( = 86164 s).
a) Aplicamos la Ley de la Gravitacin Universal al satlite en
rbita, teniendo en cuenta que la fuerza de atraccin gravitatoria le
imprime una aceleracin centrpeta, de mdulo X 2 R , de modo que
Mm
GM
F  G 2  macp  mX 2 R l GM  X 2 R 3 l R 3  2
R
X
Para determinar la masa de la Tierra, consideraremos un cuerpo de masa m en cada libre
cerca de su superficie, siendo RT el radio de la Tierra:

F G

gRT2
Mm
mg
M

l

RT2
G

que sustituimos en la expresin anterior para obtener


R3 

gRT2
X2

gR 2
l R  2T
X

1/3

De modo que
9.8q 6.37q106 2


6
R 
 42.14q10 m  6.62 RT
2
7.29q105

1/3

b) La velocidad orbital se puede calcular multiplicando su velocidad angular por el radio


orbital:

v  X R  7.29q105 q 42.14q106  3072 m/s  11 060 km/h

- 98 -

Fsica Universitaria: Problemas de Fsica

Fuerzas centrales. M12.3

3. Un satlite de comunicaciones describe una rbita circular ecuatorial en el mismo sentido de rotacin de la
Tierra, a una altura de 800 km sobre su superficie. Por dnde sale el satlite, por el Este o por el Oeste?
Durante cunto tiempo permanecer visible (sobre el horizonte) desde un lugar llano y despejado situado
en el Ecuador?
Datos: Radio ecuatorial de la Tierra: 6371 km; Masa de la Tierra: 5.979u1024 kg; G = 6.672u10-11 (SI)

Radio de la rbita del satlite


Rs  6371 800  7171 km

ngulo central correspondiente al orto (salida) y


ocaso (ocultacin) del satlite en el horizonte:
6371
R R
cos  T 
 0.888
2 Rs 7171

RT
T
Hemisferio
Norte

R  54.64  0.9537 rd

Rs

s

rel

La velocidad angular asociada con el movimiento del


satlite en su rbita alrededor de la Tierra se deduce a
partir de la Ley de la Gravitacin Universal:
G

M T ms
 ms acp  ms Xs2 Rs
Rs2

l Xs 

GM T
 1.04q103 rd/s
Rs3

2Q
 7.29q105 rad/s .
86400
Puesto que estamos observando el satlite desde la Tierra, su velocidad angular relativa a la
Tierra ser:

La velocidad de angular de la Tierra es XT 

Xrel  Xs  XT  9.67q104 rad/s


donde el signo positivo indica que la salida (orto) se produce por el OESTE. Dicho de otro
modo, la velocidad angular del satlite es superior a la terrestre, por lo que el satlite se
adelanta a sta.
El satlite estar visible mientras recorre un arco de circunferencia de ngulo central  con
una velocidad angular relativa a la Tierra rel; esto es,
R
0.9537

 986 s  16 min 26 s
t
Xrel 9.67q104

- 99 -

Fsica Universitaria: Problemas de Fsica

Fuerzas centrales. M12.4

4. Masa del Sol. Conocidos los semiejes mayores de las rbitas de la Tierra y de la Luna, 149.6106 km y

384.0103 km, respectivamente y los correspondientes periodos de revolucin, 1 ao y 27.32 das, calcular
la masa del Sol en unidades de la masa de la Tierra.

A partir de la Ley de la Gravitacin Universal deducimos la expresin de la Tercera Ley de


Kepler para rbitas circulares:
F G

Mm
4Q 2
4Q 2 3
 mX 2 R l GM  X 2 R 3  2 R 3 l T 2 
R
2
R
T
GM

Aplicamos la Tercera Ley de Kepler a los sistemas Sol-Tierra y


Tierra-Luna y dividimos las expresiones miembro a miembro:
4Q 2 3
TT2 
RT
2
3
GM S
M S TL RT
TT2 M T RT3

l

l


TL2 M S RL3
M T TT RL
4Q 2 3
TL2 
RL
GM T
Finalmente, sustituimos los valores dados en el enunciado del
problema:
M S 27.34


M T 365.24

149 600  5.60q103 q59.129q106  331q103


384
3

Conocida la masa de la Tierra, podemos calcular la del Sol:


Masa de la Tierra:
5.9736 1024 kg
Masa del Sol:
1.9891 1030 kg

- 100 -

RL
T
S

L
RT

Fsica Universitaria: Problemas de Fsica

Fuerzas centrales. M12.5

5. Lunas de Marte. Los semiejes mayores de las dos Lunas del planeta Marte, Phobos y Deimos, miden

9.408103 km y 23.457103 km, respectivamente. El periodo de revolucin orbital de Phobos es de


4.65 horas. Con esos datos se deben calcular la masa del planeta Marte y el periodo de revolucin de
Deimos.

A partir de la Ley de la Gravitacin Universal deducimos la expresin de la Tercera Ley de


Kepler para rbitas circulares:
F G

Mm
4Q 2
4Q 2 3
 mX 2 R l GM  X 2 R 3  2 R 3 l T 2 
R
2
R
T
GM

Aplicamos la Tercera Ley de Kepler al sistema Marte-Phobos:

9.408q10

4Q 2 RP3
4Q 2
MM 

 1.758q1024 kg
2
11
G TP
6.693 x 10 4.65q3600
2
6 3

Aplicamos la Tercera Ley de Kepler a los sistemas Marte-Phobos y Marte-Deimos y


dividimos las dos expresiones miembro a miembro:
4Q 2 3
TP2 
RP
3/2
3/2
RD
GM M
23.457
TP2
RP3

l

l


 18.31 h
T
T
4.65

D
P

9.408
TD2 RD3
RP
4Q 2 3
TD2 
RD
GM M

P
RP
M
RD
D

- 101 -

Fsica Universitaria: Problemas de Fsica

Movimiento armnico simple. M13.1

1. Una partcula realiza un movimiento armnico simple con una frecuencia angular de 10 rad/s. En el instante
inicial, la partcula presenta una elongacin positiva de 8 cm y se mueve con una velocidad de 60 cm/s
acercndose a la posicin de equilibrio. a) Determinar la ecuacin del movimiento (elongacin en funcin
del tiempo). b) Determinar el primer instante en el que la partcula pasa por la posicin de equilibrio.
c) dem en el que la partcula se encuentra instantneamente en reposo.

a) Comenzamos escribiendo las ecuaciones del m.a.s. e imponiendo las condiciones iniciales:
x02  A2 sen 2 G

x  A sen Xt G

x
A
sen
G


l t  0
l v 2

0  A2 cos 2 G
v0  X A cos G
v  X A cos Xt G

de modo que
v
60
A2  x02 0  82
 64 36  100 l
10
X
2

tag G 

A  10 cm = 0.10 m

X x0 10q8

 1.33 l G  126.87  0.705Q rad  2.215 rad
v0
60

y la ecuacin pedida es
x  0.10sen 10t 0.705Q
l v  cos 10t 0.705Q

b) Deber transcurrir menos de medio periodo o


ciclo de oscilacin; esto es, para una fase de S rad,
como se ilustra en el diagrama fasorial:

Diagrama
fasorial

0  0.10sen 10t1 0.705Q


l

x0

a)

10t1 0.705Q
 Q l

(S.I.)

t1  0.0295Q  0.093 s
c) Anlogamente, el instante corresponde a una
fase de 3S/2 rad, como se ilustra en el diagrama
fasorial:
0  cos 10t2 0.705Q
l

3Q
l
2
t2  0.0795Q  0.250 s

10t2 0.705Q


I
b)

v
v0
c)

- 102 -

Fsica Universitaria: Problemas de Fsica

Movimiento armnico simple. M13.2

Q 2
x , donde
16

2. Una partcula de 4 kg de masa se mueve a lo largo del eje x bajo la accin de la fuerza F  

x se expresa en metros y F en newtons. Cuando t = 2 s, la partcula pasa por el origen, y cuando t = 4 s, su


velocidad es de 4 ms-1. Determinar la frecuencia, la amplitud del movimiento y el ngulo de fase inicial.

Q2
, se trata de un movimiento
16
armnico simple, sobre el eje x, alrededor de la posicin de equilibrio x = 0, de modo que x
representa la elongacin. La frecuencia angular de tal movimiento es

Puesto que la fuerza es de la forma F  kx, con k 

X

Q2
Q rad
k


m
16q 4 8 s

l O

X
1

Hz
2Q 16

l T  16 s

La expresin de la elongacin y de la velocidad en funcin del tiempo es


x  A sen Xt G0

v  X A cos Xt G0

Con las condiciones impuestas en el enunciado, ser

2X G0
 0 l G0  2X   rad

t  2 s l A sen 2X G0
 0 l

3Q

2X G0
 Q l G0  Q  2X  rad

32
32
32 2
m

A


 14.4

Q
s
Q
Q cos Q 2

2
t  4 s l X A cos 4X G0
 4 l

32
32
32 2
m

A


 14.4

5Q

s
Q
2

Q cos Q

- 103 -

Fsica Universitaria: Problemas de Fsica

Movimiento armnico simple. M13.3

3. Un objeto de 12 kg cuelga del extremo de un muelle de constante elstica 300 N/m. Iniciamos las
oscilaciones libres del sistema comunicando al objeto una velocidad inicial hacia abajo de 50 cm/s.
a) Calcular la frecuencia y el periodo de las oscilaciones. b) Determinar la amplitud de las oscilaciones.
c) Expresar la elongacin en funcin del tiempo. d) Si inicialmente hubiramos comunicado al objeto una
velocidad mayor, indquense cuales de las magnitudes anteriores se modificarn y de que forma, justificando
las respuestas.

a) La frecuencia de las oscilaciones del sistema masa-muelle est definida por la magnitud de
la masa y el valor de la constante elstica del muelle:

k
300

 25  5 rad/s
m
12
X
5
1
O=

 0.80 Hz l T   1.26 s
2Q 2Q
O
b) De conformidad con el enunciado de este problema, al instante inicial
(t = 0) le corresponde una elongacin nula, por lo que la fase inicial
tambin ser nula. De la expresin de la elongacin se obtiene por
derivacin la de la velocidad
x  A sen Xt l v  X A cos Xt
X

En el instante inicial (t = 0) sern:


t  0 l x0  0 l v0  X A

k
m

x=0

de modo que

v0 50
  10 cm
X
5
c) Elongacin en funcin del tiempo:
x  0.10sen 5 t (SI)
A

v0

d) La frecuencia de las oscilaciones es una propiedad intrnseca del sistema masa-muelle, por
lo que no depende de las condiciones iniciales, de modo que no se modificara en el supuesto
del enunciado.
La amplitud de las oscilaciones depende de las condiciones iniciales, de tal modo que, si la
velocidad inicial impuesta aumenta, tambin lo hace la amplitud.
Aunque la fase inicial depende de las condiciones iniciales, en este caso continuar siendo
nula, ya que tan solo cambia el mdulo de la velocidad inicial, pero no su direccin.

- 104 -

Fsica Universitaria: Problemas de Fsica

Movimiento armnico simple. M13.4

4. Disponemos de tres muelles idnticos. a) Los unimos en serie, uno a continuacin de otro, y fijamos uno de
los extremos libres al techo, en tanto que del otro
duplicamos la masa suspendida, el extremo inferior
Cunto vale la constante elstica de cada muelle?
paralelo (cada muelle tiene un extremo unido al
frecuencia de las oscilaciones de este sistema?

extremo suspendemos un bloque de masa m. Cuando


del conjunto serie desciende una distancia adicional h.
b) Con los tres muelles disponemos ahora un montaje
techo) y suspendemos una masa 3m. Cul ser la

Sea k la constante elstica de cada uno de los muelles.


1
1
a) Asociacin de muelles en serie:

kserie
ki
Los tres muelles idnticos equivalen a un muelle nico cuya
constante elstica sera
k
1
3

l kserie 
l k  3kserie
kserie k
3

La tensin F (carga, en este caso) que soporta un muelle es


directamente proporcional a la deformacin x mismo (ley de
Hooke):
F
F  kx l F  kx l k 
x
As, para el muelle equivalente serie tenemos:

2mg  mg

mg
3mg
l k  3kserie 
h
h
h
b) Asociacin de muelles en paralelo: kpar  ki
kserie 

Los tres muelles idnticos equivalen a un muelle nico de


constante elstica
9mg
kpar  3k 
h
La frecuencia de las oscilaciones de una masa sujeta a un muelle
viene dada por
ksist
m
que en nuestro caso nos conduce a
X

X

kpar
3m

9mg / h
3g

3m
h

l O

X
1 3g

2Q 2Q h

- 105 -

k
k
k
m

Fsica Universitaria: Problemas de Fsica

Movimiento armnico simple. M13.5

5. Un arquero tensa la cuerda de su arco desplazando su punto medio 60 cm hacia atrs; para ello, debe ejercer
una fuerza sobre la cuerda que se incrementa uniformemente desde cero hasta 120 N. a) Calcular la
constante elstica del arco. b) Qu trabajo se ha realizado para tensar el arco? En esas condiciones, se
dispara horizontalmente una flecha de 20 g hacia un blanco situado a 30 m. c) Determinar la velocidad con
la que sale impulsada la flecha. d) Cunto se desviar verticalmente del blanco?

a) En el supuesto de que la fuerza ejercida sea directamente proporcional a la elongacin que


experimenta el punto medio de la cuerda del arco, i.e., que se cumpla la ley de Hooke, ser
F 120
F  kx l k  
 200 N/m
x 0.60
b) El trabajo realizado ser igual a la energa elstica almacenada en el sistema
1
1
W  Eelast  kx 2 l W  q 200q 0.602  36 J
2
2
c) La energa elstica se convierte en energa cintica de la flecha, que saldr impulsada con
una velocidad v0:
2 Eelast
1
2q36
Eelast  Ek  mv02 l v0 

 60 m/s
2
0.020
m
d) Bajo la accin gravitatoria, la flecha describe una trayectoria parablica.
x  v0t
g

l y   2 x2

y   1 gt 2
2v0

2
de modo que, para x = 30 m, ser
9.8
h 
302  1.225 m
2q 602

y
P

v0

h
v

- 106 -

Fsica Universitaria: Problemas de Fsica

Movimiento armnico simple. M13.6

6. Un bloque de masa m se deja caer desde una altura h sobre un muelle de constante elstica k. Determnese el
acortamiento del muelle justamente antes de iniciarse la subida de la masa.

Dado que el sistema es conservativo, aplicamos el Principio de Conservacin de la Energa.


Tanto en el instante inicial como en el de mximo acortamiento del muelle son nulas la
energa cintica, ya que el bloque se encuentra instantneamente en reposo, de modo que tiene
lugar una conversin de energa potencial gravitatoria en elstica; esto es,
1
mgh  mgx kx 2 l kx 2  2mgx  2mgh  0
2
Resolvemos la ec. cuadrtica,
mg o m 2 g 2 2kmgh
k
de modo que tenemos dos soluciones

x

mg
x1 

k

mg
2h mg  0
k
k

mg

k

mg
2h mg  0
k
k

x2 

En reposo

Ep,grav=0

x2
m

x1
k
k

En reposo

una positiva y otra negativa, que corresponden a los instantes en los que el bloque se
encuentra instantneamente en reposo, tal como se indica en la figura.
Puesto que nos piden el mximo acortamiento del muelle, ste ser

x1 

mg

k

mg
2h mg  0
k
k
2

Obsrvese que el acortamiento del muelle correspondiente a la posicin de equilibrio, con el bloque encima de
l, vale x0 

mg
, y que la amplitud de las oscilaciones del sistema, en el caso de que el bloque quede unido al
k
mg
mg
2h
k
k

muelle, vale A 

Caso particular: Si fuese h = 0, sera: x1 

2mg
k

x2  0

- 107 -

x0 

mg
k

A

mg
k

Fsica Universitaria: Problemas de Fsica

Movimiento armnico simple. M13.7

7. En el interior de un tubo vertical, cerrado por su base inferior, hay un muelle de


100 espiras y constante elstica k. Sobre dicho muelle se apoya, (por simple contacto, sin
ningn tipo de unin), un bloque de masa m. Determinar: a) La amplitud mxima con
que puede oscilar el bloque sin despegarse del muelle y la frecuencia angular Z de las
oscilaciones. b) Si decidisemos recortar el muelle para que la misma masa oscilase con
frecuencia angular 2Z cuntas espiras deberamos dejar?

a) Puesto que el muelle y el bloque tan solo estn en contacto (sin unin fija), ambos
permanecern en contacto en tanto que el muelle est comprimido. Esto es, el muelle nunca
podr alargarse ms all de su longitud natural, ya que no puede ser traccionado por el
bloque.
Durante las oscilaciones del sistema, la energa permanece constante (sistema conservativo).
Tomamos como nivel de referencia para la energa gravitatoria la correspondiente a la
posicin ms baja del bloque y sabemos que la distancia entre las dos posiciones extremas es
el doble de la amplitud de las oscilaciones (i.e., 2A) y que en ellas el bloque se encuentra
instantneamente en reposo.
Conservacin de la energa
x
En la elongacin mxima positiva (longitud natural del muelle):
Ecin  0
Egrav  mgA
Eel  0 l E  mgA
+A

-A

En la elongacin mxima negativa (muelle comprimido):


1
2
Ecin  0
Egrav  mgA
Eel  k 2 A
l E  mgA 2kA2
2
De donde:
mg
E  mgA  mgA 2kA2 l 2mgA  2kA2 l A 
k
b) La frecuencia de las oscilaciones del sistema masa-muelle es
X  k / m , por lo que, para una masa dada, ser

Xa
k

 2 l k a  4k
X
k
Por otra parte, es bien sabido que dos muelles idnticos conectados uno a continuacin de otro
equivalen a un muelle de constante elstica dada por
1
1 1 2
k
 
l keq 
l k  2keq
keq k k k
2

A la inversa, si tenemos un muelle de constante k y lo cortamos en dos partes iguales,


dispondremos de dos muelles de constante k = 2k ; si volvemos a cortar en dos parte iguales
cada una de estas mitades, tendremos cuatro muelles de constante k = 2k = 4k, que es lo que
pretendemos. Por consiguiente, puesto que el muelle original tiene 100 espiras, deberemos
quedarnos con un muelle de tan solo 25 espiras.

- 108 -

Fsica Universitaria: Problemas de Fsica

Movimiento armnico simple. M13.8

8. El movimiento del pistn de un automvil de 500 g de masa podemos considerarlo vibratorio armnico
simple. Si la carrera del pistn (doble de la amplitud) es 10 cm y la velocidad angular del cigeal es de
3600 r.p.m., calcular: a) Aceleracin del pistn en el extremo de la carrera. b) Fuerza resultante que se
ejerce sobre l en el extremo de la carrera. c) Velocidad mxima del pistn.

Comenzamos escribiendo las ecuaciones del m.a.s.:


x  A sen(Xt G ) l v  X A cos(Xt G) l a  X 2 A sen(Xt G)  X 2 x
a) La aceleracin del pistn en el extremo de su recorrido es la que corresponde a la
elongacin mxima x=A (amplitud), de modo que
a  AX 2

con A = 0.05 m y X  3600q

2Q
rad
 376.99
, resulta
60
s

a  0.05q376.992  7.11q103

m
s2

b) Aplicando la segunda ley de Newton, tenemos

F  ma  0.5q 7.11q103  3.55q103 N


c) La velocidad mxima en el m.a.s. se alcanza en la posicin de elongacin nula y viene
dada por
m
vmax  A X  0.05q376.99  18.85
s

- 109 -

Fsica Universitaria: Problemas de Fsica

Movimiento armnico simple. M13.9

9. Un reloj de pndulo que ha sido cuidadosamente ajustado para marcar el tiempo correcto en un lugar donde
g = 9.823 m/s2 retrasa 40 s por da cuando se lleva a otro lugar geogrfico. Cunto vale g en ese lugar?

El periodo de las oscilaciones del pndulo (simple) viene dado por la expresin:

T  2Q

l
,
g

de modo que en un lugar donde el valor de la gravedad sea menor, el periodo ser
mayor (el reloj atrasa).
Para expresar cuantitativamente esta circunstancia, relacionaremos el cambio T
en el periodo con el g en la intensidad gravitatoria mediante el mtodo de la derivada
logartmica:
T
1 l 1 g
1 g



T
2 l
2 g
2 g

g
T
 2
g
T

lo que pone de manifiesto que un aumento del periodo (el reloj retrasa) est asociado con una
disminucin del valor de la intensidad de la gravedad.
Sustituyendo los valores dados en el enunciado:
T
40
g  2
g  2
9.823  0.009 m/s 2
T
86400
g  g a  g l g a  g g  9.823  0.009  9.814 m/s 2
Otro mtodo
Relacionamos los periodos y las intensidades gravitatorias en los dos lugares dividiendo
miembro a miembro las expresiones del periodo del pndulo:

T  2Q

2
0
g
T

T
g
0

l 0
l g  g 0 0

T
g0
l

2
T
Q


A partir del dato del retraso por da, podemos obtener la relacin (cociente) entre los periodos
en los dos lugares:
T T  T0 T
40

 1 
l
T0
T0
T0
86400

86440
T

T0 86400

Combinando estos dos resultados, calculamos la intensidad gravitatoria pedida:

T
86400
g  g 0 0  9.823
 9.814 m/s 2
86440
T
2

- 110 -

Fsica Universitaria: Problemas de Fsica

Movimiento armnico simple. M13.10

10. Introducimos agua en un tubo en U de seccin constante, colocado en posicin vertical, y desnivelamos
momentneamente las dos ramas de la U. a) Calcular la frecuencia y el periodo de las oscilaciones que se
producen; b) Variarn los resultados del apartado anterior si en lugar de agua se colocara mercurio?
Datos: densidad del mercurio, 13.6 g/cm3.

En la situacin de equilibrio, el lquido alcanza el mismo nivel en las dos ramas. Supongamos
que, por algn procedimiento, se desnivela el lquido en las dos ramas. Consideremos un
instante genrico, ilustrado en la figura, cuando el nivel del lquido en cada una de las ramas
presenta una elongacin x (desplazamiento respecto del nivel de equilibrio).
Mtodo de las fuerzas: Sobre la totalidad de lquido acta una fuerza F, no compensada,
igual el peso de la columna de lquido de seccin S y altura 2x; i.e,
F  mag  S S 2 x
g
de sentido contrario a la elongacin x. La masa de todo el lquido es SL.
La ecuacin del movimiento se escribe
2g
F  mx l  S S 2 x
g  S SLx l x
x0
L
que es la ecuacin diferencial correspondiente a un m.a.s. con

X

2g
l

l T

2x

2Q
l
 2Q
X
2g

Mtodo de la energa:. Expresamos la energa del sistema correspondiente a una elongacin


genrica, teniendo en cuenta que la porcin lquida contenida en el codo no cambia su energa
potencial. Los centros de masas de las ramas verticales de lquido se han marcado mediante
un punto, cuya altura es la mitad de la longitud de dicha columna. Tenemos,
h  x m
h x
m
1
E  Ek Ep  mx 2 h  x
g
h x
g


2 l
2
l
2

m
m
1
1
2
2
 mx 2 g h  x
h x
 mx 2 g h 2 x 2

2
2l
2
l
Puesto que el sistema es conservativo, su energa permanece constante,
de modo que

dE
2 g
m
 2 gxx  mx x
x  0
 mxx

dt
l
l

x
x
h

Ep=0

y como esta relacin debe satisfacerse en cualquier instante, cualquiera que sea la velocidad
del sistema, podemos escribir
2g
x
x0
L
que es la ecuacin diferencial correspondiente a un m.a.s. (la misma obtenida anteriormente).
b) La frecuencia tan solo depende de la longitud de la columna lquida. Si el tubo contiene un
mismo volumen de agua, de mercurio,... la frecuencia de las oscilaciones sern las mismas,
con independencia de la densidad del lquido.

- 111 -

Fsica Universitaria: Problemas de Fsica

Movimiento armnico simple. M13.11

11. Una balsa de madera de 3 m de largo, 2 m de ancho y 0.5 m de grosor tiene una densidad de 0.6 g/cm3. Esta

balsa est flotando en agua de densidad 1 g/cm3, en ella se sube una persona que pesa 75 kg; en el momento
de subirse se producen unas oscilaciones verticales. a) Calcular la frecuencia de dichas oscilaciones.
b) dem para cuando la persona se baja de la balsa.

Calculamos la superficie, el volumen y la masa de la balsa:


S  3q 2  6 m 2
V  3q 2q 0.5  3 m3
m  SmV  0.6q103 q3  1800 kg
Cuando se sube la persona, se produce un hundimiento adicional A tal que
m
75
Sm SAg  mp g l A  p 
 0.021 m  21 mm
Sm S 600q 6
hasta que se alcanza una nueva posicin de equilibrio.
Supongamos que la balsa se hunde una distancia
adicional x (elongacin) a partir de la situacin de
equilibrio de flotacin; aparecer una fuerza
restauradora igual al peso del volumen de agua
adicional desplazado
Frec  Sa Sxg

Frec
x

Entonces, aplicando la 2 ley de Newton, podemos escribir


Frec  Sa Sxg  msist x l msist x Sa Sg
x  0
que es la ecuacin diferencial de un movimiento armnico simple cuya frecuencia angular
viene dada por
X

Sa Sg
msist

l O

1 Sa Sg
2Q msist

a) Cuando la persona se encuentra sobre la balsa, ser

O

1 1000q 6q9.8
 0.89 Hz l T  1.12 s
2Q
1800 75

b) Cuando la persona se baja


O

1 1000q 6q9.8
 0.91 Hz l T  1.10 s
2Q
1800

- 112 -

Fsica Universitaria: Problemas de Fsica

Movimiento armnico simple. M13.12

12. Una moneda permanece en reposo sobre una plataforma horizontal que realiza un movimiento armnico
simple de frecuencia v. a) Si la plataforma oscila verticalmente, cul ser el valor mximo de la amplitud A
de las oscilaciones que permita a la moneda permanecer en contacto permanente con la plataforma?
b) Supongamos ahora que la plataforma oscila horizontalmente y que sea el coeficiente de rozamiento
esttico entre la moneda y la plataforma. Cul ser, entonces, el valor mximo de A que permita a la
moneda permanecer en reposo respecto a la plataforma, sin deslizar?

a) Consideramos una elongacin genrica x y escribimos la ec. del movimiento de la moneda


en el supuesto de que permanezca sobre la plataforma:
N  mg  mx l N  mg mx  mg  mX 2 x
N
x
donde hemos tenido en cuenta la relacin existente entre la
aceleracin y la elongacin en el m.a.s.; esto es, x  X 2 x ,
que presenta su valor mximo para x = A.
mg
La moneda permanecer sobre la plataforma en tanto que
N >0, de modo que
g
g
N  mg  mX 2 A  0 l A  2  2 2 .
X
4Q O
b) Ahora, interviene una fuerza de rozamiento esttico de valor
f b N N  Nmg

De nuevo consideramos una elongacin genrica x y


escribimos la ec. del movimiento de la moneda en el
supuesto de que permanezca en reposo respecto de la
plataforma,

N
f

mg

f  mx  mX x
2

de modo que la fuerza de rozamiento tiene en cada instante direccin opuesta a la elongacin
y presenta su valor mximo para x = A, tal que f mx  mX 2 A
Combinando estas dos ecuaciones tenemos:
mX 2 A  Nmg l

A

- 113 -

Ng
Ng
 2 2
2
4Q O
X

Fsica Universitaria: Problemas de Fsica

Movimiento armnico simple. M13.13

13. Un deportista que pesa 60 kg se lanza desde un puente sujeto a una cuerda elstica de 30 m de longitud
natural (practica puenting) llegando justamente a tocar la superficie del agua situada a 40 m por debajo en
la vertical de donde inici el salto. a) Calcular la constante elstica de la cuerda. b) Determinar la
aceleracin mxima a la que estar sometido el deportista y en qu punto la adquiere. c) Una vez que se
haya amortiguado la cada, de modo que la cuerda permanezca siempre tensa, determinar la frecuencia de
las oscilaciones verticales que experimentar el deportista.

de modo que
k

2q 60q9.8q 40
N
 470.4
102
m

l = 40 m

l0 = 30 m

a) Conservacin de la energa entre A y C:


1
2mgl
2
0  mgl k l  l0
l k 
2
2
l
 l0

b) En un instante genrico, la ecuacin del movimiento


del deportista se escribe en la forma
F  mg  ma

B
v0

a
C

siendo F la tensin de la cuerda.


mg
La aceleracin mxima ocurre en el instante en el que la
cuerda elstica presenta su mximo alargamiento (l = 40 m) y, por ende, su mxima tensin;
k
Fmx  mg  mamx l k l
mx  mg  mamx l amx  l
mx  g
m
o sea
470.4q10
amx 
 9.8  78.4  9.8  68.6 m/s 2  7 g
60
c) Una vez que se haya amortiguado la cada, de modo que la cuerda permanezca siempre
tensa, el deportista experimentar un m.a.s. vertical cuya frecuencia es
X

k
470.4
X

 2.8 rad/s l O 
 0.446 Hz
60
2Q
m

l T

1
 2.24 s
O

La posicin de equilibrio corresponde a un alargamiento x0 de la cuerda tal que su tensin


equilibre el peso del deportista; esto es
mg 60q9.8
F  mg  0 l kx0  mg l x0 

 1.25 m
470.4
k

- 114 -

Fsica Universitaria: Problemas de Fsica

Movimiento armnico simple. M13.14

14. Un muelle de 20 cm de longitud y 10 N/m de constante elstica est


unido por uno de sus extremos a un eje vertical y en el otro extremo a
una pesa de 100 g situada un plano horizontal. Cuando el eje rota a
razn de 1 r.p.s., la pesa describe una trayectoria circular sobre el plano
horizontal (sin friccin) al tiempo que oscila radialmente. Determinar
el alargamiento del muelle y la frecuencia de las oscilaciones.

Describimos la situacin en un sistema de referencia en rotacin en el que la pesa se


encuentra en reposo. En este sistema de referencia, la fuerza que proporciona el muelle
extendido ser igualada por la fuerza centrfuga; esto es, Fmuelle  Fcf . Siendo R la velocidad
angular de rotacin del eje y l el alargamiento que experimenta el muelle, ser:
mlR 2
lR 2
kl  mR 2 l l
l k  mR 2
l  mlR 2 l l 

2
k 2
k  mR
R
m
Y sustituyendo los valores dados en el enunciado
lR 2

0.20q 2Q

7.896
l 


 0.130 m  13 cm
10
k 2
2
60.52
R
 2Q

0.1
m
La frecuencia angular de las oscilaciones vendr dada por la bien
conocida expresin general:
X

k
10
X
10

 10 rad/s l O 

 1.59 Hz
m
0.1
2Q 2Q

- 115 -

Fsica Universitaria: Problemas de Fsica

Superposicin de m.a.s. M15. 1

1. Expresar la elongacin en funcin del tiempo para los m.a.s. siguientes: a) Movimiento con periodo de 1 s
tal que para t = 0 son v = 3 cm/s y a = 0. b) Movimiento con frecuencia 0.25 Hz tal que para t = 0 son v = 0
y a = 16 cm/s2. c) Es la superposicin de los m.a.s. cuyas ecuaciones, en el S.I. de unidades, son:

x1  0.06sen 2Qt

x2  0.08sen(2Qt )

Ecuaciones del m.a.s.:


x  A sen(Xt G )

v  X A cos(Xt G )
l t0

2
2

a  X A sen(Xt G )  X x

x0  A sen G

v0  X A cos G

a0  X 2 A sen G  X 2 x0

2Q
 2Q rad/s
T
Como a0 = 0, ser x0 = 0 y senI = 0, de modo que I = 0 o 180 y
cosI = 1.
Como v0 = +3 cm/s (positiva), ser cosI >0, de modo que I = 0, de
donde resulta
v
3
 0.48 cm
v0  X A l A  0 
X 2Q
= x  0.0048 sen(2Qt )
(S.I.)
a) T  1 s l X 

b) Q

0.25 Hz o Z

2SQ

2S u 0.25

x0
a0

rad/s
2
Como v0 = 0, ser cosI = 0, de modo que I = 90 y senI = 1.
Como a0 = 16 cm/s2 (negativa), deber ser senI >0, de modo que
I = +90, resultando
a
16
16 u 4
x0 A  02 
 2
6.48 cm
(S / 2) 2
Z
S
? x

v0

x0

I
v0

a0

0.0648sen( t  ) 0.0648 cos( t )


(S.I.)
2
2
2
c) Se trata de superponer (sumar) dos m.a.s. de la misma direccin y de la misma frecuencia.
Recurrimos a la representacin fasorial (vide figura):
A2  A12 A22 2 A1 A2 cos G1  G2  0.062 0.082 2q 0.06q 0.08q cos 45  0.01679

tg G 

A1

A1 sen G1 A2 sen G2
0.06qsen 45

 0.3465
A1 cos G1 A2 cos G2 0.06q cos 45 0.08

A
45

G  19.11  0.33 rad


A  0.13 m
= x  0.13sen(2Qt 0.33)
(S.I.)


A2

- 116 -

Fsica Universitaria: Problemas de Fsica

Superposicin de m.a.s. M15. 2

2. Calcular la ecuacin del movimiento armnico resultante de la composicin de:


x1

3sen Z t 
6

x2

4sen Zt 
4

Ambos m.a.s. tienen la misma frecuencia, lo que nos permite utilizar de modo inmediato la
representacin de Fresnel, compleja o fasorial de tales movimientos. Esto es,
x1  3sen Xt 30

%1  3 30  2.60 1.50 j

Im
l

x2  4sen Xt 45

% 2  4 45  2.83 2.83j

% %1 % 2  5.43 4.33j  6.94 38.6  6.94 0.67 rad


A2
y la ecuacin del movimiento resultante es
x

6.94sen Zt  0.67

I1

I2

A1

I
Re

Otro mtodo
A partir de la representacin geomtrica de Fresnel (fasorial) del m.a.s., resulta inmediato la
determinacin del mdulo de m.a.s. resultante:

A12  A22  2 A1 A2 cos I1  I2

32  42  2 u 3 u 4 u cos15

as como la fase inicial del mismo:


A1 sen I1  A2 sen I2 3 u sen 30  4 u sen 45
tg I
A1 cos I1  A2 cos I2 3 u cos 30  4 u cos 45

y la ecuacin del movimiento resultante es


x

6.94sen Zt  0.67

- 117 -

0.80 o I

6.94

38.6 0.67 rad

Fsica Universitaria: Problemas de Fsica

Superposicin de m.a.s. M15. 3

3. Una partcula se mueve en el plano xy de modo que las componentes cartesianas de su aceleracin vienen
dadas por ax = -9x y ay = -9y (S. I.) a) Qu tipo de movimiento se produce sobre cada eje? b) Determinar
los vectores de posicin y de velocidad de la partcula sabiendo que en el instante inicial el punto pasa por el
origen de coordenadas con una velocidad: v0 = i + 2j (S. I.) c) Determinar la trayectoria del punto y el
mximo alejamiento del origen de coordenadas.

a) Las ecuaciones que describen el movimiento de la partcula sobre


los ejes x e y son:

x  9 x l x 9 x  0 l x  A sen(3t B)

y  9 y l y 9 y  0 l y  B sen(3t C )

ya que, por ser de la forma, x X 2 x  0 , representan sendos m.a.s.


simples en direcciones perpendiculares, con la misma frecuencia
angular X  3 rad/s .

y
B
v0
A
x

b) Imponemos las condiciones iniciales a las soluciones de

x  A sen(B )  0 l B  0
x  A sen 3t
t  0

y  B sen(C )  0 l C  0
y  B sen 3t

Derivando con respecto al tiempo, obtenemos las componentes de la


velocidad, a las que imponemos las condiciones iniciales:

x  3 A cos 3t
x  3 A  1 l A  1/ 3 m

t0

y  3B cos 3t
y  3B  2 l B  2 / 3 m

de modo que podemos escribir

x  sen 3t

2
y  sen 3t

x  cos 3t

y  2 cos 3t

x  3sen 3t

y  6sen 3t

c) Se trata de dos m.a.s. de la misma frecuencia, en fase y en direcciones perpendiculares, de


modo que la trayectoria de la partcula es rectilnea, como resulta fcil comprobar eliminando
el tiempo entre la ecuaciones paramtricas de la misma. Esto es,

x  3 sen 3t
x 1/ 3 1

l


l y  2x

2
y 2/3 2

y  sen 3t
3

El mximo alejamiento del origen ser: D  A2 B 2 

- 118 -

1 4
5

9 9
3

Fsica Universitaria: Problemas de Fsica

Geometra de masas. M16.1


y

1. Una pieza de maquinaria, de masa m, est constituida por un disco de material


homogneo, de radio R, al que le falta una porcin circular de radio R/2, tal como se
indica en la figura. a) Determinar la posicin del centro de masa de la pieza.
b) Calcular los momentos de inercia de la pieza con respecto a los ejes coordenados
(x,y,z) indicados en la figura. Expresar los resultados en funcin de R y de la masa
m de la pieza.

R
x

Comenzamos determinando las masas del disco completo (sin merma alguna) y de la porcin
suprimida (negativa) en funcin de la masa m de la pieza:
m1 4

4
m1  TQ R 2


m  m

1 3
m  TQ ( R / 2) 2   1 TQ R 2 l m 3
l
2

m2

1
1

2
3
m2   m
m  m1 m2  4 TQ R
 

3
3

m
a) Centro de masa. Aplicamos el teorema correspondiente a la determinacin del centro de
mas de un cuerpo compuesto por otros cuerpos:
R
0  13 m
m1 x1 m2 x2
2  R
xcm 

m1 m2
m
6
b) Momentos de inercia. Con carcter general, determinamos los
y
momentos de inercia de un disco de masa m y radio R con respecto a
y
los ejes que se indican:
1
1
R
I zz  mR 2
I xx I yy  I zz l I yy  mR 2
2
4
x
y aplicando el Teorema de Steiner
1
3
1
5
I z ' z '  mR 2 mR 2  mR 2
I y ' y '  mR 2 mR 2  mR 2
2
2
4
4
Ahora, aplicamos estos resultados para el clculo de los momentos de inercia pedidos en el
enunciado del problema:

1
1
14
1 1 R 2 1 1 2 15
I xx  m1 R12 m2 R22 
mR 2 
m
  mR  mR 2
4
4
43
4 3 4 3 48
48
2

1
5
14
51 R
1 5
11
I yy  m1 R12 m2 R22 
mR 2 
m
  mR 2  mR 2
4
4
43
4 3 4 3 48
48
1
3
14
3 1 R 2 2 1 2 26
I zz  m1 R12 m2 R22 
mR 2 
m
  mR  mR 2
2
2
23
2 3 4 3 8
48
de modo que
I xx 

5
mR 2
16

I yy 

11
mR 2
48

- 119 -

I zz 

13
mR 2
24

Fsica Universitaria: Problemas de Fsica

Geometra de masas. M16.2

2. Una semiesfera y un cono, ambos macizos y homogneos, construidos con el mismo material y
del mismo radio, estn soldados por sus bases. Calcular el valor mximo de la altura del cono
que permita el conjunto comportarse como un tentetieso (i.e., que no vuelque) al apoyarlo
sobre una superficie horizontal.

Centro de masa del cono: z 

zdV
dV

Descomponemos el cono el rodajas (discos) perpendiculares al eje de revolucin, de modo


que
r
z
R

l r z
dV  Qr 2 dz
z
R H
H
R
R2 H
R2 H 3 1 2
V  dV  Q r 2 dz  Q 2 z 2 dz  Q 2
 QR H
H 0
H 3
3
r
R2 H 3
R2 H 4 1 2 2
zdV  Q H 2 0 z dz  Q H 2 4  4 Q R H
z
1 2 2

QR H
zdV
3

4
= z

 H (medido desde el vrtice)
dV 13 Q R 2 H 4
Centro de la hemisfera.
r  R sen R
z  R cos R l dz  R sen R dR

dV  Qr 2 dz  Q R sen 2 R (R sen R )dR  Q R 3 sen 3 RdR


0
2
V  Q R 3 sen 3 RdR  Q R 3 (como es sabido)
Q/2
3
0
4
0
QR4
4
3
4 sen R

zdV  Q R sen R cos R dR  Q R



4
Q/2
4

Q /2
4
QR
zdV
3

= z
 4  R
2
dV 3 Q R 3 8

Centro de masa del cuerpo compuesto


Deber estar situado por debajo del punto O para que el cuerpo se
comporte como un tentetieso (equilibrio estable). En las condiciones
crticas ser:
V z Vhemisf zhemisf
zcm  cono cono
 0 l Vcono zcono Vhemisf zhemisf  0
Vcono Vhemisf

1 2 3
2
3
Q R H H  Q R 3 R l H 2  3R 2 l H  R 3
3
4
3
8

- 120 -

r


H
O

Fsica Universitaria: Problemas de Fsica

Sistemas de partculas. M17.1

1. Una varilla ligera de longitud l puede girar sin rozamiento alrededor de su


centro. Se colocan en sus extremos sendas masas 2 m y m y se abandona el
sistema desde la posicin de la varilla horizontal. En el instante en que la
varilla alcanza la posicin vertical, determinar: a) la celeridad de las
masas; b) la vector cantidad de movimiento del sistema; c) la velocidad del
centro de masa del sistema.

2m

Posicin del c.m. del sistema constituido por las dos masas:
l
l
m 2m
  l
2
l
2
ycm 
 2 
3
6
m 2m
a) Puesto que el sistema es conservativo, aplicamos el Principio de Conservacin de la Energa, tomando como nivel de referencia la posicin inicial:
l
l 1
gl
0  2mg mg 3m
v 2 l v 
2
2 2
3
b) La cantidad de movimiento del sistema es la suma de
las cantidades de movimiento de las dos masas:
gl
i
3
c) La cantidad de movimiento del sistema es igual al
producto de su masa por la velocidad de su centro de
masa; esto es,

y
mv

l/2

p  p1 p 2  2mv i  mv i  mv i  m

p  4 mi
v cm

l v cm 

p
mv
1
1 gl

i  vi 
i
3m 3m
3
3 3

- 121 -

x
Ep=0

cm
l/2
v

2 mv

Fsica Universitaria: Problemas de Fsica

Sistemas de partculas. M17.2

2. Un muchacho est situado en el centro de una plataforma circular


de radio R = 3 m. La plataforma, inicialmente en reposo, puede
girar sin friccin alrededor de su eje. El muchacho hace girar
alrededor de su cabeza una masa de 2 kg sujeta con una cuerda de
longitud R/2, con una velocidad tal que la masa da una vuelta cada
3 segundos. a) Debido a la conservacin de qu magnitud fsica se
pone en movimiento la plataforma? Razonar la respuesta.
b) Determinar la velocidad y sentido del movimiento de la
plataforma y la energa cintica del sistema en movimiento,
sabiendo que el momento de inercia del conjunto plataforma y
muchacho, respecto al eje de la plataforma, es 600 kgm2.

Datos: Xm 

2Q
 2.094 rad/s
3

R
I m  m  2q1.52  4.5 kg.m 2
2
2

Por tratarse de un sistema aislado, el momento angular del sistema completo con respecto al
eje de rotacin permanecer constante, de modo que la plataforma adquiere una rotacin en
sentido opuesto a la de la masa de 2 kg.
Conservacin del momento angular del sistema aislado:
I
4.5
I m Xm  I p Xp l Xp  m Xm 
q 2.094  0.0157 rad/s
Ip
600
Energa cintica:
1
1
1
1
Ek  I m Xm2 I p Xp2  4.5q 2.0942 600q0.0157 2  9.870 0.074  9.994 J
2
2
2
2

- 122 -

Fsica Universitaria: Problemas de Fsica

Sistemas de partculas. M17.3

3. Dos prismas triangulares, de masas M y m, y anchuras a y b, estn en


reposo, tal como se indica en la figura adjunta, sobre un tablero horizontal liso. Las superficies de contacto entre los dos prismas son, tambin, perfectamente lisas. Determinar el retroceso del prisma inferior
hasta el instante en que la cara vertical del prisma superior alcanza el
tablero horizontal. Aplicacin numrica: M = 10 kg, m = 2 kg, a = 40 cm
y b = 10 cm.

b
m
M
a

Mtodo 1. Centro de masas.


Todas las fuerzas externas que actan sobre el
b
sistema constituido por los dos prismas (pesos y
m
reaccin normal en la base del prisma inferior)
x2
tienen direccin vertical. Puesto que no hay fuerza
M
externa alguna que de componente en la direccin
x1
a
horizontal, se conserva la cantidad de movimiento
del sistema en esa direccin (aunque no en la
a-b
X
direccin vertical). En consecuencia la
componente horizontal de la velocidad del centro
del sistema permanece constante en el transcurso
del movimiento. Puesto que partimos del reposo,
x2
la posicin horizontal del c.m. del sistema
x1
permanece
invariable.
Expresamos
esta
circunstancia en la forma siguiente.
En la figura adjunta hemos representado las
posiciones de los c.m. de cada uno de los prismas mediante las distancias x1 y x2 que se
indican. En cada una de las dos situaciones, inicial y final, se determina la posicin del c.m.
del sistema mediante el Teorema de Varignon:

xcm 

Mx1 mx2 M x1  X
m x2 a  b  X


M m
M m

de modo que ser


Mx1 mx2  M x1  X
m x2 a  b  X

De donde se sigue
Mx1 mx2  Mx1  MX mx2 m a  b
 mX

M m
X  m a  b

X

m
a  b

M m

Y sustituyendo los valores dados en el enunciado


2
X
40 10
 5 cm
10 2
(sigue)

- 123 -

[1]

Fsica Universitaria: Problemas de Fsica

Sistemas de partculas. M17.4

Mtodo 2. Conservacin de la cantidad de movimiento.


Todas las fuerzas externas que actan sobre el
sistema constituido por los dos prismas (pesos y
reaccin normal en la base del prisma inferior)
b
x
tienen direccin vertical. Puesto que no hay fuerza
m
externa alguna que de componente en la direccin
vx
M
horizontal, se conserva la cantidad de movimiento
v
V
del sistema en esa direccin (aunque no en la
a
direccin vertical).
Siendo
a-b
X
dX
la velocidad del prisma inferior,
dt
dx
la componente horizontal de la velocidad relativa del prisma superior respecto del
dt
inferior,
dx dX

la componente horizontal de la velocidad absoluta del prisma superior,
dt
dt

la conservacin de la componente horizontal de la cantidad de movimiento del sistema se


expresa en la forma:
dx dX
dX
MV m vx V
 0 l M
m
[2]
 0
dt
dt
dt
De modo que
MdX  m dx dX
l M m
dX  mdx l
dX  

m
dx l
M m

dX  

ab
m
dx l

0
M m

m
a  b

M m
Y sustituyendo los valores dados en el enunciado
2
X 
40 10
 5 cm
10 2
X 

- 124 -

Fsica Universitaria: Problemas de Fsica

Sistemas de partculas. M17.5

4. Una cua de masa M se encuentra en reposo sobre un tablero horizontal, como se muestra en la figura. En la
parte ms alta de la cua reposa un pequeo bloque de masa m, a una altura h sobre el tablero horizontal.
Todas las superficies son perfectamente lisas. Abandonamos el sistema, de modo que el bloque desciende y
la cua retrocede. Encontrar la velocidad de retroceso de la cua en el instante en que el bloque toca el
tablero horizontal.

Todas las fuerzas externas que actan sobre el sistema formado por la cua y el bloque
(pesos y reacciones normales en la base de la cua) tienen direccin vertical, por lo que se
conserva la componente de la cantidad de movimiento en la direccin horizontal. Esta
circunstancia se expresa mediante la ecuacin
mvh MV  0
[1]
Adems, al no existir rozamientos, el sistema es conservativo y podemos escribir la ecuacin
que expresa la conservacin de la energa,
1
1
mgh  MV 2 m vh2 vv2

[2]
2
2
donde V es la velocidad final de la
cua y vh y vv son las componentes
m
horizontal y vertical, respectivamente,
vh
de la velocidad del bloque en el
h
v
momento en que ste abandona a la
V
vv
cua.
M
Disponemos de dos ecuaciones y tres

incgnitas (V, vh y vv). Necesitamos
una tercera ecuacin que obtenemos de
imponer que, en el sistema de referencia de la cua, la velocidad del bloque vh V , vv
es

siempre tangente al plano inclinado, es decir


vv
 tg R l vv  vh V
tg R
vh V

[3]

Despejando vh en la ecuacin [1] y sustituyndola en la ecuacin [3] tenemos


M m
M
vh  
V
vv  
V tg R

m
m
Sustituyendo estas dos expresiones en la ecuacin [2] nos queda una ecuacin con V como
incgnita nica, de la que despejamos V para obtener:
V2 

2m 2 gh
2m 2 gh

2
M m
M M m
tg R M m
M 1 tg 2 R
m tg 2 R

Multiplicando numerador y denominador por cos2 y extrayendo la raz cuadrada de la


expresin anterior, obtenemos finalmente la velocidad pedida:
V

2m 2 gh cos 2 R
M m
M m sen 2 R

- 125 -

Fsica Universitaria: Problemas de Fsica

Sistemas de masa variable M18.1

1. Una balanza de resorte est ajustada para leer el cero. Desde una altura de 5 m sobre el platillo de la balanza,
dejamos caer un chorro de perdigones, a razn de 20 perdigones por segundo, que chocan contra el platillo,
rebotan hacia arriba con la misma velocidad y salen abandonando definitivamente el platillo. Si cada
perdign pesa 200 mg, )cul ser la lectura de la balanza?

La velocidad que tiene cada uno de los perdigones cuando


chocan contra el platillo se calcula a partir del principio de
conservacin de la energa:
1
mgh  mv 2 l v  2 gh  2q9.8q5  9.9 m/s
2
Cuando un perdign colisiona elsticamente con el platillo,
experimenta un cambio en su cantidad de movimiento expresado
por:
p  p final  pinicial  mv  (mv )  2mv

+
F
v
v

Este cambio est dirigido hacia arriba, de modo que el platillo


tendr que proporcionar impulso en esa direccin; i.e., ejercer
una fuerza hacia arriba sobre cada perdign.
Como al platillo llegan n = 20 perdigones por segundo, el cambio de la cantidad de
movimiento por unidad de tiempo, esto es, la fuerza, ser
dp
F   np  2nmv
dt
Y sustituyendo los valores tenemos

F  2nmv  2q 20q 200q106


q9.9  79.2q103 N  8.1 g
Esta ser la indicacin de la balanza, ya que los perdigones ejercen sobre el platillo una fuerza
igual y opuesta a la que el platillo ejerce sobre ellos (Tercera Ley de Newton).

- 126 -

Fsica Universitaria: Problemas de Fsica

M.19 Colisiones

1. Un automvil de 1200 kg, que inicialmente viaja con una velocidad de 27 m/s, choca contra la parte
posterior de un camin que pesa 9000 kg y se desplaza en la misma direccin y sentido a 22 m/s. La
velocidad del camin en el instante inmediato posterior al choque es de 23 m/s. a) Determinar el coeficiente
de restitucin. b) Calcular la energa mecnica que se ha perdido en el choque. Cmo se explica esta
prdida de energa?

a) Conservacin de la cantidad de movimiento durante el choque:


mA vA mBvB  mA vAa mBvBa l
vAa  vA

mB
9 000
(vB  vBa )  27
(22  23)  19.5 m/s
mA
1200

Aplicamos la definicin del coeficiente de


restitucin:
v a  vBa
19.5  23
e A

 0.70
vA  vB
27  22

vB
vA
B

de modo que se trata de una colisin parcialmente


elstica.
b) Durante el choque disminuye la energa
mecnica (cintica) del sistema.

vA
A

vB
B

Ek  mvA2 mvB2  1200q 27 2 9000q 222  2 615 400 J


1

Eka  mvAa 2 mvBa 2  1200q19.52 9000q 232  2 608 650 J


%Ek  Eka  Ek   6750 J
La energa mecnica (cintica) que ha desaparecido se ha convertido en otras formas de
energa, asociadas a la deformacin de los vehculos colisionantes.

- 127 -

Fsica Universitaria: Problemas de Fsica

M.19 Colisiones

2. Un coche de 1500 kg que viaja hacia el este con una velocidad de 25 m/s choca en un cruce con una
furgoneta de 2500 kg que viaja al norte con una velocidad de 20 m/s. Hallar la direccin y el mdulo de la
velocidad de los vehculos despus de la colisin, suponiendo que los vehculos sufren una colisin
perfectamente inelstica (es decir, quedan unidos).

Puesto que la colisin es perfectamente elstica, los dos cuerpos permanecen unidos despus
del choque y tan solo se conserva la cantidad de movimiento del sistema coche-furgoneta.
Cantidad de movimiento del coche:
p1  1500q 25  37 500 kg.m/s
Cantidad de movimiento de la furgoneta:
p2  2500q 20  50 000 kg.m/s

p2

Cantidad de movimiento de sistema coche-furgoneta:


p

p1 p2  37 5002 50 0002  62500 kg.m/s

Velocidad comn despus del choque es


p
62500
v

 15.625 m/s
m1 m2
4000
En una direccin que forma un ngulo , como se indica en la figura, tal que
p
50
R  arctg 2  arctg  53
p1
37

- 128 -


p1

Fsica Universitaria: Problemas de Fsica

M.19 Colisiones

3. Consideremos dos partculas, de masas respectivas m1 y m2, que efectan una colisin perfectamente elstica
frontal, de modo que sus velocidades antes de la colisin sean v1 y v2 =
v1, con
0. Supongamos que
fuesen iguales las energas cinticas iniciales de las partculas. Calcular el valor (o valores) que deber tener
el parmetro
para que la partcula "1" quede en reposo despus de la colisin y la relacin entre las masas
de ambas partculas para que sea posible esa situacin.

Ambas partculas tienen inicialmente la misma energa cintica, con v2 =


v1; por
consiguiente
1
1
m1v12  m2 v22
2
2

m1 v2
  H2
m2 v1
2

l mv m v
2
1 1

2
2 2

Conservacin de la cantidad de movimiento, con v2 =


v1:
m1
m1v1 m2v2  m1v1a m2 v2a l
v1 v2  v2a l H 2 v1 H v1  H 1 H
v1  v2a
m2
Regla de Huygens-Newton, con v2 =
v1:

v2a  v1a   v2  v1
l v2a   v2  v1
 1 H
v1
Igualando las expresiones de v2 en las dos ltimas ecuaciones:

H 2 H
v1  1 H
v1

l H 2 H  1  H l H 2 2 H 1  0

Resolvemos la ecuacin de segundo grado:

m1
1
0.4142 (alcance) l
 0.172 

m2
5.83
1 o 1 1
 1 o 2 
H

1
m1
 5.83
2.4142 (frontal) l
m2

v1

v1

v2

>0

v2

<0
v2

v2

reposo

reposo

- 129 -

Fsica Universitaria: Problemas de Fsica

M.19 Colisiones

4. Una pelota de ping-pong rebota escaleras abajo, escaln por escaln, de tal modo que todos los rebotes son
idnticos. El coeficiente de restitucin o percusin entre la pelota y las baldosas vale 0.9 y cada escaln
tiene una altura de 19 cm. Determinar la altura de rebote de la pelota sobre cada escaln.

Consideremos un rebote aislado; por definicin de coeficiente de


restitucin (e), ser

v12  2 gh1
v2 h
v2  ev1 l con
l 22  2  e 2
2

v1
h1

v2  2 gh2
La pelota de ping-pong realiza el bote desde una altura h1, respecto al
escaln; despus de botar alcanza una altura h2. Como todos los rebotes son
idnticos, despus del bote estar a la misma altura respecto al escaln
siguiente, o sea
h1  h2 a

h1

h2
v2

v1

de modo que

h1

h1
h2

h2
 e2
h2 a

l h2 

e2
a
1 e 2

de donde

0.92
0.81
q 0.19 
q 0.19  0.81 m = 81 cm
2
1 0.9
0.19
h1  81 19  100 cm
h2 

- 130 -

Fsica Universitaria: Problemas de Fsica

M.19 Colisiones

5. Dejamos caer una pelota de ping-pong desde una altura h0 sobre un suelo duro, liso y horizontal.
Observamos que despus del quinto rebote la pelota slo asciende hasta una altura h0/2. a) Determinar el
coeficiente de restitucin de los rebotes. Es el mismo en todos ellos? b) Calcular la fraccin de energa que
se disipa en los rebotes. Es la misma en todos ellos? Por qu? c) Cuntos rebotes debern transcurrir para
que la altura de rebote se reduzca a la centsima parte de h0?

a) En la figura representamos los rebotes sucesivos. Obviamente, la relacin existente entre


las velocidades indicadas y las respectivas alturas es:

vn  2 ghn

h0

h1

v0

donde el subndice n se refiere a la velocidad y a la altura


alcanzadas tras el n-simo rebote.
Designamos por e el coeficiente de restitucin y
aplicamos la regla de Huygens-Newton a cada uno de los
rebotes sucesivos:
v1  ev0

v  ev  e 2 v
1
0
2
l vn  evn1  e n v0
v3  ev2  e3v0

...

h2

v1

para n  0,1, 2,...

v2
v1

El valor del coeficiente de restitucin, que es el mismo en todos los rebotes, lo calculamos a
partir de los datos para el quinto rebote:
v5  e5v0

l e5 

v5
h
1
 5 
v0
h0
2

l e  (0.5)1/10  0.933

b) La prdida de energa en el n-simo rebote ser:

E  En 1  En  12 mvn2 1  12 mvn2  12 m(e 2 vn2  vn2 )  12 mvn2 (e2 1)


= E  (1 e 2 ) En

l 

E
 (1 e 2 )  cte
En

La fraccin de energa perdida en cada rebote es la misma en todos ellos y viene dada por
E

 (1 e2 )  0.13  13%
En
c) A partir de la expresin de la velocidad tras el n-simo rebote, vn  e n v0 , tenemos:

en 

vn
h
 n
v0
h0

l e2 n 
= n

hn
h0

l 2n log e  log

hn
h0

l n

log 0.01
 33.2 rebotes
2q log 0.933

- 131 -

log(hn / h0 )
2 log e

Fsica Universitaria: Problemas de Fsica

M.19 Colisiones

6. Sea un sistema formado por una masa de 300 g y un muelle


1
2
de constante elstica igual a 120 N/m, situados en reposo
k
v
sobre un plano horizontal liso, como se indica en la figura.
m
m
Sobre esta masa, inicialmente en reposo, choca
elsticamente otra masa idntica a la anterior con una
velocidad de 5 m/s. Determinar: a) La velocidad de cada una
de las masas inmediatamente despus del choque. b) La amplitud del movimiento armnico subsiguiente.
c) El tiempo que tardan en volver a chocar las dos masas. d) Variar el resultado del apartado anterior si la
velocidad con la que choca la masa incidente fuese el doble? Justifquese la respuesta.

a) Aplicamos el Principio de Conservacin de la Cantidad de Movimiento y la Regla de


Huygens-Newton (con e = 1) :
mv1a mv2a  mv
v1a v2a  v
2v1a  0
v1a  0

l
l
l

v1a  v2a  v
v1a  v2a  v
2v2a  2v

v2a  v  5 m/s
por lo que el primer bloque queda en reposo y el otro adquiere la
1
2
velocidad del primero.
v
b) Despus de la colisin, la energa cintica del bloque unido al (reposo)
muelle se convierte ntegramente en energa potencial elstica cuando este bloque alcanza su
elongacin mxima (amplitud, A); esto es,
1 2 1 2
m
0.3
mv  kA l A  v
 5q
 0.25 m  25 cm
2
2
k
120
c) Calculamos la frecuencia de las oscilaciones y, a partir de ella, el periodo:

k
2Q 2Q

 20 rad/s l T 

 0.31 s

X
m
20
Obviamente, el tiempo que tardan en reencontrarse los
A
dos bloques es el correspondiente a media oscilacin,
i.e., un semiperiodo, de modo que
1
2
T
%t   0.16 s
2
d) No variar el resultado, puesto que el tiempo que tardarn en reencontrase los dos bloques
ser un semiperiodo, en cualquier caso (en tanto que sea m1=m2), siendo T independiente de la
amplitud de las oscilaciones.
X

- 132 -

Fsica Universitaria: Problemas de Fsica

M.19 Colisiones

7. Un bloque de masa m1 = 1 kg desliza sobre una superficie horizontal lisa con una velocidad de 6 m/s. El
bloque choca con otro de masa m2 = 2 kg que est en reposo unido a un resorte horizontal de constante
k = 1200 N/m. Determinar la frecuencia con la que oscilar el sistema tras el choque y la amplitud del
movimiento en los siguientes casos: a) El choque es totalmente inelstico y las dos masas quedan adheridas
despus del choque. b) El choque es perfectamente elstico.

a) Choque totalmente inelstico:


La cantidad de movimiento se conserva:
m1
1
m1v1  m1 m2
v v ' 
v1  6  2 m/s
3
m1 m2
X

v1

m2

m1

k
1200

 20 rad/s
m1 m2
3

m1+ m2

X 10
O
 Hz O  3.18 Hz
2Q Q
Conservacin de la energa despus del choque:
1
1
m1 m2
v '2  kA2
2
2

A  va

m1 m2 v a
2
 

k
X 20

b) Choque perfectamente elstico:


El coeficiente de restitucin es e=1.

A  0.1 m
v2

v1

v '1 v '2
  v1 v '2  v1 v '1

m1

m2

La cantidad de movimiento se conserva:


m1v1  m1v1a m2 v2a  m1v1a m2 v1 v1a
m1  m2
v1  m1 m2
v1a

v1a 

m1  m2
1
v1 
6  2 m/s v '2  6  2  4 m/s
3
m1 m2

por lo que la masa 1 vuelve hacia la izquierda.


X

k
1200
X

 24.5 rad/s O 
2
2Q
m2

O  3 .9 Hz

A  0.16 m

Conservacin de la energa despus del choque:


1
1
m2 v2a 2  kA2
2
2

A  v2a

m2
2
4
1200
k

Nota:
Las masas no vuelven a chocar ya que T = 0.256 s y el tiempo que tarda m1 en
recorrer la amplitud es:
A 0.16
t 
 0.08 s
2
v1a

- 133 -

Fsica Universitaria: Problemas de Fsica

M.19 Colisiones

8. Una partcula de masa m se desplaza a velocidad v cuando choca


elsticamente con otra de igual masa que se encontraba en reposo. Despus
de la colisin, la trayectoria de la primera partcula forma un ngulo de 30
con respecto a su direccin original. a) Determinar la velocidad de cada una
de las partculas tras el impacto. b) Calcular la percusin que recibe cada
una de las partculas.

a) Conservacin de la cantidad de movimiento:


mv  mv1 mv 2 l v  v1 v 2
[1]

Conservacin de la energa cintica:


1
2

mv 2  12 mv12 12 mv12

l v 2  v12 v22

[2]

v2
Las ecuaciones [1] y [2] corresponden a una suma vectorial
en cuadratura, por lo que las direcciones de las partculas
despus del choque determinan un ngulo recto, de modo que
R1 R2  90 l R2  90 30  60
1

2

v1
30
60

30
v

v1
30
60

v1

y las velocidades de las partculas despus del choque son


v1  v cos 30  0.87v
v2  v cos 60  0.50v

b) La percusin que experimenta cada una de las partculas es


igual a la variacin de su cantidad de movimiento durante el
choque; i.e.,  p  m v , de modo que

2   mv
 mv2  mv2  mv 2 1  2  mv 2 1  2  12 mv

en las direcciones en que cambia la velocidad de cada una de las partculas.


Otro mtodo
Escribimos de nuevo las ecuaciones [1] y [2] que expresan, respectivamente, la conservacin
de la cantidad de movimiento y de la energa cintica durante el choque, descomponiendo la
primera de ellas en sus componentes longitudinal y transversal:
v  3 v v cos R l v cos R  v  3 v
2
2
2
2
2 1
2 1

v  v1 cos 30 v2 cos R2

v
v2
2
0  v1 sen 30 v2 sen R2 l
12 v1  v2 sen R2 l sen R2  1 l sen R2  1 2

2v2
4v2

2
2
2

v 2  v 2 v 2 l v 2  v 2  v 2
v  v1 v2
1
2
2
1

que constituyen un sistema de tres ecuaciones con tres incgnitas (v1a, v2a , R2 ) . Resolviendo
dicho sistema de ecuaciones se obtienen de nuevo los resultados que ya conocemos.
v 2 cos 2 R  v 2 3 v 2  3vv

2
2
1
4 1

v
4v 2  v 2
2
2
2
2
2
2

cos R2  1 sen R2  1 1 2  2 2 1 l v2 cos R2  v2  14 v1 l

v
v
4
4
2
2

2
2
2

v2  v  v1
De modo que v 2 34 v12  3vv1  v 2  v12  14 v12

- 134 -

l 2v12  3vv1 l v1 

3
2

Fsica Universitaria: Problemas de Fsica

M.19 Colisiones

9. Una partcula de masa m se desplaza con una velocidad v cuando choca


elsticamente con otra de igual masa que se encontraba en reposo.
Despus de la colisin, la trayectoria de la primera partcula forma un
ngulo de 60 con respecto a su direccin original. a) Determinar la
velocidad de cada una de las partculas tras el impacto. b) Calcular la
percusin que recibe cada una de las partculas.

a) Conservacin de la cantidad de movimiento:


mv  mv1 mv 2 l v  v1 v 2
[1]

60
v

v1

60

Conservacin de la energa cintica:


1
2

mv 2  12 mv12 12 mv12

l v 2  v12 v22

[2]

30

v2
Las ecuaciones [1] y [2] corresponden a una suma vectorial
en cuadratura, por lo que las direcciones de las partculas
despus del choque determinan un ngulo recto, de
modo que
1 v1
v1
60
R1 R2  90 l R2  90 60  30
30

2

y las velocidades de las partculas despus del choque


son
v1  v cos 60  0.50v
v2  v cos 30  0.87v

b) La percusin que experimenta cada una de las partculas es igual a la variacin de su


cantidad de movimiento durante el choque; i.e.,   p  m v , de modo que
2   mv
 mv2  mv2  mv 2 1  2  mv 2 1  2  0.87mv

en las direcciones en que las que cambia la velocidad de cada una de las partculas.
Otro mtodo (nada recomendable)
Escribimos de nuevo las ecuaciones [1] y [2] que expresan, respectivamente, la conservacin
de la cantidad de movimiento y de la energa cintica durante el choque, descomponiendo la
primera de ellas en sus componentes longitudinal y transversal:

v  12 v1 v2 cos R2 l v2 cos R2  v  12 v1

v  v1 cos 60 v2 cos R2

3v1
3v 2
3

l sen 2 R2  12
0  v1 sen 60 v2 sen R2 l 2 v1  v2 sen R2 l sen R2 

2v2
4v2

2
2
2

v  v1 v2

2
2
2
2
2
2

v  v1 v2 l v2  v  v1

que constituyen un sistema de tres ecuaciones con tres incgnitas (v1a, v2a , R2 ) . Resolviendo
dicho sistema de ecuaciones se obtienen de nuevo los resultados que ya conocemos.
v22 cos 2 R2  v 2 14 v12  vv1

3v 2 4v 2  3v 2

3 2
2
2
2
2
2

cos R2  1 sen R2  1 12  2 2 1 l v2 cos R2  v2  v1

4v2
4v2
4

2
2
2

v2  v  v1
De modo que v 2 14 v12  vv1  v 2  v12  34 v12

l 2v12  vv1 l v1  0.5v

- 135 -

Fsica Universitaria: Problemas de Fsica

M.19 Colisiones

10. En una mesa de billar, la bola 1 se mueve con una velocidad de 5 m/s y choca
con la bola 2 de modo que sta se introduce en la tronera de la esquina, como
se indica en la figura. Determinar la velocidad y la direccin de cada bola
despus del choque, considerndolo: a) completamente elstico; b) con un
coeficiente de restitucin de 0.95.

30
2

a) Es bien sabido que cuando una partcula incidente colisiona


elsticamente contra otra partcula de la misma masa que se encuentra en
reposo, las partculas se mueven despus de la colisin en direcciones que
son perpendiculares entre s. As, el esquema de colisin es el que se ilustra
v1 1 2
en la figura, con 1 = 30 y 2 = 60. Puesto que en esta colisin se
v2 conservan tanto la cantidad de movimiento como la energa cintica,
podemos escribir:
mv1a cos 30 mv2a cos 60  mv1
v1a sen 60 v2a cos 60  v1

v1
l v1a cos 60 v2a sen 60  0
mv1a sen 30 mv2a sen 60  0

a2
2
2
2
2
2
1
1
1

2 mv1a 2 mv2a  2 mv1


v1 v2a  v1

de modo que disponemos de un sistema de tres ecuaciones con dos


incgnitas (v1 y v2). A partir de las dos primeras se obtiene fcilmente
2
v1a sen 60 v2a sen 60 cos 60  v1 sen 60
v1a  v1 sen 60
v1a  4.33m/s
l
l

v2a  2.50 m/s


v2a  v1 cos 60

v1a cos 60  v2a sen 60 cos 60  0


b) Para resolver este problema de choque oblicuo analizamos por separado sus componentes
frontal y transversal (tangencial). Para ello, tomamos una base vectorial apropiada definida
por los ejes x e y: el eje x sobre la recta que une los centros de las esferas en el instante de la
colisin (componente frontal) ; el eje y es normal al anterior (componente transversal), como
se muestra en la figura.
Componente frontal: Aplicamos el Principio de Conservacin de la Cantidad de
Movimiento y la Regla de Huygens-Newton,...

1 e

v1ax  2 v1 cos R2
mv1ax mv2a x  mv1 cos R2
v1ax v2a x  v1 cos R2

l
l

v1ax  v2a x  e v1x  v2 x

v1ax  v2a x  e v1 cos R2

1 e

v2a x 
v1 cos R2

Componente transversal: Suponiendo que las bolas


sean lisas, las fuerzas impulsivas que actan durante la
v1
colisin son normales a las superficies en el punto de
v1
contacto de modo que no tiene componentes sobre el
x (frontal)
y
eje y. En consecuencia, las componentes de las veloci
1
(transversal)
2
v2
dades normales a la lnea de los centros no se alteran
durante la colisin; i.e.,
bola 1
v1ay  v1 sen R2
v2a y  0
v1

(incidente

bola 2
(blanco)

Para calcular el ngulo 1 bajo el que sale dispersada


la esfera incidente, calcularemos primero 1+2; esto
es,

- 136 -

Fsica Universitaria: Problemas de Fsica

M.19 Colisiones

v1ay
v1 sen R2
2
tg R2


1 e
v1ax 1 e v cos R
1
2
2
Sustituimos los valores dados en el enunciado del problema para obtener los resultados
numricos correspondientes a cada caso:
e = 1 (colisin elstica)

1 1
v2a x 
5cos 60  2.5 m/s

v

0

1
x
2

v1ay  5sen 60  4.33 m/s


v2a y  0

v a  2.5 m/s
a

v
4.33
m/s

2
tg(R1 R2 ) 
tg R2  d l R1 R2  90 l R1  30
1 1
e = 0.95 (colisin parcialmente elstica)

1  0.95
1 0.95
v1ax 
v2a x 
5cos 60  0.0625 m/s
5cos 60  2.44 m/s

2
2
v a  5sen 60  4.33 m/s
v a  0
1 y
2 y
v a  0.06252 4.332  4.33 m/s
v2a  2.44 m/s
1

2
tg(R1 R2 ) 
tg 60  69.28 l R1 R2  89.2 l R1  29.2
1 0.95
tg(R1 R2 ) 

- 137 -

Fsica Universitaria: Problemas de Fsica

Esttica del slido rgido. M20.1

1. Una escalera AB, de 10 kg de masa, 2.5 m de longitud y centro de masa situado a mitad de
la misma, se encuentra apoyada sobre una pared lisa y un suelo rugoso, cuando su extremo
inferior se encuentra a una distancia de 1.5 m de la pared. a) Calcular las reacciones en los
apoyos A y B. b) Determinar el valor mnimo del coeficiente de rozamiento entre el suelo y
la escalera para que sta pueda permanecer en equilibrio en la posicin indicada en la
figura.

A
1.5m

NB

Determinamos el lado vertical del tringulo:

b  l 2  a 2  2.52 1.52  2 m

a) Aplicamos las Ecuaciones Cardinales de la Esttica, descomponiendo las fuerzas en las direcciones horizontal y vertical y tomando
momentos en el punto A.
G
b
f  N B  0
l f  N B  3.75 kg = 36.8 N

NA
mg
N A  mg  0 l N A  mg  10 kg = 98.0 N

1.5
a
a
mg  N Bb l N B  mg 
q10  3.75 kg = 36.8 N
A
a
2
2
2
b
q2

b) El valor mnimo del coeficiente de rozamiento es

f b NNA

mg
f
a 1.5
l Np
 2b


 0.375
2b
4
NA
mg

- 138 -

Fsica Universitaria: Problemas de Fsica

Esttica del slido rgido. M20.2


A

2. Una barra uniforme pesa P y se mantiene en equilibrio, apoyada en una pared vertical y

2l/3

en un suelo horizontal, ambos lisos, en la posicin indicada en la figura, gracias a la


cuerda horizontal CD. Determinar las reacciones en los apoyos de la barra y la tensin en
la cuerda en funcin del ngulo .


G
C

En la figura mostramos el diagrama de fuerzas que actan sobre la varilla.


Aplicamos las ecuaciones cardinales de la esttica tomando momentos en el punto C:

(1) R  T
l


(2) N  P  100 kg
A

A z

l
2

(3) R cos R  PCG sen R N CBsen R

2l/3
Con

2 1
l
CG   l ) 
3 2
6

de modo que la tercera ecuacin se escribe en la forma


2l
l
l
(3) R cos R  P sen R N sen R
3
6
3
Y, teniendo en cuenta las dos primeras ecuaciones, resulta:
4 R  P tg R 2 N tg R l
l 4 R  3P tg R l

R T 

3
P tg R
4

- 139 -

l/3

N
B

l/3

Fsica Universitaria: Problemas de Fsica

Esttica del slido rgido. M20.3


B

3. La varilla homognea AB, que tiene una longitud de 1.6 m y pesa 2 kg, se apoya por su
extremo inferior en un muro vertical y, por un punto intermedio C, en otra varilla fija,
horizontal y paralela al muro, a una distancia de 10 cm de ste. Suponiendo que no existan
rozamientos entre la varilla AB y los dems elementos en contacto con ella, determnense en
la posicin de equilibrio el ngulo formado por la varilla y el muro y las reacciones en los
apoyos de la varilla.

C
A

a) En la figura mostramos el diagrama de fuerzas que actan sobre la varilla. Aplicamos las
ecuaciones cardinales de la esttica tomando momentos en el punto A:

(1) N  R cos R
P
l
tg R 

(2)
P
R
sen
R

N

l
a
B

(3) P sen R  R
z
A

2
sen R

Que constituye un sistema de tres ecuaciones con tres


G
incgnita (N, R y ).
Sustituyendo la (2) en la (3):


l
a
l
a
2
P
R sen R sen R  R
l
sen R 
2
sen R
2
sen R
R
l/2
2a 2q 0.1 1
l sen 3 R 


E a
C
l
1.6
8
1
l sen R 
l R  30
A N 
2
D
De la ecuacin (2):
P
P  R sen R l R 
 2 P  4 kg
sen R
Y de la (1), se sigue:

3
 2 3  3.46 kg
2
Las direcciones y sentidos de estas reacciones son las indicadas en la figura.
N  R cos R  4

Otro mtodo
Puesto que sobre la varilla solo actan tres fuerzas (P, N y R), stas deben ser concurrentes en
un punto tal como el D. En consecuencia, el problema se reduce a una simple condicin
geomtrica de que el centro de gravedad (G) de la varilla se encuentre en la vertical del punto
D. Resolvemos considerando los tringulos ACE, ACD y ADG:
a
l
2a
AC 
 AD senR  AGsenR
sen R  sen 2 R l sen 3 R 
sen R
2
l
Que es el mismo resultado obtenido anteriormente.

- 140 -

Fsica Universitaria: Problemas de Fsica

Esttica del slido rgido. M20.4


B

4. La barra AB de la figura tiene seccin recta uniforme, de masa m y longitud 1.


Determinar el ngulo T correspondiente al equilibrio.

T
30

45

De la figura, se siguen fcilmente las siguientes relaciones entre ngulos:


B B a R  90 l B a  90 (B R )
C C a  R  90 l C a  90 (C  R )
Aplicando las ecuaciones cardinales de la esttica, tomando
momentos en G :

NB E

x NA

Fx  0 l [1] N A sen B  N B sen C

G
T

B
D D
Fy  0 l [2] N A cos B N B cos C  P

M  0 l [3] N l sen B a  N l sen C a


A

P
G
A
B

2
2

D
E
Puesto que tan slo estamos interesados en determinar el
valor del ngulo T correspondiente a la posicin de equilibrio, podemos eliminar fcilmente
las reacciones en los apoyos de entre las ecuaciones [1] y [3]:
N A sen B  N B sen C

sen B
sen C

u l

N A cos(B R )  N B cos(C  R )
cos(B R ) cos(C  R )
y

de modo que, desarrollando las expresiones trigonomtricas, tenemos


sen B
sen C

l
cos B cos R  sen B sen R cos C cos R sen C sen R
sen B cos C cos R sen B sen C sen R  cos B sen C cos R  sen B sen C sen R l
2sen B sen C sen R  (cos B sen C  sen B cos C ) cos R l
sen(C  B )
sen15

 1.3660 l R  20.1
tg R 
2sen B sen C 2sen 30 sen 45

- 141 -

Fsica Universitaria: Problemas de Fsica

Esttica del slido rgido. M20.5

5. Una barra de longitud l se apoya sin rozamiento sobre una pared de perfil
circular tal como se indica en la figura. a) Determinar la fuerza horizontal
que se debe aplicar en B para mantener la barra en equilibrio. b) Si en lugar
de aplicar una fuerza en B se aplicase un par en B, qu valor debera tomar
dicho par?

l
B

a) Las fuerzas que actan sobre la barra son las que se representan en la
figura. Tomamos momentos en O:

l
3
3
Fl  P sen 60 
Pl l F 
P
2
2
4
b) En este caso las fuerzas que actan sobre la barra sern las que se
representan en la figura. De nuevo tomamos momentos en O, para
obtener:
l
3
3
M  P sen60 
P l M
Pl
2
2
4
Lo que resulta obvio, ya que el momento del par (invariante) debe ser igual
al de la fuerza F inicialmente aplicada con respecto al punto O.

O
NA
60

NB
F

O
NA
NB 60
M
B

Clculo de reacciones
Ecuaciones cardinales de la esttica:

F
1



 P
N

A
j F  N A cos 30

cos
30
2
3
l

7 N B N A sen 30  P

1 1 3

N B  P  N A sen 30  P  P  P

2 2 4

- 142 -

Fsica Universitaria: Problemas de Fsica

Esttica del slido rgido. M20.6

6. Un bloque prismtico de seccin cuadrada y masa m se encuentra apoyado sobre


un plano liso horizontal y otro inclinado rugoso. Sabiendo que la situacin
representada corresponde a la de lmite de equilibrio: a) Dibujar el diagrama de
fuerzas. b) Determinar el coeficiente de rozamiento.

60

30

a) En la figura adjunta hemos representado el diagrama de fuerzas.


b) Escribimos las ecuaciones cardinales de la esttica, descomponiendo en las direcciones
horizontal y vertical y tomando momentos en B:

j [1] N 2 cos 30  f sen 30 l N 2  f tg 30

7 [2] N1 N 2 sen 30 f cos 30  P

B { [3] N1l sen 30  Pl 2 cos15


l
con
[4] f  N N 2

Puesto que tan solo estamos interesados en el valor


del coeficiente de rozamiento, es suficiente
combinar las ecuaciones [1] y [4] para
determinarlo:
1
1

 1.73
N 2  N N 2 tg 30 l N 
tg 30 0.5773

- 143 -

N2

60
B
60

15
P
30

N1
l

Fsica Universitaria: Problemas de Fsica

Esttica del slido rgido. M20.7

7. Un bloque rectangular de dimensiones a = 1 m y b = 3 m se sita sobre un plano

inclinado tal como se indica la figura. Una cuerda sujeta la parte superior del bloque
para evitar que caiga por el plano, cul ser el ngulo  mximo para que el bloque
no deslice por el plano?
b

P = 0.1
T

Cuando el ngulo tiene el valor crtico, el bloque est a punto para volcar
deslizando sobre la arista A de su base y las fuerzas que actan sobre el mismo son las
indicadas en la figura.
Aplicando las ecuaciones cardinales de la esttica en las condiciones crticas, tomando
momentos en A, tenemos:

1 T f  P sen R

4 N  P cos R
T N P cos R  P sen R

z
1
a
b
a
A Tb  P sen R P cos R
T  2 P sen R 2b P cos R

2
2
T

N
con
f
N

a

1
a
l
P sen R P cos R N P cos R  P sen R
2
2b
a
b G
l sen R cos R 2N cos R  2sen R
N
b
Psen T
f
a

a
l sen R  2N cos R l tg R  2N
P
b
Pcos
T

b
A
T
Sustituyendo los valores dados en el enunciado, resulta

1
tg R  2q 0.1  0.53 l R  28.1
3

f
P

Otro mtodo.- Tengamos en cuenta que:


1) La reaccin resultante R (suma de la reaccin normal N y de
la fuerza de rozamiento f), en las condiciones de movimiento
inminente, forma un ngulo  con la normal al plano inclinado,
tal que
f
f  N N l N   tg K
N
2) Puesto que sobre el bloque solo actan tres fuerzas (P, T y R),
stas deben ser concurrentes en un punto tal como el D.
De la figura, correspondiente a la posicin crtica, se sigue

tg R 

BC+CD a / 2 b tg K a

 2N
b/2
b
BG

- 144 -

Fsica Universitaria: Problemas de Fsica

Esttica del slido rgido. M20.8

8. Una caja de embalaje contiene un frigorfico, pesa 300 kg y tiene forma de paraleleppedo rectangular de

2 m de alto por 80 cm + 80 cm de base. El coeficiente de rozamiento entre la caja y el suelo vale 0.30. Si
deseamos arrastrarla sobre el suelo mediante la aplicacin de una fuerza horizontal: a) Cul debe ser la
magnitud de esa fuerza? b) A qu altura sobre el suelo podemos aplicar esa fuerza sin riesgo de vuelco?

En la figura hemos representado el diagrama de fuerzas que actan sobre la caja. Obsrvese
que actan dos pares de fuerzas: el par [F,f] provoca el vuelco sobre E y el par [P,N] se
opone a dicho vuelco.
Escribimos las Ecuaciones Cardinales de la Esttica, tomando momentos en B (punto de
aplicacin de la reaccin normal N):
NP
l
l F f
B

De las dos primeras ecuaciones se sigue:

O
F
h

z Fh  Px

H
N

F  f p NN  NP l

F p NP

Esto es

F p 0.3q300  90 kg

La tercera ecuacin nos permite relacionar la distancia h


B E
(punto de aplicacin de la fuerza externa) con la distancia x
(punto de aplicacin de la reaccin normal N):
P
P
x
h xb
x
F
NP
N
de modo que el valor mximo de h que no produzca el vuelco ser el que corresponda al valor
mximo posible de la distancia x: esto es,
x
l / 2 40
hmx b mx 

 133 cm  1.33 m
0.3
N
N

- 145 -

Fsica Universitaria: Problemas de Fsica

Esttica del slido rgido. M20.9

9. Deseamos transportar en una carretilla un bloque homogneo, de masa m, cuyas dimensiones se especifican en la figura. Sea  el coeficiente de rozamiento entre la base del
bloque y la plataforma de la carretilla. Determinar los valores mximos de la aceleracin
de la carretilla (acelerando y frenando) para que no haya movimiento relativo entre el
bloque y la carretilla.

4l
l

a) Cuando la carretilla est acelerando, si la aceleracin es excesiva,


el bloque puede volcar rotando alrededor del eje A. En esas
circunstancias, el diagrama de fuerzas en el referencial de la
carretilla, que es no inercial, es el que se india en la figura.
Tomando momentos en A, tenemos

ma0

z mg
2  ma
l

4l

a0 

A
l

mg

B a0

b) Cuando la carretilla est frenando, si la aceleracin es excesiva,


el bloque puede volcar rotando alrededor del eje B o resbalar hacia
delante (respecto de la carretilla). En esas circunstancias, el
diagrama de fuerzas en el referencial de la carretilla, que es no
inercial, es el que se indica en la figura.
b.1) Vuelco. Tomando momentos en B, tenemos

z
B

4l

ma0
A

mg
f N B
a0

1
g
2

l
2

1
4

mg
 ma0a
2l l a0a  g

b.2) Resbalamiento.
ma0aa  f
f  Nmg

l
N  mg l
f

a0aa  m  N g
f
N
N


Por consiguiente,

a0aa  N g

Si  < 0.25, tiene preferencia el resbalamiento.

Si  > 0.25, tiene preferencia el vuelco

- 146 -

Fsica Universitaria: Problemas de Fsica

Esttica del slido rgido. M20.10

10. Una placa rectangular y homognea, de dimensiones 30 cm x 20 cm, pesa 2 kg y est unida a un eje vertical
de modo que en A est articulada con el eje y en B tan solo se apoya en l, como se indica en la figura.
a) Determinar las reacciones en A y en B cuando el sistema est en rotacin con una velocidad angular de
30 r.p.m. b) A partir de que valor de la velocidad angular no se apoyar en B?

Reconducimos el problema a un problema de esttica


(equilibrio esttico) analizndolo en un referencial en
rotacin en el que la placa se encuentra en reposo. En
estas condiciones, el diagrama de fuerzas es el que se
muestra en la figura, incluida la fuerza centrfuga.
Clculo de la fuerza centrfuga: Sea un elemento de
masa dm  Tbdx , de modo que

RA
NA

dx

a=30 cm

b=20 cm

Fcf

G
NB
B

dFcf  X 2 x
dm  X 2Tbx dx

a
1
1
1
Fcf  X 2Tb x dx  X 2Tba 2  X 2 Tba
a  maX 2
0
2
2
2
a) Escribimos las ecuaciones cardinales de la esttica, tomando momentos en A:

l [1] RA  P

b
a

} [3]
Fcf bN B  P

2
2

[2] N B Fcf  N A

de modo que disponemos de tres ecuaciones con tres incgnitas (NA, NB y RA)
De la primera, se sigue que RA  P
De la tercera, obtenemos
NB 

ma
1
1
1
bX 2

aP  bFcf
 mga  mX 2 ab  g 
2b
2b
2b
2
2

y sustituyendo en la segunda ecuacin


N A  N B Fcf 

mga 1
1
mga 1
ma
bX 2

 maX 2 maX 2 
maX 2 
g
2b
4
2
2b
4
2b
2

b) Para una cierta velocidad angular crtica, crt, desaparece la ligadura en B (i.e., NB=0):
bX 2
bX 2
ma
2g
NB 
g  crt 0 l g  crt  0 l Xcrt 
b
2b
2
2

Sustituyendo los valores dados,


RA  2 kg  19.6 N

2 0.30
0.20q Q 2
N A  q
 16.2 N  1.65 kg
9.8
a  0.30 m

2q 0.20
2

0.20
m
b


2
l

N B  2q 0.30 9.8  0.20q Q  13.2 N  1.35 kg


m  2 kg

2q 0.20
2

X  30 r.p.m.  Q rad/s

2q9.8
 9.9 rad/s  94.5 r.p.m.
Xcrt 
0.2

- 147 -

Fsica Universitaria: Problemas de Fsica

Esttica del slido rgido. M20.11

11. Una puerta de garaje pesa 60 kg y est montada como se muestra en la figura.
Las ruedas estn enmohecidas de modo que no ruedan sino que deslizan en la
gua, siendo 0.4 el coeficiente cintico de rozamiento. La distancia entre las h
ruedas es de 2 m y cada una de ellas dista 50 cm de los bordes verticales de la
puerta. Se empuja la puerta mediante una fuerza horizontal constante de modo
F
que se mueva uniformemente. a) Si la lnea de accin de dicha fuerza dista 1
m de la gua, cul es la fuerza ejercida por cada una de las ruedas sobre el
carril? b) Encontrar la mxima distancia h a la que se puede aplicar la fuerza horizontal F sin que ninguna
rueda se separe del carril.

a) Aplicamos las ecuaciones cardinales de la esttica, tomando momentos en el punto A:

F  N P
F  N P
j F  f A f B  N( N A N B )

7 P  NA NB
P  NA NB
N A  P  N B

h
1
A }
N B  2 P  b F
Fh N Bb  P b
Fh N Bb  P b

2
2
de modo que resulta
F  0.4q 60  24 kg
N A  42 kg
f A  0.4q 42  16.8 kg
N B  30  12 24  18 kg f B  0.4q18  7.2 kg
b) En las condiciones del enunciado, ser NB = 0; i.e.,
2h
2h
b
2
P
F  N P 2hN  b h 

 2.5 m
2N 2q 0.4
b
b

NA
fA A

h
F

NB

fB B

- 148 -

Fsica Universitaria: Problemas de Fsica

Esttica del slido rgido. M20.12

12. Deseamos apilar un cierto nmero de ladrillos uno sobre otro, como se
muestra en la figura, de modo que obtengamos el mximo saliente.
Deseamos apilar un cierto nmero de ladrillos uno sobre otro, como se
muestra en la figura, de modo que obtengamos el mximo saliente.
a) Obtener el criterio que debemos seguir para conseguir nuestro objetivo. b) Demostrar que se puede conseguir un saliente tan grande como queramos sin ms que apilar un
nmero suficientemente grande de ladrillos.

a) En la figura, hemos representado mediante un circulito la posicin del centro de gravedad


de cada uno de los ladrillos. Mediante un triangulito, hemos representado la posicin del
centro de gravedad del ladrillo que sirve de base y de todos los que tiene encima.
El criterio que debemos seguir para apilar los ladrillos es que el centro de gravedad de los
ladrillos que se encuentre encima de uno dado no
G123 G12 G1
sobresalga sobre el borde de este ltimo, tal como
se ilustra en la figura para una condiciones crticas
(1)
de equilibrio.
(2)
l/2
La mayor longitud de la parte saliente del ladrillo
(3)
l/4
superior (1) es igual a l/2. Los ladrillos sucesivos
(4)
l/6
sobresalen sobre los que les sirven de base una
distancia dada por
m 0 m(l / 2)
l

ladrillo 2 l G (12) =
2m
4
2m 0 m(l / 2)
l

ladrillo 3 l G (123) =
3m
6
3m 0 m(l / 2)
l

ladrillo 4 l G (1234) =
4m
8
deducindose de modo obvio la regla a seguir en el caso de que hubieran ms ladrillos.
b) La distancia mxima en que la parte derecha del ladrillo superior (1) sobresale sobre el
ladrillo inferior (n-simo) que sirve de base, se expresa en la forma:
1 1 1 1
l 1 1 1 1

l !  !
2 4 6 8

2 1 2 3 4

que es la bien conocida serie armnica, que es divergente. As, para un nmero ilimitado de
ladrillos, esta suma tiende hacia infinito, con lo que queda demostrado el aserto propuesto.

- 149 -

Fsica Universitaria: Problemas de Fsica

Esttica del slido rgido. M20.13


C

13. La barra homognea AB, de longitud L y peso P, mantiene en equilibrio gracias a una
articulacin en el punto A y a una cuerda, de longitud tambin L, que acta unida a B y
C. a) Qu tensin tendr la cuerda BC? b) Qu reaccin habr en A?

L
B
L

30
Comenzamos determinando el ngulo  que forma la cuerda con la A
horizontal:
L  L cos 30 L cos R l cos R  1 cos 30  0.1340 l R  82.3

Escribimos las Ecuaciones Cardinales de la Esttica, tomando momentos en A:

l T cos82.3 R  0
T sen 82.3 N  P  0

A z P cos 30 TL sen(82.3 30 )  0


2

y resolviendo este sistema de tres ecuaciones con tres incgnitas (T, N y R) resulta:
0.1340 T  R  0 l R  0.1340 T  0.0733 P

0.9910 T N  P  0 l N  P  0.9910 T  0.4577 P

0.4330 P  0.7912 T  0 l T  0.5473 P


de modo que la tensin de la cuerda es
C
T  0.5473 P
y la reaccin en A (que acta sobra la barra) tiene
de componentes
N  0.4577 P
R  0.0733 P

L
T

lo que representa una resultante

F  0.4577 2 0.07332 P  0.4635 P


0.4577
tg K 
 6.24 l K  80.9
0.0733

F
R

N


30
A

- 150 -

Fsica Universitaria: Problemas de Fsica

Esttica del slido rgido. M20.14

14. El extremo superior de una varilla, de masa m y longitud l, est articulado a una
deslizadera que desliza a lo largo de una gua vertical lisa (vide figura), en tanto que
la varilla no pierde contacto en B con el apoyo liso. Determinar el valor del ngulo 
correspondiente al equilibrio y las reacciones en los apoyos de la varilla. Aplicacin
numrica: m = 10 kg, l = 100 cm y a = 5 cm.

A

a

B
l

Escribimos las Ecuaciones Cardinales de la Esttica del Slido Rgido,


descomponiendo en las direcciones horizontal y vertical y tomando momentos en A:
j N A  N B cos R
A
NA

7 P  N B sen R

a l

A z N B
=P sen R
sen R 2


NB

a
B

l/2

C
P

De modo que disponemos de tres ecuaciones con tres


incgnitas. Combinando la tercera con la primera:
l
l
2a
NB 
P sen 2 R 
N B sen 3 R l sen 3 R 
l
2a
2a
1/3
1/3
2a
10
sen R  
 0.4642 l R  27.7
100
l
NA 

10
P

 19.04 kg  187 N
tag R tag 27.7

NB 

10
P

 21.54 kg  211 N
sen R sen 27.7

Mtodo de la energa
Expresamos la energa potencial de la varilla (nivel de referencia en B) en funcin del ngulo
que forma con la vertical (1 grado de libertad):
l
cos R l

cos R
Ep  mg cos R  a
 cos R
 mg a
2
sen R 2

sen R

En la posicin de equilibrio estable, la energa de potenciar debe tener un valor mnimo. As,
determinamos el valor del ngulo correspondiente a dicho valor mnimo:
 sen 2 R  cos 2 R l

dEp
l
a
 mg a
sen R  mg sen R 
 0
2

2
dR
sen R
2
sen 2 R

De modo que
l
a
sen R 
2
sen 2 R

l sen 3 R 

- 151 -

2a
l

Fsica Universitaria: Problemas de Fsica

Esttica del slido rgido. M20.15

F 

15. Una viga AB, de 5 m de longitud y 60 kg de masa (distribuida


uniformemente), est apoyada en el suelo. Se levanta el extremo
B, a una altura de 3 m, mediante una fuerza aplicada en B,
siempre perpendicular a la viga. Hllense: la fuerza aplicada en
B, la reaccin del suelo en A y el coeficiente de rozamiento
mnimo necesario para que A no deslice.

B
l
G
h

N

A

Datos: m= 60 kg, l = 5 m, h = 3 m

h 3
  0.6 l R  36.87
l 5
Escribimos las ecuaciones cardinales de la esttica, tomando momentos en el punto A

2 3
6

[3]
f  F sen R  P 
P  14.4 kg  141.1 N

5
5
25
j f  F sen R

7 N F cos R  P
[2] N  P  F cos R  P  8 P  17 P  40.8 kg = 399.8 N

25
25

l
1
2

R
Fl
P
cos

[1] F  P cos R  mg  24 kg = 235.2 N


2

2
5

sen R 

Para que no exista deslizamiento en esta posicin deber ser


f
6
N    0.353
N 17

- 152 -

Fsica Universitaria: Problemas de Fsica

Esttica del slido rgido. M20.16

16. La viga AB, de masa 10 kg y 1 m de longitud, est cargada y apoyada


como se indica en la figura. Determinar la reaccin en el apoyo A y la
tensin del hilo BC cuando m1 = 2 kg y m2 = 7 kg.

m1
m2
30

A
B
0.4 m

En la figura presentamos el diagrama de fuerzas del cuerpo


libre correspondiente a la viga, con

F  7  2  5 kg

Ry

P  10 kg

Rx

30

A
F
a

b  0.4 m

a  0.5 m

Escribimos las Ecuaciones Cardinales de la Esttica, tomando momentos en A:


T cos 30  Rx

T sen 30 R  F P
y

l T sen 30  bF aP

de modo que disponemos de 3 ecuaciones con 3 incgnitas (T, Rx, Ry). Sustituyendo valores y
resolviendo el sistema de ecuaciones, tenemos:
T cos 30  Rx
Rx  T cos 30 = 14 cos 30  12.1 kg  119 N
l

Ry  15  T sen 30  15  7  8 kg  78 N
T sen 30 Ry  5 10  15

T sen 30  0.4q5 0.5q10  7 l T  7  14 kg  137 N

sen 30
El mdulo y la direccin de la reaccin en el apoyo A son:
R  Rx2 Ry2  12.12 82  14.5 kg  145 N

R =arctg

Ry
Rx

 arctg

8
 33.4
12.1

- 153 -

Fsica Universitaria: Problemas de Fsica

Esttica del slido rgido. M20.17

17. Sobre un canaln cilndrico AB de 10 kg se enrolla un hilo que se engancha


en el extremo B, tal como se representa en la figura. a) Determinar la
magnitud mnima de la fuerza F con que debemos tirar del otro extremo del
hilo para levantar el canaln. b) Calcular el valor mnimo del coeficiente de
rozamiento que debe tener el canaln con el suelo para que el punto A no
deslice.

30
F
B

Cuando la fuerza F es la mnima necesaria para levantar el canaln, la reaccin en B es nula y


el diagrama de fuerzas del cuerpo libre para el canaln
es el que se indica en la figura.
y
Aplicamos las ecuaciones cardinales de la esttica,
x
tomando momentos en el punto O:
T
G
F
Fx  0 (1) F cos R  N N A  0

mg T R
NA
Fy  0 (2) N A  F sen R  mg  0

B
P NA
O
M  0 (3) N R  FR  0 N  F
O
A
A

de modo que disponemos de un sistema de tres ecuaciones con tres incgnitas (F, NA y P).
De la primera ecuacin, teniendo en cuenta el resultado de la tercera, se sigue:
F cos R  N F  0 l N  cos R  cos 30  0.87
y, anlogamente, de la segunda ecuacin obtenemos:
mg
10q9.8
F  F sen R  mg  0 l F 

 196 N
1 sen R 1 sen 30

- 154 -

Fsica Universitaria: Problemas de Fsica

Esttica del slido rgido. M20.18

18. Un cilindro homogneo de 1.2 m de dimetro


pesa 1 t y descansa sobre la plataforma de un
camin segn se indica en la figura. Los
bloques representados se utilizan para impedir
que ruede el cilindro cuando acelere el camin.
Determinar la aceleracin de ste que hara que
el cilindro rodara sobre el bloque.

60cm

Supongamos que el camin frena con una aceleracin


constante -a0. Sobre el cilindro aparece una fuerza
de inercia dirigida hacia delante (en el sentido de la
marcha) dada por -m(-a0) = ma0. En la figura hemos
representa el diagrama del cuerpo libre o diagrama de
fuerzas que actan sobre el cilindro, en un referencial
solidario con el camin en el que, en las condiciones
crticas, el cilindro aun permanece en equilibrio,
aunque manifiesta una tendencia a rodar sobre el
borde A del bloque indicado.
Aplicamos tan slo la segunda ecuacin del equilibrio,
tomando momentos en A:

10cm

Fuerza de
inercia

ma0
B

mg

 A
b

b
g
Rh
A partir de la figura, por aplicacin del teorema de Pitgoras, tenemos:
ma0 ( R  h)  mgb l a0 

b 2  R 2  ( R  h) 2  2 Rh  h 2  h(2 R  h)
= b  10(120 10)  33.17 cm
De este modo, la aceleracin pedida es
33.17
a0 
g  0.66 g  6.5 m/s 2
60 10
Obviamente, las mismas consideraciones nos llevarn a los mismos resultados en el caso de
que el camin acelere, solo que entonces la fuerza de inercia tendr sentido opuesto al
indicado en la figura y la rodadura se presentar sobre el borde B del bloque trasero.
Otro mtodo: Aplicamos tan slo la primera ecuacin del equilibrio, en las direcciones
horizontal y vertical; i.e.,
j ma0  N cos R
a
u 0  cotg R l a0  g cotg R

7 mg  N sen R
g
R  h 50 5


l R  56.4 l cotg R  0.66
con sen R 
R
60 6
de modo que

a0  g cotg R  0.66 g  6.5 m/s 2

- 155 -

Fsica Universitaria: Problemas de Fsica

Esttica del slido rgido. M20.19


F

19. Calcular la fuerza horizontal tangencial F mnima necesaria para que el disco de la
figura, de masa 25 kg, ruede por el plano inclinado a 60. Hllese tambin la fuerza
que imprime el plano inclinado al disco, y el coeficiente de rozamiento mnimo para
que en esas condiciones no deslice.

Aplicamos las ecuaciones cardinales de la esttica,


tomando momentos con respecto al eje que pasa por el
punto A; i.e.,
j [1] F f cos R  N sen R

[2] N cos R f sen R  P


7

{ A [3] PR sen R  F ( R R cos R )  FR (1 cos R )

Disponemos de tres ecuaciones con tres incgnitas


(F, N , f).

F
N

O
60

60

P
[3] l F 
[1]
l
[2]

sen R
P
1 cos R

[2] l

f 

3
P
3

F

N sen R  f cos R  F

N cos R f sen R  P

De modo que N 

(q sen R )
(q cos R )

F  14.43 kg = 141.5 N

(+ m.a.m.)

l N  F sen R P cos R

sen 2 R
P P cos R  P
1 cos R

P  N cos R
sen R

1  cos R
sen R

60

P

3
3

PF

NP
f F

N  25 kg = 245.0 N
f  14.43 kg = 141.5 N

A partir de la definicin del coeficiente de rozamiento, tenemos


N

f
( 3 / 3) P
3


 0.58
N
P
3

La ecuacin [3] tambin puede escribirse tomando momentos con respecto al eje que pasa por
el punto O; de modo que las ecuaciones cardinales de la esttica quedan en la forma:

F f cos R  N sen R
j [1]

[2] N cos R f sen R  P


7

{ O [3]
fR  FR f  F

La resolucin de este sistema de tres ecuaciones con tres incgnitas (F, N, f) nos lleva a los
mismos resultados anteriormente obtenidos.

- 156 -

Fsica Universitaria: Problemas de Fsica

Esttica del slido rgido. M20.20

20. Una esfera uniforme de radio R y masa M, se mantiene en reposo sobre un

plano inclinado de ngulo T mediante una cuerda horizontal como se


muestra en la figura, siendo el coeficiente de rozamiento entre las
superficies P . Determinar: a) La tensin de la cuerda. b) La fuerza normal
ejercida sobre la esfera por el plano inclinado. c) La fuerza de rozamiento
que acta sobre la esfera. Aplicacin numrica: M = 3 kg, T = 30 y P =
0.5.

Aplicamos las ecuaciones cardinales de la esttica,


tomando momentos con respecto a un eje en O:
Fx  0 [1] T cos R f  Mg sen R
Fy  0

[2] N  Mg cos R T sen R

M O  0 [3] TR  f R l T  f

de modo que disponemos de 3 ecuaciones con


3 incgnitas (T, N y f ).
Rescribimos la ec. [1] con T = f :
sen R
T (1 cos R )  Mg sen R l T 
Mg
1 cos R

Mg

y de la ec. [2] se sigue:


2
2
2

sen R
cos R cos R sen R
1 cos R
N  Mg cos R T sen R  cos R
Mg
Mg 
Mg  Mg


1 cos R
1 cos R
1 cos R

de modo que
T

sen R
Mg
1 cos R

N  Mg

f 

sen R
Mg
1 cos R

y sustituyendo valores
sen 30
q3  0.804 kg
N  3 kg
f  0.804 kg
1 cos 30
Como la magnitud del la fuerza de rozamiento es tal que
f b f max  N N  0.5q3  1.5 kg
T

efectivamente hay equilibrio.

- 157 -

Fsica Universitaria: Problemas de Fsica

Esttica del slido rgido. M20.21


A

21. Dos tableros pesan 10 y 20 kg y miden 2 y 4 m de longitud,


respectivamente. Los tableros estn articulados en sus extremos, entre ellos
y en el techo y soportan un cilindro de 1 m de dimetro y 30 kg de peso. No
hay rozamiento en ningn contacto. a) Qu reacciones soportarn las
articulaciones A y C? (exprsense en forma vectorial) b) Qu fuerza se
ejercen los dos tableros en la articulacin B?

Geometra del problema


2
sen R   0.5 l R  30
4
AC  BC cos 30  4q 3 / 2  2 3  3.46 m

90

2m

4m

YA
A

YC
XA

90

C
=30

N1

l
O
l2
BD  BE  OD cotg 30  1/ 2q 3  3 / 2  0.87 m 1 D
N
2
Aplicamos las ecuaciones cardinales de la esttica
P1 30 30 E
P2
30
al cilindro y a cada uno de los tableros, por X
B
B
separado:
P
YB
Cilindro

j N  N sen 30 l N  P tg 30  30 3  10 3 kg
1
2
1

P
30

 20 3 kg
7 N 2 cos 30  P l N 2 
cos 30
3/2

Tabla vertical

j X A X B  N1
X A X B  10 3 l X B  10 3  7.5  9.82 kg

7 YA YB  P1
l YA YB  10

2 xA  3 / 4 10 3  7.5 kg
z 2 xA  3 / 2 N1

B
Tabla oblicua

j X C N 2 sen 30  X B

7 Y  P N cos 30 Y
l

C
2
2
B

B z 2 X C 3 P2 2 N 2  2 3 YC

X C  X B  N 2 sen 30  9.82 10 3  7.5 kg

YB  YC  P2  N 2 cos 30  14.33  20  20 3 2  35.67 kg

YC  33 X C 12 P2 14 N 2   33 7.5 10 5 3  14.33 kg

= R A  (7.5, 45.7) kg
R B  (9.82,  35.7) kg
R C  (7.5, 14.3) kg

o bien, en newtons:
= R A  (73.5, 448) N

R B  (96.2,  350) N

- 158 -

R C  (73.5, 140) N

XC

Fsica Universitaria: Problemas de Fsica

Esttica del slido rgido. M20.22

22. Una barra homognea AB de 5 N de peso y 4 m de longitud est


articulada en A una pared vertical y mantenida en su extremo
superior mediante un hilo horizontal BD, formando un ngulo
de 45 con la vertical. La barra soporta un disco, de 3 N de peso
y 1 m de dimetro, que se encuentra tambin en contacto con la
pared. Considerando despreciables los rozamientos, determinar
la tensin del hilo y la reaccin en la articulacin A.

B
r

N2
N1

En primer lugar determinamos las distancias a y b del NY 45


a
punto de contacto C a los extremos de la barra:
N2
Pdisco
0.5
r  a tg 22.5 l a 
 1.21 m
A NX
0.4142
Pbarra
En la figura adjunta hemos representado los diagramas
de fuerzas que actan sobre cada uno de los dos cuerpos (el disco y la barra).
Aplicamos las ecuaciones cardinales de la esttica al disco,

2
N  Pdisco  3 N
l N1  N 2
1
2

l
2

N 2 
Pdisco  2 Pdisco  3 2  4.24 N
2
m Pdisco  N 2

2
y a la barra, tomando momentos en A,

2
2

l N X N2
 T l N X  T  N2
 4.31 3  1.31 N

2
2

 5 3  8 N
m NY  Pbarra N 2

P
2
1.21 2
l 2
a 2

z N 2 a Pbarra
 Tl
l T
3 2 2.5  4.31 N
N 2 barra 

2 2
2
2
4
l

- 159 -

Fsica Universitaria: Problemas de Fsica

Esttica del slido rgido. M20.23

23. Un semicilindro homogneo, de peso P1 y radio R, se apoya en su base sobre un


plano horizontal rugoso. Una varilla homognea AB, de longitud l y peso P2, est
articulada a la pared por su extremo A y se apoya en la superficie lisa del
semicilindro formando un ngulo de 60 con la vertical. a) Determinar el valor
mnimo del coeficiente de rozamiento del semicilindro con el plano horizontal
para que la posicin indicada sea de equilibrio. b) En dicha posicin, calcular la
reaccin en la articulacin A.

60

De la geometra de la figura se sigue:


h
l h  R tg 60  R 3
R
a) En la figura se muestra el diagrama de fuerzas que actan sobre cada uno de los dos
cuerpos.
Aislamos la varilla y tomamos momentos en A:
tg 60 

l sen 60
l 3
l
hN 21  P2 sen 60 l N 21 
P2 
P2 
P2
2

2h
4
R
4R 3
Aislamos el semicilindro y aplicamos la primera condicin de la esttica:

l f  N cos 60  l P
12
2

8R
lP2
f

l Np


N1 8 RP1 3lP2
N1  P1 N12 sen 60  P1 l 3 P2

8R
b) Aislamos la varilla y aplicamos la primera condicin de la esttica:

l T  N cos 60  l P
h
21
2

8R

l 3

P2
Tv N 21 sen 60  P2 l Tv  P2  N 21 sen 60  1
8R

Tv
A
N21
60

Th

60
30

P2
C

h
N12
f

N1
O

60

P1

- 160 -

Fsica Universitaria: Problemas de Fsica

Esttica del slido rgido. M20.24

24. Una escalera se apoya contra un cilindro liso de radio R, fijo sobre una superficie horizontal. La escalera
forma un ngulo de 60 con la superficie horizontal y su longitud es 5 R/2. Determinar: a) La fuerza que el
cilindro ejerce sobre la escalera. b) La fuerza de rozamiento que evita que la escalera deslice. c) La fuerza
normal que la superficie horizontal ejerce sobre la escalera. d) El valor mnimo del coeficiente de
rozamiento entre el suelo y la escalera para mantener el equilibrio.

Geometra del problema:

5
A  AB  R  2.5 R

A
AG   5 R  1.25R

2 4

AC  R cotg 30  3R  1.73R

Ecuaciones cardinales de la esttica:


[1]
Fx  0 l N C cos 30  f

[2] Fy  0 l N C sen 30 N A  mg

[3]
M A  0 l NC AC  mg AG sen 30

y
x

NC
R
O

Resolvemos el sistema de tres ecuaciones con tres incgnitas (NA, NC y f):


[3] N C 

AG sen 30
(5 / 4) R(1/ 2)
5 3
mg 
mg 
mg  0.36 mg
24
AC
3R

5 3 3
5
mg  mg  0.31 mg
24 2
16
5 3
mg  0.82 mg
[2] N A  mg  N C sen 30  1
48

[1]

f  N C cos 30 

En las condiciones de deslizamiento inminente es:


f
0.31
Nmin 

Nmin  0.38
N A 0.82

- 161 -

30

C
G
mg
30
30

NA

f
A

Fsica Universitaria: Problemas de Fsica

Esttica del slido rgido. M20.25

25. Un tablero rectangular uniforme, de longitud 25 cm, se apoya sobre un


cilindro de 5 cm de radio y sobre el suelo, como se indica en la figura.
Tanto el tablero como el cilindro pesan 5 kg. Cunto deben valer, como
mnimo, los coeficientes estticos de rozamiento entre cilindro y tablero,
entre cilindro y suelo y entre tablero y suelo para que el sistema
permanezca en equilibrio?

25 cm
5 cm

30

Determinamos la posicin del punto D de contacto del tablero con el cilindro:


R
5
AD 

 18.66 cm
tg15 tg15
Para que el sistema est en equilibrio, debern estarlo el tablero y el cilindro por separado.
Aplicamos las ecuaciones cardinales de la esttica al tablero, tomando momentos en A:
l 1
N 3 sen 30  f1 f 3 cos 30

N3

f3

2
N1 N 3 cos 30 f 3 sen 30  P
L
cos 30
2
y al cilindro (momentos en C):

D f3
N2
C

N3

5 cm

N1
P

E 25 cm

l 4

15
15

f2

z 3
N 3 AD  P

f 2 f 3 cos 30  N 3 sen 30

5
N 2  P N 3 cos 30 f3 sen 30

A
f1
C

z 6

f3 R  f 2 R

de modo que disponemos de 6 ecuaciones con 6 incgnitas (N1, N2, N3, f1, f2 y f3) que
resolvemos para obtener:
PL cos 30 5q 25cos 30

 2.90 kg
3
N3 
2q18.66
2AD
6
f 3  f 2
sen 30
sen 30
q 2.90  0.78 kg
N3 
1 cos 30
1 cos 30
5
N 2  P N3 cos 30 f3 sen 30  5 2.90 cos 30 0.78sen 30
 7.90 kg

4
f 2 1 cos 30
 N3 sen 30 l f 2  f3 

2
N1  P  N3 cos 30 f3 sen 30
 5  2.90 cos 30 0.78sen 30
 2.10 kg
1
f1  N3 sen 30  f 2 1 cos 30
 f3 cos 30  f3  0.78 kg
=

f1  f 2  f 3  0.78 kg

N1  2.10 kg

N 2  7.90 kg

y los coeficientes de rozamiento pedidos sern:


f
f
0.78
0.78
N1 p 1 
 0.37
N2 p 2 
 0.10
N1 2.10
N 2 7.90

(sigue)

- 162 -

N3 p

N 3  2.90 kg

f3
0.78

 0.27
N 3 2.90

Fsica Universitaria: Problemas de Fsica

Esttica del slido rgido. M20.26

Aplicando las ec. cardinales de la esttica al sistema completo (tablero+cilindro), tomando


momentos en A:
l i

f 2  f1

ii
N1 N 2  2 P
A

z iii
N 2 AB  P AB P

L
cos 30
2

De modo que
f 2  f1
N1  2 P  N 2  10  7.90  2.10 kg
L/2

12.5
cos 30 P  1
cos 30q5  7.90 kg
N 2  1

18.66

AB

Y estas tres ecuaciones, junto con el sistema de ecuaciones (1)-(2)-(3) o el (4)-(5)-(6) nos
conduce a los mismo resultados que antes.

- 163 -

Fsica Universitaria: Problemas de Fsica

Esttica del slido rgido. M20.27

26. Un cilindro de radio r, y peso P, se apoya sobre un suelo y una pared rugosas
(coeficiente de rozamiento, ). Determinar el momento mnimo que hay que aplicar al
eje del cilindro para que dicho cilindro deslice.

Puesto que estamos interesados el momento mnimo, se entiende que el cilindro permanece
en reposo, tanto de traslacin (lo impide la pared), como de rotacin, en tanto que no se
supere dicho valor mnimo.
En la figura se representan las fuerzas que actan sobre el
cilindro (peso, rozamientos y reacciones normales en los
apoyos), con
f1
f1  N N1
f2  NN2
Mpar
A

N1

Las Ecuaciones Cardinales de la Esttica, descomponiendo en las direcciones horizontal y vertical y tomando
momentos con respecto al eje del cilindro, nos permiten
escribir:
l N1  f 2  N1  N N 2  0

f1 N 2  N N1 N 2  P

O { M par  f1r f 2 r  N( N1 N 2 )r

O
N2

f2
B

De las dos primeras se sigue:

%

1 N
 1 N2
N 1

N1 

1 N2

N2 

1 N2

N
0 N
P

P 1
1 N2

1
1 0
P

N P 1 N2

y sustituyendo en la tercera:
N
1
M par  N
P
1 N2
1 N 2

- 164 -

N (1 N)
P r 
Pr
1 N2

Fsica Universitaria: Problemas de Fsica

Esttica del slido rgido. M20.28

27. Una semiesfera hueca de 10 kg descansa sobre un plano horizontal. Sobre un


punto de su borde se coloca una masa m, inclinndose la semiesfera un ngulo de
45. Calclese el valor de la masa m.

45
m

Determinacin del centro de masa de una capa hemiesfrica:


d S  (2Qr ) R d R  2Q R 2 sen R d R

r  R sen R
zdS

zcm 

z
z  R cos R
S

S  12 (4Q R 2 )  2Q R 2
r
RdT
T

zcm 


1
S

2Q R 3
2Q R 2

Q /2

( R cos R )2Q R

sen R d R 

Q /2

sen R cos R d R R

sen 2 R
2

Q /2


0

Condicin de equilibrio:
Tomamos momentos en O.

R
mg

45

 0 mgR cos G  MG
m  12 M tg G

G
N

con I = 45,

Mg

ser m  12 M

con M =10 kg, ser m  5 kg

- 165 -

R
sen G
2

R
2

Fsica Universitaria: Problemas de Fsica

Esttica del slido rgido. M20.29

28. Dos placas rectangulares, de espesores despreciables, se apoyan por sus aristas de igual

2a

longitud sobre sendos realces de un plano horizontal separados una distancia a. Una
placa tiene la otra arista de longitud a y la otra placa de longitud 2a y masa doble que la
anterior. Despreciando los rozamientos, hllense el ngulo de equilibrio que forma la
placa mayor con el suelo horizontal y el mdulo y direccin de la reaccin que se
ejercen ambas placas.

C
a

A

Consideraciones geomtricas de inters:


sen R1  sen 2R2
R1 2R2  180 l
cos R1   cos 2R2
Teorema senos:

AC
a

sen R1 sen R2

l AC 

sen R1
a
sen R2

En la figura mostramos el diagrama de fuerzas


actuantes sobre cada una de las placas. Aplicaremos la
2 condicin del equilibrio ( M  0 ) a cada una de las
placas, tomando momentos en los puntos A y B
respectivamente.
Placa grande (momentos en A)
sen R1
R AC  2 Pa cos R2 l R
a  2 Pa cos R2 l
sen R2
2sen R2 cos R2
sen 2R2
R
P
PP l
sen R1
sen 2R2

RP

Placa pequea (momentos en B)


a
P cos R1 R cos R2
a cos R1  R sen R2
a sen R1 l
2
cos R1 2 cos R1 cos R2  2sen R1 sen R2  0 l

2a
R

2

G1

C
F2,y
A

R
2

2

2P

F2,x H

G2

a F1,y

P 1
a

F1,x

Ahora, tenemos que resolver esta ecuacin trigonomtrica teniendo en cuenta las relaciones
geomtricas establecidas inicialmente para eliminar las referencias al ngulo 1:
cos 2R2 2 cos 2R2 cos R2 2sen 2R2 sen R2  0 l
cos 2 R2  sen 2 R2 2 cos3 R2  2sen 2 R2 cos R2 4sen 2 R2 cos R2  0 l
cos 2 R2 cos 2 R2 1 2 cos3 R2 2sen 2 R2 cos R2  0 l
2 cos 2 R2 1 2 cos3 R2 2 cos R2  2 cos3 R2  0 l
2 cos 2 R2 2 cos R2 1  0
cos R2  0.3660 l

R2  68.53

l R1  42.94 l AC  0.73a

- 166 -

Fsica Universitaria: Problemas de Fsica

Esttica del slido rgido. M20.30

29. Dos bolas lisas de igual tamao y de peso P estn contenidas en el interior de una cavidad
cilndrica, tal como se indica en la figura. Determinar las reacciones en todos los
contactos.

2a
El diagrama de fuerzas del cuerpo libre para cada una de las dos bolas es el
a
que se indica en la figura.
Aplicamos las ecuaciones cardinales de la esttica al sistema completo (las dos bolas),
tomando momentos con respecto al punto O2. Tenemos
l N1  N 2

c
N1

N3  2 P
O 2 z Pa  N1 2a l N1  12 P

O1
P

de modo que

N12
N12

O2
P
N3

N2

N1  N 2  0.5 P

2a

N3  2 P

Para determinar las reacciones N12, es suficiente con


expresar la condicin de que la resultante de las tres
fuerzas (concurrentes) que actan sobre la bola superior
sea nula:
l N12 sen R  N1
5
l N12  N12 P 2 
P

N12 cos R  P
2

- 167 -

Fsica Universitaria: Problemas de Fsica

Esttica del slido rgido. M20.31

30. Dos cilindros idnticos se apoyan en una pared vertical y un suelo horizontal, tal
como se indica en la figura. Qu inclinacin mnima T debe tener la cua para que
haya equilibrio, no existiendo rozamiento en ninguno de sus contactos.

2a

En la figura presentamos el esquema de las fuerzas que actan sobre cada


a
cilindro en posicin genrica, con T > T min. Entonces aplicando las
ecuaciones cardinales de la esttica al sistema c+d y tomando momentos en O2,
obtenemos
N1  N 2 sen R

2 P  N N cos R
l
3
2

c
2aN1  aP l P  2 N1
O1
N1

N 2 sen R  12 P
A
P

[u] tg R 
4 P  2 N3
N 2 cos R  2 P  N 3
N12

P
N1  12 P
d
N21
En las condiciones crticas de equilibrio, el cilindro
T
O2
inferior se encontrar a punto de rodar alrededor de
N2
C y perder contacto con la superficie horizontal,
P
C
por lo que ser N3 = 0. El correspondiente valor del
N3
T
ngulo T, esto es T min, se obtiene a partir del
resultado anterior
B
1
P
tg Rmin 

l Rmin  14
4
4 P  2 N3

- 168 -

Fsica Universitaria: Problemas de Fsica

Esttica del slido rgido. M20.32


3r

31. Dos bolas idnticas, de masa m y radio r, estn colocadas en el interior de un tubo
cilndrico (abierto en sus bases) de dimetro 3r. El conjunto descansa sobre un plano horizontal, como se muestra en la figura. Determinar la masa mnima que deber tener el tubo
cilndrico para que el sistema no vuelque.

m,r

m,r

r
1
sen 60  3 / 2

l R  60 l tg 60  3
2r 2
Aplicamos la primera ecuacin cardinal de la esttica a cada una de las bolas:

3
3r
mg
N1 

cos
R

N
N

mg
3
2
1

tg
R
l

l

N1
2 3
N 2 sen R  mg

N 2 
mg
N1
N1
L
3

N2

N  3 mg
N3  N 2 cos R

2r
mg

3
3
N 4  N 2 sen R mg

N2
N 4  2mg
N3
N3
 Mg
Las fuerzas que actan sobre el tubo son N1, N3, N5 y
N5
r
Mg. Las dos primeras (N1,N3) constituyen un par de
mg
fuerzas
que tienden a volcar el tubo, cuyo momento
N4
A
es:
cos R 

3
mg  rmg
3
En las condiciones crticas de vuelco, el tubo estar sometidos a dos pares de fuerzas de
sentidos opuestos: el par de vuelco (N1,N3) y el par recuperador (N5,Mg). Aplicamos la
segunda ecuacin cardinal de la esttica al tubo, tomando momentos en A:
M par  2r sen R
N1  3 r

3r
rmg b Mg l
2

- 169 -

2
Mp m
3

Fsica Universitaria: Problemas de Fsica

Esttica del slido rgido. M20.33

32. Un tractor de 2 t de peso, cuyas medidas ms significativas se indican


en el esquema, arrastra una carga de la que tiene que tirar con una
fuerza horizontal de 1 t (a 1.5 m de altura): a) En tales condiciones,
determinar las componentes vertical y horizontal de las fuerzas que
actan en el contacto de cada una de las ruedas con el terreno. b) Qu
arrastre mximo horizontal puede realizar el tractor sin que se
levanten sus ruedas delanteras? c) Determinar el ngulo mximo de
elevacin que puede tener el terreno para que el tractor, sin arrastre, lo
suba sin que se levanten sus ruedas delanteras.

F
G

1.5 m
1m
2m

a) En los tractores, tan solo las ruedas traseras


son tractoras por lo que se agarran al
NB F
terreno; por el contrario, las ruedas delanteras,
en tanto que se mantenga constante la velocidad
G
del tractor, mantendrn constante su velocidad
1.5 m angular, por lo que no necesitan agarrarse al
NA
fB
terreno. En definitiva, el diagrama de fuerzas
P 1m B
A
del cuerpo libre, aplicadas al tractor, es el que se
indica en la figura.
2m
Aplicando las ecuaciones cardinales de la
esttica, en las direcciones vertical y horizontal y tomando momentos en B, tenemos
NA NB  P

N A N B  2 l N B  1.75 t (por eje)

l f B  1 (por eje)
fB  F

2 N A 1.5F  P
2 N A 1.5  2 l N A  0.25 t (por eje)

b) Rescribimos las ecuaciones anteriores para la condicin crtica (NA = 0):

NA NB  P
N B  2 l N B  2 t (por eje)

l f B  1.3 t (por eje)


f B  Fmx

2 N A 1.5Fmx  P
1.5Fmx  2 l Fmx  1.3 t

vertical

NB
G

fB
A

1m

1.5 m
B

2m

terreno

c) Para la pendiente crtica ser NA = 0 y, al no


arrastrar carga alguna, ser F = 0, por lo que tan
solo quedan las tres fuerzas indicadas en la figura,
que debern ser concurrentes en B (para que el
momento sea nulo). En consecuencia, el problema
se reduce a una simple condicin geomtrica de
que el centro de gravedad (G) se encuentre en la
vertical del punto B:

1
 0.6 l R  33.7
tg R 
1.5

horizontal

- 170 -

Fsica Universitaria: Problemas de Fsica

Esttica del slido rgido. M20.34

33. En el mecanismo que se esquematiza en la figura,


todas las cotas estn expresadas en milmetros y se
supone despreciable el peso del propio mecanismo.
a) Determinar el valor de la fuerza F que permite el
equilibrio. b) Calcular las reacciones en los cojinetes
A y B.

400
100
300
F
A

200
B

400 kg

37
250

a) Clculo de la fuerza aplicada.


La condicin de equilibrio de la polea mvil (inferior) implica
que:
P  2T T  200 kg

z
37

Tomando momentos con respecto al eje x, obtenemos


F q 250  T q50

F

50q 200
 40 kg
250

F
y

Las componentes de la fuerza F son


Fy  F cos 37  32 kg
Fz  F sen 37  24 kg
b) Calculo de las reacciones en los apoyos del eje.
Tomamos momentos con respecto al eje z
(perpendicular al papel) en A y en B (Fig. inferior
izquierda):

(A) RB q300 32q500  0


(B) RA q300  32q 200

T
P
P = 400 kg

RB  53 kg

RA  21 kg

Tomamos momentos con respecto al eje y (perpendicular al papel) en A y en B (Fig.


inferior derecha):
(A) 400q 400 N B q300  24q500

N B   493 kg

(B) 400q 700  N A q300 24q 200

N A  917 kg

y
RA
400

300
A

24 kg
NA

RB
200
B

32 kg

400

NB
300

200
x

x
A
P = 400 kg

- 171 -

Fsica Universitaria: Problemas de Fsica

Esttica del slido rgido. M20.35

34. En el mecanismo que se representa en la figura se aplica

Cy

un par mediante dos fuerzas de 100 N aplicadas en los


puntos D y E de la aleta. Todas las cotas estn expresadas
en milmetros (mm). Determinar la fuerza F necesaria
para establecer el equilibrio y las reacciones en los
apoyos fijos B y C. (Se desprecia el peso de la aleta).

y
C

30

Cx

200

By
120

Las fuerzas que actan sobre la estructura son:


B  Bx i By j
B  (0, 0, 0)
C  Cx i C y j
C  (0, 0,  0.2)
FF i
F  (0,  0.04, 0.04)
y el par de fuerzas aplicadas en D y E, cuyo
momento es

40

Bx

40

M par  0.120q100 cos 30 i sen 30 k


 12 cos 30 i sen 30 k
 6 3i 6k m N

Aplicamos las ecuaciones cardinales de la esttica, tomando momentos en B:


JJJG
JJJG
F  F B C  0
M B  M par BAqF BCqC  0

Esto es:

F Bx Cx  0
Fx  0
l
l

By C y  0

Fy  0
6 3 0 F 0 C 6 3 0.2C y

M B  0 l 0 0.04q 0 0 qC y  0.04F  0.2Cx  0


6 0.04 0 0.2 0 6 0.04 F

F  0

Y resolviendo el sistema de cinco ecuaciones con cinco incgnitas,


tenemos:

6 3

 30 3  52 N
6 3 0.2C y  0 l C y  

0.2

0.04 F  0.2C  0 l C  0.04 F  0.2 F  30 N

x
x

0.2

 150 N
6 0.04 F  0 l F  

0.04

Bx  F  Cx  150 30  180 N
F Bx Cx  0

By C y  0
By  C y  30 3  52 N
En definitiva:
F  150 i N

B  180 i 52 j
N

C  30 i  52 j
N

- 172 -

C
30
E

30
Mpar

D
B
x
A
z

Fsica Universitaria: Problemas de Fsica

Esttica del slido rgido. M20.36

Otro mtodo (mucho ms cmodo)


Una vez calculado el momento del par de fuerzas, como ya hemos hecho en el mtodo
anterior, imponemos la 2 condicin de la Esttica anulando los momentos con respecto a los
tres ejes coordenados que se indican en la figura, teniendo en cuenta que el momento de una
fuerza con respecto a un eje es nulo si la lnea de accin de la fuerza es coplanaria con el eje
(i.e., lo corta o es paralela al eje):
M par cos 30

6 3

 52 N
0.2
BC
F BA
150q 0.04

 30 N
M y  0 l F BA  Cx BC  0 l Cx 
0.2
BC
M par sen 30
6

 150 N
M z  0 l M par sen 30 F AF  0 l F  
0.04
AF
Ahora, imponemos la 1 condicin de la Esttica para determinar las reacciones en el punto B:
Bx  F  Cx  150 30  180 N
F Bx Cx  0
l
F  F B C  0 l By C y  0
By  C y  30 3  52 N

M x  0 l M par cos 30 C y BC  0 l C y  

B  180 i 52 j
N

En definitiva: F  150 i N

Cy

C  30 i  52 j
N

y
C

Cx

30

30

200

By

30

120

40

Bx

40

Mpar

A
z

- 173 -

Fsica Universitaria: Problemas de Fsica

Dinmica del slido rgido. M21.1

1. Dos masas m1 y m2 (m2 > m1), unidas mediante una cuerda inextensible y ligera,
penden de una polea de masa M, radio R y momento de inercia I respecto a su eje
de rotacin. Sobre el eje de la polea se ejerce una fuerza F vertical y hacia arriba,
superior al peso del sistema, de tal magnitud que no produce aceleracin sobre la
masa mayor m2. Considrese que la cuerda no resbale sobre la polea. Determinar
las aceleraciones de la masa pequea m1 y del eje de la polea, la aceleracin
angular de sta y la magnitud de la fuerza F.

+
+
A

m2

m1

Aplicamos las ecuaciones cardinales de la dinmica a cada una


de las masas y a la polea, con el convenio de signos indicado en
la figura y tomando momentos en el eje de la polea:
(1) T1  m1 g  m1a1

F
B

T1

(2) T2  m2 g  0

T2

(3) F  Mg  T1  T2  Ma
(4) T2 R  T1 R  IB

Mg

T1

T2

m1g

m2g

Disponemos de cuatro ecuaciones con seis incgnitas (T1, a1, T2,


F, a, ).
Obtenemos una nueva ecuacin a partir de la condicin de
inextensibilidad de la cuerda (condicin de ligadura)

x  x1
x  x2
Q R  l l 2 x  x1  x2  l  Q R

que la derivamos dos veces con respecto del tiempo para obtener una
relacin entre las aceleraciones:

2 x  x1  x2  0 l 2a  a1 a2

x1

x2

l (5)

a1  2a

La condicin de que la cuerda no resbala en la polea nos proporciona


otra ecuacin:
(6) a1  a  B R l a  B R l

B

a
R

Resolvemos el sistema de ecuaciones:


(1) T1  2m1a m1 g
(2) T2  m2 g

(4) m2 g  2m1a  m1 g 

I
a l
R2

a

(3) F  Ma Mg 2m1a m1 g m2 g  M m1 m2
g M 2m1
a

M 2m1
m2  m1

F  M m1 m2
g
I

m
2

1

R2

- 174 -

m2  m1

2m1

I
R2

Fsica Universitaria: Problemas de Fsica

Dinmica del slido rgido. M21.2

3L/4

2. La barra uniforme de masa m y longitud L de la figura est articulada en su


extremo superior A. Inicialmente se halla en reposo en la posicin vertical.
Se tira de la cuerda con una fuerza T. En el instante inicial, determinar la
aceleracin angular de la barra y la reaccin en el pasador en A.

3L/4

Las fuerzas que actan sobre la barra se indican en rojo en la


figura adjunta.
El movimiento de la barra consiste en una rotacin pura alrededor del eje que pasa por A.
Aplicando la ecuacin fundamental de la dinmica de la rotacin, tomando momentos con
respecto al eje A, se sigue:
Ry
Rx
A
G
P

3
1
9 2 T
LT cos 45  mL2B B 
4
3
8 mL
El centro de masa de la barra describe una trayectoria circular. En el
instante indicado, el c.m. tiene una aceleracin centrpeta nula (por se
nula su velocidad) y una aceleracin tangencial (horizontal) dada por

 IA

l 9 2T
ax  B 
2
16 m
Aplicando la ecuacin fundamental de la dinmica de traslacin al
centro de masa de la barra, se obtienen las dos ecuaciones siguientes:
l T cos 45 Rx  max

F  macm  P T sen 45 Ry  0

de donde
9 2
2
2
2
Rx  max  T cos 45 

T
Ry  mg 
T
T 
2
2
16
16
45

Las reacciones en el pasador A son iguales y opuestas a las calculadas y se indican en azul en
la figura.

- 175 -

Fsica Universitaria: Problemas de Fsica

Dinmica del slido rgido. M21.3

3. En el esquema que se representa en la figura, el plano carece de

m2
rozamiento. La cuerda es de masa despreciable, pasa a travs del
centro de masas de cada bloque y no desliza en la polea. La polea
tiene forma de cilindro de radio R y masa m2. Siendo m1 = m; m2 =
v
2m; m3 = 4m y T = 30. Determinar: a) El momento de inercia de la
polea respecto a su eje. b) El momento resultante de las fuerzas m
1
que actan sobre el sistema (las dos masas, la cuerda y la polea)
T
respecto al centro de la polea. c) El momento cintico (o momento
angular) del sistema respecto al centro de la polea cuando las
masas se mueven con velocidad v. d) La aceleracin de las masas. e) Las tensiones en la cuerda.

a) El momento de inercia viene dado por


1
1
I  m2 R 2  2mR 2  mR 2
2
2
b) Las fuerzas que intervienen en el movimiento del
sistema son las que se representan en la figura.
El momento neto con respecto al eje que pasa por O es
M O  (m3 g sen R ) R  m1 gR  4mg 12 R  mgR  mgR
c) El momento angular respecto a O vendr dado por
LO  m1vR I X m2 vR  mvR mvR 4mvR  6mvR

ya que X  v R .

O
T1

m3
v

T3
T3 N3

T1

m1g

m3g

v
d) A partir de los resultados anteriores, podemos determinar
v
directamente las aceleraciones. En efecto, teniendo en cuenta
dL
que O  M O , resulta
dt
dLO
d
dv
1
 (6mrv)  6mr  6mra  mgr l a  g  1.63 m/s 2
dt
dt
dt
6
e) Para calcular las tensiones en las cuerdas debemos escribiremos las ecuaciones del
movimiento para cada uno de los bloques:
7
T1  m1 g  m1a l T1  m1 (a g )  m g  76 mg
6
m3 g sen R  T3  m3a l T3  m3 ( g sen R  a )  (4m) g ( 12  16 )  43 mg

- 176 -

Fsica Universitaria: Problemas de Fsica

Dinmica del slido rgido. M21.4

4. Una caja de 15 kg est sujeta al extremo de una cuerda inextensible

Mpar

arrollada sobre un tambor uniforme de 40 kg y 600 mm de dimetro, segn


se indica en la figura. En el instante representado, la caja est cayendo a
9 m/s. Determinar el par constante de frenado que hay que aplicar al tambor
para que la caja quede en reposo tras descender 3 m.

300 mm
40kg

9m/s

15kg

Mtodo dinmico: El movimiento del bloque es un movimiento


uniformemente acelerado, por lo que aceleracin (de frenado) que detendr al bloque se
calcula a partir de la expresin:
v2
v 2  v02
92
m
 0 
 13.5 2
2x
2x
2q 3
s
Llamando a la aceleracin angular del tambor, la condicin de
ligadura que relaciona el movimiento de ste y con el del bloque se
expresa por
a  BR
Escribimos las ecuaciones del movimiento:
Bloque (movimiento de traslacin):
mg  T  ma l T  m( g  a )
v 2  v02 2ax l a 

Mpar
M,R

+
T

Tambor (movimiento de rotacin):


1
1
1
m
TR  M par  I B  MR 2B l M par  TR  MR 2B  TR  MRa
2
2
2
mg
Sustituyendo los valores dados en las expresiones anteriores,
calculamos la tensin T de la cuerda y el momento del par pedido:
T  m( g  a)  15q (9.8 13.5)  349.5 N

Ma
40q13.5
M par  (T 
) R  349.5
q 0.3  185.9 N.m

2
2

Mtodo de la energa: Mientras que el bloque desciende una distancia h, hasta detenerse, el
tambor gira un ngulo ; adems, en cada instante, la velocidad del bloque est relacionada
con la velocidad angular del tambor:
v
h 3000
9
 10 rad
 30 rad/s
h  RR l R  
v  X R l X0  0 
R
300
R 0.3
El trabajo realizado por el par de frenado, Wpar  M par R , es igual a la disminucin de la

energa del sistema, que viene dada por:

%E  mgh 12 mv02 12 ( 12 MR 2 )X02 


 mgh 12 mv02 14 Mv02  15q9.8q3 12 15q92 14 40q92  1858.5 J
Y el momento de frenado ser:

Wpar  M par R l M par 

Wpar
R

%E 1858.5

 185.9 N.m
10
R

- 177 -

Fsica Universitaria: Problemas de Fsica

Dinmica del slido rgido. M21.5

5. Una varilla de longitud L y masa m puede girar sin rozamiento alrededor de un eje

a) Tomamos el origen en O:

m L / 2
m 0

L
xcm 

l OG  h  14 L
2m
4
b) El momento de inercia del sistema es la suma de los momentos de
inercia de la masa y de la varilla:

horizontal O que pasa por su punto medio. En uno de los extremos de la varilla hay
adherida una masa puntual m. Se abandona el sistema en posicin horizontal.
Determine: a) La posicin del centro de masa. b) El momento de inercia del sistema
respecto al eje O. c) La velocidad angular cuando la varilla alcance la posicin vertical.
d) La aceleracin angular cuando la varilla forma un ngulo T con la vertical. e) La
fuerza que ejerce el eje sobre la varilla cuando sta alcanza la posicin vertical.

O
h
L

L
1 1
1
1
I O  m mL2  mL2  mL2
2 12
4 12
3
2

c) El movimiento del sistema consiste en una rotacin pura alrededor


del eje O. Puesto que el sistema es conservativo, la conservacin de la
energa se expresa en la forma:
L 1
1
3g
I O X 2 l mgL  mL2 X 2 l X 
L
2 2
3
d) Aplicamos la Ecuacin Fundamental de la Dinmica de la Rotacin,
tomando momentos en el eje de rotacin:
L
2mg sen R
2mgh sen R
3g
4


2mgh sen R  I OB l B 
sen R
1
2L
IO
mL2
3
e) Aplicamos la Ecuacin Fundamental del Movimiento del c.m. del
sistema:


m

0  mg

R  2mg  2m
acm

T
2mg

l R  2m g acm

y teniendo en cuenta que las componentes tangencial y normal del


c.m., para la posicin vertical ( T = 0 ), son:
3g L 3g
L 3g

sen R  0
at  B h  B 
an  X 2 h 
4
8
4
L 4
resulta que acm = an , de modo que
3 7
R  2m g acm
 2mg 1  mg
4 2

- 178 -

O
acm
G


m

Fsica Universitaria: Problemas de Fsica

Dinmica del slido rgido. M21.6

puntual A de 2.5 N de peso comienza a deslizarse lentamente hacia fuera a lo largo de


miembro horizontal. Determinar la disminucin de la velocidad angular del rbol cuando
la masa A se desliza desde 75 mm hasta 600 mm a partir del eje del rbol.

525 mm

75 mm

6. El rbol vertical de la figura gira con una velocidad inicial de 20 rad/s cuando la masa

Puesto que no existe momento dinmico externo con respecto al eje de rotacin (vertical), se
conserva el momento angular o cintico con respecto a dicho eje. Esto es:
I
I1X1  I 2 X2 X2  1 X1
I2
Despreciando la masa de la varilla y el momento de inercia del dispositivo, sern:
I1  mE12

I 2  mE22

I1 E1

I 2 E2

525 mm

75 mm

de modo que
2
E
75
X2  1 X1 
q 20  0.31 rad/s
600
E2
2

Por consiguiente, ser


%X  X2  X1  19.69 rad/s

- 179 -

Fsica Universitaria: Problemas de Fsica

Dinmica del slido rgido. M21.7

7. Considere una barra delgada, con masa m = 4 kg y longitud l = 1.2 m, que oscila sin rozamiento en un plano
vertical alrededor de un eje horizontal que pasa por un punto de la barra situado a 1/4 de uno de los
extremos de la misma. a) Expresar la aceleracin angular de la barra en funcin del ngulo que forma con la
vertical. b) Calcular el periodo de las pequeas oscilaciones de rotacin de la barra.

a) La nica fuerza que produce momento respecto al eje de rotacin es el peso de la barra que
est aplicado en el centro de gravedad de la misma.
Planteamos la ecuacin para la dinmica de la rotacin de la barra para una posicin genrica
en la que sta forma un ngulo  con la vertical: i.e.,
l
M O  I OR l  mg sen R  I OR
4
M
l/4
donde IO es el momento de inercia de la barra respecto al eje de de
rotacin y R la aceleracin angular de la barra.
O
G l
Calculamos I aplicando el teorema de Steiner:

l
1
1
7
I O  I cm m  ml 2 ml 2  ml 2
4
12
16
48
2

Por lo que la ecuacin para la aceleracin angular resulta ser


l
7
12 g
mg sen R  ml 2R l R 
sen R
4
48
7l
b) La ecuacin anterior resulta ser la de un pndulo compuesto. Haciendo la aproximacin
de pequeas oscilaciones, i.e., sen R x R , tenemos la ecuacin del movimiento de rotacin,
que podemos escribir en la forma
12 g
R 
R0
7l
Que corresponde a oscilaciones armnicas simples cuya frecuencia angular y periodo vienen
dados por
mg

X

12 g
12q9.8
2Q
2Q

 3.74 rad/s l T 

 1.68 s
X
7l
7 q1.2
3.74

- 180 -

Fsica Universitaria: Problemas de Fsica

Dinmica del slido rgido. M21.8

8. a) Calcular la frecuencia de las pequeas oscilaciones de un aro de radio R colgado de la pared mediante un
clavo horizontal. Cul es la longitud reducida de este pndulo fsico? b) Repetir el apartado anterior si se
suprime la mitad inferior del aro.

a) El Teorema de Huygens para los puntos conjugados en un pndulo compuesto nos permite
escribir

I cm mR 2

R
mh
mR
de modo que el conjugado de O es O (obvio) y la distancia entre ellos
es la longitud reducida del pndulo fsico constituido por el aro en sus
pequeas oscilaciones; eso, es
M  h h '  R R  2R
La frecuencia de las pequeas oscilaciones ser:
mhh '  I G

l h'

O

1 M
1 2R

2Q g 2Q g

O
h
G
h
O

b) Aplicamos el Primer Teorema de Pappus-Guldin para determinar la posicin del c.m. (G)
de medio aro, siendo CG=
:
JJJK
2
Q2
S  sL l 4Q R 2  Q R q 2QE l E  R l h  GO  R  E 
R
Q
Q
El Teorema de Steiner nos permite determinar el momento de inercia de medio aro con
respecto a su c.m.:
2
4 2 Q  4


R
mR 2
Q2
Q2
De nuevo, el Teorema de Huygens nos permite determinar el conjugado O de O:

I C  I G mE 2

mhh '  I G

l I G  I C  mE 2  mR 2  m

JJJK I cm Q 2  4
Q
Q 2

R
R
l h '  GO' 

mh Q 2 Q  2

Q 2

Q2
R
R  2R
Q
Q
de modo que, de nuevo el conjugado de O es el punto O
diametralmente opuesto.
La frecuencia de las pequeas oscilaciones ser:
M  h h'

O

1 M
1 2R

2Q g 2Q g

- 181 -

O
h
G

Fsica Universitaria: Problemas de Fsica

Dinmica del slido rgido. M21.9

9. Un disco uniforme, de radio R y masa m, posee una pequea perforacin practicada en el


punto A, a una distancia h del centro del disco. a) Determinar la frecuencia de las
pequeas oscilaciones que se producen cuando el disco est suspendido en el punto A. b)
A qu distancia h hay que hacer el agujero para que la frecuencia sea mxima?

A
M,R

a) Escribimos la ecuacin fundamental del movimiento de


rotacin alrededor de un eje fijo, tomando momentos respecto al
eje de suspensin que pasa por el punto A
mgh
mgh sen R  I A R l R
sen R  0
IA

A
+
h

G

mg

y, como para valores pequeos de T podemos establecer la


aproximacin sen T = T , resulta
mgh
R
R0
IA
que es la ecuacin del movimiento, que corresponde a un
m.a.s.de rotacin (por ser de la forma la forma R X 2R  0 ) en
el que la frecuencia angular es,

mgh
mgh
2 gh
2 gh

 2
l X  2QO 
2
2
1
IA
R 2h 2
mR 2 mh 2 R 2h
2
b) La frecuencia angular ser mxima cuando lo sea Z 2; esto es,
X2 

d X 2

dh

0 l

d X 2

dh

 2g

R 2 2h 2
 h 4h

R 2  2h 2
 2g
0
2
2
R 2 2h 2

R 2 2h 2

lo que se consigue cuando el numerador es nulo


R 2  2h 2  0 l h 

- 182 -

R
2

R
2
2

Fsica Universitaria: Problemas de Fsica

Dinmica del slido rgido. M21.10

10. Consideremos un disco, de masa m y radio R, que oscila sin rozamiento en un plano vertical alrededor de un
eje horizontal y perpendicular al disco y que pasa por un punto del mismo situado a R/2 del centro del disco.
a) Expresar la aceleracin angular del disco en funcin del ngulo que forma con la vertical. b) Calcular el
periodo de las pequeas oscilaciones de rotacin del disco.

a) La nica fuerza que produce momento respecto al eje de rotacin es el peso del disco que
est aplicado en el centro de gravedad del mismo.
Planteamos la ecuacin para la dinmica de la rotacin del disco para una posicin genrica
en la que sta forma un ngulo  con la vertical: i.e.,
R
M O  I OR l  mg sen R  I OR
2
donde I0 es el momento de inercia del disco respecto al eje de de
rotacin
y R la aceleracin angular del disco.
m
O R/2
G
Calculamos I0 aplicando el teorema de Steiner:

R
1
1
3
I O  I cm m  mR 2 mR 2  mR 2
2
2
4
4
2

R


Por lo que la ecuacin para la aceleracin angular resulta ser


R
3
2g
mg sen R  mR 2R l R 
sen R
2
4
3R
b) La ecuacin anterior resulta ser la de un pndulo compuesto. Haciendo la aproximacin
de pequeas oscilaciones, i.e., sen   , tenemos la ecuacin del movimiento de rotacin, que
podemos escribir en la forma
2g
R 
R0
3R
Que corresponde a oscilaciones armnicas simples cuya frecuencia angular y periodo vienen
dados por
mg

X

2g
3R

l T  2Q

- 183 -

3R
2g

Fsica Universitaria: Problemas de Fsica

Dinmica del slido rgido. M21.11

11. Un cilindro, de masa m y radio R, desciende por un plano inclinado un ngulo  respecto de la horizontal.
Determinar el valor mximo de dicho ngulo para que el cilindro ruede sin deslizar sobre el plano, siendo 
el coeficiente de rozamiento entre el plano y el cilindro.

Escribimos la ecuacin fundamental del movimiento de rotacin, tomando momentos con


respecto a la generatriz del cilindro en contacto con el plano inclinado:
1

3
2 g sen R
mgR sen R  I OB  mR 2 mR 2 B  mR 2B l B 
2

2
3R
Rescribimos la ec. fundamental de la rotacin tomando momentos con respecto al eje de
simetra del cilindro (que pasa por el c.m.):
1

1
R f  I cmB  mR 2 B l f  mRB

2
N
m, R
Sustituyendo en esta ecuacin el resultado obtenido en la
primera, tenemos
1
f
f  mgR sen R
O
3
mg
de
modo
el
que
rozamiento
esttico
entre el cilindro y el plano
T
inclinado tendr que ser tanto mayor cuanto mayor sea el
ngulo de inclinacin de ste, a fin de mantener la rodadura (sin deslizamiento) del cilindro.
Puesto que el rozamiento esttico no puede superar un cierto valor mximo, ser
1
f b NN l
mg sen R b Nmg cos R l tg R b 3N l R b arctg 3N

3
y la rodadura sin deslizamiento tan solo ser posible si se cumple esta ltima condicin. Por
consiguiente, el ngulo pedido es
cm

Rmax = arctg 3N

- 184 -

Fsica Universitaria: Problemas de Fsica

Dinmica del slido rgido. M21.12

12. Un cilindro macizo baja rodando sin resbalar por un plano inclinado. a) Calcular la aceleracin del centro de
masa del cilindro. b) Determinar el valor mnimo de la fuerza de rozamiento (esttico) entre el plano y el
cilindro a fin de que ste ruede sin resbalar. c) Calcular el valor mnimo del coeficiente de rozamiento para
que el cilindro no resbale. d) Estudiar el movimiento del cilindro en funcin de diversos valores del coeficiente de rozamiento. e) Se conserva la energa total del cilindro cuando ste rueda sin resbalar?

abc) Aplicamos las ecuaciones cardinales de la dinmica de traslacin y rotacin al cilindro,


tomando momentos en el c.m.:
/ [1] N  mg cos R  0
2 [2] mg sen R  f  ma
N
1

z [3] fR  mR 2 B

que, junto con la condicin de rodadura,


[4] a  B R
mg

constituyen un sistema de cuatro ecuaciones con cuatro incgnitas (N, f, a, ).

1
Sustituyendo la [4] en la [3] tenemos f  mR B  ma
2

2
3
2
De la [2] se sigue mg sen R  ma f  ma l a  g sen R que es la aceleracin
2
3
pedida.
El valor mnimo de la fuerza de rozamiento necesaria para la rodadura es
1
1 2
1
f  ma  m g sen R  mg sen R
2
2 3
3
De la definicin del coeficiente de rozamiento esttico, se sigue
f b NN

l Np

1
mg sen R 1
f
3
 tg R l
N
mg cos R
3

1
Nmn  tg R
3

d) Tipo de movimiento del cilindro:


x Si  = 0, el cilindro no puede rodar, simplemente desliza (traslacin pura).
x Si 0 <  < mn , el cilindro rueda y resbala.
x Si   mn , el cilindro rueda sin resbalar (rodadura).
e) La energa se conservar en tanto que no haya resbalamiento (rozamiento cintico) entre el
cilindro y el plano; esto es, que sea   mn .

- 185 -

Fsica Universitaria: Problemas de Fsica

Dinmica del slido rgido. M21.13

13. a) Hallar la aceleracin del centro de masa de un cilindro macizo de 20 cm de radio y 10 kg de masa, que
rueda sin deslizar sobre un plano inclinado 30. b) Si el cilindro parte del reposo, que velocidad alcanzar al
descender 5 m de altura.

N
f

c.m.

5m
mg

a) Escribimos las Ecuaciones Cardinales de la dinmica


de traslacin (c.m) y de rotacin alrededor de un eje
que pasa por el c.m., as como la condicin de
rodadura:
mg sen R  f  macm

N  mg cos R  0
1

con I  mR 2
Rf  I B
2

acm  RB

que constituyen un sistema de cuatro ecuaciones con cuatro incgnitas (acm, , f, N).
Operando con las dos ltimas ecuaciones, tenemos

1
Rf  mR 2B
1
1
2 acm
l f  macm
2
Rf  mR

R
2
2
acm  RB

y sustituyendo este valor en la primera ecuacin


1
2
1
mg sen R  macm  macm l acm  g sen R  g  3.27 m/s 2
2
3
3
b) Se trata de un movimiento de traslacin con aceleracin constante a lo largo del plano.
Descender una altura de h = 5 m implica un desplazamiento x a lo largo del plano
h
5
x

 10 m
sen R sen 30
y la velocidad alcanzada, partiendo del reposo, ser
v  2acm x  2q3.27 q10  8.08 m/s

- 186 -

Fsica Universitaria: Problemas de Fsica

Dinmica del slido rgido. M21.14

14. Una bola esfrica, maciza y homognea, de radio r y masa m, rueda sin resbalar por un plano inclinado un
ngulo T con la horizontal. a) Determinar la fuerza de rozamiento acta sobre la bola, indicando
grficamente su direccin y sentido. b) Calcular coeficiente de rozamiento mnimo que se requiere entre el
plano y la bola pera evitar que resbale.

a) Aplicamos las ecuaciones cardinales del movimiento de la bola, tomando momentos en C


(c.m. de la bola), de modo que:
[1] N  mg cos R  0 l N  mg cos R

[2] mg sen R  f  ma
N

2
2
[3] fr  I B  mr 2B l f  mrB
C
f

5
5
+
[4] a  Br
+

A
que junto con la condicin de rodadura [4] constituyen
mg
un sistema de cuatro ecuaciones con cuatro incgnitas

(N, f, a, ).
Sustituyendo la ec. [4] en la ec. [3] y resolviendo el sistema de ec. [2] y [3], tenemos
[1] N  mg cos R

[2] mg sen R  f  ma

2
7
5
[3] f  2 mrB  2 ma ( ) mg sen R  ma 5 ma  5 ma l a  7 g sen R

5
5

de modo que
2
2
f  ma  mg sen R
5
7
b) El coeficiente de rozamiento mnimo que se requiere ser
2
mg sen R
f
2
 tg R
f b NN l N p  7
N
mg cos R
7

- 187 -

Fsica Universitaria: Problemas de Fsica

Dinmica del slido rgido. M21.15

15. Un carrete est constituido por dos discos de radio R unidos por un eje de radio r,
siendo m la masa e I el momento de inercia del conjunto con respecto a su eje de
simetra de rotacin. Alrededor del eje est enrollada una cuerda o cinta ligera,
sujeta por su otro extremo a un punto fijo. Calcular la tensin de la cuerda y la
aceleracin de carrete cuando desciende por un plano inclinado perfectamente
liso, tal como se indica en la figura.

R
I

En la figura mostramos el esquema de fuerza que actan


sobre el carrete. Aplicando las ecuaciones cardinales de la
N
+
Dinmica del Movimiento Plano del Slido Rgido,
r
R tomando momentos con respecto al eje del carrete y con el
T
convenio de signos indicado en la figura, tenemos:
I

2 mg sen R  T  ma

mg
F  ma


l / N  mg cos R  0
M  I

z Tr  I B

O
Por otra parte, como el punto de aplicacin de la fuerza T se encuentra instantneamente en
reposo, dicho punto constituye el CIR del movimiento en ese instante, por lo que entre la
aceleracin del c.m. y la aceleracin angular establecemos la condicin de rodadura:
aBr
+

En definitiva, tenemos

mg sen R  T  ma l m 2 a  mg sen R
r

N  mg cos R

I
I

Tr  I B  a l T  2 a
r
r

de modo que
a=

1
g sen R
I / mr 2
1

T

- 188 -

I / mr 2
mg sen R
I / mr 2
1

Fsica Universitaria: Problemas de Fsica

Dinmica del slido rgido. M21.16

16. Un disco delgado de 600 mm de dimetro y masa 60 kg se mantiene sobre un plano


inclinado gracias a un bloque y a un cable arrollado en su superficie segn se indica en
la figura. Determinar la tensin del cable y la aceleracin del centro de masa del disco
una vez suprimido el bloque, con lo que el disco podr deslizarse libremente por el
plano inclinado. El coeficiente de rozamiento entre disco y plano vale 0.20.
50

Al suprimir el bloque, el disco inicia un movimiento acelerado partiendo del reposo. El


movimiento del disco es un movimiento plano en el que el centro instantneo de rotacin
(CIR), i.e., el punto de velocidad instantnea nula, se encuentra en la posicin que se indica
en la figura. En estas condiciones, el punto A de contacto del disco con el plano inclinado
desliza sobre ste, por lo que, al tratarse de un contacto rugoso, aparece una fuerza de
rozamiento (f).
Ecuaciones del movimiento:
y

mg sen R  T  f  maG
[1] eje x

[2] eje y
N  mg cos R  0
CIR

[3] < rotacin en G >

G
T R  f R  12 mR 2 B

f


aG

Ecuaciones complementarias:
[4] rozamiento en A

[5] rodadura en el CIR

f  N N

50
aG  B R

Disponemos de cinco ecuaciones con cinco incgnitas (T, f,


aG, N, ). Resolvemos sustituyendo [4] y [2] en [1] y [5] en [3]; obtenemos dos ecuaciones
con dos incgnitas (T, aG):
mg sen R  T  Nmg cos R  maG

T  Nmg cos R  12 maG


Sumando miembro a miembro estas dos ecuaciones obtenemos:
mg (senR  2NcosR )  32 maG l aG  32 (senR  2NcosR ) g

mg

= aG  23 sen50 2q 0.2q cos50 ) g  0.34 g  3.33 m/s


1
T  12 maG NmgcosR  < 13 (sen R  2N cos R ) N cos R > mg  (sen R N cos R ) mg
3
= T  13 (sen 50 0.2q cos 50 )q 60  17.89 kg =175.3 N

- 189 -

Fsica Universitaria: Problemas de Fsica

Dinmica del slido rgido. M21.17

17. Deseamos acelerar una esfera homognea de 2 kg de masa y 5 cm de radio,


situada sobre un plano horizontal rugoso, ejerciendo sobre ella una fuerza
horizontal constante cuya lnea de accin pasa por el centro de la esfera. El
coeficiente de rozamiento esttico entre la esfera y el plano vale 0.3. a) Cul
ser la mxima aceleracin que podemos comunicar a la esfera con tal que
ruede sin resbalar sobre el plano? b) Cul ser la magnitud de la fuerza que
produzca esa aceleracin mxima?

Aplicamos las ecuaciones fundamentales de la dinmica del slido rgido, tomando momentos
en el centro de masa de la esfera, escribimos la ecuacin correspondiente a la rodadura y
resolvemos:

l
F  f  macm

F  f  macm

l
z f R  52 mR B
2

cm

f  5 macm

F
acm  B R (condicin de rodadura)

P
f

De modo que

F  75 macm

f  5 macm

En tanto que la esfera no resbale sobre el plano, tenemos rozamiento esttico entre la esfera y
el plano, de modo que

f d PN

P mg o

2
5

macm

acm d 52 P g

d P mg o F d 72 P mg
f d P mg

Sustituyendo los valores dados en el enunciado:


acm b 5 0.3q9.8  7.35 m/s 2
2

F b 7 0.3q 2  2.1 kg  20.6 N


2

f b 0.3q 2  0.6 kg  5.9 N

- 190 -

Fsica Universitaria: Problemas de Fsica

Dinmica del slido rgido. M21.18

18. Una esfera maciza y homognea, de masa m y radio r resbala sin rodar sobre una
superficie horizontal rugosa bajo la accin de una fuerza F dirigida
horizontalmente y aplicada a una altura h < r, como se indica en la figura.
Determinar la aceleracin de la esfera y el coeficiente de rozamiento entre sta y
el plano.

m,r
F
h

Puesto que en el enunciado del problema nos aseguran que resbala sin rodar, el movimiento
de la esfera consiste en una traslacin pura (no hay rotacin).
Interviene una fuerza de rozamiento cintico en el punto de contacto de la esfera con el plano
sobre el que desliza sin rodar, tal que
f k  Nk mg
m,r
Escribimos las Ecuaciones Cardinales de la Dinmica
C
del movimiento plano del Slido rgido, tomando
momentos en el centro de masa de la esfera:
F
F  f k  macm

fk
h
F r  h
 f k r

De la segunda ecuacin se sigue:


h
h F
f k  1 F  Nk mg l Nk  1
r
r mg
De la primera ecuacin obtenemos la aceleracin de la esfera:
F  f k F h hF
 1 1 
acm 
m
m r mr

- 191 -

Fsica Universitaria: Problemas de Fsica

Dinmica del slido rgido. M21.19

19. Una esfera maciza y homognea, de masa m y radio R rueda (sin resbalar) sobre una superficie horizontal rugosa bajo la accin de una fuerza F dirigida
horizontalmente y aplicada a una altura h < R, como se indica en la figura. Determinar la aceleracin de la esfera y el coeficiente de rozamiento mnimo
entre sta y el plano que impida el resbalamiento.

m,R
F
h

Aplicamos las ecuaciones fundamentales de la dinmica del slido rgido, tomando momentos
en el centro de masa de la esfera (C), y tenemos en
cuenta la condicin de rodadura:
+ +
[1] F  f  ma

F
[2] fR  F R  h
 2 mR 2 B

m,R

h
f
[3] a  B R
I

de modo que disponemos de tres ecuaciones con tres


incgnitas (f, a, ).
Sustituyendo las ecuaciones [3] y [1] en la [2], tenemos:
[2]

fR  F R  h


2
2
Rma  R F  f
l
5
5

5h
f  1 F
7 R

Sustituyendo este resultado en la [1], obtenemos


[1] ma  F  f 

5h
F
7R

a

5h F
7R m

a
5h F
 2
R 7R m
en las direcciones indicadas en la figura.
En las condiciones de rodadura, existir rozamiento esttico entre la esfera y el plano.
Aplicando la definicin del coeficiente de rozamiento esttico, ser
f
f b N N  Nmg l N p
mg
[3] B 

De modo que deber ser


5h F
Nmn  1
7 R mg

Otro mtodo: Como antes, pero tomamos momentos en el CIR (situado en I).
[1] F  f  ma

5h

f  F  ma  1 F

7 R

5h F
f
[2] Fh  7 mR 2 B l
l Np
 1

mg 7 R mg
7R
5h

F
ma
ma
F


l


5h
7R
[3] a  B R

- 192 -

Fsica Universitaria: Problemas de Fsica

Dinmica del slido rgido. M21.20

20. Un disco, de 300 g de masa, 10 cm de radio y de pequeo espesor, pivota (gira)


alrededor de un dimetro vertical sobre el centro de una plataforma circular horizontal
de mayor tamao, 450 g de masa y 20 cm de radio, que puede girar alrededor de su eje
de simetra vertical, comn con el de rotacin del disco pequeo. Cuando el disco
pequeo se encuentra girando a 130 r.p.m., se libera el disco grande para que pueda
empezar a girar. Una vez que, por efecto del rozamiento entre ambos, se igualen sus
velocidades de rotacin, cul ser esa velocidad angular de rotacin comn?
Determnense, tambin, los momentos de inercia necesarios para la resolucin de este
ejercicio.

Z
(1)

(2)

Para determinar el momento de inercia de un disco circular con


respecto a uno de sus dimetros, aplicamos el teorema de los ejes
perpendiculares:
y
1
1
I zz  I xx I yy  2 I dim l I dim  I zz  mR 2
x
2
4
de modo que los momentos de inercia de los discos que
intervienen en este problema, con respecto a sus respectivos ejes de rotacin, son

1
I1  m1 R12
2
2
4
l I 2  2m2 R2  2q 450q 20  12

1
I1
m1 R12
300q102
I 2  m2 R22

2
La rotacin por pivotamiento del disco pequeo sobre el grande transmite un par a este ltimo
y lo acelera; la reaccin de ese par retarda la rotacin del disco pequeo. Finalmente, se
igualan las velocidades angulares a un valor comn . Puesto que no intervienen momentos
exteriores al sistema, se conservar el momento angular del mismo; esto es,
I1
X
130
X

 10 r.p.m.
I1X  I1 I 2
X a l X a 
I2
13
I1 I 2
1
I1
z

- 193 -

Fsica Universitaria: Problemas de Fsica

Dinmica del slido rgido. M21.21

21. En la figura se muestra esquemticamente el tren de aterrizaje de un avin visto desde


atrs. El radio de la rueda es de 40 cm y su momento de inercia es de 2.5 kgm2. El avin
despega a una velocidad de 180 km/h. Despus del despegue, se recoge el tren de aterrizaje girndolo lateralmente a razn de 45 por segundo. Determinar la magnitud del
par ejercido sobre la rueda por su soporte e indicar las direcciones de las magnitudes
vectoriales implicadas.

45/s

Datos
Momento de inercia:

I = 2.5 kgm2

Radio de la rueda:

R = 0.40 m

Velocidad:

v = 180 km/h = 180/3.6 m/s = 50 m/s

Velocidad angular rueda:

Z = v/R = 50/0.40 = 125 rad/s

Momento angular rueda:

L = IZ = 2.5 u 125 = 312.5 kgm2/s

Velocidad angular tren:

: = 45 /s = S/4 rad/s = 0.7854 rad/s

:
Durante la recogida del tren de aterrizaje, el modulo del momento
angular de la rueda permanece constante, pero su direccin cambia, ya
que gira con una velocidad angular :. El par ejercido sobre la rueda
es igual a la variacin de su momento angular por unidad de tiempo;
esto es,
dL
Q
M
 q L M  8 L  q312.5  245.4 N m
dt
4
y la direccin de M es la que se indica en la figura.

- 194 -

M
L

Fsica Universitaria: Problemas de Fsica

Slido Rgido: Trabajo y energa. M22.1

1. La barra homognea de la figura puede girar sin rozamiento alrededor de un

m, L

eje horizontal que pasa por uno de sus extremos. Se coloca en posicin
horizontal y se abandona. Determinar: a) La aceleracin angular de la barra en
el instante en que se deja en libertad. b) La fuerza ejercida por el eje sobre la
barra en ese instante. c) La velocidad del centro de masa de la barra cuando sta alcanza la posicin vertical.

a) Aplicamos la ecuacin fundamental de la dinmica de rotacin, tomando momentos con


respecto al eje de rotacin:
L
1
3g
mg  I B  mL2B l B 
2
3
2L
b) El c.m. de la barra describe una trayectoria circular, de radio L/2, de modo que las
componentes intrnsecas de su aceleracin, en el momento
Ny
inicial, son
+

L 3
v2
= g
an 
0
2 4
L/2
Nx
m, L
Aplicando las ecuaciones del movimiento del c.m, tenemos:
N x  0
N x  man  0

mg

3
1

mg
N
ma
m
g



y
t

N y  mg
4
4

c) Puesto que el sistema es conservativo, y solo estamos interesados en la velocidad final, nos
serviremos del Principio de Conservacin de la Energa para

Ep=0 determinar la velocidad angular de la barra en el instante pedido:
L 1
mgL
mgL
3g


0  mg I X 2 l X 2 
L/2
1
I
L
2
2
2
vcm
mL

at  B

y la velocidad de su centro de masa, en ese instante, ser:


vcm  X

- 195 -

L L 3g 1


3gL
2
2 2 L

Fsica Universitaria: Problemas de Fsica

Slido Rgido: Trabajo y energa. M22.2

2. Una barra de longitud L y masa m puede girar alrededor de un eje fijo que

m, L

pasa por uno de sus extremos O. Dejamos caer la barra desde la posicin
horizontal (T = 0), partiendo del reposo. Determinar: a) La aceleracin y
velocidad angulares de la barra en funcin del ngulo T . b) La reaccin
en O cuando T = /2.

1
Momento de inercia de la barra respecto de O: I O  ml 2
3
a) Conservacin de la energa:

l
1 1
3g
0  mg ( sen R ) ml 2 R 2 l R 2 
sen R

l
2
2 3

m, L

Ep=0

l/2

A partir de la velocidad angular, por derivacin determil/2


mg
namos la aceleracin angular:
3g
  3g R cos R l R  3g cos R
sen R l 2RR
R 2 
l
l
2l
g
Q
3
2
b) Para R 
l R 
R  0
2
l
O Ry
Escribimos la ecuacin del movimiento del centro de masa:

l
Rx
ax  R  0
l Rx  max
2

F  ma cm
l con
l/2
Ry  mg  ma y

3g l 3
2 l
 g
a y  R 
l 2 2
2

v
de modo que
mg
Rx  0

Ry  mg ma y  mg 3 mg  5 mg

2
2

- 196 -

Fsica Universitaria: Problemas de Fsica

Slido Rgido: Trabajo y energa. M22.3

3. Una varilla homognea est apoyada en posicin vertical sobre uno de sus extremos en
contacto con un plano horizontal, de modo que inicialmente se encuentra en equilibrio
inestable. La desplazamos ligeramente de esa posicin para que comience a caer. Calcular
su velocidad angular cuando alcanza la posicin horizontal sobre el plano horizontal:
a) Suponiendo que la varilla no resbale sobre el plano. b) Suponiendo que el plano sea
perfectamente liso.

m,l

a) La varilla presenta una rotacin pura alrededor de su extremo


inferior en E. Aplicamos el principio de Conservacin de la Energa a
los instantes inicial y final:

l 1
1 1
mg  I E X 2  ml 2 X 2
[1]

2 2
2 3

m,l
G

De modo que
3g
l
b) Como no actan fuerzas que den componente en la direccin
horizontal, el centro de masas de la varilla no puede desplazarse en esa
direccin, por lo que tan solo desciende verticalmente. El movimiento
de la varilla consiste en una traslacin (vertical) del c.m. simultnea
con una rotacin alrededor de un eje horizontal que pasa por el c.m.
(G).
Aplicamos de nuevo el principio de Conservacin de la Energa a los

X

E


vG

instantes inicial y final:


l 1 2 1
 mvG I G X 2
[2]
2 2
2
Puesto que el punto E tan solo se desplaza horizontalmente, podemos
determinar la velocidad del c.m. (G) localizando previamente el C.I.R. (I)
correspondiente a la posicin final de la varilla, tal como se indica en la
figura, de modo que coincide con el extremo inferior (E) de la varilla, de
modo que vG  X l / 2
. Sustituimos este valor, as como el valor del
mg

m,l
G

I

momento de inercia, en la expresin de la conservacin de la energa, para


obtener
1 1

l 1 1
mg  m l 2 X 2 ml 2 X 2
[3]

2 12

2 2 4

3g
l
Obsrvese que el resultado es el mismo que en los dos casos. Se debe a
que, en el segundo caso, en el instante final el movimiento consiste en una
rotacin pura en el CIR=E, de modo que hubiramos podido escribir
directamente la misma ecuacin [1] en este segundo apartado.

X

- 197 -

De donde resulta:

G
I

Fsica Universitaria: Problemas de Fsica

Slido Rgido: Trabajo y energa. M22.4

4. Un cubo homogneo est apoyado sobre una de sus aristas en contacto con un plano
horizontal, de modo que inicialmente se encuentra en equilibrio inestable. Lo desplazamos
ligeramente de esa posicin para que comience a caer. Calcular su velocidad angular
cuando una de sus caras choca con el plano horizontal: a) Suponiendo que la arista no
resbale sobre el plano. b) Suponiendo que el plano sea perfectamente liso.

2
2
a) El cubo presenta una rotacin pura alrededor de su arista en E.
Aplicamos el principio de Conservacin de la Energa.
Determinamos la distancia EG  l

l
G

mgl

2
l 1
 mg I E X 2
2
2 2

Con

l 2
1

2
1
1
2
 ml 2 ml 2  ml 2
I E  I G m EG  2 ml 2 m
12

2
6
2
3
2

De modo que

3 2 1 g
l 12 2 2
g
2
 mg
ml X l X 2 
 0.62
l
2
2 23
2l

b) Como no actan fuerzas que den componente en la direccin
horizontal, el centro de masas del cubo no puede desplazarse en esa
direccin, por lo que tan solo desciende verticalmente. El movimiento del cubo consiste en
una traslacin (vertical) del c.m. y una rotacin alrededor de un eje horizontal que pasa por el
c.m. (G).
Aplicamos de nuevo el principio de conservacin de la energa:

mgl

l
G

E
CIR

l
G

2
l 1
1
 mg mvG2 I G X 2
2
2 2
2
Puesto que el punto E tan solo se desplaza horizontalmente, podemos
determinar la velocidad del c.m. (G) localizando previamente el C.I.R.
correspondiente a la posicin final del cubo, tal como se indica en la
figura, de modo que vG  Xl / 2 , que sustituimos en la expresin de la
conservacin de la energa para obtener

2
l 1 l 2 1 1
mgl
 mg m X 2 ml 2 X 2

2
2 2 4 2 6
mgl

De modo que
X2 

12

2 1 g
5l

 0.99

g
l

- 198 -

Fsica Universitaria: Problemas de Fsica

Slido Rgido: Trabajo y energa. M22.5

5. Dadas dos esferas, una maciza y la otra hueca, describir detalladamente un experimento que, sin daar las
esferas, nos permita averiguar cual es la maciza y cual la hueca. Hacer los clculos necesarios para justificar
los resultados del experimento.

Si dejamos rodar las dos esferas por un plano inclinado, abandonndolas simultneamente
partiendo del reposo, la que llegue antes al pie del plano ser la esfera maciza, ya que
presenta menos inercia a la rotacin que la esfera hueca.
Mtodo de la energa. El Principio de Conservacin de la Energa nos permite determinar las
velocidades que alcanzan las esferas cuando llegan al pie del plano inclinado, en el supuesto
de que haya rozamiento suficiente para que se produzca la rodadura (sin resbalar):
1
1
1
1 v2
1
I
mgh  mv 2 I X 2  mv 2 I 2  m 2 v 2

2
2
2
2 R
2
R

2mgh
2 gh

m I 2 1 I
R
mR 2
donde hemos tenido en cuenta la condicin de rodadura, v
N
= R. Puesto que la expresin del momento de inercia es
f
de la forma kmR2, resulta que la velocidad es
2 gh
h
v2 
mg
1 k

de modo que es independiente de la masa y del radio, pero
depende del momento de inercia, siendo tanto mayor cuanto menor sea en momento de
inercia. Por tanto, la esfera que llega antes al pie del plano es la esfera maciza.

2 gh 10

2
2

vesf.
 gh  1.43 gh
I esf. maciza  mR 2
maciza 

1 52
7


vesf. maciza  vesf. hueca

2
2 gh 6
2
v 2
1.20
gh
gh



I esf. hueca  mR
esf. hueca

3
1 23 5

Mtodo dinmico. Podemos demostrar que la aceleracin que adquiere cada una de las
esferas, en el supuesto de que haya rozamiento suficiente para que se produzca la rodadura
(sin resbalar), tan solo depende del momento de inercia de las misma, i.e., de la distribucin
de la masa en las mismas.
mg sen R  f  ma

g sen R
I

fR  I B
I mg sen R  m 2 a l a 
I

R
f  2 a
1

a  B R (condicin de rodadura)
mR 2
Resulta que la aceleracin es independiente de la masa y del radio, dependiendo tan solo del
ngulo de pendiente y del momento de inercia. As, conocidos los momentos de inercia
implicados, tenemos las aceleraciones respectivas:
g sen R 5
aesf. maciza 
 g sen R  0.71g sen R
1 52
7
aesf. hueca 

l v2 

g sen R 3
 g sen R  0.60 g sen R
1 23
5

de modo que la esfera maciza se acelera ms que la hueca.

- 199 -

Fsica Universitaria: Problemas de Fsica

Slido Rgido: Trabajo y energa. M22.6

6. Un rodillo macizo, de seccin circular, de radio r y masa m, descansa

m,r

sobre un borde horizontal de un escaln y empieza a rodar hacia fuera,


sin resbalar, con velocidad inicial despreciable. Calcular el ngulo que
girar el rodillo antes de que pierda contacto con el borde del escaln, as
como su velocidad angular en ese instante.

En tanto que no pierda contacto con el borde del escaln, el movimiento del rodillo es una
rotacin pura alrededor de un eje que coincide con dicho borde.
Conservacin de la energa: (nivel de referencia a la
altura del escaln)
T

m,r

mgr  mgr cos R 12 23 mr 2


X 2
mg (1 cos R )  34 mr X 2

f
N
mg

rX 2  43 g (1 cos R )
Movimiento radial del centro de masas:
N  mg cos R  man  mr X 2

Combinamos las dos ecuaciones anteriores para obtener:


N  mg cos R  43 mg (1 cos R )  mg cos R  43 43 cos R
 mg 73 cos R  34

de modo que
N  13 mg (7 cos R  4)
El rodillo pierde contacto con el borde del escaln cuando N=0, ya que entonces desaparece la
ligadura, lo que equivale a

7 cos R  4  0 cos R  4 7

R  55.2

La velocidad angular del rodillo en ese instante ser:


X2 

4
4
4
4g
g (1 cos R )  g (1 ) 
3r
3r
7
7r

- 200 -

X

4g
7r

Fsica Universitaria: Problemas de Fsica

Slido Rgido: Trabajo y energa. M22.7


R

7. En el interior de un semicilindro hueco de radio R, una esfera de radio r y masa m,


parte del reposo desde la posicin A. Calcular la velocidad y aceleracin de la
esfera cuando pasa por el punto ms bajo, suponiendo que en todo instante rueda
sin deslizar.

m, r
A

a) Para determinar la velocidad del c.m. de la esfera, aplicamos el Principio de conservacin


de la energa,
m, r
1
1
Ep =0
0  mg R  r
mv 2 I cm X 2
2
2
R
Con la condicin de rodadura v  X r obtenemos

v
1
1 2
7
mg R  r
 mv 2 mr 2  mv 2
r
2
2 5
10
2


v

de modo que
I
10
R  r
g
7
b) Cuando la esfera pasa por la posicin ms baja, que corresponde a la posicin de
equilibrio (energa potencial mnima), su velocidad es mxima y la aceleracin tangencial de
su c.m. es nula.
El c.m. de la esfera tan solo presenta aceleracin normal o centrpeta, dirigida hacia el centro
de la trayectoria que describe, de radio (R r) y
R
cuyo mdulo es

v

v2
10
 g
7
Rr
Y la reaccin normal (N) ser:
10
17
N  mg  mg l N  mg
7
7
an 

N
an
I
mg

- 201 -

Fsica Universitaria: Problemas de Fsica

Slido Rgido: Trabajo y energa. M22.8

8. Una bola hueca, de espesor despreciable, masa m y radio r, se lanza rodando


por un plano horizontal con velocidad de su centro v0 y recorre un bucle
situado en un plano vertical, siempre rodando, de radio R, del cual se despega
cuando forma un ngulo  con la horizontal, tal como se indica en la figura.
Hllese: a) El momento de inercia de la bola con respecto a un eje diametral.
b) La velocidad v0 que debe de tener para que se despegue en .

m,r
v0

a) Puesto que toda la masa de la bola se encuentra situada a la misma distancia de su centro,
el momento de inercia con respecto a este es I O  mr 2 . Entonces, aplicando el teorema que
nos dice que la suma de los momentos con respecto a tres ejes perpendiculares que se
interceptan en un punto es igual al doble del momento de inercia con respecto a dicho punto,
tenemos
2
I xx I yy I zz  2 I O l 3I D  2 I O l I D  I  mr 2
3
b) Puesto que la bola rueda sin deslizar en todo momento, no hay disipacin de energa por
rozamiento, de modo que la energa total de la bola permanece constante. Podemos escribir
1 2 1 2
1
1
mv0 I X0  mgR 1 sen R
mv 2 I X 2
2
2
2
2
con v0  r X0 y v  rX , de modo que nos queda
1 2 12 2 2
1
12 2 2
mv0
mr X0  mgR 1 sen R
mv 2
mr X l
2
23
2
23
6
5 2
5
mv0  mgR 1 sen R
mv 2 l v 2  v02 gR 1 sen R

6
5
6

N
T

Escribimos la componente radial de la ec. del movimiento del centro de


masa de la bola:

v2 6
v2
 m 0 mg 1 sen R

R
R 5
En el momento crtico en el que la bola se despega de la pista, ser N = 0, de modo que
N mg cos R  macp  m

g cos R 

v02 6
6
g 1 sen R
l v02  gR cos R  1 sen R

R 5
5

- 202 -

mg

Fsica Universitaria: Problemas de Fsica

Slido Rgido: Trabajo y energa. M22.9

9. Los ejes de dos cilindros de masas m1 y m2, y radios r1 y r2

m1, r1

m2, r2
respectivamente, estn unidos por una cinta inextensible que
pasa a travs de una polea de masa despreciable. Los cilindros
ruedan sin deslizar sobre sendos planos, inclinados un ngulo


. a) Calcular las aceleraciones de ambos cilindros. b) Si el
sistema parte del reposo, determinar la energa cintica de ambos cilindros tras recorrer 1 m sobre los
planos.
Datos: m1 = 1 kg, m2 = 1.5 kg, r1 = 15 cm y r2 = 20 cm,  = 30.

a) Escribimos las ecuaciones del movimiento de rotacin para cada uno de los cilindros,
tomando momentos con respecto a las generatrices de contacto con los planos (E.I.R.) para
cada uno de los cilindros:
3
3

2
Tr1  (m1 g sen R ) r1  2 m1r1 B1
T  m1 g sen R  2 m1a
l

3
3
2
(m2 g sen R ) r2  Tr2  m2 r2 B2
m2 g sen R  T  m2 a
2
2

donde hemos tenido en cuenta que, por ser la


+
cinta
inextensible,
las
velocidades
y
+
aceleraciones de ambos cilindros son idnticas
T
T
N1
N2
en cada instante. Tambin hemos considerado la
condicin de rodadura para cada uno de los
f2
f1
cilindros.


Sumando m.a.m. estas dos ltimas expresiones,
m
g
m
g
1
2
se tiene
2(m2  m1 )
3
g sen R
(m2  m1 ) g sen R  (m1 m2 )a l a 
2
3(m1 m2 )

y sustituyendo los datos

a

2(1.5 1.0)
9.8sen 30  0.653 m/s
3(1.0 1.5)

y las aceleraciones angulares de los cilindros son


a 0.653
a 0.653
B1  
 4.35 rad/s 2
B2  
 3.27 rad/s 2
r1
r2
0.15
0.20
b) Puesto que la aceleracin es constante, calculamos las velocidades de cada cilindro a partir
de la expresin v 2  v02 2as :
v1  v2  v  2as  2q 0.653q1  1.14 m/s

La energa cintica de cada cilindro viene dada por la expresin:

1
1 1
1
1
3
Ek  mv 2 mr 2 X 2  mv 2 mv 2  mv 2

2
22
2
4
4
y la energa cintica que adquiere cada uno de los cilindros es:
3
3
Ek,1  1.0q1.142  0.98 J
Ek,2  1.5q1.142  1.47 J
4
4

- 203 -

Fsica Universitaria: Problemas de Fsica

Slido Rgido: Trabajo y energa. M22.10

10. La rueda representada en la figura consiste en un semicrculo de madera que pesa

100 N contenido en un aro circular de acero de 45cm de dimetro y peso y grosor


despreciables. Si rueda sin deslizamiento por un piso horizontal y tiene una
velocidad angular de 15 rad/s en sentido horario cuando su centro de masa G se
halla directamente debajo del centro C de la rueda. a) Determinar la velocidad
angular de la rueda cuando G se halle directamente a la izquierda de C.
b) Calcular las componentes normal y de rozamiento de la fuerza que el suelo
ejerce sobre la rueda cuando G se halla directamente a la izquierda de C.

Determinacin del c.m. de un semicrculo:


2 teorema de Pappus: V=2SycmS

1 V
1 43 Q R 3 4 R


l
2Q S 2Q 12 Q R 2 3Q
4 R 4q 0.225

 0.0955 m
E
3Q
3q Q
Clculo del momento de inercia:
IC=mR2=0.258 kg.m2
IG=IC-mG2= 0.165 kg.m2
IO=IG-m(R-G) 2=0.336 kg.m2
IP=IG+m(R2+G2)=0.774 kg.m2
a) Conservacin de la energa:

l XP2 

ycm 

mg E 12 I O XO2  12 I P XP2

G
C

G
P

I O XO2  2mg E
IP

0.336q152  2q100q 0.0955


 73.0035 (rad/s) 2
0.774
l XP  8.54 rad/s

Xp2 

b) Ecuaciones del movimiento:


 f  max
ax  B R EX 2

N  mg  ma y con
a y  BE

fR  E N  I G B
ya que

mg
f

N
P


JJJG
JJJG B R 0 E
E B R X E
2
2

aG  aC  q CG  X CG  0 0 q 0  X 0  BE

0
0 B 0

De modo que
 f  m(B R X 2E )

N  mg  mBE

fR  E N  I G B

- 204 -

f  6.08 N
N  72.4 N
B = - 33.6 rad/s 2

Fsica Universitaria: Problemas de Fsica

Slido Rgido: Trabajo y energa. M22.11

11. En el sistema representado en la figura, el rodillo rueda sin resbalar

m,R

sobre el suelo horizontal rugoso y los dos muelles tienen la misma


constante elstica k. a) Determinar la frecuencia de las oscilaciones del
sistema. b) Si separamos el rodillo una distancia A de su posicin de
equilibrio y lo abandonamos partiendo del reposo, calcular la velocidad
mxima que adquiere.

a) Mtodo de la energa
Consideremos un desplazamiento genrico del sistema a partir de su posicin de equilibrio
(elongacin, x):

1
1 2
1 2
1 1
1 2
3
E  mvcm
mR 2 X 2 2 kx 2  mvcm
mvcm
kx 2  mx 2 kx 2

2
2
2
2 2
4
4

ya que vcm  X R (rodadura), con vcm  x .


Como el sistema es conservativo, la energa total permanece constante, de modo que,
derivando con respecto del tiempo, tenemos:
3

dE 3
4k
 2kxx  x mx 2kx  0 l x
x0
 mxx

2
dt
2
3m
que es la ecuacin diferencial de un m.a.s. cuya frecuencia angular es
4k
1 4k
l O
3m
2Q 3m
b) La velocidad mxima se presenta cuando el rodillo pasa por su posicin de equilibrio y
podemos calcularla a partir de la frecuencia angular y de la amplitud:
Xfrec 

4k
3m
Al mismo resultado llegaremos a partir de la conservacin de la energa:
1
1 2
2
1 1
2 kA2  mvmax
mR 2 Xmax
l
2

22
vmax  Xfrec A  A

1 2
1 2
3 2
mvmax
 mvma
kA2  mvmax
x
2
4
4

- 205 -

2
l vmax


4k 2
A
3m

Fsica Universitaria: Problemas de Fsica

Slido Rgido: Trabajo y energa. M22.12

12. En el dispositivo que se ilustra en la figura, constituido por un muelle ideal de


constante k, una polea de radio R y momento de inercia I y una pesa de masa m,
determinar la frecuencia de las oscilaciones de la pesa y escribir la expresin de su
elongacin en funcin del tiempo.

I, R

m
Mtodo de la energa: Puesto que el sistema es conservativo, la energa k
total del mismo permanece constante, de modo que, para una elongacin x
x
genrica, podemos escribir:
1
1
1
2
E  mx 2 I R 2 k x0 x
 mgx  cte.
2
2
2
En el supuesto de que la cuerda no resbale sobre la polea, podemos establecer la siguiente
condicin de ligadura:
x  RR

que, sustituida en la expresin de la energa, nos permite expresar la energa en funcin de la


elongacin x y de sus derivada temporal:
1
1 I 2 1 2 1 2
E  mx 2
x kx0 kx kx0  mg
x  cte.
2
2 R2
2
2
Derivamos esta expresin con respecto al tiempo:
dE
I
I
 2 xx
 kxx kx0  mg
x  0 l mx 2 x kx kx0  mg  0 l
 mxx
dt
R
R

m I x kx  mg  kx0
2

R
que es la ecuacin diferencial correspondiente a un movimiento armnico simple con
mg
x0 
I
k
m 2
R
La expresin de la elongacin en funcin del tiempo es
X

I, R
T2

x  x0 A sen Xt G

T1
T2

T1

donde la amplitud de las oscilaciones (A) y la fase inicial (I) se k


x
determinan a partir de las condiciones iniciales.
m
Mtodo de las fuerzas: Para una elongacin genrica, escribimos las
mg
ecuaciones del movimiento para cada uno de los tres elementos
T
2
implicados (bloque, polea y muelle):
mg  T1  mx

mg  T1  mx

T1 R  T2 R  I R
I

l T1  T2  I / R 2
x l mg  kx0  m 2 x kx

T2  kx

T  k x x

0
2
x  RR (condicin rodadura)
que es la misma ec. dif. de un m.a.s. obtenida por el mtodo de la energa.

- 206 -

Fsica Universitaria: Problemas de Fsica

Slido Rgido: Trabajo y energa. M22.13

13. El cilindro macizo y homogneo que se muestra en la figura, de masa m y radio R, est
suspendido del techo mediante una cuerda ligera. Uno de los extremos de la cuerda
est unido directamente al techo; el otro lo est a un muelle de constante elstica k. Determinar la frecuencia de las oscilaciones del sistema si la cuerda no resbala sobre el
cilindro.

Mtodo de la energa: Puesto que el sistema es conservativo, la energa


total del mismo permanece constante, de modo que, para una elongacin x
genrica, podemos escribir:

1
1 1
1
2
E  mx 2 mR 2 R 2 k 2 x
 mgx  cte.

2
2 2
2

k
m,R

En el supuesto de que la cuerda no resbale sobre la polea, podemos establecer la siguiente


condicin de ligadura:
x  RR
que, sustituida en la expresin de la energa, nos permite expresar la energa en funcin de la
elongacin x y de sus derivada temporal:
1
1
3
E  mx 2 mx 2 2kx 2  mgx  mx 2 2kx 2  mgx  cte.
2
4
4
Derivamos esta expresin con respecto al tiempo:
3

dE 3
 4kxx  mgx  0 l x mx 4kx  mg  0
 mxx

dt
2
2
3
8k
2
l
mx 4kx  mg l x
x g
2
3m
3
que es la ecuacin diferencial correspondiente a un movimiento armnico simple con
8h
2g / 3
mg
y x0 

8k / 3m 4k
3m
Mtodo de las fuerzas: Para una elongacin genrica, escribimos las
ecuaciones del movimiento para cada uno de los tres elementos
implicados (polea y muelle):
k
mg  F1  F2  mx
F2

m,R
mg  2kx  F2  mx
F R F R  1 mR 2R
F1
2
1
2
l

2kx F2  1 mx
F1  k 2 x
 2kx


x
x  RR (condicin rodadura)

mg
3
8k
2
x g
l mg  4kx  mx l x
2
3m
3
que es la misma ec. dif. de un m.a.s. obtenida por el mtodo de la energa.
X

- 207 -

Fsica Universitaria: Problemas de Fsica

Slido Rgido: Trabajo y energa. M22.14

14. En el dispositivo que se muestra en la figura, el collarn ligero por el que pasa la
varilla y al que estn unidos dos muelles idnticos, permite que stos permanezcan
horizontales. Los muelles trabajan a extensin-compresin y poseen su longitud
natural cuando la varilla est en posicin vertical. Determinar la frecuencia de las
pequeas oscilaciones de la varilla.

m,l

k
k
Mtodo dinmico
Cuando desplazamos la barra de su posicin de equilibrio, aparecen las fuerzas recuperadoras
en las direcciones que se indican en la figura. Tomamos momentos en A y obtenemos:
1

l
A}
con x  h tg R
 2kxh  mg sen R  ml 2 R

2
Para oscilaciones pequeas es sen R x tg R x R de modo que
1
1
2kh 2R  mglR  ml 2R
2
3
6kh 2 3 g
R 2 R  0
ml
2l

T mg

y la frecuencia angular de las pequeas oscilaciones es

kx

6kh 2 3g
X

ml 2
2l

kx
x

Mtodo de conservacin de la energa


Expresamos la energa total del sistema para una elongacin arbitraria:
1
l
E  13 ml 2
R 2  mg cos R 2 12 kx 2 con x  h tg R
2
2
1 2 2 1
E  ml R  mgl cos R kh 2 tg 2R  cte.
6
2
y derivamos con respecto al tiempo:
dE 1 2  1
2kh 2 tg R 
 ml RR mglR sen R
R0
dt
3
2
cos 2 R
Para pequeas oscilaciones es sen R x tg R x R

y cos 2 R x 1 , de modo que

1 2  1
1 2  1
ml R mglR 2kh 2R  0 l
ml R mgl 2kh 2 R  0
2

3
2
3

que es la ecuacin diferencial de un m.a.s. con


1
mgl 2kh 2
3 g 6kh 2
2
2
X 


1 2
2l
ml 2
ml
3

- 208 -

Fsica Universitaria: Problemas de Fsica

Slido Rgido: Trabajo y energa. M22.15

15. Un pndulo compuesto est formado por una varilla de masa despreciable y longitud
4R, que est unida a una esfera de masa m y radio R. En el punto medio de la varilla se
conecta un muelle ideal, con constante elstica k, en posicin horizontal, como se
muestra en la figura, que est relajado cuando la varilla est en posicin vertical.
Determinar la frecuencia de las pequeas oscilaciones del pndulo en el plano de la
figura.

2R
k

2R

2R

2R


F

5Rcos

Mediante el teorema de Steiner, calculamos el momento de inercia de la esfera con respecto al


eje de rotacin en O:
2
127
I O  mR 2 m(5R ) 2 
mR 2
5
5
Mtodo de Newton. Consideramos una posicin genrica, caracterizada por un
desplazamiento angular T del pndulo, y planteamos la ecuacin fundamental de la rotacin
tomando momentos con respecto al eje de rotacin:
O
Ep=0
M O  mg (5 R sen R )  k (2 R sen R )(2 R cos R ) 
+
 5mgR sen R  4kR 2 sen R cos R  I R

2R
2Rsen

que, en la aproximacin de pequeas oscilaciones, con


sen R x R y cosR  1 , se reduce a
127
mR 2R (5mgR 4kR 2 )R  0 l
5
25mg 20kR

R
R0
127 mR
que es la ecuacin diferencial de un m.a.s. en el que la
frecuencia angular vale

mg

X

25mg 20kR

127 mR
Mtodo de la energa. Expresamos la energa total del pndulo (cintica + potencial) en
funcin del ngulo :
1
1
127
E  I OR 2  mg (5 R cos R ) k (2 R sen R ) 2 
mR 2R 2  5mgR cos R 2kR 2 sen 2 R  cte
2
2
10
como la energa permanece constante, su derivada temporal ser nula, lo que nos conduce a
dE 127
 5mgRR sen R 2kR 2R sen R cos R 

mR 2RR
dt
5
127

 R
mR 2R 5mgR sen R 2kR 2 sen R cos R  0
5

que, en la aproximacin de pequeas oscilaciones, con sen R x R y cosR  1 , se reduce a

25mg 20kR

127
R0
mR 2R (5mgR 4kR 2 )R  0 l R
5
127 mR
que es la ecuacin diferencial de un m.a.s. cuya frecuencia angular vale
X

25mg 20kR

127 mR

- 209 -

Fsica Universitaria: Problemas de Fsica

Slido Rgido: Trabajo y energa. M22.16

16. En un tubo cilndrico horizontal hay dos discos, de masa m, unidos por un hilo

L
que los mantiene a una distancia l uno del otro, y a distancia l/2 del centro del
tubo O. Este dispositivo est girando libremente alrededor de un eje perpenl
dicular al tubo que pasa por O, con una velocidad angular Z. En un cierto
instante se rompe el hilo que une ambos discos (dejndolos libres). El
O
rozamiento entre los discos y el tubo se considera despreciable, el momento de
inercia del tubo respecto al eje de giro es It = ma2 (donde a es un parmetro con
dimensiones de longitud) y el momento de inercia de cualquiera de los discos
Z
respecto a un eje paralelo al de giro que pasa por el centro de masa del disco es
2
Id = (ma )/2. a) Hllese la velocidad angular de giro del dispositivo descrito
cuando ambas masas llegan a sus sendos extremos del tubo. b) Con qu velocidad se aleja del eje en el
momento que abandonan el tubo?

l
1
1
1
I d  ma 2 m  ma 2 ml 2
2
2
2
4
2

I t  ma 2

1
l I  I t 2 I d  2ma 2 ml 2
2

a) Por no existir momento externo, el momento angular del sistema se conserva:


L  cte. l I i Xi  I f Xf

1
2ma 2 ml 2
Ii
4a 2 l 2
2
= X f  Xi 
X 2
X
1
If
4a L2
2ma 2 mL2
2
b) Por no existir fuerzas disipativas (rozamiento) se conserva la energa mecnica;
concretamente, se conserva la energa cintica del sistema:
1
2

I i Xi2  12 I f Xf2 2 12 mvr2 l 2mvr2  I i Xi2  I f Xf2 l

2mvr2  I i Xi2  I f

I i2 2 I i 2
1 I i 2
Xi  1 I i Xi l vr2 
1 I i Xi
2
I f
If
2m I f

de modo que sustituyendo los valores de Ii e If y operando se obtiene

1 I i 2
1 4a 2 L2  4a 2  l 2
1
2ma 2 ml 2 Xi2 
vr2 
1 I i Xi 

2
2

2m I f
2m
4a L
2

1 L2  l 2 2 2 2
4a l
Xi
 2
4 4a L2
De modo que
vr2 

1 2 2 4a 2 l 2 2
L  l
4a 2 L2 X
4

- 210 -

Fsica Universitaria: Problemas de Fsica

Slido Rgido: Trabajo y energa. M22.17

17. Determinar la aceleracin angular que adquieren los tres cilindros idnticos de
la figura cuando sobre uno de ellos se ejerce un par M. Los mencionados
cilindros pueden girar alrededor de sus respectivos ejes, sin que exista
deslizamiento en los contactos entre diferentes cilindros.

En la figura hemos representado las fuerzas de rozamiento entre lo cilindros. Por se idnticos
y no existir deslizamiento entre ellos, las aceleraciones angulares de los tres cilindros tendrn

D
f21

f23
f32

f12

el mismo mdulo y las direcciones indicadas en la figura.


Aplicamos la ecucin fundamental de la dinmica de la rotacin alrededor de un eje fijo a
cada uno de los cilindros:
Cilindro 1:
z
M  r f12  I B
Cilindro 2:

r f12  r f 23  I B

Cilindro 3:

r f 23  I B

Sumando las tres ecuaciones resulta


M  3I B l B 

M
M
2M
 1 2
3I 3 2 mr
3mr 2

siendo m la masa y r el radio de cada cilindro.

- 211 -

Fsica Universitaria: Problemas de Fsica

Slido Rgido: Trabajo y energa. M22.18

18. Un cilindro de masa total m que tiene una mitad de densidad doble que la otra se
abandona con velocidad nula sobre un plano sin rozamiento en la posicin indicada en
la figura. Determinar la velocidad y aceleracin angulares en el instante inicial y en el
instante en que el dimetro AA sea horizontal.

Clculo del centro de masa


Semicilindro macizo: Aplicamos el 2 Teorema de Pappus.
Vesf  2QE1
Ssemicirculo

4 3
QR2
Q R  2QE1
3
2

E1 

4R
3Q

Cilindro completo:
E

m1E1 m2E2 2m2E1  m2E1 E1




m1 m2
2m2 m2
3

E

4R
9Q

O GG
mg
N

x
Clculo de momentos de inercia
Momentos de inercia con respecto a los ejes longitudinales en
E
los puntos O y G (centro de masa)
1
1
2
I O  mR 2 I G  mR 2  mE 2
2
2
Estudio dinmico
O
Puesto que no existe rozamiento, se produce resbalamiento en
G
Z2
los puntos de contacto del cilindro con el plano. Siendo a la
aceleracin del centro de masa y aplicando la 2 Ley de Newton
0  ma a  0
E
F ma N  mgx  ma x
y

Por ser siempre nula la componente horizontal de la aceleracin, el centro


de masa no adquiere velocidad en la direccin horizontal; i.e., tan solo cae verticalmente.
Posicin 1: Obviamente es Z1 =0 (condicin inicial). Tomamos momentos en el punto O (ya
que su aceleracin es horizontal y, en consecuencia, est dirigida hacia el c.m.)
mg E  I OB1

B1 

mg E
8g

IO
9Q R

1
1
0  mg E mvG2 2 I G X22
2
2
pero vG2=0 (su movimiento es vertical y ha alcanzado el punto ms bajo de su trayectoria).
Por consiguiente

Posicin 2: Principio de conservacin de la energa:

mg E 

1
I G X22
2

X22 

Tomando momentos en el centro de masa (punto G):

- 212 -

2mg E
IG

0  I G B2

B2  0

Fsica Universitaria: Problemas de Fsica

Dinmica impulsiva del slido rgido. M24.1

1. Un slido rgido, de masa m, est suspendido de un eje horizontal situado a una distancia h
de su centro de masa, como se indica en la figura. Sabemos que cuando aplicamos una
percusin , horizontal y perpendicular al eje, cuya lnea de accin pasa a una distancia 2h
de eje de suspensin, dicho eje absorbe la mitad de dicha percusin. Determinar el
momento de inercia del slido con respecto al eje de suspensin.

h
G

Escribimos la Ecuaciones Cardinales de la Dinmica Impulsiva (movimiento plano) del


Slido Rgido, tomando momentos en el eje de suspensin:
12 1  mvG
1  1 a  mvG

1
1a 
l

2h1  I X

2
2h1  I X
Dividiendo miembro a miembro estas dos ecuaciones, tenemos
4hmvG
IX
O
4h 
l I

mvG
X

La velocidad del punto G, en la rotacin pura  alrededor del eje de


suspensin, viene dada por
vG  hX
De modo que sustituyendo esta expresin en la del momento de
inercia, nos queda
4hmX h
2
I
 4mh 2  m 2h

Si en el enunciado se omitiera la palabra absorbe en dicho eje absorbe la mitad de dicha percusin, el
planteamiento y el resultado sera distinto.

Escribimos la Ecuaciones Cardinales de la Dinmica Impulsiva (movimiento plano) del


Slido Rgido, tomando momentos en el eje de suspensin:
1 1 a  mvG
32 1  mvG
1
l
1a 

2h1  I X

2
2h1  I X
Dividiendo miembro a miembro estas dos ecuaciones, tenemos
4hmvG
4
IX
h
l I
3
mvG
3X

La velocidad del punto G, en la rotacin pura  alrededor del eje de


suspensin, viene dada por
vG  hX
De modo que sustituyendo esta expresin en la del momento de
inercia, nos queda
4hmX h 4 2 1
4
2
I
 mh  m 2h
 mh 2
3X
3
3
3

- 213 -

h
P

Fsica Universitaria: Problemas de Fsica

Dinmica impulsiva del slido rgido. M24.2

2. Una varilla uniforme de longitud l y masa m cuelga verticalmente y est sujeta por una articulacin en su
extremo superior. Golpeamos la varilla en su extremo inferior aplicando una fuerza horizontal F que dura un
tiempo muy pequeo t. a) Determinar momento angular que adquiere la varilla respecto al extremo
superior. b) Qu percusin habra que dar en la forma indicada para que la varilla llegar a alcanzar una
posicin vertical hacia arriba? c) Siendo l = 1 m; m = 2.5 kg; F = 100 N y t = 1/50 s, averiguar si la varilla
alcanzar la posicin vertical hacia arriba.

a) La percusin que recibe la varilla viene dada por


1 = F t

De las leyes de la dinmica impulsiva se sigue:


l1  L  L l L  l1  lF t
b) Expresamos la energa cintica que adquiere la varilla
inmediatamente despus de la percusin en funcin de la magnitud de
sta:

N.R.
1 2 L2 3l 21 2 31 2


Ek  I X 
2
2I
2ml 2
2m
m,l
ya que entre el momento angular y la velocidad angular existe la
relacin:
=Ft
L
1 2
L  IX l X 
con I  ml
I
3
La energa (cintica + potencial gravitatoria) se conserva en el movimiento de la varilla
posterior a la percusin:
l 31 2
l
31 2
mg
 mg
l
 mgl l 1  m
2 2m
2
2m
c) Con los datos del problema, sern:

1
kg m 2
L  1q 2  2
 2 Ns
50
s
El valor mnimo de la percusin, calculado en el apartado b) sera
1  100q

2q9.8q1
 6.39 N s
3
de modo que no alcanza la posicin vertical
1min  2.5

- 214 -

2
3

gl

Fsica Universitaria: Problemas de Fsica

Dinmica impulsiva del slido rgido. M24.3

3. Sobre una varilla homognea, de masa M y longitud l, que puede girar alrededor

de un eje fijo horizontal A, impacta elsticamente una partcula de masa m con


velocidad v0 perpendicular a la varilla. a) Calcular la velocidad de la partcula y la
velocidad angular de la varilla justamente despus de la colisin. b) Determinar la
percusin en el eje A.

mv0
a) La cantidad de movimiento del sistema no se conserva, por tener el
sistema un eje impuesto, en el que, en principio, aparecern percusiones
reaccionales..
Conservacin del momento angular con respecto al eje de suspensin
que pasa por A:

2l
2l 1
mv0  mv Ml 2 X l 2m(v0  v)  Ml X

3
3
3
Conservacin de la energa cintica:
1 2 1 2 1 1 2 2
mv0  mv Ml X

2
2
2 3

cm l
cp
l/3

[1]

l 3m(v02  v 2 )  Ml 2 X 2 [2]

Dividimos m.a.m. las dos ecuaciones anteriores:


3m(v02  v 2 ) Ml 2 X 2

2m(v0  v)
Ml X

3(v0 v)
 lX
2

[3]

que sustituida en la ecuacin [1] nos conduce a


3(v0 v)
4 m  3M
l v
v0
2m(v0  v)  M
2
4 m 3M
y sustituyendo este resultado de nuevo en la ecuacin [1] obtenemos:
4 m  3M
12m v0
) l X
Ml X  2mv0 (1
4 m 3M
4 m 3M l
b) La resultante de las percusiones que actan sobre el sistema es igual al cambio que
experimenta la cantidad de movimiento el sistema completo; esto es,


con vcm  X

eje

 p l 1eje  mv Mvcm  mv0

l
6m

v0 , de modo que
2 4 m 3M
4m 2  3mM

6mM

 m v0  0
1eje  mv Mvcm  mv0 

4m 3M
4 m 3M

como era de esperar ya que el impacto tiene lugar en el centro de percusin, por lo que la
varilla gira libremente alrededor del eje A.
Otro mtodo
Podemos llegar directamente a la expresin [3] a partir de la regla de Huygens-Newton con un
coeficiente de restitucin e = 1 (elstico):
v  vB
2
e
 1 l v  vB  v0  l X  v0 l 3(v0 v)  2l X [3]
v0  0
3

- 215 -

Fsica Universitaria: Problemas de Fsica

Dinmica impulsiva del slido rgido. M24.4

4. Una viga uniforme, de longitud 2l y masa m, est sostenida hori-

zontalmente por dos apoyos, A y B, a una distancia x del centro


G de la viga. a) Determinar la distancia x para que, al suprimirse
sbitamente uno de los apoyos, no vare en ese instante la reaccin en el otro. b) Justamente despus de suprimir el apoyo B,
calcular la aceleracin del punto G de la viga.

a) Los puntos A y B deben ser conjugados; de modo que sus distancias al punto G tienen
cumplir la condicin de Huygens para el centro de percusin, mhh '  I cm , de modo que ser:

mxx  mx 2 

1
1
2
m 2l
 ml 2
12
3

l x

l
3
l  0.58 l

3
3

b) Antes de retirar el apoyo B, sern


FA  FB  12 mg

FA

Justamente despus de retirar el apoyo B, en las


condiciones del apartado anterior, la viga estar
sometida a las fuerzas
FA  12 mg
P  mg
Aplicamos la primera ecuacin cardinal de la dinmica
del slido rgido para calcular la aceleracin del centro
de masas de la viga:
P  FA  macm l mg  12 mg  macm

FB

G
x

P
FA
A

G
x
P

acm  g
1
2

Otro mtodo
Antes de retirar el apoyo B, sern

FA  FB  12 mg
Justamente despus de retirar el apoyo B, en las condiciones que impone el enunciado del
problema, la viga estar sometida a las fuerzas
FA  12 mg
P  mg
Aplicamos las ecuaciones cardinales de la dinmica del slido rgido, tomando momentos en
G, y tenemos en cuenta que tendr lugar una rotacin instantnea alrededor de A:

1
acm  2 g
mg  12 mg  macm

1
2
2
2 mgx  13 ml
B l B  23 gx / l

acm  B x
1 g  3 gx 2 / l 2 l x 2  1 l 2 l x  3 l

2
3
3
2

- 216 -

Fsica Universitaria: Problemas de Fsica

Dinmica impulsiva del slido rgido. M24.5

5. Una varilla rectilnea y uniforme, de masa m y longitud l, que dispone de

m, l

gancho

un gancho en uno de sus extremos, cae libremente en posicin horizontal.


En el instante en que su velocidad es v0, la varilla queda enganchada en
v0
eje
un eje horizontal fijo. a) Determinar el estado de movimiento de la varilla
inmediatamente despus de quedar enganchada en el eje. b) Calcular la reaccin percusional en el eje.
c) Determinar el cambio que experimenta la energa cintica de la varilla.

a) Durante el proceso impulsivo, la varilla est sometida a una percusin, por lo que no se
conservan ni su cantidad de movimiento ni su energa cintica. En cambio, se conserva el
momento angular de la varilla con respeto al eje, por ser
m, l
nulo el momento percusional con respecto al mismo; esto
es,
v0

+
l 1
3v

mv0  ml 2 X l X  0

2 3
2 l
+
m, l
y la velocidad del c.m. de la varilla ser
l 3
vcm

vcm  X  v0
2 4
b) La percusin que recibe la varilla es igual al cambio que experimenta su cantidad de
movimiento; esto es,
3

1
1  m(vcm  v0 )  m v0  v0   mv0 (hacia arriba)
4

4
c) Cambio de la energa cintica:

E0  mv02

2
1 2

l E  E  E0   mv0
2

3 v
8
1 1
1 1
3
E  ml 2 X 2  ml 2 0  mv02

2 l
2 3
2 3
8

E 1/ 8

 0.25  25% .
de modo que hay una prdida de energa cintica igual al 
E0 1/ 2
Otro mtodo
a) y b) Aplicamos las ec. cardinales de la dinmica impulsiva, tomando momentos en el c.m.
de la varilla:
1  m vcm  v0

u l  l  1 l2 X
l 6vcm l X  6v0

1 2
l
2 12 vcm  v0
1  ml X  0

2
12
l
y la condicin de ligadura (rotacin alrededor del eje): vcm  X
2
Resolviendo este sistema de 2 ec. con dos incgnitas, tenemos
3v
3
1
X 0
vcm  v0
l
1   mv0
2 l
4
4

- 217 -

Fsica Universitaria: Problemas de Fsica

Dinmica impulsiva del slido rgido. M24.6

6. Una varilla rectilnea y uniforme, de masa m y longitud l cae libremente en

m, l

cuchilla

posicin horizontal. En el instante en que su velocidad es v0, la varilla


golpea elsticamente el borde de una cuchilla rgida y fija. a) Determinar
el estado de movimiento de la varilla inmediatamente despus del golpe.
b) Calcular la percusin sobre la varilla. c) Determinar el cambio que
experimenta la energa cintica de la varilla.

a) Durante el proceso impulsivo, la varilla est sometida a


una percusin externa en su extremo izquierdo, por lo que no A
se conservan ni su cantidad de movimiento, ni su energa
cintica.

Aplicamos las ecuaciones cardinales de la dinmica
impulsiva, tomando momentos en el c.m. de la varilla, y las A
dividimos m.a.m. para eliminar :

v0

m, l

C
v0

+
+

m, l

vcm

1  m vcm  v0

1
l
X

l   l2
l 6vcm l X  6v0
(1)
l
1
2
m.a.m.
u
1  ml X  0

2 12 vcm  v0

12

2
Aplicamos la Regla de Huygens-Newton con e =1 (colisin perfectamente elstica), siendo vA
la velocidad del punto A de la varilla inmediatamente despus de la colisin y obtenemos
dicha velocidad a partir de la del centro de masa C de la varilla:

vA  v0

l vcm  X  v0 l 2vcm  Xl  2v0


(2)

2
v
v
X


A
cm

Resolvemos el sistema de ecuaciones (1) y (2):


vcm  0.5v0

6vcm l X  6v0

l 8vcm  4v0 l

3v0

m.a.m.

2vcm  l X  2v0
X 
l

Mtodo 2. Conservacin del momento angular en A:

l
l 1
mv0  mvcm ml 2 X l 6v0  6vcm l X

2
2 12
Mtodo 3. Aplicamos la Regla de Huygens-Newton en A.
Adems, sabemos que el conjugado del punto de percusin
A se encuentra en A, a una distancia 2l/3 del punto A y que A
su velocidad despus de la percusin no se altera (v0), por lo
que obtenemos la velocidad de A a partir de ella:
v0


vA  v0

3v0

A
l X

vA  v0  2l / 3
X
A partir de la velocidad de A determinamos la del centro de
masa

- 218 -

(1)

m, l
v0
m, l

C A
vcm

+
+

v0

Fsica Universitaria: Problemas de Fsica

Dinmica impulsiva del slido rgido. M24.7

l
l 3v0
vcm  vA X  v0
 0.5v0
2
2 l
b) La percusin que recibe la varilla es igual al cambio que experimenta su cantidad de
movimiento. Calculamos la percusin a partir de la primera ec. de (1) en la que sustituimos
los resultados anteriores:
1
1  m vcm  v0
 m 0.5 1
v0  mv 0 (hacia arriba)
2
c) Variacin de la energa cintica:

E  E  E0   mv02
E0  mv02

l
2

1 2
1 1 2 2 1 v02 1 1 2 3v0
1 2 E  0.5  50%
E  mvcm ml X  m ml  mv0

l
2
2 12
2 4 2 12
4

E0

de modo que la prdida de energa cintica representa el 50% de la inicial.

- 219 -

Fsica Universitaria: Problemas de Fsica

Dinmica impulsiva del slido rgido. M24.8

7. Una barra uniforme de longitud l1 y masa M puede girar libremente en el


plano vertical alrededor de una bisagra O. De la bisagra pende una
partcula de masa m colgada de un hilo de longitud l2. La barra se
abandona a partir del reposo en la posicin indicada y al chocar con la
partcula, sta queda adherida a la barra. Determinar la velocidad angular
que adquiere el conjunto inmediatamente despus del choque.

l1
M

l2
m

1.- Movimiento inicial de la barra. Aplicamos el Principio de


conservacin de la energa (nivel de referencia en O):
l
3g
0  Mg 1 12 13 Ml12
X 2 X 2 
2
l1
2.- Colisin inelstica manteniendo un eje fijo. Se produce una percusin en el eje, por lo
que no se conserva la cantidad de movimiento del sistema. En cambio, el momento angular
respecto del eje (O) permanece constante.
13 Ml12 X  13 Ml12X a mv al2
13 Ml12 X  13 Ml12X a ml2 2 X a  13 Ml12 ml2 2
X a

v a  X al2 (condicion de ligadura)

o sea
Xa 

Ml12
Ml12
X

2
2
1
Ml12 3ml2 2
3 Ml1 ml2
1
3

- 220 -

3g
l1

Fsica Universitaria: Problemas de Fsica

Dinmica impulsiva del slido rgido. M24.9

m, l

8. Una placa rectangular, de lado l y masa m uniformemente distribuida,


puede girar alrededor de un eje fijo horizontal que coincide con uno de sus
bordes, como se indica en la figura. Separamos la placa hasta la posicin
horizontal y la abandonamos partiendo del reposo. Cuando alcanza la posicin vertical, la placa colisiona (coeficiente de restitucin, 0 < e < 1) contra
el borde de otra placa idntica que se encontraba en reposo sobre un plano
horizontal. a) Determinar las velocidades de cada placa justamente despus
de la colisin. b) Calcular la reaccin percusional suministrada por el eje.

m, l

Primera fase: Durante la rotacin de la primera placa se conserva


la energa de sta, de modo que

l 1 1
0  mg ml 2 X02

2 2 3

l X02 

3g
l

l X0 

N.R.

3g
l

l/2

Segunda fase: Durante la colisin entre las dos placas,


x No se conserva la cantidad de movimiento del sistema (por
existir una reaccin externa en el eje),
x Se conserva el momento angular con respecto al eje:
1 2
1 2
ml X0  ml X mvl l l X 3v  l X0
E
3
3
x

No se conserva la energa cintica del sistema (por


tratarse de una colisin parcialmente elstica). Sin
embargo, puesto que se supone conocido el coeficiente
de restitucin, podemos aplicar la Regla de HuygensNewton en el punto A de contacto durante la colisin
(frontal); esto es,
l X  v  e l X0  0
l l X  v  el X0

m, l


+

m, l

l
-

m, l v

Disponemos de dos ecuaciones con dos incgnitas (, v):

1
X  1 3e X

X   X0

4
2
l X 3v  l X0
l
l e  1

l X  v  el X0

1 e
1

v 

l X0
v  l X0

4
2

b) La percusin que suministra el eje es igual al cambio que experimenta la cantidad de


movimiento del sistema.
l
Antes de la colisin: p0  mvcm  mX0
2

1
l
 3e l 1 3e
p1  mvcm  mX  m
X0 
ml X0
3 e
2
4
2
8
Despus:
ml X0
l p  p1 p2 

1 e
8
ml X0
p2  mv 
4

De modo que

1eje  p  p0 

3 e
1
1 e
1 e
ml X0  ml X0  
ml X0  
m 3 gl
8
2
8
8

- 221 -

Fsica Universitaria: Problemas de Fsica

Dinmica impulsiva del slido rgido. M24.10

9. Un tren de engranajes de radios r1 y r2 y masas m1 y m2 se acoplan del


modo en que se indica en la figura, permaneciendo los ejes sobre los
puntos fijos O1, y O2. Supongamos que los dientes sean lo
suficientemente pequeos como para poder considerar cada engranaje
como un cilindro homogneo. a) Determinar la velocidad angular con
que cada engranaje inicia el movimiento al incidir una percusin
tangencialmente en el punto A. b) Calcular las reacciones
percusionales en los ejes.

O2

1

O1

2

a) Durante la percusin en el punto A, aparecen las percusiones, 1 , 2, 12 y 21 que


actan tal como se indica en la figura, con 12 = 21 (accin-reaccin).
Para cada cilindro, el momento de percusional con respecto del eje de rotacin es igual al
incremento del momento angular; i.e.,
1
21

1 r1  112 r1  I1X1  12 m1r12 X1

121r2  I 2 X2  m r X2
1
2

O2

2
2 2

y como la condicin de rodadura da lugar a que


X1r1  X2 r2

1

O1

2

12

de modo que tenemos tres ecuaciones con tres incgnitas ( , 1 y 2).


Resolviendo este sistema de ecuaciones tenemos:

21
X1 
1
1  112  2 m1r1X1

(
m
m2 )r1

l 1  12 (m1 m2 )r1X1 l

1
1

21

121  2 m2 r2 X2  2 m2 r1X1
X2 
(
m
m2 )r2

1
b) Podemos calcular 12

121  112  12 m2 r2 X2 

m21
m1 m2

Clculo de las reacciones percusionales en los ejes: Para cada cilindro, la resultante de
todas las percusiones es igual al incremento de su cantidad de; i.e.,
41  m vcm  0

puesto que el centro de masa de cada cilindro permanece en reposo. As, escribimos las
ecuaciones
11  1  112  0 l 11  1 112
112  12  0 l 1 2  112
de modo que
11 

m1 2m2
1
m1 m2

12 

en las direcciones que se indican en la figura.

- 222 -

m2
1
m1 m2

Fsica Universitaria: Problemas de Fsica

Elementos de elasticidad. M27.1

1. Una barra de seccin rectangular 100 u 50 mm y 2 m de longitud, sometida a una traccin de 50 t


experimenta un alargamiento de 1 mm y una contraccin lateral de 0.007 mm en la arista de 50 mm.
Calcular: a) El mdulo de Young de la barra. b) El valor del coeficiente de Poisson. c) La contraccin que
experimenta la arista de 100 mm de la seccin recta. d) Dimensiones de la seccin recta si se somete a la
barra a una traccin de 40 t.

a) A partir de la definicin del mdulo de Young, se sigue:


T
l F
F/S
E l 
 0
Fl %l / l0 %l S

2000 50 u103 u 9.8


N
u
1.96 u 1011 2
1
0.05 u 0.100
m
b) A partir de la definicin del coeficiente de Poisson, tenemos:
0.007 / 50
b
l
b / b
 N
l N 

 0.28
l / l0
1 / 2000
b
l0
E

c) La deformacin transversal en la arista de 100 mm ser:


a
l
1
Fa 
 N   0.28
  1.4q104
2000
a
l0

a  Fa a  1.4q104 q100   0.014 mm


d) Teniendo en cuenta la proporcionalidad directa entre fuerzas y deformaciones,
'ac F c
Fc
40
o 'ac
'a
u ( 0.014)  0.0112 mm
'a F
F
50
a 100  0.0112 99.9888 mm
'bc F c
Fc
40
o 'bc
'b
u ( 0.007)  0.0056 mm
'b F
F
50
b 50  0.0056 49.9944 mm

- 223 -

Fsica Universitaria: Problemas de Fsica

Elementos de elasticidad. M27.2

2. Una barra de seccin rectangular a u b = 100 mm u 50 mm y 2 m de longitud experimenta un alargamiento

de 1 mm y una contraccin lateral 'b =  0.007 mm cuando se la somete a una traccin de 50 t. a) Calcular
el mdulo de Young y el coeficiente de Poisson de la barra. b) Calcular la variacin de la seccin recta de la
barra.

a) A partir de la definicin del mdulo de Young, se sigue:


3
Tl
F / S 50q10 q9.8
/ 0.100q 0.050

N
E 

 1.96q1011 2
m
Fl
l / l0
1/ 2000

A partir de la definicin del coeficiente de Poisson, N  


N 

F
b
a

FD
, tenemos:
Fl

b / b
0.007 / 50

 0.28
l / l0
1/ 2000

b) La deformacin transversal en la arista de 100 mm ser:


a
l
1
Fa 
 N   0.28
  1.4q104
2000
a
l0

l
l

a  Fa a  1.4q104 q100   0.014 mm


El cambio unitario que experimenta la seccin recta de la varilla es:
S a b


 1.4q104 1.4q104  2.8q104
S
a
b
Y el cambio que experimenta la seccin transversal es

S  2.8q104 q100q50  1.4 mm 2

- 224 -

Fsica Universitaria: Problemas de Fsica

Elementos de elasticidad. M27.3


300 cm

3. Un cable de acero, de 1 mm2 de seccin, se sujeta horizontalmente, sin


tensin, a dos soportes distantes 300 cm. Se suspende una carga P en el
punto medio del cable producindose en el mismo un esfuerzo de
1000 kg/cm2. a) Calcular el alargamiento que experimenta el cable. b) Determinar el peso de la carga. Datos: E = 2u106 kg/cm2

a) A partir de la definicin del mdulo de Young (ley de Hooke) tenemos:


T F/S
l F
E 
l l 
F l / l
E S
De modo que
300 cm
l 
q1000 kg/cm 2  0.15 cm
2q106 kg/cm 2
b) Determinaremos el ngulo T a partir de las longitudes inicial y final del cable:
l/2
l
300
cos R 
=

 0.9995 R  1.81o
l l
/ 2 l l 300.15

Tras la deformacin, el sistema constituido por el alambre y la carga suspendida queda en


equilibrio, por lo que ser 6F = 0; esto es,
P  2 F sen R
Y sustituyendo los datos

kg
P  2 1000 2 q 0.01 cm 2 sen (1.81 )  0.63 kg

cm

l/2

F

F
P

- 225 -

(l+l)/2

Fsica Universitaria: Problemas de Fsica

Elementos de elasticidad. M27.4

4. Un peso cuelga de un cable de acero de 2 m de longitud 1 mm2 de seccin. Se desea sustituir dicho cable
por otro de cobre de 3 m de longitud. a) Qu seccin deber tener el cable de cobre para que se alargue lo
mismo que el de acero? b) dem para que se rompan a la misma tensin?
Datos:

acero:

E  2.0q1011 N/m 2

Trup  4q108 N/m 2

cobre:

E  1.1q1011 N/m 2

Trup  2q108 N/m 2

Fl
T F/S

l l 
ES
F l / l
a) Imponemos la condicin de que ambos cables experimenten el mismo
alargamiento:
Flac
FlCu
E l
lac  lCu l

l SCu  ac Cu Sac
Eac Sac ECu SCu
ECu lac
Definicin del mdulo de Young: E 

y sustituyendo valores
2.0 3
q q1  2.73mm
1.1 2
b) Imponemos la condicin de que sean iguales las tensiones de ruptura en los dos
cables:
SCu 

Facrup  FCurup

rup
l Tacrup Sac  TCu
SCu

l SCu 

y sustituyendo valores
4
SCu  q1  2 mm 2
2

- 226 -

Tacrup
S
rup ac
TCu

Fsica Universitaria: Problemas de Fsica

Elementos de elasticidad. M27.5

5. De dos hilos de longitud l, seccin S y mdulos de Young E y 2E se cuelga una masa m


inicialmente en reposo, como se indica en la figura. Determinar la frecuencia angular y la
amplitud de las oscilaciones verticales del sistema.

2E
m

En cada instante los dos cables tienen igual deformacin x, de modo que
F1 

ES
x
l

F2 

2 E
S
l

3ES
x  keq x
l
F1
F2
de modo que la constante elstica de los dos
A
hilos es
x
m
posicin de
m
3ES
equilibrio
keq 
l
El sistema equivale a una masa suspendida de un muelle de constante elstica keq, por lo que
la frecuencia de sus oscilaciones es
l

2E

F  F1 F2 

keq

3ES

m
ml
Dado que la energa mecnica se conserva, igualamos la energa del sistema en la posicin
inicial (1) (cuando se cuelga la masa) y la de mxima deformacin (2)
X

Ep1 Ek1 Eel1  Ep2 Ek 2 Eel2

2A
N.R.

posicin
inicial

2
posicin de
equilibrio
elongacin
mxima

- 227 -

1
2
mg 2 A  keq 2 A

2
mg mgl
A

keq 3ES

Fsica Universitaria: Problemas de Fsica

Elementos de elasticidad. M27.6

6. Una barra homognea de 1000 N de peso se cuelga en posicin horizontal de tres


hilos de igual seccin y longitud inicial como se indica en la figura. Determinar en
que relacin se alargan los hilos (unos respecto a otros) y que fuerza soporta cada
uno de ellos en los siguientes casos: a) El hilo central es de acero y los de los
extremos de cobre. b) El hilo de acero es el de uno de los extremos y los otros dos
de cobre. Datos: El mdulo de Young del acero es doble que el del cobre.

a/2

a/2
a

a) En virtud de la simetra que presenta el problema, los tres hilos experimenta el mismo
alargamiento, por lo que de la definicin del mdulo de Young y de las ecuaciones de la
esttica se obtiene:
T
T
Eacero Tacero Facero


 2 l Facero  2 Fcobre
F  cobre  acero l
Ecobre Eacero
Ecobre Tcobre Fcobre

cobre

- 228 -

cobre

F1  363.6 N

2 F1 F2  1000

l F2  272.7 N

F2  0.75 F1

F3  363.6 N

cobre

As que el hilo 3 experimenta una deformacin doble que el 1. La


deformacin del hilo 2 ser F2  1.5 F1 , de modo que las
deformaciones (alargamientos) se encuentran en la relacin
2:3:4
Al comparar las deformaciones de los hilos de cobre (2 y 3) se obtiene:
F2 T2 F2 1.5
  
 0.75 l F2  0.75 F3  0.75 F1
F3 T3 F3
3
De modo que

FCu

acero

ya que F1 y F3 presentan el mismo brazo con respecto a B. A


partir de este resultado, teniendo en cuenta la definicin del
mdulo de Young, se sigue:
F1
F1 T1 F1 Eacero
F
 
 2 1  1 l F3  2F1
x1
F3
F3 T3 F3 Ecobre

Fac

acero

cobre

Fcobre  250 N
2 Fcobre Facero  1000

Facero  2 Fcobre
Facero  500 N

F
b) Cuando colocamos el hilo de acero en un extremo, la barra se Cu
inclina, ya que, a igualdad de esfuerzos, el acero se alarga menos
que el cobre. Las ecuaciones de la esttica, tomando momentos en
A
B, se escriben en la forma:
F1 F2 F3  P

2 F1 F2  P

F1b  F3b

F1  F3

F3

F2
x2
B
P

x3
C

Fsica Universitaria: Problemas de Fsica

Elementos de elasticidad. M27.7

7. Una barra prismtica est constituida por dos materiales diferentes A y B y se somete a compresin en sus
extremos, segn se indica en la figura. Determinar la variacin de volumen que se produce en la barra en
funcin de los mdulos de Young y coeficientes de Poisson de los materiales.

F
S

Se trata de una compresin longitudinal pura (sin esfuerzos laterales externos) en


la que ambos segmentos de la barra (A y B) estn sometido a un mismo esfuerzo: lA
F
(compresor)
T 
S
lB
En cada uno de los segmentos, la deformacin unitaria longitudinal vale
T
F
Fzz   
(acortamiento)
E
SE
F
y la deformaciones unitarias transversales son
z
NF
Fxx  F yy  NFzz 
(ensanchamiento)
ES
x
y
de modo que el cambio de volumen ser
V
F
 Fxx F yy Fzz  (1 2N)Fzz   1 2N

V
ES
V
Fl
V 
Sl   1 2N

(disminuye)
V
E
En definitiva
Segmento A:

V
A   1 2NA

FlA
EA

(disminuye)

Segmento B:

V
B   1 2NB

FlB
EB

(disminuye)

Total:

V
 V
A V
B

(disminuye)

- 229 -

Fsica Universitaria: Problemas de Fsica

Elementos de elasticidad. M27.8

8. Un pilar de hormign armado y seccin cuadrada debe soportar una carga, a compresin pura, de
2.5 u 106 N. Se sabe que el 10% del rea de la seccin cuadrada del pilar est ocupado por los redondos
(varillas cilndricas) de acero y el resto por hormign en masa. Las caractersticas mximas de trabajo
permitidas y los mdulos de Young tienen los valores siguientes:

Tmax H  25 N mm 2

Tmaxacero  400 N mm 2

EH  30 000 N mm 2

Eacero  200 000 N mm 2

a) Calcular la deformacin unitaria mxima del pilar. b) Determinar el lado de la seccin cuadrada del pilar.
c) Siendo cuatro los redondos del acero de la armadura del hormign, determnese el dimetro de cada uno de
ellos.

a) Las deformaciones mximas experimentadas en las condiciones crticas de


trabajo son
Tmax H
25

  0.83q103
hormign: Fmax H 
EH
30 000
Tmax acero
400
acero: Fmax acero 

  2q103
Eacero
200 000

Como

Fmax H  Fmax acero , y ambos materiales experimentan la misma

deformacin, tendrn el lmite del ms crtico; i.e.,


F  Fmax H   0.83q103

b) La carga soportada por cada uno de los materiales ser


FH  0.9 S EH F

6
F Facero  0.9 EH 0.1 Eacero
S F  2.5q10
Facero  0.1 S Eacero F H

S

2.5q106
 63830 mm 2
27 000 20 000
(0.83q103 )
a  63830  253 mm

c) La seccin del acero ser el 10% de la del pilar

Sacero  6383mm 2
y cada uno de los cuatro redondos tendr una seccin de
S redondo  1595.75 mm 2

r  22.3 mm

- 230 -

 44.6 mm

F
a

Fsica Universitaria: Problemas de Fsica

Elementos de elasticidad. M27.9

9. Se cuelga una bola de 20 kg de un hilo de acero de 1 mm de dimetro y 3 m de longitud que se supone sin
peso. El sistema se comporta como una masa unida a un resorte. Cuando se le hace vibrar verticalmente,
mantenindose el hilo siempre tenso, la frecuencia resulta ser 8.14 Hz. a) Determinar la constante elstica
(k) del hilo. b) Deducir la relacin existente entre el mdulo de Young del hilo y la constante k.
c) Determinar el mdulo de Young de este acero.

a) La frecuencia angular del m.a.s. que realiza la masa unida al hilo viene dada por

X2 

k
m

k  mX 2  20 (2Q q8.14) 2  52 316.48

N
m
l

b) A partir de la definicin del mdulo de Young, tenemos

E

T F /S

F %l / l

F

ES
ES
%l  k %l k 
l
l

E

kl
S

c) Aplicando la expresin anterior a los datos que nos dan en el enunciado, se sigue

E

kl
4kl
4q52 316.48q3
N


 19.98q1010 2
S
m
QD2
Q q (103 ) 2

- 231 -

Fsica Universitaria: Problemas de Fsica

Elementos de elasticidad. M27.10

10. a) Calcular el aumento de longitud que experimenta un cable de acero ordinario de 100 m de longitud
cuando se le suspende verticalmente por uno de sus extremos, de modo que se estira elsticamente bajo la
accin de su propio peso. Despreciar las variaciones originadas en la seccin recta y en la densidad como
consecuencia del alargamiento. b) Se supera el lmite elstico en alguna seccin recta del cable? c) Qu
longitud mxima de cable puede suspenderse sin que ocurra su ruptura?
Datos para el acero: densidad, 7.87 g/cm3; mdulo de Young, 20u1010 N/m2; lmite elstico, 25u107 N/m2;
esfuerzo de ruptura, 50u107 N/m2.

a) La tensin no es constante a lo largo del todo el cable, sino que va


aumentando conforme nos alejamos del punto de suspensin. La
tensin y el esfuerzo tensor en un punto situado a una distancia x del
punto de suspensin corresponden al peso de la porcin de cable
situada por debajo de dicho punto; esto es,
F ( x)
 S l  x
g
F ( x)  S l  x
Sg l T ( x) 
S
Consideremos un elemento infinitesimal de cable, de longitud dx,
situado a una distancia x del punto de suspensin. El alargamiento
dY
unitario experimentado por dicho elemento ser F 
, donde d es el
dx
aumento de longitud que experimenta el elemento. Aplicando la
definicin del mdulo de Young, T  E F , resulta
dY
Sg
T ( x)  E  S l  x
g l dY 
l  x
dx
E
dx
El aumento de longitud que experimenta el cable completo ser la suma
de las contribuciones de todos los elementos de cable,

dx

dx+d
F(x)
l

Sg l
S g
x2
S g 2 l 2 S g 2
l  x
dx 
lx  
l

l  

E 0
E
E
2 0
2 2 E
l

l  dY 

y sustituyendo los valores numricos


7.87q103 q9.8
q1002  1.93q107 q104  1.93q103 m  1.93 mm
2q 20q1010
b) La tensin y el esfuerzo tensor en el cable son mximos en el punto de suspensin (x=0),
por lo que
l 

Tmx  T (0)  S gl  7.87 q103 q9.8q100  0.771q107 N/m 2


que es considerablemente menor que el lmite elstico del acero, Trup  25q107 N/m 2 .
c) Puesto que el esfuerzo tensor no puede superar el valor del esfuerzo de ruptura, la longitud
del cable deber ser inferior a
Trup
50q107
lmx 

 6 483 m  6.5 km
Sg
7.87 q103 q9.8

- 232 -

Fsica Universitaria: Problemas de Fsica

Elementos de elasticidad. M27.11

11. Un paraleleppedo de cierto material elstico (coeficiente de Poisson P = 0.3 y

mdulo de Young E = 2u107 N/cm2), que a cierta temperatura tiene dimensiones


a = 10 cm, b = 30 cm y c = 40 cm, se introduce entre dos mordazas rgidas,
c
planas, paralelas y horizontales separadas por una distancia c. Por efecto de un
a
aumento de temperatura, en ausencia de tensiones mecnicas, el paraleleppedo
b
-4
experimenta una dilatacin trmica unitaria de 2.5u10 en cada una de las tres
dimensiones dadas. a) Calcular el valor de la tensin y de la fuerza ejercida por
las mordazas que impiden la dilatacin trmica del material en la direccin
vertical. b) Determinar las dilataciones unitarias y absolutas que, en estas condiciones, tendrn realmente
cada una de las aristas. c) Hallar la variacin unitaria de volumen del paraleleppedo.

a) Como consecuencia de la dilatacin trmica, aparecen esfuerzos


mecnicos compresores en la direccin del eje z, que contrarrestan
la dilatacin trmica; esto es,

zz

T zz  EFtrm  2q1011 q 2.5q104  5q107 N/m 2


c

y, por ser compresor, ser negativo:

T zz  5q10 N/m
7

La fuerza compresora ejercida sobre las caras ser:


F  T zz S  ab T zz  0.10q 0.30q5q107 =1.5q106 N

zz

b) Escribimos las Ecuaciones Elsticas para valorar las deformaciones unitarias debidas a los
efectos puramente mecnicos:

Fxx  1 T xx  NT yy  NT zz  N T zz
5q107

E
E
 0.75q104
Fxx  F yy  0.3q

11
1
T
2
q
10
F   NT T  NT  N zz
zz
xx
yy
yy E
E

T
1
F   NT  NT T  zz  F
 2.5q104
xx
trm
zz
yy
zz E
E
Las deformaciones unitarias reales sern la superposicin (suma) de las de origen elstico y
de origen trmico. Esto es,
*
*
4
4
4

Fxx  F yy  0.75q10 2.5q10  3.25q10


*
4
4

Fzz  2.5q10 2.5q10  0


y las deformaciones absolutas pedidas son
a*  aF*xx  0.1q3.25q104  3.25q105  32.5 m
*
*
4
5
b  bF yy  0.3q3.25q10  9.75q10  97.5 m

c*  cF*  0
zz

c) La variacin unitaria del volumen es:

V *
 F*xx F*yy F*zz  6.5q104
V

- 233 -

Fsica Universitaria: Problemas de Fsica

Elementos de elasticidad. M27.12

12. Un cuerpo, cuya forma es la de un paraleleppedo rectangular, se somete a


compresin normal uniforme sobre dos de sus caras opuestas, en tanto que
se impide la expansin transversal, de modo que su anchura permanezca
constante, utilizando una armadura rgida como la que se muestra en la
figura adjunta. a) Determinar las deformaciones unitarias en las aristas del
paraleleppedo y los esfuerzos sobre las otras caras. b) Determinar los valores efectivos del mdulo de Young (Eef) y del coeficiente de Poisson (ef)
correspondientes a esta forma de compresin (definidos por

E'

T xx
Fxx

y N'  

Fzz
).
Fxx

a) Escribimos las ecuaciones elsticas:

1
Fxx  (T xx  NT yy  NT zz )

F  1 (T  NT  NT ) con
xx
zz
yy E yy

F  1 (T  NT  NT )
xx
yy
zz E zz

z
F

bloque
y
x

armadura

Fxx  (T xx  NT yy )

l 0  (T yy  NT xx )

Fzz  (NT xx  NT yy )

F yy  0

T  0
zz

Resolvemos este sistema de 3 ec. con 3 incgnitas ( Fxx , Fzz , T yy ):

1
1 N 2

Fxx  (T xx  N 2T xx ) 
T xx

E
E

T yy  NT xx

1
N(1 N)

Fzz  (NT xx  N 2T xx )  
T xx

E
E

z
F

b) Valores efectivos del mdulo de Young y del


coeficiente de Poisson:

T xx
E
E  T xx 

2
ef F
N
1

N2
1

xx

T xx
E

(1 N)
F
N
N

zz
Nef   F  1 N 2  1 N
xx

- 234 -

bloque
y
x

armadura

Fsica Universitaria: Problemas de Fsica

Elementos de elasticidad. M27.13

13. Sobre un bloque cbico, de 25 cm de arista, se ejerce una fuerza de 625 t, mediante

una plancha rgida. Dicho bloque se encuentra perfectamente encajado entre dos
paredes rgidas. Determnense: a) Las deformaciones sufridas por las aristas (en las
tres direcciones principales). b) La presin que se ejerce sobre las paredes
laterales. c) La variacin de volumen que experimenta el bloque.
Datos: mdulo de Young, E = 1011 N/m2 ; coeficiente de Poisson, P = 0.4.

a) y b) Escribimos las ecuaciones elsticas:

Fxx  (T xx  NT yy  NT zz )

T xx  0

F yy  0
F yy  (T yy  NT xx  NT zz ) con

1
F
625 000q9.8
N

(
)



F
T
NT
NT
  9.8q107
T zz    

zz
zz
xx
yy
2
2

E
S
0.25
m

de modo que

1
N

[1] Fxx  (NT yy  NT zz )   (T yy T zz )


z

E
E

[2] 0  (T yy  NT zz )

1
x

[3] Fzz  (T zz  NT yy )

Resolvemos este sistema de 3 ec. con 3 incgnitas:


[2] l T yy  NT zz  0.4q (9.8q107 )   3.92q107

N
(compresor)
m2

0.4
(3.92  9.8)q107  5.49q104
1011
= x  Fxx x0  5.49q104 q 0.25  1.37q104 m  0.137 mm

[1] l Fxx  

1
1 0.42
(1 N 2 )T zz 
(9.8q107 )   8.23q104
E
1011
= z  Fzz z0  (8.23q104 )q 0.25  2.06q104 m   0.206 mm

[3] l Fzz 

c) Para el cambio de volumen tenemos:


V
 Fxx F yy Fzz  (5.49 0  8.23)q104   2.74q104
V0
= V  (2.74q104 )q 0.253  4.28q106 m3   4.28 cm3

- 235 -

Fsica Universitaria: Problemas de Fsica

Elementos de elasticidad. M27.14

14. Un pilar zunchado (i.e., recubierto de otro material que impide dilataciones transversales) soporta una
carga de 1500 N/cm2. a) Qu esfuerzos transversales soportar el zuncho? b) Cul ser la deformacin
vertical del pilar, si ste tiene 3 m de altura?

Datos: mdulo de Young, 2u106 N/cm2; coeficiente de Poisson, 0.25.

Datos:

N
N

E  2q106
 2q1010 2

cm
m

N  0.25

N
N

T zz  1500 2  15q106 2

cm
m

a) Escribimos las ecuaciones elsticas, con las condiciones Hxx= Hyy= 0; esto
es, :

Fxx  (T xx  NT yy  NT zz )  0

F yy  (T yy  NT xx  NT zz )  0

Fzz  (T zz  NT xx  NT yy )

zz

z
y
x

Las dos primeras nos conducen a


T xx  NT yy  NT zz
N

T zz
l T xx  T yy 
NT xx T yy  NT zz
1 N

y sustituyendo valores
0.25
N
(15q106 )   5q106
T xx  T yy 
0.75
cm 2
b) La tercera ecuacin elstica nos permite calcular la deformacin unitaria longitudinal del
pilar:
1
Fzz  [T zz  N(T xx NT yy )] 
E
1
1
 [T zz  2NT xx ] 
(15 2q 0.25q5)q106   6.3q104
E
2q1010
y el acortamiento total ser:
z  Fzz z   6.3q104 q3  1.89q103 m=  1.89 mm

- 236 -

Fsica Universitaria: Problemas de Fsica

Elementos de elasticidad. M27.15


pilar

15. Un pilar cilndrico, de 20 cm de dimetro y 3 m de altura, de hormign (E = 2.8+1010 N/m2,


 = 0.4), est revestido exteriormente con un zuncho de acero que impide las dilataciones transversales. a) Determinar la contraccin longitudinal que experimenta el pilar cuando soporta una
carga vertical de 8000 kg. b) Calcular el esfuerzo transversal que soporta el zuncho.

Escribimos las ecuaciones elsticas, imponiendo las condiciones de


imposibilidad de dilataciones transversales debidas al zuncho rgido:

Fxx  (T xx  NT yy  NT zz )  0

F yy  (T yy  NT xx  NT zz )  0

Fzz  (T zz  NT xx  NT yy )

pilar

zuncho

de modo que disponemos de tres ecuaciones con tres incgnitas


( Fzz , T xx , T yy )
Resolviendo las dos primeras y sustituyendo los resultados en la tercera,
tenemos:
T xx  NT yy  NT zz
N

T zz
l T xx  T yy 
NT xx T yy  NT zz
1 N

Y sustituyendo en la tercera,:

H zz

1
(V zz  PV xx  PV yy )
E

2 P 2 V zz
1 

1 P E

Sustituyendo los datos en los resultados algebraicos anteriores, tenemos:


E  2.8q1010 N/m 2 ; N  0.4; S  14 Q q 0.202  0.0314 m 2 ;
8000q9.8
 2.50q106 N/m 2
0.0314
2q 0.42 2.50q106

Fzz  1
 4.16q105

1 0.4 2.8q1010
T xx  

x  Fzz h  4.16q105 q3  0.125 mm


T xx  T yy 

0.4
2
2
T zz  T zz  2.50q106
 1.67 q106 N/m 2
1 0.4
3
3

- 237 -

zuncho

y
x

Fsica Universitaria: Problemas de Fsica

Elementos de elasticidad. M27.16

16. a) Expresar el mdulo de compresibilidad de un material en funcin del mdulo de Young y del coeficiente
de Poisson del mismo. b) Calcular el incremento de presin al que debemos someter un cierto volumen de
agua (K = 22 u 108 N/m2) para que su densidad aumente en un 0.1%.

a) El mdulo de compresibilidad se define como


p
V / V
Para obtener V/V aplicamos una compresin uniforme p sobre las seis caras de un elemento
de volumen de forma paralelepipdica. Las deformaciones unitarias de las aristas, con
xx = yy = zz = p, vienen dadas por las ecuaciones elsticas:
1
1
(1 2N)
Fxx  (T xx  NT yy  NT zz )  (1 2N)(p )  
p
E
E
E
y expresiones anlogas para yy y zz .
El cambio unitario en el volumen ser
V
3(1 2N)
 Fxx F yy Fzz  
p
V
E
Que, sustituida en la expresin de definicin de K, nos conduce a
E
K 
3(1 2N)
K 

b) Como la masa (constante) de un cierto volumen de agua se expresa por m = V, siendo  la
densidad, la diferencial logartmica de esta expresin nos conduce a
dS dV
S
V

0 l

S
S
V
V
que sustituida en [1] nos lleva a
S
V p
S


l p  K
 22q108 q 0.1q102  22q105 Pa x 22 atm
S
S
V
K

- 238 -

Fsica Universitaria: Problemas de Fsica

Elementos de elasticidad. M27.17

17. Un cubo metlico que tiene de longitud de 20 cm de arista se sumerge en el mar a una profundidad

de 400 m. Conociendo el mdulo de Young del metal E = 2.1u1010 N/m2, el coeficiente de Poisson P = 0.3 y
el valor de la densidad del agua del mar U = 1.06 g/cm3, calcular la variacin de volumen que experimenta el
cubo sumergido.

Escribimos las ecuaciones elsticas:

Fxx  E T xx  N(T yy T zz )

F  1 T  N(T T ) l F  F  F  1 <T  N(T T ) >  1 2N T


xx
zz
xx
yy
zz
yy E yy
E
E

1
F  T  N(T T )
yy
xx
zz E zz
y por tratarse de una compresin uniforme debida a la presin, ser  = p, de modo que
la deformacin unitaria en cada arista vendr dada por
V
1 2N
F
p
E
V
y el cambio unitario en el volumen ser
V
3(1 2N)
V
 Fxx F yy Fzz  3F  
p
V
E
con
p  S gh  1.06q103 q9.8q 400  41.552q105 Pa  41 atm
de modo que
V
3(1 0.6)

q 41.552q105   2.37 q104
V
2.1q1010
V   2.37q104 q 0.23  1.9q106 m3  1.9 cm3

- 239 -

Fsica Universitaria: Problemas de Fsica

Elementos de elasticidad. M27.18

18. Un cubo macizo y de acero, de 25 cm arista se sumerge a una profundidad de 500 m en agua marina
(densidad del agua constante U = 1.05 g/cm3). Calcular la variacin de volumen que experimenta el cubo.

Datos para el acero: mdulo de Young = 2u106 kg/cm2 y coeficiente de Poisson = 0.3.

Escribimos las ecuaciones elsticas, teniendo en cuenta que, al tratarse de una compresin
uniforme debida a la presin, ser T xx  T yy  T zz  T  p , de modo que la deformacin
unitaria en cada arista vendr dada por
1

Fxx  E T xx  N(T yy T zz )

F  1 T  N(T T ) l
xx
zz
yy E yy

F  1 T  N(T T )
yy
xx
zz E zz

1
1  2N

Fxx  F yy  Fzz  E <T  N(T T ) >  E T

1  2N
p
F  
E

El cambio unitario en el volumen ser


V
3(1 2N)
 Fxx F yy Fzz  3F  
p
V
E
con
p  S gh  1.06q10 q9.8q500  51.94q10 Pa  51.3 atm
3

V
V
V

2
kg
N
N
4 cm
q
q
 19.6q1010 2
9.8
10
E  2q106
2
2
cm
kg
m
m

de modo que
V
3(1 0.6)

q51.19q105   3.30q104
V
19.6q1010
V  3.30 u104 u 0.253 5.2 u 106 m3 5.2 cm3

- 240 -

Fsica Universitaria: Problemas de Fsica

Elementos de elasticidad. M27.19

19. El esfuerzo de ruptura por cortadura para el cobre laminado ordinario es de


aproximadamente 16107 N/m2. Qu fuerza F debe aplicarse para cortar con una
cizalla una tira de cobre de 60 mm de ancho y 3 mm de espesor?

Cizalla

Plancha
de cobre

Seccin de corte: Sc  60q103 q3q103  180q106 m 2


Esfuerzo de corte: Tc 

Fc
Sc

l Fc  Tc Sc

Por consiguiente, la fuerza de corte que debe aplicarse debe ser superior a la correspondiente
al esfuerzo de ruptura por cortadura; esto es,
Fc,rupt  Tc,rupt Sc  16q107 q180q106  28800 N  2939 kg

Cizalla

Plancha
de cobre

- 241 -

Fsica Universitaria: Problemas de Fsica

Elementos de elasticidad. M27.20

20. El acero ordinario requiere un esfuerzo de aproximadamente 35 u 107 N/m2 para que se produzca la ruptura
por cortadura. Determinar la fuerza que es preciso ejercer sobre el punzn para hacer un agujero de 20 mm de
dimetro en una plancha de ese acero de 6 mm de espesor.

El esfuerzo de corte al que se encuentra sometida la plancha cuando el punzn ejerce una
fuerza F, est distribuido en la superficie lateral de un cilindro de 20 mm de dimetro y 6 mm
de altura, esto es, sobre una superficie S  Q Dh , de modo que
F
F
Tcorte  
F
S Q Dh
Para conseguir el corte de la plancha, el esfuerzo de corte
aplicado deber ser superior al esfuerzo de ruptura por
cortadura, de modo que
Tcorte p Trupt

Fmin  Q DhTrupt  35q107 q Q q 0.020q 0.006 


 131 947 N = 13 464 kg

- 242 -

punzn
plancha
estampa

Fsica Universitaria: Problemas de Fsica

Elementos de elasticidad. M27.21

21. Un pndulo de torsin est formado por un alambre de acero ordinario, de 80 cm de


longitud y 1 mm de dimetro, que lleva en su extremo inferior un disco homogneo
de plomo, de 12 cm de dimetro y 1 cm de espesor. Se gira el disco un cierto ngulo
y despus se abandona de modo que efecte oscilaciones de rotacin en un plano
horizontal. El tiempo empleado en realizar 100 oscilaciones completas es 315 s.
a) Qu esfuerzo tensor soporta el alambre? Se supera el lmite elstico?
b) Calcular la constante de torsin del pndulo. c) Determinar el mdulo de rigidez
del acero del alambre.
Datos: densidad del plomo, 11.35 g/cm3; lmite elstico del acero, 25u107 N/m2; relacin
entre el coeficiente de torsin y el mdulo de rigidez de un alambre, U 

Periodo de las oscilaciones: T 

QR4
G.
2l

315
2Q
rad
 3.15 s l X 
 1.995
100
T
s

Masa del disco : m  SV  11.35q103 qQ 0.06


q 0.01  1.284 kg
2

1
1
Momento de inercia del disco: I  mR 2  1.284q 0.062  2.31q103 kg.m 2
2
2
a) El esfuerzo tensor que soporta el alambre es:

Tl 

mg
1.284q9.8
N

 1.6q107 2 (no se supera el lmite elstico)
Q R 2 Q 0.5q103
2
m

b) El momento recuperador de torsin es proporcional al ngulo o elongacin de torsin; esto


es, M  UG . Por otra parte, tenemos la ecuacin fundamental de la dinmica de rotacin
(aplicada al disco), M  IG ; de modo que

M  UG  I G l I G UG  0


que es la ecuacin diferencia de un m.a.s. de rotacin (torsin) cuya frecuencia angular es
U
I
de donde podemos calcular el valor del coeficiente de torsin
X

2Q
2Q
3
3 m.N
U  X 2 I  I 
q 2.31q10  9.19q10
T
3.15
rad
2

c) Calculamos el valor del mdulo de rigidez del acero a partir del coeficiente de torsin y de
las dimensiones del alambre:

U

2l
2q 0.80
N
QR4
q9.19q103  7.49q1010 2
G l G
U
2l
m
QR4
Q q 0.54

- 243 -

Fsica Universitaria: Problemas de Fsica

Elementos de elasticidad. M27.22

22. El eje de transmisin de un automvil es de acero y mide 1.80 m de longitud por 2.5 cm de dimetro.
a) Qu ngulo se tuerce uno de sus extremos con respecto al otro cuando el eje est transmitiendo una
potencia de 30 CV a 2400 r.p.m.? b) Qu energa elstica est entonces almacenada en el eje?
Datos: mdulo de rigidez del acero, G = 7.8u1010 N/m2; coeficiente de torsin para un eje macizo,

U

QR4
G ; 1 CV = 736 kW.
2l

Datos:
l  1.80 m
D  2.50 cm  0.025 m
R  1.25 cm  0.0125 m
X  2400 r.p.m.  40 r.p.s.  80Q rad/s
P  30 CV  30q 736  22 080 W

G  7.8q1010 N/m 2
a) Determinamos el par (momento) transmitido por el eje a partir de la potencia transmitida
a unas revoluciones conocidas; esto es,
P 22080
P  MX l M  
 87.9 m.N
80Q
X
El par o momento de torsin (M) es proporcional al ngulo de torsin (I),
M  UG

siendo W el coeficiente de torsin, cuyo valor es

Q 0.0125

QR4
G
q 7.8q1010  1.66q103 m.N/rad
2l
2q1.80
4

U

As pues, el ngulo de torsin correspondiente a las condiciones de trabajo del enunciado,


vale:
M
87.9
G

 0.0529 rad  3.03
U
1.66q103
b) Energa elstica de torsin:
1
1
U  UG 2  1.66q103 q 0.05292  2.32 J
2
2

- 244 -

Fsica Universitaria: Problemas de Fsica

Esttica de los fluidos. M29.1

1. Demostrar que el centro de presiones que un lquido ejerce sobre una pared rectangular,
plana y vertical se encuentra situado una profundidad igual a dos tercios de la altura de
la pared.

2/3 H

Aplicamos el teorema del centro de presiones:

hc hcp 

I
S

siendo:
x

hc la profundidad a la que se encuentra el centro geomtrico de la pared:


1

hc  H
2

I el momento de reas de segundo orden de la superficie de la pared con


respecto a la lnea definida por la interseccin del plano de la pared con la
1

superficie libre del lquido: I  SH 2


3

hcp la profundidad a la que se encuentra el centro de presiones;

de modo que
hcp 

1
SH 2 2
I
 31
 H
hc S
3
2 HS

- 245 -

c.q.d.

Fsica Universitaria: Problemas de Fsica

Esttica de los fluidos. M29.2

2. a) Dibujar el perfil o diagrama de presiones sobre la compuerta de la

2a

figura, de dimensiones 2a y b, cuando retiene agua y aceite en el


modo que se representa. b) Calcular el empuje al que se encuentra
sometida la compuerta, c) Determinar la posicin del centro de
empuje. Aplicacin numrica: a = 1 m; b = 2 m.

Aceite

a (U =900kg/m3)
a

Agua

a) En la figura, presentamos la distribucin de fuerzas de presin sobre cada una de las dos
porciones de la pared.
b) Determinamos el empuje sobre cada porcin de la pared correspondiente a cada una de las
distribuciones de fuerzas representadas en la figura y el empuje resultante.
Mitad superior de la compuerta:
a
1
2
aceite
F1  S1 g ab  S1 ga 2b  8820 N
con h1  a  0.67 m
(U1 =900kg/m3)
2
3
2
Mitad inferior de la compuerta:
1
con h2a  a  0.50 m
F2a  (S1 ga)ab  S1 ga 2b  17 640 N
(U2 =1000kg/m3)
2
agua
1
a
2
F2aa (S2 g )ab  S2 ga 2b  9800 N con h2aa  a  0.67 m
2
2
3

2
1
1
2
2
F2  S1 ga b S2 ga b  S1 S2 ga b  27 440 N

2
2
Empuje total sobre la compuerta:
3

F  F1 F2  ( S1 S2 ) ga 2b  (3S1 S2 ) ga 2b  36 260 N
c) Aplicamos el teorema de Varignon, tomando momentos en A:

11
2a
a
2a
5
HF 
F1 a F2a a F2aa S1 S2 ga 3b

3
2
3
6
6

Determinamos la posicin H del centro de presiones:


1
11S1 5S2
ga 3b 11S 5S

1
2
6

H
a
1

S
S
3
(3
2
1
2)
(3S S ) ga b

F1

H
F

h1

F2cc

F2c

h2c

h2cc

y sustituyendo valores, con 1 = 0.9 y 2 = 1.0, resulta


11q 0.9 5
14.9
H
1
 1.35 m
3q (3q 0.9 1)
11.1

- 246 -

Fsica Universitaria: Problemas de Fsica

Esttica de los fluidos. M29.3

3. Un canal rectangular de 2 m de ancho est cortado por una pared transversal


que separa dos lquidos. A un lado de la pared hay agua que alcanza una
altura de 1,8 m. Al otro lado un aceite de densidad 0,9 g/cm3. a) Qu altura
debe de alcanzar el aceite para que sobre la pared, los dos lquidos, ejerzan
unos empujes equivalentes a un par de fuerza? b) Valor del par de fuerza y
sentido del mismo. c) Qu fuerzas (valor, direcciones y sentidos) deben
aplicarse a la pared separadora, una en B al nivel de la superficie del aceite y
la otra a la de su fondo, en A, para equilibrar los empujes.

a) Para que la accin conjunta de ambos empujes sea


equivalente a un par es necesario que se igualen los
mdulos de ambos empujes: i.e.,
Eagua  Eaceite
h1
h
(h1 L)  S2 g 2 (h2 L) l S1h12  S2 h22
2
2
de modo que
S1 g

h2 

S1
1
h1  1.8
 1.897 m
0.9
S2

B
aceite
agua
A

h1

Eaceite

Eagua

0.6 m

h2

0.6324 m

Cada empuje vale


h1
1
1
(h1 L)  S1 gh12 L  1000q9.8q1.82 q 2  31752 N
2
2
2
b) Los centros de presiones distan de la solera un tercio de la profundidad, como se indica en
la figura, por lo que el brazo del par de empujes y el par valen:
h2 h1 h2  h1 1.897  1.8
b

0.032 m
3 3
3
3
M emp  bE  0.032q31752  1030.5 Nm
E  S1 g

c) Para compensar dicho par actuando sobre los puntos A y B ser necesario ejercer el par
opuesto, de modo que
M emp 1031
F AB M emp o F
543 N
AB 1.897

- 247 -

Fsica Universitaria: Problemas de Fsica

Esttica de los fluidos. M29.4

4. La compuerta vertical de la figura mide 3 m de altura, est engoznada por su borde superior y separa dos
recipientes que contienen agua y aceite de densidad relativa 0.85. Si la altura H del agua alcanza 2 m,
calcular la altura que debe alcanzar la superficie libre del aceite para que la compuerta se mantenga vertical.

Las fuerzas que ejercen el agua y el aceite sobre la compuerta


vienen dadas por

H
Fagua  S g HL  12 S gH 2 L
2

Ha
H aL  12 S a gH a 2 L
Faceite  S a g
2

Sabemos que las lneas de accin de esas fuerzas son


horizontales y estn situadas a una altura sobre la solera igual a
un tercio de la profundidades H y H respectivas.
En efecto,
hc hcp 

I
S

hcp 

O
aceite

H agua

1
SH 2 2
I
 3H
 H
3
hc S
2 S

medida desde la superficie libre del lquido.


Sea A la altura de la compuerta (i.e., desde la solera hasta el borde superior engoznado).
Tomando momentos con respecto al eje que pasa por O, tenemos

H
H a
H
H a
Fagua A   Faceite A  12 S gH 2 L A   12 S a gH a 2 L A 

3
3
3
3

H
H a
S
S H 2 A   S a H a 2 A  H a3  3 AH a 2 (3 AH 2  H 3 )  0

Sa
3
3
En la que sustituimos los datos para obtener
H a3  3q3H a 2

1
(9q 22  23 )  0
0.85

Resolviendo esta ecuacin cbica se obtiene H a  2.2 m

- 248 -

H a3  9 H a 2

28
0
0.85

Fsica Universitaria: Problemas de Fsica

Esttica de los fluidos. M29.5

5. a) Determinar la fuerza total debida a la presin del agua sobre la compuerta


inclinada, de 3 m de anchura, que se muestra en la figura. b) Calcular el momento de dicha fuerza respecto a la bisagra (B). c) Localizar la lnea de accin de
dicha fuerza resultante. d) Determinar la reaccin de la solera sobre el borde
inferior (A) de la compuerta.

agua
1m
B
30
1.5 m

a) Aplicamos la expresin: F  S ghc


S , con
hc  1 0.75sen 30  1.375 m; S  1.5q3  4.5 m 2
F  1000q9.8q1.375q 4.5  60 638 N  6188 kg
c) Aplicamos la expresin: yc ycp  I xx / S , con

I xx  I cc SD 2  121 S 1.5
S 2.75

ycp 

I xx
2
2
 121 1.5
2.75
 7.75 m 2
S

JJJJK
I xx / S 7.75

 2.82 m l BCp  2.82  2.00  0.82 m
yc
2.75

b) Aplicamos el Teorema de Varignon para una distribucin de fuerzas paralelas, tomando


momentos en B:
M B  BCp F  0.82q 60 638  49 723 m N
d) Puesto que la compuerta est en equilibrio, los momentos en B de F y de FA deben ser
iguales y opuestos; esto es,
MB
49 723
M B BA cos 30 FA o FA
38 277 N 3906 kg
BA cos 30 1.5 u 0.866

x
30

agua

hcp

F
FA
A

30

2m
2.75 m

1m

hc

1.5

- 249 -

ycp

Fsica Universitaria: Problemas de Fsica

Esttica de los fluidos. M29.6

6. La compuerta representada en la figura est formada por dos superficies

3m

rectangulares unidas rgidamente entre si y puede girar alrededor del eje


AA. a) Calcular el empuje que ejerce el agua sobre la compuerta.
b) Calcular el momento del par que hay que aplicar al eje AA para que
la compuerta no se apoye en la solera.

2m
2m

30

Panel vertical:
F1  S ghC1S1  1000q9.8q1q 2q3
 58800 N
hC1hP1 

I xx
S

l hP1 


I xx / S 13 22 4

  1.3 m
hC1
1
3

Panel inclinado:
F2,hor  F2 sen 30  73500 N
F2  S ghC2 S 2  1000q9.8q 2.5q 2q3
 147 000 N l
F2,vert  F2 cos 30  127 306 N

hC2  2 1qsen 30  2.5 m



I
I
1
76
 25.3 m 2
hC2 hP2  xx sen 2 30
con xx  22 52 
S
S
12
3


I xx / S
25.3
hP2 
sen 2 30 
0.25  2.53 m
hC2
2.5

Las componentes de la resultante, su mdulo y direccin son:


F  1323002 127 3062  183603 N

Fhorz  132300N
l

R =arctg 21218  43.9


Fvert 127 306 N

127 306
b) El momento del par que deberemos aplicar deber ser igual a la diferencia de los
momentos de las fuerzas F1 y F2 con respecto al eje AA; esto es,
2
2

| F1 q 3  58800q 3  39 200 N m
l | M par  116 620 N m

0.53
}
AP
147
000
155820
N
m
F
q

q


2
2
sen 30

O
xx

F1

C1
P1

F
A

F2
P2 C2

- 250 -

Fsica Universitaria: Problemas de Fsica

Esttica de los fluidos. M29.7

7. Cuando sube el nivel del agua en la alberca, la compuerta rectangular, doblada en


L, se abrir automticamente. Cul deber ser el nivel mnimo (h) del agua por
encima de la bisagra para que no se produzca la apertura? (Despreciar el peso de la
compuerta).

h
1.22 cm

agua

Calculamos las fuerzas que actan sobre cada una de las secciones de la
compuerta:
Fv  S gh
aL  S gLah

Fh  S g h hL  1 S gLh 2

2
2
Determinamos la posicin del centro de presin de la seccin vertical de la compuerta:
hc hcp 

h
Fh

1.22 cm

agua

Fv

I xx
S

l hcp 

1
Sh 2 2
I xx
 31
 h
hc S
3
2 hS

medida desde la superficie libre del agua.


La compuerta permanecer cerrada mientas que el momento
de la fuerza horizontal sea mayor que el de la fuerza
vertical, ambos respecto de la bisagra; esto es,
h
a
h
a
1
Fh  Fv l
S gLh 2  S gLah
3
2

3
2
2
o sea,
1 2
h  a
3

- 251 -

l h  a 3  1.22 3  2.11 cm

Fsica Universitaria: Problemas de Fsica

Esttica de los fluidos. M29.8

8. Una viga de madera, de seccin cuadrada de lado a, apoyada sobre una de sus aristas, bloquea el extremo de
un canal de fondo plano y horizontal, alcanzando la superficie libre del agua la arista superior de la viga.
Calcular el empuje que el agua ejerce sobre la viga y ngulo que ste forma con la horizontal.

H  a2 a2  a 2

F1

Mtodo 1 (dos caras):


Cara superior:
2
H
2
F1  S g aL  S g
a aL 
S ga 2 L

4
4
4

F2

Cara inferior:

3H
3 2 a aL  3 2 S ga 2 L
F2  S g
aL  S g

4
4
4
El mdulo de la resultante es:
2
2
1 9
5
F 2  S ga 2 L
 S ga 2 L

8 8
4

Calculamos el ngulo: tg G 

F

5
S ga 2 L
2

F2
 3 G  71.6
F1

R  71.6  45  26.6

Mtodo 2 (Arqumedes):
Determinamos directamente las componentes horizontal y vertical de la fuerza total que acta
sobre la viga.
Empuje sobre la proyeccin vertical:

H
a 2
Fhor  S g HL  S g
a 2 L  S ga 2 L

2
2

a
Fver

H
Fhor

Empuje de Arqumedes:
a2 1
Fver  S gV  S g L  S ga 2 L
2 2

1
2
2
2
Fver
 1 S ga 2 L

Mdulo de la resultante: F 2  Fhor


4

ngulo que forma con la horizontal: tg R 

Fver 1


Fhor 2

- 252 -

F

R  26.6

5
S ga 2 L
2

9. Un tonel de completamente lleno de vino (de densidad, 0.92 g/cm3) tiene las
dimensiones que se indica en la figura y est tumbado en la bodega. Calcular el
empuje que ejerce el vino sobre cada una de las tapas del tonel y determinar la
posicin del centro de presiones sobre las mismas medidas respecto al centro de
las tapas.

100 cm

Esttica de los fluidos. M29.9

140 cm

Fsica Universitaria: Problemas de Fsica

200 cm

Clculos previos
Medimos las profundidades a partir del
eje xx indicado en la figura. El centro
R
geomtrico (centroide) de la tapa se encuentra
a una profundidad hc=R.
hc
D
D
Determinamos el momento de rea de la
hcp
tapa con respecto al eje DD (Teorema
F
r
Ejes Perpendiculares):
F
1
1
y
2 I DD  I  Sr 2 l I DD  Sr 2
2
4
Ahora, mediante el Teorema de Steiner,
determinamos el momento de rea de la tapa respecto al eje xx definido en la figura:
1

I
1
I xx  I DD SR 2  Sr 2 SR 2  r 2 R 2 S l xx  r 2 R 2

4
4
4
S
x

La fuerza resultante sobre la tapa ser

F  (S ghc ) S  S gR
Qr 2
 QS gRr 2
La profundidad a la que se encuentra el centro de presiones se determina mediante el teorema
del centro de presiones:
hc hcp S  I xx

l hcp 

r2
I xx / S 1 1 2
 r R 2 
R
4R
hc
R 4

O bien, medida respecto a centro de la tapa:


ER 

r2
4R

Sustituyendo los valores numricos se obtiene:


F  Q q920q9.8q 0.70q 0.52  4957 N  506 kg
E

0.52
 0.0893 m  8.93 cm
4q 0.7

- 253 -

Fsica Universitaria: Problemas de Fsica

Esttica de los fluidos. M29.10

10. Un barril de cerveza tiene forma de cilindro de revolucin de 1.2 m de dimetro y 1.8 m de longitud y se
apoya sobre una generatriz. Si el barril contiene la mitad de su capacidad, calcular la resultante de las
fuerzas de presin de la cerveza (densidad, 0.939 g/cm3) sobre la tapa del barril y determinar la posicin del
centro de presiones sobre la misma.

Clculos previos
Determinamos la posicin del centro geomtrico (centroide) de un semicrculo sirvindonos
del Segundo Teorema de Pappus-Guldin:
4 3 1 2
4R
V  SL l
Q R  Q R 2Qhc l hc 
3
2
3Q
Determinamos el momento de rea de un semicrculo con respecto a su dimetro, sabiendo
que el momento con respecto a un eje perpendicular a su plano y que pasa por su centro es
igual a
1
I zz  SR 2
2
de modo que, aplicando el Teorema de los Ejes Perpendiculares, resulta
1
2 I xx  I zz l I xx  SR 2
4

La fuerza resultante sobre la tapa ser


4R QR2 2
 S gR 3
3Q 2
3
La profundidad a la que se encuentra el centro de presiones se determina mediante la
expresin
hc hcp S  I xx
y
F  (S ghc ) S  S g

hcp 

I xx / S

hc

R 2 3Q R

 0.59 R
4
16
3Q R

Sustituyendo los valores numricos se obtiene:


2
F  939q9.8q 0.63  1325 N = 135 kg
3
hcp  0.58q 0.60  0.353 m = 35.3 cm

- 254 -

R
x

x
hc
hcp
y

Fsica Universitaria: Problemas de Fsica

Esttica de los fluidos. M29.11

11. Determinar el empuje y el centro de empuje sobre una compuerta

circular de radio R, como la que se representa en la figura, cuando el


nivel del lquido coincide con el punto ms elevado de la misma.
60

Determinamos la posicin del centro geomtrico (centroide) de la compuerta y calculamos el


momento de rea de la misma con respecto al eje determinado por la interseccin del plano de
la compuerta con la superficie libre del lquido (eje x):
x

S  QR2

O
x

y
yc

hc hcp

C
ycp

5
4

3
2

La fuerza resultante sobre la compuerta se


determina multiplicando la presin en su
centro por la superficie de la compuerta:
F  (S ghc ) S  S g

I xx  SR SR  SR
1
4

yc  R hc  R sen 60 

3
2

RQ R 2 

3
2

S g Q R3

La posicin del centro de presin o empuje se


determina a partir del Teorema de Centro de
Presin:
yc ycp 

- 255 -

I xx
S

ycp 

5
SR 2 5
I xx
4
4R
yc S
RS

Fsica Universitaria: Problemas de Fsica

Esttica de los fluidos. M29.12

12. Un depsito de base cuadrada de lado 1 m, tiene una de sus paredes inclinada 30
respecto a la vertical. a) Demostrar que el empuje sobre la pared rectangular vertical
est a 1/3 de la altura h que alcanza el agua medido desde la base. b) Calcular el
empuje y centro de empuje sobre la pared inclinada. c) Puede producirse el vuelco
del depsito al ascender el nivel del agua?

30

a
a

Aplicamos los dos teoremas bsicos de la hidrosttica en las dos paredes.


a) Pared vertical:

h
1
F1  S ghc1S1  S g ha  S gah 2

2
2

1 2
a

h
I xx
I /S
2
l hp1  xx
 3  h
hc1hp1 
(xx)
S
hc1
h/2 3

b) Pared inclinada:
La longitud de la pared inclinada baada por el
agua es

l

hp1

a
(xx)

hp2

30

F2

P1

F1

h
h
2h 2 3



h
cos 30
3
3/2
3

P2

a
y

h
3
2
F  S gh S  S g la 
S gah
c2
2
2
3

1 2

l
I
I /S
2
 3  l
yc2 yp2  xx l yp2  xx
S
yc2
l/2 3

c) Calculamos el momento neto en el punto O, ya que, de producirse el vuelco, ocurrira


alrededor de ese lado de la base.

2
1 1
1
M O
1  h  h F1  h S gah 2  S gah3

3
3 2
6

2 3

12 3

S gah 2  S gah3
h
M O
2  OP2 F2  l  l S gah2 
3 3
3 3
3
9
2 1
1
M O  M O
2  M O
1   S gah3  S gah3
9 6
18

Como el momento resulta positivo, cualquiera que sea el valor de h, se producir el vuelco en
todo caso, a menos que lo impida el propio peso del depsito, que en este problema hemos
supuesto despreciable.

- 256 -

Fsica Universitaria: Problemas de Fsica

Esttica de los fluidos. M29.13

13. Determinar la fuerza resultante (mdulo y direccin) que acta sobre la compuerta cilndrica AB, cuya seccin recta es la de un cuarto de circunferencia. La anchura de la compuerta es 1.20 m.

60 cm

agua
B

90 cm
A

Determinamos las componentes horizontal y vertical del empuje hidrosttico sobre la


compuerta.
Componente horizontal = Empuje sobre la proyeccin de la compuerta sobre un plano
vertical:
Fh  pc S  S ghc RL
y sustituyendo valores
Fh  1000q9.8q 0.60 0.45
q 0.90q1.20  11.1q103 N  1135 kg

Componente vertical = Peso del volumen de lquido situado por encima de la compuerta:

Q R 2
Q
L  S g H  R RL
Fv  S gVencima  S g HR 

4
4

y sustituyendo valores

Q
Fv  1000q9.8q1.50  q 0.90q 0.90q1.20  8.40q103 N  857 kg

4
La lnea de accin del empuje resultante pasa por el punto O, ya que las diferentes
contribuciones son perpendiculares a la superficie y, por tanto, tienen direccin radial.
El mdulo y la direccin del empuje hidrosttico resultante se calculan fcilmente
2
2
F  Fhorz
Fvert
 11.12 8.402 q103  13.9q103 N  1421 kg

tg R 

Fvert 8.40

 0.7554 l
Fhorz 11.1

R  37.1

60 cm

agua
Fh
B

H
Fv

90 cm

CP

- 257 -

Fsica Universitaria: Problemas de Fsica

Esttica de los fluidos. M29.14

14. Un canal de agua, de 2 m de ancho y 1 m de profundidad, est cerrado


mediante una compuerta cilndrica de radio igual su profundidad.
a) Calcular la magnitud del empuje hidrosttico sobre la compuerta y el
ngulo que forma su direccin con la horizontal. b) Determinar la
posicin del eje de giro (normal al dibujo) de la compuerta para que dicho
eje no tenga que soportar ningn momento.

1m

Determinamos las componentes horizontal y vertical del empuje hidrosttico sobre la


compuerta.
Componente horizontal = Empuje sobre la proyeccin de la compuerta sobre un plano
vertical:
R
1
1
Fh  (S ghc ) S  S g lR  S glR 2 l Fh  1000q9.8q 2q12  9800 N
2
2
2
Componente vertical = Empuje de Arqumedes (peso del volumen de lquido desalojado):
1
Q
Q
Fv  S gV  S g Q R 2l  S glR 2 l Fv  1000q9.8q 2q12  15394 N
4
4
4
La lnea de accin del empuje resultante pasa por el punto O, ya que las diferentes
contribuciones son perpendiculares a la superficie y, por tanto, tienen direccin radial. El
mdulo y la direccin del empuje hidrosttico resultante se calculan fcilmente

F  Fh2 Fv2  98002 153942  18249 N


tg R 

Fv Q
  1.57 l R  57.5
Fh 2

La posicin del eje de giro de la compuerta se determina mediante la interseccin de la lnea


de accin de la fuerza de empuje resultante con la compuerta; i.e., el punto A que se indica en
la figura.

F
Fv

O Fh

Fv
1m

Fh

- 258 -

Fsica Universitaria: Problemas de Fsica

Esttica de los fluidos. M29.15

15. La compuerta de la figura est compuesta de dos chapas soldadas, una rectangular y otro
cilndrica. Determinar el empuje sobre toda la compuerta y el ngulo que ste forma
respecto a la horizontal cuando la compuerta contiene un lquido de densidad U .

b
a
a
a

Determinamos las componentes horizontales sobre las dos partes de la compuerta:


a
1
F1,hor  S gh1,c S1  S g ab  S ga 2b
2
2
b
a
3
F2,hor  S gh2,c S2,proy  S g (a )ab  S ga 2b
2
2
La componente vertical, dirigida hacia arriba, es igual al
a
F2
peso del fluido situado encima de la porcin cilndrica
F1
F2,vert
de la compuerta:
Qa 2
Q
b)  (1 )S ga 2b
F2,horz
4
4
a
Las componentes horizontal y vertical de la fuerza
resultante son:
1
3
Fhorz  F1,horz F2,horz  S ga 2b S ga 2b  2S ga 2b
2
2
Q
2
Fvert  F2,vert  (1 )S ga b
(hacia arriba)
4
El mdulo y la direccin de dicha fuerza son:
F2,vert  S gV  S g (a 2b

Q
4 1  2.86 S ga 2b

4
2

2
2
Fvert
 S ga 2b
F  Fhorz

Fvert

F


tg R 

Fvert 1 Q4

 0.893 l R  41.8
Fhorz
2

Fhorz

- 259 -

(hacia arriba)

Fsica Universitaria: Problemas de Fsica

Esttica de los fluidos. M29.16

16. La compuerta representada en la figura tiene una anchura L y est formada por un

tramo AB rectangular de altura R y la cuarta parte (BC) de una superficie cilndrica de


seccin circular de radio R. La compuerta puede girar alrededor de un eje
perpendicular al plano del dibujo y que pase por A. Determinar la fuerza vertical
mnima F que se debe aplicar en C para mantener cerrada la compuerta.

R
R
R

Sobre la porcin rectangular (AB) de la compuerta acta una fuerza F1 horizontal, aplicada a
una profundidad h1  2 R / 3 (centro de presin), cuyo mdulo es:

Nx

R
1
F1  S g RL  S gR 2 L
2
2

Ny
R

F1

F2v

F2
O

R
F
F2

F2h

Sobre la porcin cilndrica (BC) de la compuerta


acta una fuerza F2 cuya lnea de accin pasa por el
eje de simetra de revolucin del cilindro (punto O,
en la figura). Las componentes horizontal y vertical
de esta fuerza son
3R
3
F2h  S g RL  S gR 2 L

2
2

Q
Q R 2
L  1 S gR 2 L
F2v  S g R 2

4
4

El eje fijo en A, alrededor del cual puede girar la compuerta, ejerce sobre sta una fuerza N
cuyas componentes horizontal y vertical se indican en la figura.
Puesto que la compuerta debe permanecer cerrada y en equilibrio, el momento resultante
sobre ella debe ser nulo; as, tomando momentos en A, podemos escribir:
2
2
M A  F1 R F2h R F2v R  FR  0 l F  F1 F2h F2v
3
3
y sustituyendo valores
1 3
34 3Q
2
Q
2
F  F1 F2h F2v  1 S gR 2 L 
S gR L

3 2
12
4
3
esto es,
F  3.62 S gR 2 L

- 260 -

Fsica Universitaria: Problemas de Fsica

Esttica de los fluidos. M29.17

17. Debemos obstruir un canal de seccin rectangular mediante un


cuarto de cilindro de radio R = 0.5 m y cuya altura coincide con
la altura del agua en el canal. a) Calcular el empuje y centro de
empuje en los casos ilustrados en la figura. b) Cul sera la
densidad mnima del cuarto de cilindro en ambos casos para
evitar el vuelco respecto al eje 00?

R
Caso A

Caso B

Caso A. Determinamos las componentes horizontal y vertical del empuje hidrosttico:

Fh  (S ghc ) S  S g R R 2  1 S gR 3  612.5 N

2
2

F  S g ( R 3 V )  S g R 3  Q R R  4  Q S gR 3
v

4
4

hc

Fv

Fh


FA

F  S gR 3 1 (4  Q )  666.5 N
A

4
16
l

tg R  Fv  2  Q  0.429 l R  23.3

Fh
2

Como la lnea de accin del empuje hidrosttico corta al eje


OO, no se produce momento de vuelco, por lo que el peso del
cuarto de cilindro es irrelevante, al menos a los efectos de
vuelco.
Caso B. Empuje hidrosttico horizontal sobre una superficie
plana
R
1
FB  (S ghc ) S  S g R 2  S gR 3  612.5 N
2
2
y el centro de presiones o de empuje se encuentra situado a
una profundidad
hc hcp S  I xx

hcp

hc
G
FA

x
P

l hcp 

I xx / S

hc

R2 2
 R  33 cm
1 R
3
2

El momento que produce el empuje hidrosttico respecto


del eje OO deber estar contrarrestado por el que produce
el peso con respecto al mismo eje: i.e.,
R
R
FB  P( R  x) l P 
FB
3
3( R  x)

Determinamos x mediante el 2 teorema de Pappus-Guldin:


2 3
QR2
4R
Q R  2Q x
l x
3
4
3Q
De modo que el peso y la densidad del cuarto de cilindro debern ser:
R
R
Q
1
P
FB 
S gR 3 
S gR 3  354.7 N  36.2 kg
R
4
3( R  x)
2
2(3
4)
Q

3( R 
)
3Q
Q
S R3
m P/g
2
Scil  
S  0.369S = 0.369 g/cm3


3
1
V
V
2(3Q  4) 4 Q R
(3Q  4)
V  LS l

- 261 -

Fsica Universitaria: Problemas de Fsica

Esttica de los fluidos. M29.18

18. El final de un canal est taponado por una compuerta semicilndrica, de 1 m de radio
y 2 m de longitud, cuyo dimetro est inclinado 45. a) Calcular el empuje
hidrosttico sobre la compuerta, especificando su mdulo y su recta de accin.
b) Determinar la fuerza mnima que deberemos aplicar perpendicularmente sobre la
compuerta, as como el punto de aplicacin de dicha fuerza, para impedir que la
compuerta vuelque sobre el eje A.

45

2
R 2
2
a) La componente horizontal del empuje es igual al que ejercera el agua sobre la superficie
proyectada de dimensiones HuL; esto es,
H
1
Fh  S ghc
S  S g HL  S gH 2 L  S gR 2 L  19 600 N
2
2
Como consecuencia del Teorema de Arqumedes, la componente vertical del empuje es igual
al peso del volumen de agua desalojado (contenido) por la compuerta (indicado en la
figura); esto es, el volumen de medio cilindro ms el de un prisma de base triangular:
1

Q
1
Fv  S gV  S g Q R 2 R 2 R 2 L  S g 1 R 2 L  50388 N
2
2

2
H  2 R cos 45  2 R

Conocidas las componentes del empuje hidrosttico, calculamos su mdulo y su direccin de


F  Fh2 Fv2  S gR 2 L 1 ( Q2 1)2  2.76S gR 2 L  54 065 N  5517 kg
tag G 

Fv Q
 1  2.5708 l G  67.8
Fh 2

y su lnea o recta de accin pasa por el punto O (centro de la semicircunferencia), por ser la
resultante de una distribucin continua de fuerzas perpendicular a la compuerta en cada uno
de sus puntos (direccin radial).
b) El momento del empuje E con respecto al eje de la compuerta deber ser igual (y de signo
opuesto) al de la fuerza F. La magnitud de dicha fuerza ser mnima cuando su brazo, con
respecto al eje A, sea mximo; esto es, cuando sea igual a R, de modo que su lnea de accin
est inclinada 45. As, tenemos
M A  ER cos G  45
 FR l F  E cos 67.8 45
 49841 N  5086 kg

L
R 2

HR 2
O

Eh

R
A

Ev
45

Eh

E
F

- 262 -

Fsica Universitaria: Problemas de Fsica

Esttica de los fluidos. M29.19

19. Un depsito contiene un lquido de densidad U hasta su borde. Una compuerta


hecha con una chapa cuya forma es un cuarto de esfera de radio R est
adosada a un lateral del depsito, tal como se indica en la figura. Determinar
el empuje que acta sobre la compuerta, as como la lnea de accin del
mismo.

Determinamos la posicin del centro geomtrico (centroide) de un


semicrculo por medio del Segundo Teorema de Pappus-Guldin:
4 3 QR2
4R
QR 
q 2Qhc l hc 
3
2
3Q
Componente horizontal del empuje = empuje sobre la
superficie proyectada sobre el plano vertical:
V  SL l

4R QR2 2
q
 S gR 3
3Q
2
3
Componente vertical del empuje = peso del lquido situado
encima:

hc

Fh  (S ghc ) Sproy  S g

4 Q R3 Q
 S gR 3
3 4
3
El mdulo y la direccin del empuje, cuya lnea de accin pasa por el
centro de la esfera, sern:
1
F  Fh 2 Fv 2 
4 Q 2 S gR 3  1.24S gR 3
3
F
Q/3 Q
  1.571 l R  57.5
tg R  v 
Fh 2 / 3 2

S proy

Fv  S gV  S g

- 263 -

Fh

T
Fv
F

Fsica Universitaria: Problemas de Fsica

Esttica de los fluidos. M29.20

20. En una de las paredes verticales de un acuario hay un mirador de vidrio, de forma
hemisfrica, de radio R = 50 cm, cuyo centro est situado a una profundidad 3R. a)
Determinar el mdulo (newtons) y la direccin (en grados) de la fuerza que ejerce el
agua sobre el mirador, as como el punto de aplicacin de dicha fuerza. b) dem en el
caso de que el mirador fuese plano, de forma circular de radio R, contenido en el plano
de la pared.

a) Determinamos directamente las componentes horizontal y vertical de la


fuerza total que acta sobre el mirador.
Empuje sobre la proyeccin vertical (crculo):

3R

agua

Fh  S ghc S  S g (3R)(Q R 2 )  3QS gR 3  3Q q1000q9.8q 0.503  11.5q103 N


Empuje de Arqumedes:
2

2
2
Fv  SVg  S Q R 3 g  QS R 3  Q q1000q9.8q 0.503  2.57q103 N
3

3
3
Mdulo de la resultante:

2
85
F  Fh2 Fv2  32 QS gR 3 
QS gR 3  11.8q103 N
3
3
2

Fv

Fh

y su lnea de accin pasa por el centro de la hemisfera.


ngulo que forma con la horizontal:
F
2/3 2
tg R  v 

l R  12.5
3
9
Fh
b) Como antes, determinamos la fuerza horizontal que acta

37 2
SR
4

eje x sobre el mirador circular:


hcp

Fh  11.5q103 N

hc=3R

Calculamos el momento de rea de la superficie del mirador


con respecto al eje x:
1
37
I xx  SR 2 S (3R) 2  SR 2
4
4
Aplicamos el Teorema del Centro de Presiones:

1 2
SR
4

hc hcp 

I xx 37 2
37 R 2 37 R 2 37
 R l hcp 

 R  3 121
R  1.54 m
S
4
4 hc
4 3R 12

- 264 -

Fsica Universitaria: Problemas de Fsica

Esttica de los fluidos. M29.21

21. Una semiesfera hueca se halla sumergida en un lquido de densidad U1,

contiene en su interior otro lquido de densidad U2 (U2 ! U1) y descansa sobre


r
el fondo horizontal de un depsito. La superficie libre del lquido de menor
A
densidad se encuentra a una distancia por encima de la semiesfera igual al
r
radio de sta. Ambos lquidos, no miscibles, estn en contacto a travs de un
orificio (A) situado en la cima de la semiesfera. a) Calcular la presin en el
B
C
punto B, situado en el fondo y centro de la semiesfera. b) La presin en el
punto C, situado en el fondo y exterior a la semiesfera. Es mayor o menor
que en el punto B? c) Qu masa debera tener, como mnimo, la semiesfera para que el lquido interior no
la levante?

a) y b) La presin en los puntos B y C, teniendo en cuenta que la presin ejercida por el


lquido externo se transmite a travs del orificio A, es
pB  patm S1 gr S2 gr  patm S1 S2
gr
l pB  pC  S2  S1
gr  0 l pB  pC

pC  patm 2S1 gr

c) En general, la presin a una profundidad h, medida desde A, en los puntos interiores y


exteriores de la semiesfera son

pint (h)  patm S1 gr S2 gh

l pint (h)  pext (h)  S2  S1


gh

ext (h)  patm S1 gr S1 gh


de modo que la situacin es equivalente a la reflejada en la
A
figura, en la que hemos eliminado el lquido externo y
F
hemos sustituido el lquido interno por otro de densidad
(2 - 1). En definitiva, tenemos que determinar la fuerza
2- 1
vertical ejercida hacia arriba por el lquido interno (2 - 1),
en ausencia del lquido externo.
La distribucin de presiones sobre la cara interna de la semiesfera origina una fuerza
resultante vertical hacia arriba que podemos calcular mediante el Principio de Arqumedes,
ya que dicha distribucin es anloga a la que se
producira sobre la cara exterior (incluida la base) de
una semiesfera maciza sumergida en un fluido:
2- 1
EArq
2
FArq  Qr 3 S2  S1
g
3
Como la semiesfera hueca no tiene base, hay que
sustraer la fuerza ejercida sobre sta, i.e.,
Fbase  Qr 2 S2  S1
rg  Q S2  S1
gr 3

Fbase

de modo que
2
1
1
Q S2  S1
gr 3  Q S2  S1
gr 3   QS2 gr 3 l
F  Q S2  S1
gr 3
3
3
3
de modo que la masa de la semiesfera deber de ser, como mnimo,
1
mmn  Q S2  S1
r 3
3

- 265 -

Fsica Universitaria: Problemas de Fsica

Esttica de los fluidos. M29.22

22. Un bloque cbico, homogneo, de 20 cm de arista y densidad 0.7 g/cm3, est

sumergido en un recipiente que contiene aceite de densidad 0.8 g/cm3. La cara


inferior del cubo se apoya sobre una caera de 200 cm2 de seccin, que penetra
2 cm en el fondo del recipiente. La cara superior del cubo dista 40 cm de la
superficie libre del aceite. Calcular la presin manomtrica del aire que deberemos
insuflar por la caera para que el cubo se desprenda y comience a ascender.

40 cm

20 cm
2 cm
aire

P  S gV  700q9.8q 0.23  54.88 N


F1  Sac gh1
S1  800q9.8q 0.40q 0.202  125.44 N
40 cm

F1

F2  Sac gh2
S 2  800q9.8q 0.60q 0.02  94.08 N
Faire  paire
S3  0.02 paire

P
F2

F3
aire

20 cm
2 cm

En las condiciones crticas de equilibrio deber ser:


P F1  F2 Faire
de modo que
Faire  P F1  F2  86.24 N

As, la presin manomtrica del aire insuflado a travs de la caera es


F
86.24
 4312 Pa
paire  aire 
Saire
0.02

- 266 -

Fsica Universitaria: Problemas de Fsica

Esttica de los fluidos. M29.23

23. Un cuerpo de 1260 kg flota en agua marina (densidad 1.05 g/cm3) emergiendo 0.24 m3 sobre la superficie
libre del agua. a) Determnese la densidad del cuerpo. b) Realmente este cuerpo es poroso y est formado
por una masa slida de densidad 1.2 g/cm2 con pequesimas burbujas de aire atrapadas. Cul es el
porcentaje en volumen del aire existente en el cuerpo?
Nota: Despreciar la densidad (peso) de aire contenido en el cuerpo.

a) Como el cuerpo est en equilibrio de flotacin, podremos aplicar el principio de


Arqumedes para determinar el empuje vertical, que ser igual al peso del cuerpo. El empuje
est relacionado con el volumen sumergido VS, del modo
m 1260

 1.2 m3
E  Sm gVS  mg l VS 
E
Sm 1050
P

El volumen del cuerpo es


Vcuerpo  1.2 0.24  1.44 m

y la densidad de este cuerpo es


Scuerpo 

m
Vcuerpo

1260
kg
g
 875 3  0.875
1.44
m
cm3

b) El volumen de la masa slida, Vms, viene dado por


m 1260

 1.05 m3
Vms 
Sms 1200

de modo que el volumen de aire viene dado por


Vaire = 1.44 1.05 = 0.39 m3
que en porcentaje volumtrico representa
Vaire
0.39

 0.27  27%
Vtotal 1.44

- 267 -

Um

Fsica Universitaria: Problemas de Fsica

Dinmica de los fluidos ideales. M32.1

1. El giser Old Faithful (Yellowstone Park) expulsa peridicamente un chorro de agua que alcanza una altura
de hasta 40 m. a) Determinar la velocidad del agua en la base del chorro. b) Calcular la presin manomtrica que debe existir en el interior de giser, a una profundidad de 100 m, para que pueda proyectar el agua
hasta esa altura.

a) Aplicamos Bernoulli de 2 a 3:
1
patm S gz2 Sv 2  patm S gz3 l v 2  2 g z3  z2

2
de modo que

3
40m
v

v  2 g z3  z2
 2q9.8q 40  28 m/s  100.8 km/h

b) Aplicamos Bernoulli de 1 a 3:
p1 S gz1 0  patm S gz3 0 l

p1  patm  S g z3  z1

100 m

de donde
p1  patm  S g z3  z1
 1000q9.8q140  1.37 q106 Pa


1.37 q106
atm  13.6 atm
101 328

- 268 -

Fsica Universitaria: Problemas de Fsica

Dinmica de los fluidos ideales. M32.2

2. En un tramo de una tubera horizontal existe un estrechamiento


que reduce su seccin a la mitad. Si por la misma circula un lquido y la diferencia de presin que se origina entre un punto de
la tubera y el punto donde existe el estrechamiento equivale a la
presin que producira una columna del mismo lquido de altura
h, qu velocidad tendr el lquido en la tubera?

B
A

Aplicamos la ecuacin de Bernoulli entre los dos puntos, supuestos a la misma cota:
pA 12 SvA2  pB 12 SvB2 l

pA  pB  12 S (vB2  vA2 )  S gh

de modo que

vB2  vA2  2 gh

[1]

Tenemos en cuenta la ecuacin de continuidad:


vA SA  vB SB l vB 

SA
vA  2vA
SB

[2]

y sustituyendo este resultado en la expresin [1], tenemos


4vA2  vA2  3vA2  2 gh l vA 

- 269 -

2
3

gh

Fsica Universitaria: Problemas de Fsica

Dinmica de los fluidos ideales. M32.3

3. Para medir la velocidad del agua que circula por una tubera, se intercala en sta un venturmetro cuyos
dimetros en el tramo principal y en el estrechamiento se encuentra en la relacin 5:1. La diferencia de presin entre el tramo principal y el estrechamiento resulta ser de 0.35 atm. Cul es la velocidad?

v2

v1
2

Aplicamos el teorema de Bernoulli entre los puntos 1 y 2:


1
1
p1 Sv12  p2 Sv22
2
2

l v22  v12 

2 p1  p2

[1]

con
kg
101325 Pa
p1  p2  0.35atm q
 25331.25 Pa
3
m
1atm
Por otra parte, de la ecuacin de continuidad, se sigue:
S  1000

2
v1 S2 D22 D2 1
1
  2   
v2 S1 D1 D1 5
25
2

v1S1  v2 S 2

Resolviendo el sistema de ecuaciones dado por [1] y [2], se obtiene


v22  v12  70.9
m
m
l v1  0.34
v2  8.43

v2  25v1
s
s

- 270 -

[2]

Fsica Universitaria: Problemas de Fsica

Dinmica de los fluidos ideales. M32.4

4. En el dispositivo de la figura, un fluido ideal (densidad, U) circula

por una tubera de seccin constante con una velocidad v. La tubera lleva acoplados dos tubos piezomtricos, que permiten medir la
presin esttica y la presin dinmica, respectivamente. Sea Um la
densidad del lquido manomtrico. Expresar la velocidad v del fluido en funcin de la diferencia de cotas h que se indica en la figura.

Aplicamos reiteradamente la ecuacin de Bernoulli entre U


los puntos AB12CA y sumamos miembro a
miembro todas las ecuaciones (hemos procedido a sumas
parciales para mayor claridad).

pA S gzA  pB S gzB
AlB

pA S gzA  p1 S gz1
B l1
pB S gzB  p1 S gz1

1l 2
p1 Sm gz1  p2 Sm gz2

1 2

1 2 p2 S gz2  pA S gzA Sv
2
C l A pC S gzC  pA S gzA Sv

Obtenemos, despus de sumar m.a.m. todas las ecuaciones,


1
Sm gz1 S gz2  S gz1 +Sm gz2 Sv 2 l
2
1 2
Sm g z1  z2
 S g z1  z2
Sv l
2
1 2
Sm  S
g z1  z2
 Sv
2
Y, finalmente, con h = z1 z2, resulta
S

v 2  2 gh m 1
S

p2 S gz2  pC S gzC

- 271 -

Um
B

2lC

v C
z

Fsica Universitaria: Problemas de Fsica

Dinmica de los fluidos ideales. M32.5

5. Por una tubera circula un caudal de 208 L/s de agua. En la tubera se ha instala-

do un medidor de Venturi, con mercurio en su interior, tal como se representa en


la figura. Si las secciones de la tubera en 1 y 2 son 800 y 400 cm2, respectivamente, calcular el desnivel h que se produce en el mercurio.

Aplicamos la ec. de Bernoulli entre 1 y 2:


1
1
1
p1 Sv12  p2 Sv22 l p1  p2  S v22  v12

2
2
2
Aplicamos reiteradamente la ec. hidrosttica en el camino 1AB2:

1 l A
l p1 S gz1  pA S gzA

A l B
l

pA Sm gzA  pB Sm gzB

B l 2
l

pB S gzB  p2 S gz2

[1]

y sumando m.a.m.
p1 Sm gzA S gzB  S gzA Sm gzB p2 l
p1  p2  Sm  S
g zB  zA
 Sm  S
gh

m

[2]

Igualamos las ecuaciones [1] y [2]


1
2

Sm  S
gh  S v22  v12
l h 

S v22  v12
Sm  S 2 g

Aplicamos las ecuacin de continuidad entre los puntos 1 y 2:




  v1S1  v2 S2 l v1 
v2 
S1
S2

[3]

[4]

y sustituyendo las velocidades en la ec. [3]:


2
2

S   1
S 1
1  2


h



Sm  S S 2 S1 2 g Sm  S S 22 S12 2 g

Aplicacin numrica:
1
1000
1 0.2082

h

 0.082 m  82 mm

13600 1000 0.04


0.082 2q9.8

- 272 -

Fsica Universitaria: Problemas de Fsica

Dinmica de los fluidos ideales. M32.6

6. Para medir la velocidad del agua que circula por un arroyo, se dispone de un tubo en
L, como se muestra en la figura adjunta. Cul ser la velocidad de la corriente si el
agua asciende por el tubo vertical hasta una altura de 40 cm por encima de la superficie
libre del agua?

h
v

Aplicamos la ecuacin de Bernuilli entre AB, BC y CD, teniendo en cuenta que C es un punto
de estancamiento:
patm  pB  S gha
1
pB  S gh a Sv 2  pC  S gha
2
pC  S gh a  patm S gh
Sumando miembro a miembro las tres ecuaciones anteriores,
tenemos
1 2
Sv  S gh l v  2 gh
2
Y sustituyendo los valores numricos:
v  2q9.8q 0.40  7.84  2.8 m/s

- 273 -

D
A

h
C

Fsica Universitaria: Problemas de Fsica

Dinmica de los fluidos ideales. M32.7

7. En un pulverizador de pesticida se sopla aire sobre el extremo superior de un tubito abierto por sus dos extremos, estando el extremo inferior sumergido en un recipiente que contiene lquido de densidad
0.92 g/cm3. Cul deber ser la velocidad mnima del aire que pueda
elevar el lquido 10 cm para ser dispersado? (Densidad del aire,
1.25 g/L).

aire

1
lquido

Aplicamos la ecuacin de Bernoulli entre A y B, a lo largo de


la corriente de aire
1
pA Saire v 2  patm (1)
2
Aplicamos la ecuacin fundamental de la esttica de fluidos entre los puntos 1 y 2 (en el lquido)
(2)
patm  p2 Sliq gh
Sumamos miembro a miembro (1) y (2) y tenemos en cuenta que pA  p2 ,
1
Saire v 2  Sliq gh
2
de modo que
v

2Sliq gh

Saire

2q920q9.8q 0.1
 38 m/ s
1.25

- 274 -

Fsica Universitaria: Problemas de Fsica

Dinmica de los fluidos ideales. M32.8

8. Un tanque de grandes dimensiones abierto a la atmsfera est apoyado en el


suelo y contiene agua, como se indica en la figura. Se hace un agujero de pequeo dimetro a una altura h en la pared del tanque. a) Deducir la velocidad
de salida del agua por el orificio. b) Determinar el alcance horizontal x del
chorro de agua. c) Calcular la altura a la que debera hacerse el orificio para
que el alcance fuese mximo.

H
h
x

a) Aplicamos el teorema de Torricelli para calcular la velocidad de salida del agua por el orificio:

y
v0

v0  2 g H  h

b) Escribimos las ecuaciones paramtricas de la


trayectoria, que corresponde a la de un movimiento uniformemente acelerado; i.e., una trayectoria parablica,
x  v0t

g
l y  h  2 x2

y  h  1 gt 2
2v0

2
El alcance se determina haciendo y = 0, de modo que

y  h

h
x
xalc

g 2
2h
x  0 l xalc  v0
 2 h H  h

2
g
2v0

c) Puesto que el alcance es funcin de la posicin h a la que se encuentra el orificio, determinaremos su valor mximo imponiendo la condicin de extremo (mximo o mnimo) de la funcin:
dvalc
H  2h
H  2h
2

 0 l h  12 H
dh
h H  h

2 h H  h

- 275 -

Fsica Universitaria: Problemas de Fsica

Dinmica de los fluidos ideales. M32.9

9. Por la tubera de la figura, de secciones S1 = 40 cm2 y


S2 = 20 cm2, circula un caudal de agua de 3u10-3 m3/s.
Los tubos piezomtricos estn llenos de aceite de densidad 800 kg/m3. Se observa que el nivel del aceite en
los piezmetros tiene igual cota. Cul es el desnivel h
entre los dos tramos de tubera?
Nota: despreciar el valor de los radios de la tubera en la
expresin de las alturas.

S1
H

S2

h
n ive l d e r e fe r e nc ia

Datos: S1 = 40 cm2, S2 = 20 cm2, Q = 3u10-3 m3/s, Um = 800 kg/m3, U = 1000 kg/m3


S
(ec. continuidad)
Q  v1S1  v2 S 2 l v2  1 v1  2v1
S2
v1 

3q103
 0.75 m/ s
40q104

v2  1.50 m/ s

Aplicamos la ec. de Bernoulli entre 1 y 2:


1
1
1
3
p1 S gh Sv12  p2 Sv22  p2 S (4v12 ) l p2  p1  S gh  Sv12
2
2
2
2
Aplicamos la ec. de Bernoulli entre A-1 y entre B-2:

patm Sm gH  p1 Sm gh

l p2  p1  Sm gh
[2]

patm Sm gH  p2
Igualando las ecuaciones [1] y [2] tenemos:

[1]

3Sv12
3
3
S gh  Sv12  Sm gh l (S  Sm ) gh  Sv12 l h 
2
2
2(S  Sm ) g
de modo que
h

3q1000q 0.752
 0.43 m  43 cm
2(1000  800)q9.8

Otro modo operatorio:


Aplicamos la ecuacin de Bernoulli a la trayectoria A-1-2-B:

A l1
patm Sm gH  p1 Sm gh

1
1
1

p1 S gh Sv12  p2 Sv22  p2 S (4v12 )


1 l 2

2
2
2

S
2
B
l


p
p
gH
2
atm
m

y sumamos m.a.m. estas tres ecuaciones:


1
1
3
S gh Sv12  Sm gh S (4v12 ) l (S  Sm ) gh  Sv12
2
2
2

- 276 -

l h

3S v12
2 (S  Sm ) g

Fsica Universitaria: Problemas de Fsica

Dinmica de los fluidos ideales. M32.10

10. Un depsito de grandes dimensiones abierto a la atmsfera desagua por una tubera troncocnica que lleva incorporado un tubo en U invertida cuyo extremo inferior est
sumergido en otro depsito que contiene el mismo lquido. Determinar la altura h a la que asciende el lquido en
funcin de la distancia x.

L
H
X

x
S1

Calculamos la velocidad de salida en 2 mediante el


teorema de Torricelli:

S2

v2  2 gH

Aplicamos la ecuacin de Bernoulli y la condicin de continuidad entre los puntos X y 2:

1
1
pX SvX2  patm Sv22

2
2
Xl2
S2

v2
S X vX  S 2 v2 l vX 
S
X

de modo que, sustituyendo vX y v2 en la primera ecuacin, tenemos


S2
1
1 S2
pX  patm  S v22  vX2  1 22 Sv22  1 22 S gH  0

2
2 SX
SX
Aplicamos la ecuacin fundamental de la Esttica de Fluidos entre B y A, teniendo en cuenta
que la presin en A es igual a la presin en X:
patm  pX S gh l pX  patm  S gh
Igualando las dos ecuaciones anteriores, obtenemos
S2
S2
S2

S gh  1 22 S gH l h   1 22 H l h  22 1 H  0
SX

SX
SX

ya que S2 < SX, por lo que realmente el agua no asciende por el tubo, sino que desciende,
como se ilustra en la figura.
Tan solo nos queda expresar SX en funcin de la distancia x indicada en la
figura. El radio de la tubera en funcin de la distancia x es
R1  R2
S
S2
x
R1  1 R2 
con
L
Q
Q
de modo que, despus de un laborioso desarrollo, obtenemos
RX  R1 

S  2 S1S 2 S 2 2
S S  S1
SX  Q R  1
x 2 1 2
x S1
L2
L
2
X

- 277 -

X
B

h
A

Fsica Universitaria: Problemas de Fsica

Dinmica de los fluidos ideales. M32.11

11. El depsito de grandes dimensiones de la figura est abierto a la atms-

2
x

12 m

fera y desagua por la tubera que se indica. La seccin transversal de la


tubera en los puntos 2 y 3 es de 300 cm2 y en el 4 de 100 cm2. Calcular: a) El caudal de agua que fluye por la seccin 4. b) La presin en el
punto 3. c) La cota del punto 2 respecto a los puntos 3 y 4 para que la
presin en aqul sea de 1.2 atm.

a) Dado que el depsito est abierto a la atmsfera y que la


tubera desagua a la atmsfera, podemos aplicar directamente el Teorema de Torricelli:

v4  2 gz1  2q9.80q12  235.2  15.3 m/s


Q4  S 4 v4  100q104 q15.3  0.153 m3 /s

b) Aplicamos la ec. de continuidad y la ec. de Bernoulli entre 3 y 4:

S
100
1

S3v3  S4 v4 l v3  4 v4 
v4  v4  5.1 m/s

S
300
3
3

1
1
1
4

p3 Sv32  patm Sv42 l p3  patm  S v42  v32


 Sv42

2
2
2
9

de modo que
4
4
p3  patm  Sv42  q1000q 235.2  104 533 Pa  1.032 atm l
9
9
c) Aplicamos la ec. de Bernoulli entre 2 y 3:
p2 S gz2  p3 l z2 

p3  2.032 atm

p3  p2 2.032 1.2
q101 328

 8.60 m
Sg
1000q9.8

- 278 -

Fsica Universitaria: Problemas de Fsica

Dinmica de los fluidos ideales. M32.12

12. Clepsidra (reloj de agua). Determinar la forma que debe darse a un recipiente con simetra de revolucin
alrededor de un eje vertical para que al vaciarse por un orificio situado en su fondo sea constante la velocidad de descenso del nivel del agua que contiene.

Aplicamos Bernoulli entre la superficie libre (1) y el orificio de salida (2):


1
1
patm S gz Sv12  patm Sv22 l 2 gz v12  v22
2
2
Y la ecuacin de continuidad en esos mismos puntos:

z
1

Sv1  S0 v2
r
S0

l Qr 2 v1  Qr02 v2

l v2 

r2
v1
r02

Despejando la cota z en la primera ecuacin y sustituyendo


el valor de v2 obtenido en la segunda, tenemos
v2 r 4
1 2
z
v2  v12
 1 4 1  ar 4  b

2g
2 g r0

Puesto que la velocidad de descenso del nivel del agua es constante, esto es, independiente del
tiempo, la clepsidra es un autntico reloj de agua.
Podemos utilizar cualquier software matemtico para representar en 3D la clepsidra.

- 279 -

Fsica Universitaria: Problemas de Fsica

Dinmica de los fluidos ideales. M32.13

13. Un depsito abierto, cilndrico, de eje vertical y seccin recta S1 est lleno de agua hasta una altura H por
encima de su fondo. Determinar el tiempo necesario para que se vace el depsito a travs de un orificio bien
perfilado, de rea S2, practicado en su fondo. Aplicacin numrica: S1 = 2 m2, S2 = 10 cm2, H = 3 m.

Aplicamos la ecuacin de Bernoulli entre la superficie libre del


agua en el depsito y el orificio de salida
1
1
patm S gz Sv12  patm 0 Sv22 l 2 gz v12  v22 [1]
2
2
Aplicamos la ecuacin de continuidad entre esos dos mismos puntos
S
S1v1  S2 v2 l v2  1 v1 [2]
S2

S1

H
v1
z
S2

Eliminamos la velocidad de salida (v2) entre estas dos ecuaciones,


tenemos

v2

S
S2
2 gz
2 gz v  1 v12 l v12 12 1  2 gz l v1 
 Az
2
2
S

S 2
S

2
1 S 2 1
2

2
1

De modo que v1  

dz
2g
 Az con A  2 2
dt
S1 S 2 1

Integramos la ecuacin diferencial para obtener t(z):

dt  
H

dz
1
1
1/2
1/2

z dz   A 2 z
Az
A H

z
H

2
H z
A
Cuando se depsito se vace, ser z = 0, lo que requiere que transcurra un tiempo t0 tal que
= t

t0  2

H
2 H S12

1
A
g S 22

Aplicacin numrica:
A

2g
2q9.8

 4.9q106 m/s 2
S12 S22 1 2q104 10
2 1
t0  2

3
 1565 s  26 min 5 s
4.9q106

- 280 -

Fsica Universitaria: Problemas de Fsica

Dinmica de los fluidos ideales. M32.14

14. Un sifn es un dispositivo que se utiliza para extraer lquido de un depsito. Su forma de operar
se muestra en la figura adjunta. El extremo del tubo que est sumergido en el lquido puede estarlo a cualquier profundidad. Naturalmente, para que el sifn funcione deber estar inicialmente lleno de agua; pero una vez que est lleno, el sifn succionar lquido del depsito hasta que
el nivel en ste descienda por debajo del nivel del extremo del tubo abierto al aire libre. Supongamos que el lquido sea agua a 15.5 (C (ps = 13 Torr) y despreciemos totalmente la friccin.
a) Determinar la velocidad de salida del lquido por el extremo inferior del tubo del sifn.
b) Cunto vale la presin absoluta en el punto ms alto del tubo? c) A qu altura mxima sobre el extremo inferior del tubo puede estar el punto ms alto del tubo sin que el sifn falle por
cavitacin?

a) Aplicamos la ecuacin de Bernoulli entre A-C


1
patm S gzA 0  patm 0 Sv 2 l v  2 gzA
2
Esto es, la velocidad de salida viene dada por el Teorema de Torricelli,
siendo h la diferencia de niveles entre la superficie libre del lquido (A) y
la salida del tubo de desage (C).
b) Aplicamos la ecuacin de Bernoulli entre B-C

B
A

pB S gzB
=

z=0

1 2
1
Sv  patm 0 Sv 2
2
2

pB  patm  S gzB  0

c) Se presentar cavitacin, esto es, la formacin de burbujas de vapor en


el seno del lquido, causada por las variaciones que este experimenta en su
presin, si la presin en B es inferior a la presin de vapor saturante del
agua:

pB  patm  S gzB  ps

l zB 

pB  patm 760 12.788


101328

 10.2 m
Sg
1000q9.8
760

- 281 -

Fsica Universitaria: Problemas de Fsica

Dinmica de los fluidos ideales. M32.15


A

15. Un depsito de grandes dimensiones desagua mediante un tubo sifn


de seccin S y terminado en un estrechamiento de seccin S/4, como se
indica en la figura. a) Determinar la presin en A. b) Calcular valor
mximo de h3 para que el depsito contine desaguando.

h1
B

h2

a) Aplicamos la ecuacin de Bernoulli entre B-C:


patm S g (h2 h3 ) 0  patm 0 12 SvC2

h3

l v  S g (h2 h3 )

Nivel de ref.

2
C

Ecuacin de continuidad entre A-C:


vA S  vC

S
4

vA  14 vC

Ecuacin de Bernoulli entre B-A:

patm S g (h2 h3 )  pA S g (h1 h2 h3 ) 12 SvA2

v
patm  pA S gh1 12 S C  pA S gh1 161 S g (h2 h3 )
4
2

pA  patm  S gh1  161 S g ( h2 h3 )  patm  S g < h1 161 (h2 h3 ) >

b) Para pA | 0, (en realidad pA | ps, presin de vapor saturante), ser:


patm
 h1 161 (h2 h3 )
Sg

- 282 -

h3 

16 patm
16h1  h2
Sg

Fsica Universitaria: Problemas de Fsica

Dinmica de los fluidos ideales. M32.16

16. Disponemos de un depsito de agua de grandes dimensiones y abierto a la atmsfera


que desagua a la atmsfera a travs de un tubo vertical, de seccin constante, que est
acoplado a su fondo, tal como se indica en la figura. a) Calcular la velocidad de salida
del agua por el tubo. b) Expresar la presin en funcin de la cota z medida a partir del
extremo inferior del tubo, representarla grficamente y explicar la cada de presin en
la entrada del tubo de desage.

a) Aplicamos la ec. de Bernoulli entre 1 y 3:


1
patm S gH  patm Sv 2 l v  2 gH
2
que es el mismo resultado que nos proporciona el Teorema de
Torricelli.
b) Aplicamos la ec. de Bernoulli entre el punto 1 y un punto
genrico contenido en el depsito (de cota h < z < H):
patm S gH  p S gz l

H
h

1
2

H
z

p  patm S g H  z

Aplicamos la ec. de Bernoulli entre el punto 3 y un punto gen3


rico contenido en el tubo (de cota 0 < z < h):
1
1
patm Sv 2  p S gz Sv 2 l p  patm  S gz
2
2
En la representacin grfica, observamos que la presin presenta una discontinuidad (cada
brusca) en la entrada del tubo, ya que
p(h )  patm S g H  h

l p(h )  p (h )  S gH

p(h )  p  S gh
atm

que es consecuencia de haber considerado despreciable la velocidad del agua en el depsito,


incluso en las proximidades de la entrada del tubo, lo que es una aproximacin que no se ajusta a la realidad.

p
patm+g(H-h)
gH

patm
patm-gh
h

- 283 -

Fsica Universitaria: Problemas de Fsica

Dinmica de los fluidos ideales. M32.17

17. Calcular la longitud x del tubo de salida del agua del depsito de la figura para que la presin en B sea 1/n
de la presin en A (presin atmosfrica), teniendo en cuenta que SA  SB.

Mtodo 1
Simplemente, aplicamos el Teorema de Bernoulli entre BC,
teniendo en cuenta que la velocidad es la misma en ambos
puntos:
patm
1
1
S gx Sv 2  patm Sv 2
n
2
2
n 1
l S gx 
patm
n
n 1 patm
= x
n Sg

h
B

z=0

C
Mtodo 2
Aplicamos el Teorema de Bernoulli entre AB y AC

p
patm S g h x
 atm S gx 1 Sv 2

2
n

1 2
patm S g h x
 patm Sv
2

Restando miembro a miembro estas dos ecuaciones:


patm
1
1
n 1
n 1 patm
S gx Sv 2  patm Sv 2 l S gx 
patm = x 
2
2
n
n
n Sg

Mtodo 3
Obsrvese que la aplicacin del Teorema de Bernoulli entre A y C equivale a la aplicacin del
Teorema de Torricelli para calcular la velocidad en el desage, de modo que el problema
tambin puede plantearse de modo que sigue:

patm 1 2

patm S gh  n 2 Sv l p S gh  patm S g (h x)

atm

n
2

2
(
)
v
g
h
x


De modo que
n 1
n 1 patm
patm  S gx l x 
n
n Sg

- 284 -

Fsica Universitaria: Problemas de Fsica

Dinmica de los fluidos ideales. M32.18

18. Desde un estanque de grandes dimensiones se desagua a un gran

depsito mediante una tubera de seccin S1. Desde este depsito, a


su vez, se desagua mediante una tubera de seccin S2, tal como se
indica en la figura. a) Determinar la relacin entre los diferentes datos para que permanezca constante el nivel del depsito intermedio.
b) dem para que sean idnticas las velocidades de desage en ambas tuberas.

h1

h2
C

Empezamos determinando las velocidades de desage en


ambas tuberas.
Aplicamos la ecuacin de Bernoulli entre
1
A-B
p0 S g (h1 h2 h3 )  pB Sv12
JJJG
2
B-C
pB  p0 S gh3
JJJG

h4

h5

que sumadas miembro a miembro, nos conducen a


1
S g (h1 h2 )  Sv12 l v1  2 g (h1 h2 )
2
Aplicamos la ecuacin de Bernoulli entre
1
p0 S g (h4 h5 )  p0 Sv22 l v2  2 g (h4 h5 )
C-D
JJJG
2
Debern ser iguales los caudales en ambos desages; i.e.,
S1v1  S2 v2 l S12 (h1 h2 )  S22 (h4 h5 )
Igualamos las velocidades; i.e.,
v1  v2

l (h1 h2 )  (h4 h5 )

- 285 -

h3

Fsica Universitaria: Problemas de Fsica

Dinmica de los fluidos ideales. M32.19

19. Trasvasando aceite. Dos depsitos de grandes dimensiones, abiertos a la atmsfera, contienen aceite de
oliva (0.918 g/cm3), existiendo un desnivel entre las superficies libres del aceite en ellos de 10 m. Los depsitos estn intercomunicados mediante una tubera horizontal, de 120 mm de dimetro, con entradas bien
perfiladas por debajo de los niveles de aceite en cada depsito. a) Determinar el caudal que circula por la tubera. b) Calcular la potencia nominal de la bomba que se necesitar (70% de rendimiento) para conseguir el
mismo caudal en sentido inverso.

a) Calculamos la diferencia de presiones entre los extremos de la tubera que comunica los
depsitos:
1
p0 S gz1  p2

l p2  p4  S gh (obvio)
h
5
p0 S gz5  p4

Puesto que no hay prdidas de energa, la cada de presin entre


(2) y (4) ser igual al incremento de energa cintica en la tubeaceite
ra; esto es,
1
p2  p4  S gh  Sv 2 l v  2 gh  14 m/s
v
2
2
4
Resultado al que tambin podemos llegar directamente aplican3
do el teorema de Torricelli.
El caudal ser
Q  Sv  Q q 0.062 q14  0.158 m3 /s  158 A /s

b) La bomba deber suministrar una potencia P a la corriente


fluida, dada por
P  ' w  SQw
p  p2
con w  (e4  e2 )  ( 4
)  gh
S
de modo que

aceite
bomba
2

P  (S gh)Q

4
w

O sea

P  918q9.8q10q0.158  14 214 W  14.2 kW


14.2
Pnom 
 20.3 kW
0.70

- 286 -

Fsica Universitaria: Problemas de Fsica

Dinmica de los fluidos ideales. M32.20

20. El depsito de grandes dimensiones

AB

CD
de la figura contiene un lquido ideal
M
que desagua por la tubera de seccin
variable que se indica situada a una
profundidad H. a) Calcular la velociH
dad del lquido en cada uno de los tres
tramos de la tubera. b) Calcular y dibujar la altura que alcanza el lquido
en cada uno de los tubos piezom2S
3S
tricos. c) Dibujar de forma aproximada la altura que tendra el lquido en los tubos piezomtricos si el lquido fuese viscoso.

a) Teorema de Bernouilli entre M y N:


1
patm S gH 0  patm 0 SvS2 l vS  2 gH
2
Ecuacin de continuidad:

1
1
v2S 2S  vS S l v2S  vS 
2 gH
2
2

1
1
2 gH
v3S 3S  vS S l v3S  vS 
3
3

b) Toma de presin esttica en X :


pX  patm
Sg

pX  patm S gh l hX 
Teorema de Bernouilli entre M y X:
1

patm S gH 0  pX 0 SvX2  l
2

Combinando las dos ecuaciones anteriores: hX  H 

pX  patm  S gH  SvX2
2

2
X

v
2g

Tubos A y B:

hA  H 

2
v3S
1
8
H H  H
2g
9
9

Tubos C y D:

hC  H 

2
v2S
1
3
H H  H
2g
4
4

Tubo E:

hA  H 

vS2
 H H  0
2g

- 287 -

N
S

Fsica Universitaria: Problemas de Fsica

Dinmica de los fluidos ideales. M32.21

21. Un depsito de grandes dimensiones desagua a

la atmsfera mediante el sistema de tuberas


que se representa. Determinar la velocidad y el
caudal del agua en cada tramo de tubera, as
como la presin en el punto A. Datos: S0 =
100 cm2.

10m

1m
S0
1 A
Consideraremos los tres tramos del sistema numerados (1, 2 y 3) tal como se S
0
representa en la figura.
Determinamos las velocidades en los tramos 2 y 3 aplicando el teorema de Torricelli:
3S0

v2  2 gh2  2q9.8q12  15.33 m/s


v3  2 gh3  2q9.8q14  16.56 m/s
y los caudales en estos tramos son
Q2  v2 S0  15.33q 0.01  0.1533 m3 / s  153.3 L/s
Q3  v3 S0  16.56q 0.01  0.1656 m3 / s  165.6 L/s
El caudal y la velocidad en el tramo 1 son:
Q1  Q2 Q3  0.3189 m3 /s = 318.9 L/s
v1 

Q1
 10.63 m/s
3S0

Aplicamos la ecuacin de Bernoulli entre A y B:


1
1
pA S gzA SvA2  pB S gzB S vB2
2
2
Despejando pA , teniendo en cuenta que pB = patm y que vB = 0, resulta
1
pA  patm  S g zB  zA
 SvA2
2
1
2
pA  patm  1000q9.8q11  1000 10.63
 107800 -56 498 = 51302 Pa = 0.506 atm
2
pA  152 627 Pa = 1.506 atm

- 288 -

2m
2
2m
3

Fsica Universitaria: Problemas de Fsica

Dinmica de los fluidos ideales. M32.22

un depsito de grandes dimensiones, eleva agua hasta los


niveles A y B a travs del sistema de tuberas representado. El agua sale en el nivel A con una velocidad de 5 m/s.
a) Determinar la velocidad de salida en B. b) Calcular los
caudales suministrados en A y B.

22. Una bomba de 5 kW de potencia, situada en el fondo de

6m

5m

15 m

2S

3S

N.R.

a) Aplicamos la ecuacin de Bernoulli entre C-A y C-B:


1
1

CA l pC SvC2 S gzC  patm SvA2 S gzA


1 2
1

2
2
SvA S gzA  SvB2 S gzB
l

1 2
1 2
2
2
CB l pC SvC S gzC  patm SvB S gzB

2
2

de modo que
vB2  vA2 S g zA  zB
l vB2  52 2q9.8q (1)  5.4 m 2 /s 2

l vB  2.32 m/s

Ecuacin de continuidad en la bifurcacin:


vA 2vB
 3.22 m/s
3
b) La potencia proporcionada por la bomba ser P =wQ, donde Q es el caudal total (en C) y
w es el trabajo especfico realizado sobre el fluido, tal que, aplicando la ec. de Bernoulli
(energas especficas) entre O y A, i.e., e0 w  eA , incluyendo la bomba, tenemos
3SvC  SvA 2SvB l vC 

1
1
1
1
w  eA  e0  patm  patm
vA2  v02 g zA  z0
 pC  p0
vA2 g zA  z0

2
2
S
S
1
1
S w  SvA2 S g zA  z0
 1000q52 1000q9.8q5  61500 Pa
2
2
El caudal principal ser
5000
P

 0.0813 m3 /s  81.3 L/s
QC 
S w 61500
La seccin de la tubera principal ser:
Q
0.0813
 0.0253 m 2  25.3q103 m 2
SC  3S  C 
3.22
vC
De modo que las secciones y caudales en las tuberas secundarias son:
S A  8.42q103 m 2

l QA  S A vA  42.1q103 m 2  42.1 L/s

S B  16.8q103 m 2

l QB  S BvB  39.1q103 m 2  39.1 L/s

- 289 -

Fsica Universitaria: Problemas de Fsica

Dinmica de los fluidos reales. M33.1

1. Una lmina de aluminio (U = 2.70 g/cm3) desliza por un plano inclinado bajo la accin de la gravedad y de

la fuerza viscosa ejercida sobre ella por una fina pelcula de aceite lubricante SAE-30 (K = 250 cP) de
0.25 mm de espesor, depositada sobre el plano. Las dimensiones de la lmina son 10u4u1 cm3 y el ngulo
de inclinacin del plano respecto de la horizontal es de 5. Calcular la velocidad lmite que alcanza la lmina
en su descenso.

Datos:
Al = 2.70 g/cm3
h = 9.25 mm = 0.025 cm
S = 4u10 = 40 cm2
 = 250 cP = 2.5 P

Fvisc

v
mg

El flujo del lquido en el espacio comprendido entre las dos superficies se reduce a un flujo
de Couette, como se ilustra en la figura inferior. Aplicamos la ley de la viscosidad de
Newton:.
v
v
V =K
o f visc V S K S
h
h
ya que el esfuerzo cortante est uniformemente distribuido sobre la cara de la lmina en
contacto con el aceite.
Bajo la accin de las fuerzas que actan sobre la lmina (indicadas en la figura superior), sta
se acelera, aumentando continuamente su velocidad de bajada y la resistencia viscosa a su
movimiento, hasta que finalmente se alcanza una cierta velocidad lmite. Las ec. del
movimiento se escribe en la forma:
v
mgh sen T
mg sen T  f visc ma 0 o mg sen T K S lm o vlm
h
KS

con m

U AlV

2.7 u (10 u 4 u 1) 108 g , de modo que


vlm

108 u 980 u 0.025 u sen 5


2.5 u (10 u 4)

- 290 -

2.31 cm

Fsica Universitaria: Problemas de Fsica

Dinmica de los fluidos reales. M33.2

2. Un cilindro macizo de radio R = 10 cm y altura h = 10 cm gira en el interior de otro cilindro hueco de radio

R + 2 mm y altura h + 4 mm. En el espacio entre ambos cilindros existe un lquido de viscosidad K = 84 cP


que rodea totalmente el cilindro interior. Determnese el momento que hay que aplicar al eje del cilindro
mvil y potencia necesaria para mantener una velocidad de rotacin constante de 1000 r.p.m.

Datos:
2Q
 104.72 rad/s
60
v  X R  104.72q10  1047.2 cm/s
X  1000

R+R
El flujo del lquido en el espacio entre los dos cilindros se

reduce a un flujo de Couette, como se ilustra en la imagen
inferior. Aplicamos la ley de la viscosidad de Newton:
v
1047.2
T=I
 0.84
 4398.23 dyn/cm 2
%R
0.2
Este esfuerzo cortante est uniformemente distribuido
sobre la superficie lateral del cilindro interior mvil. En consecuencia, el momento dinmico
con respecto al eje de rotacin ser:

M  T (2Q Rh) R  2Q R 2 hT  2Q q102 q10q 4398.23  2.76q107 dyn cm


= M  2.76q107 dyn cm  2.76 N m

que es igual al momento que deberemos aplicar para mantener el


cilindro interior en movimiento.
La potencia necesaria para mantener constante la velocidad de
rotacin ser:
P  M X  2.76q104.72  289 W

- 291 -

v=R

Fsica Universitaria: Problemas de Fsica

Flujo viscoso. M34.1

1. a) Determinar la direccin del flujo en la tubera de seccin constante representada en la figura adjunta. b) Calcular el caudal y el
nmero de Reynolds del flujo. El fluido es aceite de oliva a 20 C.
Datos: densidad, 0.918 g/cm3; viscosidad, 84 cP;

1 kg/cm2
1

12 m

45 mm

30
kg
Datos: 1 2  9.8q104 Pa; I  0.84 P  0.084 Pa.s
cm
a) Determinamos la prdida de carga entre los puntos 1 y 2:
p  p2
1
p  p2
H l  e1  e2  1
gh v12  v22
 1
gh 
2
S
S


1 2
q9.8q104
918

9.8q12sen 30  106.81 58.84  47.97

2
2 kg/cm2

J
kg

La direccin del flujo es la de la prdida de carga (disminucin de energa); por consiguiente


se dirige desde abajo hacia arriba, (2) o (1) .
b) Expresamos la prdida de carga (Hl) en trminos de presin (prdida de presin, Hp):
kg
H p  S H l  '~  918q 47.97  44 039 Pa  0.45
cm 2
Aplicamos la ley de Hagen-Pouiseuille
Q

m3
L
Q D 4  '~
Q q 0.0454 44 039

 4.4q103
 4.4
128I L
128q 0.084 12
s
s

La velocidad media del fluido en la tubera es:


4Q
QD2
V l V
 2.76 m/s
4
QD2
Calculamos el nmero de Reynolds del flujo:
S DV
 1360
R
I
Q  SV 

de modo que se trata de un flujo laminar, por ser R < 2300.

~1  p1 S gh  98000 918q9.8q 6  151978 Pa l H  ~ ~   a~  44 022 Pa


p
1
2

~ 2  p2  196 000 Pa

- 292 -

Fsica Universitaria: Problemas de Fsica

Flujo viscoso. M34.2

2. El lquido de un depsito de grandes dimensiones se vaca por medio de un tubo horizontal de 250 m de
largo y 20 mm2 de seccin, que est situado a 15 m por debajo del nivel del lquido. Sabiendo que la densidad del lquido es 1 g/cm3 y su velocidad de salida es de 4.67 cm/s, calclese su viscosidad. Determinar si el
flujo es laminar.

Datos:

S = 0.20 cm2 ; D  4S / Q  4q 0.2 / Q  0.50 cm

15 m

v = 4.67 cm/s; Q  Sv  0.934 cm3 /s

20 mm2

l = 250 m = 25u103 cm; h = 15 m = 1.5u103 cm

250 m

Mtodo 1 (simple):
Calculamos la cada de presin en el tubo:

p  S gh  1q980q1.5q103  1.470q106 barias


Aplicamos la ecuacin de Poiseuille:
Q

Q D 4 p
128Il

l I

Q D 4 p Q q 0.504 q1.47 q106



 0.097 P  9.7 cP
128Ql
128q 0.934q 25q103

Calculamos el nmero de Reynolds:


S DV 1q 0.50q 4.67

 24  2300 (laminar)

I
0.097
Mtodo 2 (avanzado):
Aplicamos la ecuacin de Bernoulli (sin prdidas) entre 0 y 1:
patm S gh

1 2
1
Sv0  p1 Sv 2
2
2

1
p1  patm  S gh  Sv 2
2

Como el tubo horizontal desagua a la atmsfera, p2 = patm, de


modo que la cada de presin entre los extremos del tubo de
desage es,
1
p1  p2  p1  patm  S gh  Sv 2 
2
1q 4.67 2
3
 1q980q1.5q10 

2
 1.47 q106 11  1.47 q106 barias

0
15m
1
250 m

Aplicamos la ecuacin Poiseuille (flujo viscoso) entre 1 y 2:


Q
I

Q D 4  ap Q D 4 p1  p2

l
128I l
128I

Q D 4 p1  p2
Q q 0.54 q1.47q106

 0.097 P  9.7 cP
128Q
l
128q 0.934q 25q103

- 293 -

Fsica Universitaria: Problemas de Fsica

Flujo viscoso. M34.3

3. El agua de un depsito de grandes dimensiones se vierte por medio de un tubo

horizontal de 250 m de largo y 20 mm2 de seccin situado a 15 m por debajo del


nivel del agua en el depsito. Calcular la velocidad y el caudal de salida del agua.
Datos: viscosidad del agua = 1 mPas

15m

250 m

(1) Aplicamos la ecuacin de Bernoulli (sin prdidas) entre 0 y 1:


patm S gh

1 2
1
Sv0  p1 Sv 2
2
2

1
p1  patm S gh  Sv 2
2

(2) Aplicamos la ecuacin Poiseuille (flujo viscoso) entre 1 y 2:


Q

Q D 4 %a p Q D 4 p1  p2

L
128I L
128I

con p2 = patm, esto es,


1
p1  p2  S gh  Sv 2
2

de modo que
Q D
1 2
S gh  Sv
128I L
2
4

Q

15m

Teniendo en cuenta que


D2 

4S
Q

l D4 

Q  vS l v 

16S 2
Q2

2
250 m

Q
S

resulta
Q

Q 16 S 2
1 Q 2 S S 2
1 Q 2
l
S gh  S 2 
gh 
2
128I L Q
2 S 8IQ L
2 S 2

8IQ L
1 Q2
Q

gh

SS 2
2 S2

16IQ L
Q  2 S 2 gh  Q 2
S

l Q2

16IQ L
Q  2 S 2 gh  0
S

y sustituyendo valores, queda la ecuacin


2
16q103 Q q 250
Q  2 20q106
9.8q15  0
1000
Q 2 12.57 q103 Q 117.6q109  0

Q2

Y resolviendo esta ecuacin de segundo grado resulta


Q  9.34q106 m3 /s  9.34 cm3 /s l v 

- 294 -

Q
 0.467 m/s
S

Fsica Universitaria: Problemas de Fsica

Flujo viscoso. M34.4

4. El agua de un recipiente cilndrico de 5 cm de dimetro


abierto a la atmsfera se vaca a travs de un tubo delgado de 0.5 mm de dimetro y 20 cm de longitud. Inicialmente la altura del agua en el depsito es de 10 cm.
a) Considerando el agua como un fluido ideal, calcular
la velocidad de salida del lquido en el instante inicial y
el tiempo necesario para que el nivel del lquido descienda a 5 cm. b) Siendo la viscosidad del agua de
1 mPas, determinar la velocidad real de salida por el tubo, en el instante inicial, despreciando la velocidad del
lquido en el depsito.

A
10 cm
B
20 cm

a) La secciones del recipiente y del tubo son:

S A  14 Q DA2  14 Q q 0.052  1.96q103 m 2


S B  14 Q DB2  14 Q q (0.5q103 ) 2  1.96q104

A
 104

m 2 S B

Bernoulli entre A-B:

patm S gH 12 SvA2  patm 0 12 SvB2 l vB2  vA2 2 gH

[1]

Ecuacin de continuidad entre A-B:


vA SA  vB SB l vA 

SB
vB  104 vB
SA

[2]

Sustituimos [2] en [1]:


2gH
 2q9.8q 0.10  1.40 m/s
1108
En un instante genrico, cuando la altura del agua en el recipiente sea z, la expresin [1] se
escribe en la forma:
vB2  108 vB2 2 gH

l vB 

vA2  vB2  2 gz  108 vA  2 gz l vA 

2 gz
dz
 104 2 gz  
8
10 1
dt

104 2 g d t  z 1/2 d z l 104 2 g d t  z 1/ 2 d z  2 z  2( H  z )

H
H

2q10 ( H  z ) 2q10 ( 0.10  0.05)



 418 s
2g
2q9.8
4

= t

b) Aplicamos Bernoulli con prdidas entre A-B y sustituimos la prdida de presin por la
expresin dada por la frmula de Hagen-Poiseuille:

patm  U gH  12 U vA2
64K l

o v  U D 4 vB  2 gH
2
B

patm  0  12 U vB2  ('cp ) visc [3] o U gH


0 o

1
2

U vB2  128K4l ( SBvB )

3
v  64u10 u0.20 4 vB  2 u 9.8 u 0.10

1000u 0.5u103
2
B

o vB2  51.2 vB  1.96 0 o vB

SD

0.0383 m/s = 3.83 cm/s

Obsrvese la gran importancia de los efectos viscosos en los tubos de descarga muy delgados.
Adems, puesto que vB2  0.03832  1.467 q103  1.96  2 gH , podemos despreciar el trmino 12 SvB2 en la expresin [3], lo que simplifica notablemente los clculos.

- 295 -

Fsica Universitaria: Problemas de Fsica

Flujo viscoso. M34.5

5. Un aparato estndar para hacer demostraciones acerca de la prdida

25 cm

de carga a lo largo de una tubera est constituido por un depsito


de grandes dimensiones que desagua a la atmsfera a travs de una
15 cm
tubera horizontal de longitud L y seccin constante, de 8 mm de
11 cm
dimetro interno. La entrada de la tubera est bien perfilada y pueden despreciarse las prdidas menores en la misma. A lo largo de la
12 cm
12 cm
tubera se han dispuesto dos tubos manomtricos verticales, como
se ilustra en la figura adjunta. En el instante en que el nivel de agua
en el depsito se encuentra a 25 cm por encima de la entrada de la tubera y los manmetros indican 15 cm y
11 cm, respectivamente. a) Cul es la longitud de la tubera? b) En el instante mencionado, cul es el caudal en el desage? c) Calcular el nmero de Reynolds del flujo en la tubera.
Datos: densidad, 1 g/cm3; viscosidad, 1.002 cP.

25 cm
15 cm
11 cm

h2
h3

h4

12 cm
2

12 cm
3

a) Cada lineal de presiones.


Una simple relacin de semejanza de tringulos en la figura
nos permite determinar la longitud de la tubera.
15 L 12

l
11 L  24
15 L  360  11L 132 l

L

228
 57 cm
4

b) La presin esttica existente a la profundidad de entrada de la tubera es


p2  patm S gh2

Aplicamos la ec. de Bernouilli sin prdidas entre 1-2:


patm S gh1  p2 12 Sv 2
Sumando m.a.m. estas dos ecuaciones:
S gh1  S gh2 12 Sv 2

l v  2 g h1  h2
 2q9.8q 0.06  1.085 m/s

La disminucin de altura piezomtrica entre 1 y 2 est asociada al aumento de velocidad


entre esos puntos.
El caudal de salida es

QD2
Q q 0.0082
m3
cm3
v
q1.085  54.5q106
 54.5
4
4
s
s
c) Nmero de Reynolds:
Q

`

S DV 1000q8q1031.085

 8663  2300 (turbulento)
I
1.002q103

La ley de Hagen-Pouiseuille no conducira a

Q

1000q9.8q 0.04
m3
cm3
Q D 4  '~
Q q 0.0084
 3.27q106
 327


3
128I l 128q1.002q10
0.12
s
s

No es aplicable por tratarse de un flujo turbulento.

- 296 -

Fsica Universitaria: Problemas de Fsica

Flujo viscoso. M34.6

6. Un depsito de grandes dimensiones abierto a la atmsfera contiene petrleo de densidad 860 kg/m3 y viscosidad 7.2 mPas. El petrleo debe transportarse mediante una tubera lisa de acero de dimetro 0.12 m y
1 km de longitud que descarga a la atmsfera. Determinar la potencia que debe suministrar la bomba que
impulse el lquido para mantener un caudal de 0.06 m3/s en los siguientes casos: a) El extremo por el que
descarga la tubera est a la misma cota que el nivel del petrleo en el depsito. b) Dicho extremo est situado a 30 m por encima del nivel del petrleo en el depsito.

Mediante la ecuacin de Poiseuille, calculamos la cada de presin debida a la friccin (viscosidad) en la conduccin:
128Il
128q 0.0072q1000
pf 
Q
q0.06  84883Pa
4
4
QD
Q 0.12

Velocidad de circulacin en la conduccin:


Q
0.06
 5.3 m/s
Q  v2 S v2  
S Q 0.062

Nmero de Reynolds del flujo:


S DV 860q 0.12q5.31
R

 76 041 (turbulento)
I
0.0072
(a pesar de ello, continuamos con el problema, aunque los resultados discreparn notablemete de la realidad).

Estableciendo el balance energtico entre los puntos 1 y 2, llamando w al trabajo especfico


(valor absoluto) que suministra la bomba y qvisc a la prdida de energa especfica (valor absoluto) debido a la friccin (viscosidad) en la conduccin, podemos escribir:

e1 w  e2 qvisc

w  e2  e1 qvisc

P  ' w  SQw

con
e2  e1 

p2  p1
1
g ( z2  z1 ) (v22  v12 ) con p1  p2  p0
S
2

= e2  e1  gh

v22
2

qvisc 

z2 - z1  h y v1 x 0

pf 84883
J

 98.70
860
kg
S

a) En el primer apartado es h=0, por lo que:


v22
5.312
J
qvisc 
98.70  14.07 98.70  112.77
2
2
kg
P  SQw  860q 0.06q112.77  5819W  5.8 kW

w

b) En este caso es h = 30 m, por lo que


w  gh

v22
5.312
J
qvisc  9.8q30
98.70  294 14.07 98.70  406.77
2
2
kg
P  SQw  860q 0.06q 406.77  20 989W  21 kW

- 297 -

Fsica Universitaria: Problemas de Fsica

Flujo viscoso. M34.7

7. a) Determinar la velocidad lmite de una esferita de acero ( G = 7.87 g/cm3) de 2 mm de dimetro que cae en

un recipiente que contiene glicerina a 20 C ( U = 1.26 g/cm3 , K = 1.49 Pas). b) Calcular el valor del nmero de Reynolds correspondiente a esa velocidad lmite para asegurarse de que fue correcto utilizar la ley
de Stokes en el apartado anterior. c) Determinar el valor mximo del dimetro de la esferita de acero que
an permita utilizar la ley de Stokes.

Ley de Stokes : F

 3QI D v . Nmero de Reynolds: } 

SD v
. Nmero de Reynolds crtico: } crtico  1
I

a) Expresamos las tres fuerzas que actan sobre la esferita en su movimiento de cada en el
seno de la glicerina, sien E el empuje de Arqumedes y F la resistencia viscosa al
movimiento:
4
4
E
F
mg  Q R 3E g
E  Q R 3S g
F  6QI Rv
3
3
Inicialmente el movimiento es acelerado; pero cuando la esferita alcanza una cierta velocidad, la resistencia viscosa es suficientemente
v
intensa como para, sumada el empuje de Arqumedes, compensar el
peso de la esferita. A partir de ese instante la velocidad no se incrementar (aceleracin nula):
mg
mg  E  F  ma  0 l mg  E F

de modo que
4 3
4
2 E S 2 E S
Q R E g  Q R 3S g 6QI Rvlm l vlm 
gR 
gD 2
3
3
9 I
18I
2
7.87 1.26
q103
q9.8q 2q103
 9.66 mm/s
vlm 
18q1.49
b) El valor del nmero de Reynolds en estas condiciones de flujo externo es
1.26q103 q 2q103 q9.66q103
 0.016  } crt
1.49
que, al ser muy inferior al valor crtico, nos asegura que fue correcto utilizar la ley de Stokes
en el apartado anterior.
c) Sustituimos la expresin de la velocidad lmite en la expresin del nmero de Reynolds
}

18I 2}

S D vlm S E  S
3

crtico

gD
D
} crtico 

l


18I
I
S E  S
g

1/3

de modo que el valor mximo del dimetro de la esferita ser:

18q1.492 q1
 7.88 mm x 8 mm
D 
6
1.26 7.89 1.26
q10 q9.8
1/3

- 298 -

Fsica Universitaria: Problemas de Fsica

Ondas mecnicas. M35.1

1. Una onda esfrica se propaga en un medio absorbente (E = 0.0231 m-1), homogneo e istropo. La intensidad de la onda a una distancia de 10 m de la fuente es de 100 nW/m2. a) Cul ser la intensidad a una distancia de 20 m de la fuente? b) dem a 100 m? c) Calcular la potencia de la fuente. d) Evaluar los apartados
anteriores en ausencia de absorcin a fin de resaltar la importancia de sta.

a) La intensidad de una onda esfrica que se propaga en un medio absorbente viene dada por
P C r
I
e
4Q r 2
siendo P la potencia emitida por el foco y r la distancia al mismo. Aplicando esta expresin a
dos distancias diferentes y dividiendo miembro a miembro:

I 20 
eC r20

4
Q
r
I
r2
r2
20

l 20  102 eC ( r20 r10 ) l I 20  I10 102 eC ( r20 r10 )

P
I10 r20
r20

eC r10
I10 

4Qr10

2
10
nW
I 20  100 e0.0231(2010)  25 e0.231  19.84
20
m2
b) Anlogamente, a una distancia de 100 m del foco, ser:

10
I100  100
100

e0.0231(10010)  e2.08  0.125

nW
m2

c) La potencia de la fuente se calcula directamente a partir de la expresin de la intensidad:

P  4Qr 2 I e C r  4Q q102 q100qe0.0231q10  158q103 nW= 158 NW


d) Supongamos que se trata de la misma fuente, i.e., emitiendo una potencia de 158 W. La
intensidad vendr dada por

I
a)

P
4Qr 2

I 20 r10

I10 r20

Intensidad a una distancia de 10 m: I 

158
nW
 126
4Q q102
m2

10
nW
Intensidad a una distancia de 20 m: I  q126  31.5
20
m2
2

10
nW
Intensidad a una distancia de 100 m: I100 
q126  1.26
100
m2
2

b)

d) Supongamos que la intensidad de la onda a una distancia de 10 m de la fuente es de


100 nW/m2.

10
nW
a) Intensidad a una distancia de 20 m: I 20  100  25
20
m2
2

10
nW
b) Intensidad a una distancia de 100 m: I100 
100  1 2
100
m
2

c) Potencia de la fuente: P  4Qr 2 I  4Q q102 q100  126q103 nW= 126 W

- 299 -

Fsica Universitaria: Problemas de Fsica

Ondas mecnicas. M35.2

2. La funcin y(x,t) correspondiente a cierta onda estacionaria sobre una cuerda tensa es
y = 0.05 sen (2.5 x) cos (500 t)
(SI)
a) Hallar la amplitud y velocidad de propagacin de las ondas que originan esta onda estacionaria. b) Cul es la
distancia entre nodos sucesivos? Cul es la longitud ms corta posible de la cuerda?

Cuando dos ondas que se propagan en direcciones opuestas se superponen, dan lugar a una
onda estacionaria, cuya funcin de onda es
y1 ( x, t )  A0 sen(Xt  kt G1 )
y ( x, t )  (2 A0 ) cos(kx +) sen(Xt G)

y2 ( x, t )  A0 sen(Xt kt G1 )
a) Identificando los valores numricos de la expresin dada en el problema, se obtiene
2 A  0.05 m

k  2.5 m -1

X = 500 s-1

de donde
A  25 mm

c

X 500
m

 200
k
2.5
s

b) La longitud de onda viene dada por

2Q 2Q

 2.513 m
k
2.5
y tanto la distancia entre nodos sucesivos como la longitud mnima de la cuerda sern iguales
a media longitud de onda; esto es
M
Lmin   1.257 m
2
M

- 300 -

Fsica Universitaria: Problemas de Fsica

Ondas mecnicas. M35.3

3. Las cuerdas de una guitarra tienen una longitud til de 66 cm. La quinta cuerda tiene una densidad lineal de
3.1 g/m y su frecuencia fundamental es de 440 Hz (La3) cuando el instrumento est bien afinado. a) Calcular
la velocidad de propagacin de las ondas en esa cuerda y la tensin de la misma. b) Escribir una expresin
general que nos relacione el incremento unitario en la tensin con el cambio unitario en la frecuencia. Utilizar esa relacin para calcular la tensin necesaria para incrementar la frecuencia fundamental en un 2%.
c) Si la tensin de ruptura de la cuerda es de 200 kg, cul ser la frecuencia fundamental ms alta a la que
podemos tensar la cuerda? d) Escribir la funcin de onda estacionaria - esto es, y(x,t) para el armnico
n-simo de una cuerda tensa, de longitud L, sujeta por ambos extremos. Particularizar para el primer y segundo armnicos de la quinta cuerda de la guitarra, si la amplitud de los vientres o antinodos es de 2 mm y
1.5 mm, respectivamente.

a) La longitud de la cuerda debe contener un nmero


entero de semilongitudes de onda:
M
2L
c
c
 K On  n
(1)
L  n n Mn 
On
n
2
2L

primer armnico fundamental (n = 1)

De donde se sigue que


2 LO n 2q 0.66q 440

 580.8 m s
c
n
1

segundo armnico (n = 2)

Por otra parte: F  Nc 2  3.1q103 q580.82  1046 N = 107 kg


b) La expresin (1) la escribimos en la forma

On 
Para

O n

On

n
2L

F
N

O n2 

 2% ser F

n2 F
4 L2 N

d On d F

On
F

%O n
%F
2
On
F

 4% , de modo que

F  0.04q107 kg  4 kg

F '  111 kg

c) La frecuencia fundamental correspondiente a la tensin de ruptura ser


O1 

1
2L

Frup
N

1
200q9.8
 602 Hz
2 0.66 3.1q103

d) La funcin general de onda estacionaria es:

yn x, t
 An sen kn x sen Xn t

2Q
2Q
nQ

k n  M  2 L n  L

n
con

nc nQ

Xn  2QO n  2Q

c

2L
L

o sea
nQ nQ
yn x, t
 An sen x sen ct
L L
n 1

y1 x, t
 0.002sen 4.76 x
sen 2765t

n2

y2 x, t
 0.0015sen 9.52 x
sen 5529t

- 301 -

Fsica Universitaria: Problemas de Fsica

Ondas mecnicas. M35.4

4. Una cuerda con una densidad lineal de 4 g/m est sometida a una tensin de 360 N y est fija en ambos
extremos. Una de sus frecuencias de resonancia es 375 Hz; la siguiente frecuencia ms alta es de 450 Hz.
a) Cul es la frecuencia de resonancia fundamental? b) Qu armnicos son los que se dan en el enunciado
de este problema? c) Cul es la longitud de la cuerda?

a) Las frecuencias de las ondas estacionarias que pueden residir en la cuerda son mltiplo de
la frecuencia fundamental o primer armnico.

O n  nO1

O  O n 1  O n

O n 1  (n 1) O1 1

En consecuencia, la frecuencia fundamental es:

O1  450  375 Hz

O1  75 Hz

b) En el enunciado del problema se dan el quinto y el sexto armnico, ya que:


O
375
450
n n
5
 6
O1
75
75
c) La longitud de la cuerda es la mitad de la longitud de onda del primer armnico

F
360

 300 m/s
4q103
N
c 300
M1  
 4 m L  2M1  8 m
O1
75
M
L 1 L2m
2
c

M

c
O

primer armnico fundamental (n = 1)

segundo armnico (n = 2)

tercer armnico (n = 3)

cuarto armnico (n = 4)

- 302 -

Fsica Universitaria: Problemas de Fsica

Ondas mecnicas. M35.5

5. Un altavoz genera en un concierto de rock 10-2 W/m2 a 20 m a una frecuencia de 1 kHz. Supongamos que la
energa procedente del altavoz se extienda uniformemente en todas las direcciones. a) Cul es el nivel de
intensidad a 20 m? b) Cul es la potencia acstica total generada por el altavoz? c) A qu distancia alcanzar la intensidad el umbral de dolor de 120 dB? d) Cul es el nivel de intensidad a 30 m?

a) Por definicin del nivel de intensidad, se sigue

C20  10 log

I 20
102
 10 log 12  10 log 1010  10q10  100 dB
I umb
10

b) Por tratarse de una onda esfrica, la potencia se reparte sobre frentes de onda cada vez ms
extensos, a medida que nos apartamos del foco.

P  I 20 S20  102 q 4Q q 202  50.3 W


c) Deberemos disponer de una intensidad de 1 W/m2, ya que entonces ser
1
C  10 log 12  10 log1012  120 dB
10
Como P = IS = 4SR2I, sera
P
50.3

 4.00  2 m
4Q I
4Q q1
d) De nuevo tenemos en cuenta que se trata de una onda esfrica.
P
50.3
I 30 

 4.44q103 W/m 2
S30 4Q q302
R

C30  10 log

4.44q103
 10 log 4.44q109  96.6 dB
1012

- 303 -

Fsica Universitaria: Problemas de Fsica

Ondas mecnicas. M35.6

6. Una onda sonora disminuye su nivel de intensidad en 30 dB cuando avanza 50 m. Determnese el coeficiente de absorcin de dicha onda en el medio en el que se propaga en los siguientes supuestos: a) Se trata de
una onda plana. b) Se trata de una onda esfrica a 100 m de su foco.

I
, siendo I la intensiI0
dad e I0 la intensidad umbral de referencia. Determinamos la relacin existente entre la variaciones del nivel de intensidad y los cocientes de intensidades:
I
I
C  C2  C1  10 lg 2 l 2  10C /10  1030/10  103
I1
I1
Sea  el nivel de intensidad de la onda sonora definido por C  10 lg

a) Onda plana: I  I 0 eB x

I1  I 0 eB x1
I 2  eB ( x2 x1 )  eB x l B   1 ln I 2

B x2
I1
x I1
I 2  I 0e

y sustituyendo valores
1
I
1
ln 1  ln103  0.138 m -1
B
x I 2 50
b) Onda esfrica:

I 2  I1

I2

I1

r12 -B (r2 r1 )


e
r22

I 2 r22
 e-Br
I1r12

l B 

y sustituyendo valores

r1 =100 m

r2

B 

2
1 3 150
ln 10
 0.122 m-1

100
50

- 304 -

I r2
1
ln 2 22
r I1r1

Fsica Universitaria: Problemas de Fsica

Ondas mecnicas. M35.7

7. Una onda plana se propaga por un medio cuyo coeficiente de absorcin es  =0.01 m-1. Qu distancia
tendr que avanzar en dicho medio para que su intensidad disminuya 10 dB?

De acuerdo con la definicin del decibelio (dB)


I
I
I
10 log10  10 dB l log10  1 l I  0
I0
I0
10
de modo que la intensidad debe reducirse a la dcima parte de su valor inicial.
La intensidad de una onda plana que se propaga en un medio absorbente decrece exponencialmente conforme avanza en dicho medio:
I
1 I
1 I
I  I 0 eC x l
 eC x l x   ln  ln 0
I0
C I0 C I
donde x es el trayecto recorrido. Sustituyendo los datos en esta expresin
1
x
ln10  230 m
0.01

- 305 -

Fsica Universitaria: Problemas de Fsica

Ondas mecnicas. M35.8

8. Para determinar la densidad lineal de un hilo tomamos un trozo del mismo, de 1 m de longitud, fijamos un
extremo y del otro colgamos una pesa de 50 kg, al someterla a vibraciones observamos que a 70 Hz se establece una onda estacionaria con un vientre entre sus extremos. a) Determinar la densidad lineal de dicho
hilo. b) A qu frecuencia conseguiremos observar dos nodos entre sus extremos. c) Qu observaramos a
una frecuencia de 100 Hz.

a) La longitud L de la cuerda corresponde a media longitud de onda (distancia internodal); por


consiguiente
M
N
N
L  1 l M1  2 L  2q1  2 m
2
Determinamos la velocidad de fase a partir de la relacin
existente entre la longitud de onda y la frecuencia:
c  M1O1  2q 70  140 m/s

La tensin del hilo ser: F  50 kg = 50q9.8  490 N


La velocidad de los ondas transversales en el hilo viene dada por:
c

F
N

l N

F
490
kg
g

 0.025
 25
2
2
c
140
m
m

b) Ahora, la longitud de la cuerda corresponde a tres distancias


internodales (tercer armnico):

M
2
L  3 3 l M3  23 L   0.66 m
2
3
c
1
490
O3  
 210 Hz

M3 0.66 0.025

M3 / 2 M3 / 2 M3 / 2

lo que resulta obvio, ya que para el tercer armnico es O 3  3O1 .


c) La frecuencia de 100 Hz no es mltiplo de 70 Hz, por lo que no corresponde a ningn armnico y la cuerda no mostrar vientres ni nodos. Esa frecuencia no se establecer en la cuerda como una onda estacionaria, sino como una onda progresiva o viajera a lo largo de la cuerda.

- 306 -

Fsica Universitaria: Problemas de Fsica

Termologa - T00.1

1. Un mol de agua a 25 C se calienta en recipiente abierto a la atmsfera hasta 100 C. Los valores medios del
coeficiente de dilatacin y del calor especfico del agua en ese intervalo de temperaturas son 4.0 u 10-4 K-1 y
1.0 cal/gK, respectivamente. Calcular las variaciones de la energa interna, entalpa y entropa en el
proceso.

Durante el proceso a presin constante, el agua absorbe una cantidad de calor


Q  mcT  18q1q (100  25)  1350 cal = 5643 J

y realiza un trabajo de expansin


W  patm %V

siendo V el incremento de volumen que experimenta el agua al incrementarse su


temperatura. De la definicin del coeficiente de dilatacin, se sigue:
V / V0
m
B
l V  V0B T  B T
T
S0
y tomando 0  1 g/cm3,

V 

18
q 4.0q104 q 75  0.54 cm3  5.4q104 L
1

de modo que
W  1 atm q5.4q104 L = 5.4q104 atm L  0.013 cal  0.055 J
que es despreciable en comparacin con el calor absorbido.
Las variaciones de energa interna y de entropa son:
U  Q W  1350  0.013 x 1350 cal  5643 J
H  Q p  1350 cal  5643 J
La variacin de entropa se calcula imaginando un proceso isobrico reversible entre el estado
inicial y el final; esto es,
Tf dT
T
Q
373
cal
J
S 
 mc
 mc ln f  18q1q ln
 4.04
 16.9
Ti
298
K
K
T
T
Ti

- 307 -

Fsica Universitaria: Problemas de Fsica

Termologa - T00.2

2. En un calormetro disponemos de 200 g de hielo a 0C. a) Determinar la cantidad de agua a 40C que
deberemos aadir al calormetro para que justamente funda todo el hielo. b) Calcular el la variacin de
entropa que experimenta el sistema (hielo+agua) durante el proceso de fusin del hielo. Discutir y explicar
el signo de dicha variacin. Se trata de un proceso reversible? Explquese.

a) Para fundir, el hielo deber absorber una cantidad de calor


Qhielo  mh A f  200q80  16 000 cal

que deber ser cedida por el agua al enfriarse desde 30C a 0C; esto es
Q
16 000
Qagua  ma ca t l ma  agua 
 400 g
ca t
1q 40
b) El proceso de fusin del hielo tiene lugar a temperatura constante (0C = 273 K) con
aumento de entropa:
Q
16000
cal
 58.6
Shielo  hielo 
Tf
273
K

Proceso de enfriamiento del agua desde 40C ( = 313 K) hasta 0C (=273 K) con aumento de
entropa:
T2 d T
Q
T
273
cal
Sagua 
 ma ca
 ma ca ln 2  400q1q ln
 54.7
T
1
T
T
T1
313
K
La variacin de entropa del sistema es:
cal
K
Donde el signo positivo nos indica que este proceso es espontneo y que ocurre siempre en
este sentido y nunca en el sentido opuesto (600 g de agua a 0C  200 g de hielo a 0C +
400 g de agua a 40 g). Se trata claramente de un proceso irreversible.
S  Sh Sa  58.6  54.7  3.9

hielo a
0C

agua a
40C

- 308 -

agua a
0C

Fsica Universitaria: Problemas de Fsica

Termologa - T00.3

3. En un recipiente abierto a la atmsfera, colocamos 1 kg de hielo a 0 C y 2 kg de agua a 90 C. Determinar:

temperatura (C)

a) La temperatura final de equilibrio. b) El trabajo realizado por cada sistema. c) Los cambios de energa
interna, entalpa y entropa de cada unos de los sistemas en la transformacin.
Datos: Calor latente de fusin del hielo, 80 cal/g; calor especfico del agua, 1 cal/g.C; densidad del hielo,
0.9 g/cm3; densidad del agua (0 a 100C), 1g/cm3.

Comenzamos tanteando el estado final del sistema:


Q1  mh lf  1q80  80 kcal

ma

90

Q3,mx  ma c%t  2q1q90  180 kcal

Q3
t
0

mh+ma

Q2

mh

Por consiguiente, al enfriarse el agua es capaz de


ceder al hielo calor ms que suficiente para su fusin.
El estado final del sistema ser el de agua (fase
lquida) a una cierta temperatura t.
Por tratarse de un sistema cerrado, ser
Q1 Q2 Q3  0 , de modo que

Q1
tiempo

mh lf mh c(t  t0,h ) ma c(t  t0,a )  0 l mh lf c(t  t0,h )  ma c(t  t0,a )


y sustituyendo los datos
1q<80 1q (t  0) > 2q1q (t  90)  0 l 80 3t 180  0 l t  33.3 C  306.5 K

Trabajo
Tan solo se produce cambio de volumen en el proceso de fusin del hielo, a presin constante,
por lo que el trabajo asociado al mismo ser
1
1
1
1
 11.26 J  2.69 cal
W  p%V  p Va Vh
 pmh   101328q1q

1000 900
S
S
a

Entalpa
Puesto que el proceso tiene lugar a presin constante, los cambios de entalpa coinciden con
los calores absorbidos o cedidos. Esto es,
%H hielo  1q80 1q1q33.3  80 kcal 33.3 kcal  113.3 kcal
%H  0

%H agua  2q1q 33.3  90


 113.3 kcal

Energa interna
El cambio de la energa interna, en cada sistema ser %U  Q W , de modo que
%U hielo  113.3 kcal  2.7 cal
 113.3 kcal

%U agua  113.3 kcal l %U  2.7 cal

Entropa
%S hielo 

Q1
Tf

mh c

Tf

%Sagua  ma c

T0

dT
T

dT
T

 ma c ln

mh lf
Tf
T
T0

mh c ln

T
Tf

 2 q1q ln

80
273

306.48
363.15

1q1q ln

 0.34

- 309 -

306.48
273.15

kcal
K

 0.29 0.12  0.41

l %S  0.07

kcal
K

kcal
K

Fsica Universitaria: Problemas de Fsica

Termologa - T00.4

4. Un recipiente adiabtico contiene 40 g de hielo a una temperatura de -20C. Aadimos 30 g de agua a 50C,
removemos y esperamos a que se alcance el equilibrio trmico. a) Determinar la composicin de la mezcla y
la temperatura de la misma. b) Calcular los cambios de entropa que experimentan el hielo y el agua.
c) Vara la entropa del sistema hielo-agua en este proceso? Aumenta o disminuye? Por qu?
Datos: calor especfico del agua, 1.00 cal/g.K; calor especfico del hielo, 0.5 cal/g.K; calor latente de fusin del
hielo, 80 cal/g.

a) Balance calorfico para pasar 40 g de hielo a 20C hasta agua a 0C:


calentamiento: Q1  mcT  40q 0.5q (0  (20)  400 cal

Qhielo  3600 cal

fusin: Qf  mA f  40q80  3200

Balance calorfico cuando se enfra el agua desde 50C hasta 0C:


enfriamiento: Q  mcT  30q1.0q (0  50)  1500 cal

Resulta obvio que no puede fundirse todo el hielo, por lo que en el estado final tendremos
hielo y agua en equilibrio a la temperatura de fusin (0C).
Sea mf la masa de hielo que funde. Escribimos la ecuacin del balance calorfico:
1100
80 mf  (1500  400)  1100 l mf 
 13.75 g
80
de modo que la mezcla final estar compuesta por (30 + 13.75) = 43.75 g de agua lquida y
(40-13.75 )= 26,25 g de hielo.
b) Determinamos los cambios de entropa que experimentan el hielo y el agua:
273 m c dT
ml
273 13.75q80
cal
h h
S hielo 
f f  40q 0.5q ln

 1.52 4.03  5.55
253
273
253
273
K
T
273 m c dT
273
cal
a a
Sagua 
 30q1q ln
 5.04
323
323
K
T
c) El cambio de entropa que experimenta el sistema hielo+agua ser
cal
S  Shielo Sagua  0.50
K
i.e., la entropa del sistema (aislado) aumenta porque el proceso que ha tenido lugar en el es
espontneo e irreversible.

- 310 -

Fsica Universitaria: Problemas de Fsica

Termologa - T00.5

5. En un recinto adiabtico disponemos de 1 kg de hielo a 0 C y exceso de vapor de agua a 100 C. Calclense


la cantidad de vapor que se condensar y los cambios de entalpa y entropa experimentados por cada
componente. Datos: lf = 80 cal/g, lv = 540 cal/g , c = 1 cal/g.K.

Puesto que tenemos exceso de vapor, el estado final de la mezcla corresponde a la


temperatura de ste (100 C), por lo que tendremos agua y vapor a 100 C.
Balance calorfico
Calor absorbido por el hielo para fundir + calor absorbido por el agua al calentarse desde 0 C
hasta 1000 C = Calor cedido por el vapor de agua al condensarse; i.e.,
l cT
80 1q100
1
mh lf mh c T  mv lv l mv  f
mh 
q1   0.333 kg
lv
540
3
Cambios de entalpa
H h  mh lf  1q80  80 kcal
H agua  mh cT  1q1q (100)  100 kcal
H v  mv lv  0.333q540  180 kcal
H total  80 100 180  0

Cambios de entropa
Sh 
Sagua
S v 

80
kcal
Q

 0.29
K
T 273
Tf
373
kcal
 mh c ln  1q1q ln
 0.31
273
K
Ti
kcal
Q 180

  0.48
373
K
T

kcal
K
de modo que la entropa crece en el proceso, como era de esperar por tratarse de un proceso
espontneo e irreversible.
H total  0.29 0.31 0.48  0.12

- 311 -

Fsica Universitaria: Problemas de Fsica

Termologa - T00.6

6. A presin constante, se suministran 500 J en forma de calor a 2 moles de un gas ideal biatmico. Calcular:
a) El incremento de temperatura. b) El trabajo realizado por el gas. c) El cociente entre el volumen final y el
inicial del gas si la temperatura inicial es 20 C. d) La variacin de entropa. Dato: R = 8.31451 J/mol K.

CV  R CV  20.786 J/ mol K

2
Por tratarse de un gas biatmico, sern
7

C p  R C p  29.101J/ mol K

a) Se trata de un proceso de expansin isobrico, por lo que Q = n Cp 'T


Q
500
T 

 8.59 K T  8.6 K
nC p 2q 29.101

b) Calculamos la variacin de la energa interna:


U  nCV T  2q 20.786q8.59 U  357 J
W  Q  %U  500  357  143 J

c)
pV1
pV
 2
T1
T2

V2 T2 T1 %T
 
V1 T1
T1

V2 293.15 8.59 301.74




 1.03
V1
293.15
293.15

d) Calculamos la variacin de entropa para un proceso isbaro


T2 nC d T
d Qp
T
301.59
J
p
S 

 nC p ln 2  2q 29.101q ln
 1.67
T
1
T
T
T1
293.15
K

- 312 -

Fsica Universitaria: Problemas de Fsica

Termologa - T00.7

7. Dos moles de un gas perfecto, que inicialmente ocupan 44.8 L a 1 atm de presin, se someten a una
transformacin isoterma reversible en la que su entropa disminuye 2.75 cal/K. Determinar el estado final
del gas (p, V y T) as como el trabajo realizado y el calor absorbido durante el proceso.

p
3

1
273 K

La ecuacin de estado del gas perfecto nos permite


determinar la temperatura.
pV
1q 44.8
pV  nRT l T 

 273 K
nR 2q 0.08205
La variacin de entropa en una transformacin isoterma
viene dada por
Q
V
S   nR ln 2
T
V1

de donde se sigue

10
20
30
40
50

Q  T S  273q 2.75
 751 cal

W  Q  U  751 cal  3138 J


La expresin de la variacin de entropa tambin nos permite determinar el volumen en el
estado final
V2
 eS nR  e2.75 2q1.987  e0.692  0.50 l V2  22.4 L
V1
Ahora determinamos la presin en el estado final
V
p1V1  p2V2 l p2  1 p1  2q1  2 atm
V2
60 V

- 313 -

Fsica Universitaria: Problemas de Fsica

Termologa - T00.8

8. Cinco moles de un gas biatmico, que se encuentran inicialmente a presin atmosfrica y temperatura de
0C, se someten a las transformaciones indicadas en el diagrama adjunto. a) Determinar la temperatura final
del gas. b) Calcular el trabajo realizado por el sistema, el calor absorbido y los cambios de energa interna,
de entalpa y de entropa.
Datos: R = 1.987 cal/molK = 0.08205 atmL/molK

En un gas biatmico (f = 5):


f 2
cal
Cp 
R  6.95
2
mol K
f
cal
CV  R  4.97
2
mol K

2patm

patm

a) Aplicando la ecuacin de estado de los gases perfectos


a los puntos 1 y 4:
p1V1
pV
pV
2 p 2V
 4 4 T4  T1 4 4  T1 1 1
T1
T4
p1V1
p1V1

V1

2V1

3V1

l T4  4T1  4q 273  1092 K


b) El trabajo realizado en la transformacin completa ser:
2

W1,4  pdV pdV pdV  p1 V2 V1


p4 V4 V3

1

 patm (3V1  V1 ) 2patm (2V1  3V1 )  2V1 patm  2V1 patm  0


El calor absorbido por el sistema, segn el primer principio de la termodinmica ser:
Q  U W  U  23353 cal
La variacin de energa interna es:
U  nCV T  5 q 4.97q (1092  273)  23353 cal
La variacin de entalpa ser:
H  nC p T  5q 6.95q (1092  273)  28460 cal

La variacin de entropa ser:


4 dQ
4 dU
4 pdV
4 dT
4 dV
T
V
S1,4 


 nCV
nR
 nCV ln 4 nR ln 4 
1
1
1
1 T
1 V
T
T
T
T1
V1
cal
 5q 4.97 q ln 4
+ 5q1.987q ln 2
 41.34
K

- 314 -

Fsica Universitaria: Problemas de Fsica

Termologa - T00.9

9. Una corriente ascendente de aire seco tiene una temperatura de 27 C, una presin de 101.3 kPa y una
densidad de 1.18 kg/m3 a nivel del suelo. Consideremos que asciende a 100 m de altura sin intercambio de
calor con su entorno. a) Determinar la presin atmosfrica a la altura de 100 m. b) Calcular la temperatura
que tendr la corriente de aire a esa altura. c) Explicar la causa de esa variacin de temperatura de la
corriente de aire. Cuantifquese para 1 mol de aire.
Nota: A los solos efectos del apartado a), considrese que la densidad de la atmsfera no vara significativamente
en una diferencia de alturas de 100 m.

a) Calculamos la presin atmosfrica a una altura de 100 m sobre el suelo despreciando el


cambio de densidad del aire con la altura ( = cte); esto es, p0  p100 S gh

p100  p0  S gh  101300 1.18q9.8q100  101300 1156  100144 Pa


b) Consideremos una cierta cantidad de aire (digamos, n moles). Durante el ascenso,
experimenta un proceso termodinmico en el que disminuye su presin, en tanto que no
intercambia calor con su entorno. En definitiva, experimenta una expansin adiabtica durante
el ascenso, lo que implica una disminucin de la temperatura (vide grfica).
A partir de la ec. de la adiabtica en funcin de (p,V) y de la ec. de estado del gas perfecto,
eliminamos el volumen para obtener la ec. de la adiabtica en funcin de (p,T).

pV H  cte


l

pV
nRT
V
nR

T
p  cte l
p

p1H T H  cte l

1H
H

T  cte

Puesto que el aire se comporta como un gas biatmico,


ser
C
1 H
2
f 2 5 2 7
H p 

  1.4 l

H
5
5
7
CV
f

suelo

p0

100 m

p100

T0
T100

Entonces, aplicando la ec. de la adiabtica entre los estados inicial y final, ser
1H
H

p100

1H
H

T100  p0

T0

p
l T100  T0 0
p

100

1H
H

2/7

101 300

T100  300
100 144

 300q1.01152/7  300q 0.997  299 K  26 C

de modo que la corriente de aire ascendente se ha enfriado un grado.


c) Aplicando el Primer Principio al proceso adiabtico, U  Q W  W , de modo que el

trabajo asociado a la expansin adiabtica del aire da lugar a la disminucin de la energa


interna de ste y, por ende, de su temperatura. As, para 1 mol de aire ser:
5

W  U  nCV T  1q q8.3143q 1


 20.8 J
2

- 315 -

Fsica Universitaria: Problemas de Fsica

Termologa - T00.10

10. Consideremos un tubo en U, de seccin uniforme S, con uno de sus extremos taponado y el otro abierto a la
atmsfera. Inicialmente, la columna de aire atrapada en la rama cerrada tiene longitud H y se encuentra a la
presin atmosfrica p0. Sean L la longitud y  la densidad de la columna lquida. Determina la frecuencia de
las pequeas oscilaciones de la columna lquida.

Iniciadas las oscilaciones de la


columna lquida, para una elongacin
genrica x, las fuerzas que actan sobre
la columna lquida, de masa
m  S SL

p0

p0

pS
p0S

son las que se indican en la figura de la


derecha, que corresponden a la presin
ejercida por el aire sobre las superficies
libres de la columna lquida y al peso
de la porcin de columna lquida en
exceso en la rama derecha. De este
modo, la fuerza neta en la direccin del
desplazamiento es

2x

S(2x)g

Fneta  p0 S  pS  S S 2 x
g  2S gSx  p  p0
S

Si suponemos que las oscilaciones son suficientemente rpidas, el aire atrapado en la rama de
la derecha experimenta compresiones y expansiones aproximadamente adiabticas, por lo que
podemos escribir
H

p0 SH
 p < S ( H  x) >

H

x
p  p0 1
H

x
x p0 1 H l

p  p0 x

H p0
x
H

donde hemos supuesto que las oscilaciones son pequeas en comparacin con la longitud de
la columna de aire , i.e., x  H .
De este modo, la fuerza neta que acta sobre la columna de lquido es

Hp
Hp
Fneta  2S gSx  0 Sx  2S gS 0 S x

H
H
Y aplicando la ec. fundamental de la dinmica, Fneta  mx , tenemos

Hp
2S gS 0

2 g H p0

S x  S SL 
x l 
x
x  0
L S HL

Que es la ecuacin diferencial correspondiente a un m.a.s. cuya frecuencia angular es


X

2 g H p0

L S HL

T

L
2Q
 2Q
2 g H p0 S H
X

Si F  1 , es 1 F
x 1 nF
, como se demuestra fcilmente a partir del desarrollo del
n

n n
n
n
n
binomio de Newton 1 F
 F F 2 F3 ! .
0 1
2
3

- 316 -

Fsica Universitaria: Problemas de Fsica

Termologa - T00.11

11. Un mol de oxgeno a 300 K se mezcla con 2 moles de oxgeno a 400 K, ambos a la presin atmosfrica.
Determinar la temperatura final de la mezcla, as como las variaciones de energa interna, de entalpa y de
entropa durante el proceso.

En el proceso de mezcla se conserva la energa interna


del sistema:
U  n1CV T  T1
n2CV T  T2

 CV n1 T  T1
n2 T  T2
 0
Esta expresin nos permite determinar la temperatura
final de la mezcla:
n T n2T2 1q300 2q 400
T 11

 367 K
1 2
n1 n2

n1=1mol
p1=1atm
V1=24.6 L
T1=300K

n2=2mol
p2=1atm
V2=65.6 L
T2=400K
n=3mol
p=1atm
V1=90.2 L
T=367K

El cambio que experimenta la entalpa del sistema ser:


H  n1C p T  T1
n2C p T  T2
 C p n1 T  T1
n2 T  T2
 0
El cambio que experimenta la entropa del sistema ser la suma de los cambios
experimentados por cada uno de los gases en los procesos isobricos correspondientes:
T dT
Q
T
7
367
cal
S1 
 n1C p
 n1C p ln  1q 1.987 ln
 1.41
T1 T
T
T1
2
300
K
S 2 

T dT
Q
T
7
367
cal
 n2C p
 n2C p ln  2q 1.987 ln
 1.21
T2 T
T
T2
2
400
K

cal
J
 0.84
K
K
Resultando un aumento de entropa por tratarse de un proceso irreversible.
S  S1 S 2  0.20

Otro mtodo:

V  V1 V2  90.26 L

T  pV  1q90.26  367 K
n RT
2q 0.08205q 400
V2  2 2 
 65.64 L
nR 3q 0.08205

1
p2

V1 

n1 RT1 1q 0.08205q300

 24.62 L
1
p1

- 317 -

Fsica Universitaria: Problemas de Fsica

Termologa - T00.12

12. En dos compartimentos de un recipiente adiabtico de paredes rgidas mantenemos separados, mediante un
tabique rgido y adiabtico, 1 mol de helio a 1 atm y -100 C y 2 moles de hidrgeno a 1 atm y 27 C.
a) Determinar la temperatura y la presin final de la mezcla cuando se retira el tabique. b) Calcular la
presin parcial de cada componente en la mezcla. c) Evaluar el cambio de entropa en este proceso de
mezcla.

1 mol
p1=1atm
V1=8.21L
T1=100K

2 mol
p2=1atm
V2=49.23L
T2=300K

3 mol
p=1.09atm
V=57.44L
T=254K

1 mol de He a 1 atm y 100 K ocupa

V1 

n1 RT1 1q 0.08205q100

 8.21 L
1
p1

2 mol de H2 a 1 atm y 300 K ocupan

V2 

n2 RT2 2q 0.08205q300

 49.23 L
1
p2

El volumen del recipiente es:

V  V1 V2  57.44 L

El proceso de mezcla es adiabtico (Q=0), iscoro (V=cte; W=0) e irreversible.


El Primer Principio de la Termodinmica establece que %U  Q W  0  0  0 , de modo
que la energa interna del sistema permanece constante, lo que nos permite determinar la
temperatura final de la mezcla
U  U1 U 2  n1C1V (T  T1 ) n2C2V (T  T2 )  0
T

n1C1V T1 n2C2V T2 1q3q100


2q5q300
3q100
10q300



 254 K= 19C
n1C1V n2C2V
3 10
1q3
2q5

nRT 3q 0.08205q 254



 1.09 atm
V
57.44
b) Las presiones parciales se calculan multiplicando la presin por las fracciones molares de
cada componente:
p1  D1 p  13 p  0.36 atm
p1  D2 p  32 p  0.73 atm
l

p

c) El cambio de entropa en el proceso de mezcla, para cada uno de los gases, viene dado por

S 

Q
T

 nCV

dU W

dT
T

nR

dU
T

dV
V

W
T

 nCV ln

 nCV

dT
T

pdV
T

Tf
V
n R ln f
Ti
Vi

254
57.44
cal

S1  1q 2 1.987q ln 100 1q1.987q ln 8.21  2.78 3.87  6.64 K


cal

l S  9.63

254
57.44
cal
K

S 2  2q 52 1.987q ln
2q1.987 q ln
 1.66 4.64  2.99

300
49.2
K

- 318 -

Fsica Universitaria: Problemas de Fsica

Termologa - T00.13

13. Un sistema termodinmico est formado por 2 moles de hidrgeno a 2 atm y 1 mol de helio a 1 atm que
estn contenidos en un cilindro rgido y adiabtico, a diferente lado de un
pistn buen conductor del calor bloqueado que los separa. Inicialmente, el
sistema se encuentra en equilibrio trmico a la temperatura de 0C. En un
instante dado desbloqueamos el pistn, de modo que el sistema evoluciona
hasta que finalmente alcanza el equilibrio. En toda esta transformacin
termodinmica: a) Se conservar inalterada alguna variable de estado del
sistema? Cul ser la temperatura final del sistema? Es reversible esta
transformacin? Raznense las respuestas. b) Calcular los cambios de
entropa experimentados por cada uno de los dos gases y por el sistema.

H2

He

estado inicial

H2

He

estado final

a) Por tratarse de un sistema aislado, que no intercambia energa con el exterior en forma de
calor o de trabajo, la energa interna del sistema permanece constante. Puesto que en el
estado final de equilibrio ambos gases tendrn la misma temperatura (y la misma presin),
podemos escribir
U  cte. l U  U H U He  nH CV ,H (Tf  T0 ) nHeCV ,He (Tf  T0 ) 

 nH CV ,H nHeCV ,He
(Tf  T0 )  0 l Tf  T0  0 C
de modo que la temperatura final es la misma que la inicial.
El proceso es irreversible, ya que al expansionarse un gas y comprimirse el otro, contra una
presin diferente a la suya propia, hasta que adquieren una presin comn pf, los estados
intermedios no sern estados de equilibrio.
b) Como los procesos implicados son irreversibles, para calcular 'S debemos imaginar unas
transformaciones reversibles que lleve al sistema del estado inicial al final. Puesto que, para
cada gas, la temperatura inicial es igual a la final, podemos considerar un proceso isotermo:
T  cte d U  Q  p d V  0 Q  p d V
Vf d V
Q
p dV
V

 nR
 nR ln f
V0 V
T
T
V
0
rev

S 

Determinamos los valores iniciales y finales ocupados por cada gas


VH

nH RT0
pH

2 RT0
2

RT0

22.4 A

VHe

nHe RT0
pHe

1RT0
1

RT0

22.4 A

o V

VH  VHe

2 RT0

44.8 A

Al final tenemos la misma presin en ambos gases


n RT
n RT
pf  H ' f  He ' f
VH
VHe

2
1
l
 '
'
VH VHe

VH'  2VHe'
l '
'

VH VHe  2 RT0

V '  RT  29.9 A

H 3 0
l

V '  2 RT  14.9 A

He 3 0

4
cal
J
 4.78
cal
J
3
K
K
 1.41
l S  0.34
2
cal
J
K
K
S
He  R ln  0.81  3.37
3
K
K
de modo que el sistema experimenta un incremento de entropa, ya que evoluciona
espontneamente hacia un nuevo estado de equilibrio.

S
H  2 R ln  1.14

- 319 -

Fsica Universitaria: Problemas de Fsica

Termologa - T00.14

14. Una mezcla de gases constituida por 1 mol de helio y dos moles de oxgeno se encuentra en un recinto de
67.2 L a 1 atm de presin. Si la mezcla se calienta isobricamente hasta duplicar su volumen, determnense
los balances de calor y trabajo y las variaciones de energa interna, entalpa y entropa de la mezcla.

C p  52 R

5
7

O 2 (biatmico): CV  2 R
C p  2 R

Calculamos las temperaturas inicial y final de la mezcla:


He (monoatmico): CV  32 R

R  1.987

cal
mol K

p1V1
1q67.2


 273 K

3q0.08205
nR
T  273 K

p2V2
1q134.4
p2  1 atm
V2  134.4 L T2 

 546 K

3q 0.08205
nR

El trabajo realizado en el proceso de calentamiento isobrico ser:


W  pV  1q 67.2  67.2 atm L = 6809 J
p1  1 atm

V1  67.2 L

T1 

El balance calorfico y las variaciones de energa interna, entalpa y entropa son:


Q  nC p T  (1q 52 q1.987q 273) (2q 72 q1.987q 273)  1356 3797  5153 cal= 21 541 J

U  nCV T  (1q 32 q1.987q 273) (2q 52 q1.987q 273)  814 2712  3526 cal= 14 739 J

H  nC p T  (1q 52 q1.987q 273) (2q 72 q1.987q 273)  1356 3797  5153 cal= 21 541 J
S  nC p ln

J
T2
cal
= 54.7
 (1q 52 q1.987q ln 2) (2q 72 q1.987 q ln 2)  3.44 9.64  13.08
K
K
T1

- 320 -

Fsica Universitaria: Problemas de Fsica

Termologa - T00.15

15. Una botella de acero, cerrada y de volumen constante, contiene en su interior 0.5 kg de oxgeno a la presin
de 10 atm y temperatura de 20 C. Hllense: a) La capacidad de la botella de acero. b) La presin que se
alcanzar en su interior, cuando se calienta el oxgeno hasta 80 C . c) El incremento de entropa del oxgeno
al realizarse el calentamiento descrito en el apartado b).

Los 0.5 kg de oxgeno representan 500/32 = 15.625 mol y, puesto que se trata de un gas
(ideal) biatmico, ser
5
CV  R
2
p2
a) Determinamos el volumen ocupado por el gas (capacidad de la
botella) a partir de la ec. de estado de los gases perfectos:
nRT1 15.625 u 0.08206 u 293
p1
p1V nRT1 o V
37.57 L
p1
10
b) Cuando lo calentamos a volumen constante hasta 80 C = 353 K, la
presin aumenta:
p1
p
T
353
 2 l p2  2 p1 
q10  12.05 atm
T1 T2
T1
293

c) En el proceso isocoro (a volumen constante) el incremento de entropa ser


S 

T2

Q
dT
T
5
353
cal
 nCV
 nCV ln 2  15.625q q1.987q ln
 14.46
T
T
T
2
293
K
1
T
1

- 321 -

Fsica Universitaria: Problemas de Fsica

Termologa - T00.16

16. Un recinto de paredes rgidas y adiabticas est dividido en dos


compartimentos mediante un tabique ligero y mvil. Uno de los
compartimentos contiene un mol de anhdrido carbnico a una presin
CO2
vaco
inicial de 50 atm y una temperatura de 300 K; en el otro compartimiento
existe el vaco. Permitimos que el gas se expansione espontneamente
hasta que su volumen se hace 20 veces superior a su volumen inicial.
Supongamos que el CO2 se comporte como un gas perfecto. a) Explicar si el proceso es reversible o no. Se
intercambia calor? Se realiza trabajo? b) Calclense los cambios de energa interna, de temperatura y de
entropa, experimentados durante el proceso.
Datos: R = 1.987 cal/(molK) = 0.082057 (atmL)/(molK).

a) El proceso es irreversible, espontneo, ya que al expansionarse contra el vaco los estados


intermedios no sern estados de equilibrio. Por consiguientes:
x No se intercambia calor (paredes adiabticas)
x No se realiza trabajo, por tratarse de una expansin contra el vaco.
b) Segn el Primer Principio de la Termodinmica
U  Q W

y, puesto que no se intercambia calor ni trabajo, es %U  0 . Entonces, dado que la energa


interna del gas ideal es tan solo funcin de la temperatura, ser
U  nCV T  0

T  0

y la temperatura permanece constante (efecto de Joule-Kelvin).


La variacin de entropa la calculamos como
Q
S 
rev T
Como el proceso que se describe en el enunciado es irreversible, para calcular 'S debemos
imaginar una transformacin reversible que lleve al sistema del estado inicial al final. Puesto
que la temperatura inicial es igual a la final podemos considerar un proceso isotermo:
T  cte d U  Q  p d V  0 Q  p d V
de modo que
20V0 d V
20V0
Q
p dV

 nR
 nR ln
 nR ln 20
V0
T
T
V
V0
rev

S 

S  1molq1.987

cal
q ln 20
mol K

- 322 -

S  5.95 cal/K

Fsica Universitaria: Problemas de Fsica

Termologa - T00.17

17. Un mol de hidrgeno a una presin de 1 atm ocupa 22.4 L y evoluciona segn las transformaciones
reversibles siguientes: 1 Un calentamiento isobrico hasta una temperatura de 819 K; 2 Un enfriamiento
iscoro hasta la temperatura inicial; 3 Una transformacin que cierra el ciclo, cuya representacin en un
diagrama pV es una recta. a) Calcular el trabajo neto y el balance calorfico en el ciclo completo, as como
las variaciones de energa interna, entalpa y entropa en el mismo. b) El intercambio de calor y de trabajo y
las variaciones de energa interna, de entalpa y de entropa en la tercera transformacin.

Estado A: pAVA  nRTA l TA 


Estado B:

Estado C:

pB VB
TB
pC VC
TC




pA VA
TA
pB VB
TB

pAVA
1q 22.4

 273 K
nR
1q 0.082

p
V
(atm) (L)

T
(K)

l VB 

TB
819
VA 
VA  3VA  67.2 L
TA
273

A 1.00 22.4 273


TC
273
pB 
pB  0.33 atm
TB
819

B 1.00 67.2 819

l pC 

C 0.33

67.2 273

a) En el proceso cclico, las variaciones de las


p
funciones termodinmicas son nulas y los balances de
(atm)
calor y de trabajo coinciden y vienen representados por
B
A
1
el rea del ciclo en el diagrama de Clayperon (pV). As
W
pues:
819 K
1
1
2
Q  W  ABq BC  44.8q  14.9 atm L 
2
2
3
C
1/3
 361.6 cal = 1512 J
273 K
W3
U  0
H  0
S  0
E
D
V (L)
67.2
22.4
b) Transformacin no-politrpica C
A. Puesto que
en esta transformacin a los estados inicial (C) y final
(A) les corresponde la misma temperatura, sern U = 0, H = 0
y, de acuerdo con el Primer Principio, Q = W, viniendo representado ste ltimo por el rea
del trapecio ACDE que se indica en el diagrama de Clayperon:
W3  (ABq AE)  W  44.9q114.9  29.9 atmL
= QCA  WCA   29.9 atm L = - 723.2 cal = - 3023 J

La variacin de la entropa la calculamos a travs de una transformacin isoterma que conecte


los estados C y A:
A dQ
A dW
A pdV
A dV
V
S 


 nR
 nR ln A 
C T
C
C
C V
T
T
VC
1
cal
J
 1q1.987 q ln  2.18
 9.12
3
K
K

- 323 -

Fsica Universitaria: Problemas de Fsica

Termologa - T00.18

18. Disponemos de una cierta cantidad de gas perfecto, a 1 atm de presin y 27 C de temperatura, y lo
sometemos a los siguientes procesos: 1 Una compresin isotrmica hasta que su volumen se reduce a la
cuarta parte de su valor inicial. 2 Una expansin adiabtica hasta devolverle su volumen inicial; entonces,
su presin es 0.4 atm. a) Determinar la atomicidad del gas (n de tomos de la molcula). b) Calcular los
cambios molares de energa interna, entalpa y entropa en el proceso total.

1
2 Transformacin isoterma, p1V1  p2V2 :
p

p2 

4.

V1
p1  4q1  4 atm
V2

2
3 Transformacin adiabtica, p2V2H  p3V3H :
H

V
2  p3
V3
p2

300 K
1
3

1.
0.4
V1/4

V1

l H

ln( p3 p2 ) ln(0.4 / 4)

 1.66
ln(V2 V3 )
ln(1/ 4)

Entre el valor del coeficiente adiabtico y el nmero


120 K de grados de libertad de la molcula (mono o
biatmicas) estn relacionados por
V
f 2
2
2
H
l f 

x3
H 1 1.66 1
f

que corresponde a molculas monoatmicas. Entonces


cal
f
cal
f 2
cal
R  1.987
CV  R  2.98
Cp 
R  4.98
K mol
2
K mol
R
K mol
b) Determinamos la temperatura correspondiente al estado 3:
p3V3
pV
pV
0.4
 1 1 l T3  3 3 T1 
q300  120 K
T3
T1
p1V1
1

Los cambios 'U y 'H tan solo dependen de las temperaturas inicial y final, de modo que
U  CV T  2.98q 120  300
 536.4 cal
H  C p T  4.98q 120  300
 892.8 cal
El cambio 'S en el proceso total se calcula a travs de la transformacin isocora 13
T
120
S  CV ln 3  2.98q ln
 2.73 cal/K
300
T
1

- 324 -

Fsica Universitaria: Problemas de Fsica

Termologa - T00.19

19. Tres moles de N2 a presin atmosfrica y temperatura de 300 K se comprimen isotrmica y reversiblemente
hasta duplicar la presin. Desde ese nuevo estado se expansiona adiabtica y reversiblemente hasta alcanzar
la presin original. a) Dibujar la transformacin indicando los estados inicial (1), intermedio (2) y final (3).
b) Calcular la temperatura, la presin y los volmenes desconocidos (resumirlo en un cuadro de resultados,
con sus unidades). c) Calcular los trabajos, calores y variaciones de energa interna, entalpa y entropa en
cada una de las transformaciones elementales y en su conjunto (resumirlo en un cuadro de resultados, con
sus unidades

Por tratarse de un gas biatmico, sern:

7
cal

n  3 mol C p  R  6.95

2
mol
K
7

5
cal
H   1.40

CV  R  4.97
5

2
mol K

nRT1 3q 0.08205q300
(1) V1 

 73.85 L
p1
1
(2)

p1V1  p2V2

V2 

(3)

p2V2H  p3V3H

p
2

p2

p1

3
V2 V3

p1
73.85
V1 
 36.92 L
p2
2
1/ H

= V3  36.92q 2

1/1.4

V1

p
V3  V2 2
p3

73.85 300

 36.92q1.64  60.58 L

36.92 300

60.58 246.1

p (atm) V (L) T (K)

p3V3
1q 60.58

 246.10 K
nR
3q 0.08205
Proceso 1-2 .- Isotermo (T = cte.)
T3 

W12  p d V  nRT

V2

V1

36.92
dV
V
 nRT ln 2  3q1.987 q300 ln
 1240 c al  5181 J
73.82
V
V1

Q12  U W  1240 c al  5181 J


U12  nCV T  0

H12  nC p T  0

S12 

Proceso 2-3.- Adiabtico (Q = 0) = U  Q W

Q12 1240
cal
J

 4.13
 17.3
T
300
K
K

y S  0

U 23  nCV T3  T2
 3q 4.97 246.10  300
 804 cal  3360 J
H 23  nC p T3  T2
 3q 6.95 246.10  300
 1124 cal  4698 J
W23  U  804 cal  3360 J

Q23  0

- 325 -

S 23  0

Fsica Universitaria: Problemas de Fsica

Termologa - T00.20

20. Disponemos de 5 moles de nitrgeno que se encuentran inicialmente a la presin atmosfrica y temperatura
de 0 C. Se les someten a un proceso ciclo compuesto por las siguientes etapas reversibles: 1 expansin
isbara, 2 compresin isotrmica y 3 enfriamiento iscoro. Sabiendo que al expandirse se realiza un
trabajo de 11346 J, calcular las variaciones de energa interna, de entalpa y de entropa, as como el trabajo
y calor producidos o consumidos en cada transformacin y en el ciclo.
Datos: R = 1.987 cal/mol K = 0.08205 atmL /mol K.

Por tratarse de un gas biatmico, ser


5

cal

CV  R  4.9675
2

mol.K

cal

mol.K

C p  R  6.9545

p
(atm)

3
2
1

546 K

Determinamos las coordenadas termodinmicas de


los tres estados:

224

112
V1 

V (L)

nRT1 5q 0.082q 273



 112 L
p1
1

V1 V2

T1 T2

l T2 

p2V2  p3V3 l

V2
224
T1 
q 273  546 K
V1
112

p3 

V2
224
q1  2 atm
p2 
V3
112

estado p (atm) V (L) T (K)

1
2 Proceso isobrico:
U  nCV T  5q 4.9675q (546  273)  6780 cal

H  nC p T  5q 6.9545q (546  273)  9493 cal


S  nC p ln

T2
546
cal
 5q 6.9545q ln
 24.1
T1
273
K

2
3 Proceso isotrmico:
U  0
H  0

S  nR ln

V2
112
cal
 5q1.987 q ln
 6.9
V1
224
K

3
1 Proceso isocoro:
U  nCV T  5q 4.9675q (273  546)  6780 cal

H  nC p T  5q 6.9545q (273  546)  9493 cal


S  nCV ln

T2
273
cal
 5q 4.9675qln
 17.2
T1
546
K

Para el ciclo completo: U  0

H  0

S  0

- 326 -

112

273

224

546

112

546

Fsica Universitaria: Problemas de Fsica

Termologa - T00.21

21. Dos litros de nitrgeno (gas biatmico), inicialmente a presin atmosfrica y 27 C, evolucionan segn las
transformaciones reversibles siguientes: 1) Un calentamiento a presin constante hasta duplicar el volumen
inicial. 2) Una expansin adiabtica hasta alcanzar la temperatura inicial. 3) Una transformacin que cierra
el ciclo, y cuya representacin es una recta en el diagrama p-V. Calcular los balances de calor y trabajo y los
cambios de energa interna, entalpa y entropa en cada una de las transformaciones.
Datos: R = 1.987 cal/molK = 0.08205 atml/molK.

CV 

5
R
2

Cp 

7
R
2

H

Cp
CV

p
(atm)

7
 1.4
5

Determinamos el nmero de moles:


pV
1q 2
pV  nRT l n 

 0.08 mol
RT 0.08205q300

600 K
0.088

Determinacin de los procesos


(1
2) Proceso de expansin isobrica:
V1 V2

T1 T2

l T2 

V2
4
T1  300  600 K
V1
2

pV H  cte. l TV H 1  cte.
T
l V3  V2 2
T

1/0.4

TV

TV

0.4
3 3

300 K
3
22.63
V(L)

 p (atm) V (L) T (K)

(2
3) Proceso de expansin adiabtica:

0.4
2 2

 4q 2

1/0.4

 22.63 L

(3
1) Proceso de compresin no-politrpico:
V
2
p3V3  p1V1 l p3  1 p1 
q1  0.088 atm
V3
22.63

Balances energticos
(1
2) Proceso de expansin isobrica:
7
Q12  nC p (T2  T1 )  0.08q q1.987 q300  170 cal  709 J
2
W12  p1 (V2 V1 )  1q 2  2 atm L  48 cal  203 J
5
U12  nCV (T2  T1 )  0.08q q1.987 q300  121 cal  506 J
2
7
H12  nC p (T2  T1 )  0.08q q1.987q300  170 cal  709 J
2
T2
7
cal
J
S12  nC p ln  0.08q q1.987 q ln 2  0.39
 1.64
2
T1
K
K

(sigue)

- 327 -

300

600

3 0.088 22.63 300

Fsica Universitaria: Problemas de Fsica

Termologa - T00.22

(2
3) Proceso de expansin adiabtica:
Q23  0

W23  U 23  121 cal  506 J


5
U 23  nCV (T3  T2 )  0.08q q1.987 q 300
 121 cal  506 J
2
7
H 23  nC p (T3  T2 )  0.08q q1.987 q 300
 170 cal  709 J
2
S23  0
(3
1) Proceso de compresin no-politrpico:
U 31  0
ya que T1  T3
H 31  0
U 31  Q31 W31  0 l Q31  W31

1
Q31  W31  rea del trapecio = 1 0.088
22.63 - 2
 11.23 atm L  272 cal  1136 J
2
cal
J
S31  S123  0.39
 1.64
K
K

- 328 -

Fsica Universitaria: Problemas de Fsica

Termologa - T00.23

22. Un mol de gas perfecto biatmico, a 27C de temperatura y 10 atm de presin, se somete a los siguientes
procesos reversibles: 1. Un calentamiento isobrico hasta que alcanza una temperatura de 227C. 2. Un
enfriamiento adiabtico hasta que se recupera la temperatura inicial. 3. Una compresin isotrmica que
cierra el ciclo. a) Representar grficamente el ciclo en un diagrama de Clayperon. b) Determinar el
rendimiento de una hipottica mquina que funcionase de acuerdo con el ciclo descrito y compararlo con el
de una mquina reversible de Carnot que funcionase entre las mismas dos temperaturas extremas (i.e., 27C
y 227C).

7
7
5
 1.40
Cp  R
CV  R
5
2
2
a) En la figura adjunta hemos representado el ciclo en un
diagrama p-V (Clayperon), recorrido en el sentido horario.
nRTA 1q0.08205q300

 2.46 L
(A) VA 
pA
10
n  1 mol

V
V
(B) A  B
TA TB

H

Q1
A

2.5

pC 

500K

Q2
C

300K

V
p (atm) V (L) T (K)

1
H 1

500
VC  4.10q
 14.71 L
300

B
W

T
500
2.46  4.10 L
VB  B VA 
300
TA

T
(C) TBVBH1  TCVCH1 l VC  VB B
TC

pAVA  pCVC

A 10

2.46 300

B 10

4.10 500

C 1.67

14.7 300

pAVA
 1.67 atm
VC

b) El rea encerrada por el ciclo representa el trabajo neto realizado (positivo). En la


transformacin isobrica AB el sistema absorbe una cantidad de calor Q1 (positivo). En
proceso adiabtico BC el sistema no intercambia energa en forma de calor. En el proceso de
compresin isotermo CA el sistema cede una cantidad de calor Q3 (negativo).

7
Q1  nC p (TB  TA )  1q 1.987 q 500  300
 1391 cal

VA d V
V
2.46

 nRT ln A  1q 2q300 ln
 1073 cal
Q3  WCA  p d V  nRT V
C
14.7
V
VC

Puesto que se trata de un ciclo ('U = 0), el Primer Principio de la Termodinmica nos permite
escribir
W  Q1 Q3  13911073  318 cal
b) El rendimiento de esa mquina trmica sera: I 

W
318

 23%
Q1 1391

El rendimiento de la mquina de Carnot sera: ICarnot 

- 329 -

T1  T2 500  300 2

  40%
T1
500
5

Fsica Universitaria: Problemas de Fsica

Termologa - T00.24

23. Un mol de gas perfecto biatmico, a 27C de temperatura y 10 atm de presin, se somete a los siguientes
procesos reversibles: 1. Un calentamiento isocrico hasta que alcanza una temperatura de 227C. 2. Un
enfriamiento adiabtico hasta que se recupera la temperatura inicial. 3. Una compresin isotrmica que
cierra el ciclo. a) Representar grficamente el ciclo en un diagrama de Clayperon. b) Determinar el
rendimiento de una hipottica mquina que funcionase de acuerdo con el ciclo descrito y compararlo con el
de una mquina reversible de Carnot que funcionase entre las mismas dos temperaturas extremas (i.e., 27C
y 227C).

Por tratarse de un gas biatmico:


7
5
7
Cp  R
CV  R
H   1.40
2
2
5
a) En la figura adjunta hemos representado el ciclo en
un diagrama p-V (Clayperon), recorrido en el sentido
horario.
Determinamos los estados A, B y C:
nRTA 1q0.08205q300

 2.46 L
(A) VA 
pA
10
(B)

pA
p
 B
TA
TB

pB 

1
H 1

500
VC  2.46q
 8.82 L
300
pC 

pA

B
Q1
W

500K

Q3
pC

300K

V
p (atm) V (L) T (K)
A

2.5

pB

TB
500
10  16.67 atm
pA 
300
TA

T
(C) TBVBH 1  TCVCH 1 l VC  VB B
TC

pAVA  pCVC

10

2.46

300

B 16.67

2.46

500

8.82

300

2.79

pAVA
 2.79 atm
VC

b) El rea encerrada por el ciclo representa el trabajo neto realizado (positivo). En la


transformacin isocrica AB el sistema absorbe una cantidad de calor Q1 (positivo). En
proceso adiabtico BC el sistema no intercambia energa en forma de calor. En el proceso de
compresin isotermo CA el sistema cede una cantidad de calor Q3 (negativo).

5
Q1  nCV (TB  TA )  1q 1.987 q 500  300
 993.5 cal

VA d V
V
2.46

 nRT ln A  1q1.987 q300 ln


 761.1 cal
Q3  WCA  p d V  nRT V
C
8.82
V
VC

Puesto que se trata de un ciclo ('U = 0), el Primer Principio de la Termodinmica nos permite
escribir
W  Q1 Q3  993.5  761.1  232.4 cal
b) El rendimiento de esa mquina trmica sera: I 

W 232.4

 23%
Q1 993.5

El rendimiento de la mquina de Carnot sera: ICarnot 

- 330 -

T1  T2 500  300 2

  40%
T1
500
5

Fsica Universitaria: Problemas de Fsica

Termologa - T00.25

24. Tres moles de nitrgeno inicialmente a 27C y presin atmosfrica se someten al ciclo constituido por las
tres transformaciones siguientes: 1) calentamiento a presin constante hasta duplicar el volumen inicial.
2) una compresin isoterma hasta alcanzar el volumen inicial. 3) un enfriamiento iscoro que cierra el ciclo.
Determinar el trabajo, el calor, y las variaciones de energa interna, entalpa y entropa en cada
transformacin y en el ciclo.

Datos: 3 moles de gas biatmico.


7
cal
5
cal
C p  R  6.95
CV  R  4.97
2
2
mol K
mol K

1 atm L  101.328 cal

R  1.987

p (atm) V (L) T (K)

cal
mol K

Determinacin de los estados 1, 2 y 3:


nRT1 3q 0.08205q300
(1) V1 

 73.85 L
p1
1

V V
(2) 1  2
T1 T2
(3)

V
l T2  2 T1  2q300  600 K
V1

p2V2  p3V3 l

p3 

V2
p2  2q1  2 atm
V3

Balances energticos:

2 atm
1 atm

74

300

148

600

74

600

(3)
(2)
(1)
74 L

(1
2) Proceso de expansin isobrica (p=0)
W  p V  1q (147.69  73.85)  73.85 atm L  1788 cal  427 J
12
1
Q  nC T  3q 6.95q (600  300)  6259 cal  1496 J
p
12
U12  nC T  3q 4.97q (600  300)  4471 cal  1069 J
V
H12  nC T  3q 6.95q (600  300)  6259 cal  1496 J
p
T
600
cal
J
S12  nC ln 2  3q 6.95q ln
 14.46
 3.46
p T
300
K
K
1
(2
3) Proceso compresin isoterma (T=0)
V
W  nRT ln 3  3q1.987 q600qln 2  2479 cal  592 J
23
V
2
Q  W  2479 cal  592 J
23
23
U 23  nC T  0
V
H 23  nC T  0
p
Q
2479
cal
J
 4.13
 0.99
S 23  23 
T
600
K
K
2

- 331 -

600 K
300 K
148 L

Fsica Universitaria: Problemas de Fsica

Termologa - T00.26

(3
1) Proceso isocoro (V=0)
W31  0

Q  nC T  3q 4.97q (300  600)  4471 cal  1069 J


V
31
U 31  nCV T  3q 4.97 q (300  600)  4471 cal  1069 J

H 31  nC T  3q 6.95q (300  600)  6259 cal  1496 J


p

T
300
cal
J
S31  nC ln 1  3q 4.97 qln
 10.33
 2.47
V T
600
K
K
3
Balances energticos en el ciclo completo:
W  W12 W23 W31  691 cal  165 J

Q  Q12 Q23 Q31  691 cal  165 J


U  0

H  0

S  0

- 332 -

Fsica Universitaria: Problemas de Fsica

Termologa - T00.27

25. Dos litros de helio (gas monoatmico) a la presin de 16 atm y 600 K se expansionan isotrmicamente hasta
que su volumen es de 8 L y luego se comprime a presin constante hasta que su volumen y temperatura son
tales que puede cerrarse el ciclo mediante una compresin adiabtica. a) Dibujar el ciclo termodinmico
reversible en un diagrama (p-V) b) Calcular la temperatura, presin y volumen en los estados que no son
conocidos. c) Calcular los trabajos, calor y variaciones de energa interna, entalpa y entropa en cada una de
las transformaciones elementales y en el ciclo.

Datos: Puesto que se trata de un gas ideal


p
monoatmico (tres grados de libertad), sern:
(atm)
f 2
5
cal
Cp 
R  R  4.97
2
2
mol.K
16
3
cal
f
CV  R  R  2.98
2
2
mol.K
4

Cp
f 2 3 2
H


 1, 6
3
CV
f

n

1
2

p1V1
16q 2

 0.65 moles
RT1 0.08205q 600

Determinacin de los estados 1, 2 y 3:

2
3 Isobara:

p V 16q 2
p2  1 1 
 4 atm
V2
8

V2 V3

T2 T3
H
1 1

l T3  T2
H
3 3

1
3 Adiabtica: p V  p V

V3
4.59
 600
 344.6 K
V2
8
V3 p1

V1 p3

16

600

600

S12 

Q12 1074

 1.79 cal
K
T1
600

(2
3) Proceso de compresin isobrica (p = 0):
Q23  nC p T3  T2
 0.65q 4.97 q 345  600
 825 cal
W23  p2 V3 V2
 4 q 3.41
 13.64 atm L  330.3 cal
U 23  nCV T3  T2
 0.65q 2.98q 345  600
 495 cal
H 23  Q23  825 cal
S 23 

3
2

3 dT
T
dQ
345
 nC p
 nC p ln 3  0.65q 4.97 q ln
 1.79 cal
K
2
T
T
T2
600

- 333 -

4.6 345

p 5
16 5
l V3  V1 1  2  4.59 L
4
p3

Balances energticos:
(1
2) Proceso de expansin isotrmica (T = 0):
2
V
8
Q12  W12  pdV  nRT1 ln 2  0.65q1.987 q 600 ln  1074 cal
1
2
V1

U12  H12  0

V (L)

p
V
T
(atm) (L) (K)

1
2 Isoterma:

p1V1  p2V2 l

600 K

Fsica Universitaria: Problemas de Fsica

Termologa - T00.28

(3
1) Proceso de compresin adiabtica (Q = 0):
Q31  0

W31  p2 V1 V3
 4 q 3.41
 13.64 atm L  330.3 cal
U 31  nCV T1  T3
 0.65q 2.98q 345  600
 495 cal
H 31  Q31  825 cal
S31  0
Q (cal)

W (cal)

'U (cal)

'H (cal) 'S (cal/K)

12 (T = cte)

1074

1074

1.79

23 (p =cte)

-825

-330

-495

-825

-1.79

31 (Q = 0)

-495

495

825

Ciclo

249

249

- 334 -

Fsica Universitaria: Problemas de Fsica

Termologa - T00.29

26. Un mol de un gas ideal monoatmico est inicialmente a 273 K de temperatura y 1 atm de presin. El gas
experimenta una expansin isotrmica hasta que alcanza un volumen 2.5 veces el inicial. Luego se extrae
calor a volumen constante, disminuyendo la presin. Finalmente se comprime adiabticamente hasta volver
al estado inicial. a) Dibujar la transformacin en un diagrama pV. b) Determinar en cada transformacin el
calor y el trabajo intercambiados as como los incrementos de energa interna, entalpa y entropa. (Presentar
los resultados en una tabla y en unidades del sistema internacional).

Gas Monoatmico:
CV  3 / 2
R

C p  5 / 2
R

H 5/3

Determinacin de los estados:


Estado (1):
nRT 1q 0.08205q 273

 22.4 L
pv  nRT l V 
p
1
Estado (2):
V2  2.5V1  56.0 L

p1V1  p2V2 l

p2 

273 K

2
3

V1
p1  0.40 atm
V2

Estado (3):
H

5/3
V
1
p3  p1 1   0.22 atm
2.5
V3
pV
0.22q56.0
T3  3 3 
 148 K
nR
0.08205

p3V3H  p1V1H

Estados p (atm) V (L) T (K)


(1)

1.00

22.4

273

(2)

0.40

56.0

273

(3)

0.22

56.0

148

Balances energticos:

Proceso (1
2) Expansin isotrmica:
V
Q12  W  nRT ln 2  497 cal  2078 J
V1
U12  H12  0
Q
S12  12  1.8 cal/K  7.6 J/K
T1

Proceso (2
3) Enfriamiento isocrico:

Proceso (3
1) Compresin adiabtica:
Q31  0
W31  U 31  375 cal  1568 J
U 31  nCV T1  T3
 375 cal  1568 J
H 31  nC p T1  T3
 625 cal  2613 J
S31  0

Ciclo completo:
Q  122 cal  510 J
W  122 cal  510 J
U  0
H  0
S  0

Q23  nCV T3  T2
 375 cal  1568 J
W 0
U 23  Q23  375 cal  1568 J
H 23  nC p T3  T2
 625 cal  2613 J
T
S 23  nCV ln 3  1.8 cal/K  7.6 J/K
T2

- 335 -

Fsica Universitaria: Problemas de Fsica

Termologa - T00.30

27. Un mol de N2 , inicialmente a una temperatura de 20 C y una presin de 5 atm, es sometido a los siguientes
procesos reversibles sucesivos: (i) expansin adiabtica hasta una presin de 1 atm; (ii) calentamiento a
presin constante hasta alcanzar de nuevo una temperatura de 20 C; (iii) calentamiento a volumen constante
hasta que la presin es de 5 atm; (iv) enfriamiento a presin constante hasta el estado inicial. a) Dibujar el
diagrama pV del ciclo. b) Calcular el trabajo realizado en el ciclo. c) Determinar las cantidades de calor
absorbidos o cedidos en cada etapa y en el ciclo completo. d) Calcular las variaciones de entropa en cada
etapa y en el ciclo completo.

Datos: 1 mol de gas biatmico.

5
5

C  R  8.314 J
 20.79 J

mol K
mol K
V 2
2

7
7

C p  R  8.314 J
 29.10 J

mol K
mol K :

2
2

C
7
H  p   1.4
CV
5

p
(atm)
5

3
2

Q =0

T=293 K

Determinacin de los estados:

V (L)

nRT1 1q 0.08205q 293



 4.81L
p1
5

(1)

V1 

(2)

p1V1H  p2V2H

1/1.4
p H
5
l V2  1 V1  4.81  15.17 L
1
p2

p2V2 1q15.17

 185 K
nR
0.08205
nRT3 0.08205q 293
V3 

 24.04 L
p3
1
T2 

(3)
(4)

T4 

p(atm) V (L)

p4V4 5q 24.04

 1465 K
nR
0.08205

1 5

4.81

2 1

15.17 185

3 1

24.04 293

4 5

24.04 1465

Balances energticos:

(12) Expansin adiabtica (Q=0):


Q12  0
S12  0
(23) Calentamiento isobaro (p=0):
Q23  nC p T3  T2
 29.10q 293 185
 3143 J
S 23 

3
2

3 dT
T
Q
293
 nC p
 nC p ln 3  29.10q ln
 13.38 J
K
2 T
T
T2
185

(34) Calentamiento isocoro (V=0):

- 336 -

T (K)
293

Fsica Universitaria: Problemas de Fsica

Termologa - T00.31

Q34  nCV T4  T3
 20.79q 1565  293
 24 360 J
4

S34 

4 dT
Q
T
1465
 nCV
 nCV ln 4  20.79qln
 2033.45 J
K
3
T
T
T3
293

(41) Enfriamiento isobaro (p=0)


Q41  nC p T1  T4
 34 104 J
S 41 

1
4

T1 d T
T
Q
293
 nC p
 nC p ln 1  29.19q ln
 46.83 J
K
T
4
T
T
T4
1465

Ciclo completo:
Qciclo  6 601.3 J

U ciclo  Qciclo Wciclo  0 l Wciclo  Qciclo  6 601.3 J


Sciclo  0

- 337 -

Fsica Universitaria: Problemas de Fsica

Termologa - T00.32

28. Una mquina trmica ideal est constituida por un cuerpo de


bomba que encierra un gas perfecto que evoluciona segn un
ciclo de Carnot. a) Deducir la expresin de su rendimiento en
funcin de las temperaturas T y T. b) Son necesariamente
iguales las reas 23322 y 14411? Por qu?

1
2
4

a) El rendimiento de un ciclo de Carnot viene dado por la


expresin
W
I
Q

1 4

T
3

T
V

en la que W es el trabajo realizado y Q el calor intercambiado con la fuente caliente.


Formulando el Primer y Segundo Principios de la Termodinmica, escribiremos:

T
Qa

W  Q Qa l I  1

Q
a
a
T T
T

Q
l I  1 

W
a
a
a
T
T
Q
Q
Q
T
0 l


a
T
T
Q
T

que es la expresin pedida.


Q
b) Las dos reas indicadas son necesariamente iguales.
T
Para demostrarlo, prestemos atencin a que las transformaciones
2o3 y 4o1 tienen lugar entre las mismas temperaturas extremas y por lo tanto
U 23  U 41
y adems, por ser adiabticas, sern
Q23  Q41  0
por lo que, en virtud del Primer Principio U  Q W

W23  W41 y, como estos

trabajos vienen representados en un diagrama de Clayperon por las reas 23322 y 14411,
dichas reas sern igual (c.q.d.).

- 338 -

Fsica Universitaria: Problemas de Fsica

Termologa - T00.33

29. Una mquina trmica reversible, trabaja segn un ciclo formado por las siguientes transformaciones
reversibles:
1. Expansin isotrmica a 300 K
2. Expansin isoentrpica (adiabtica) a 3500 cal/K
3. Compresin isotrmica a 200 K
4. Compresin isoentrpica a 1500 cal/K
a) Representar el ciclo en un diagrama TS. b) Determinar el calor absorbido en la primera transformacin.
c) dem en la tercera. d) Calcular el trabajo realizado en dicho ciclo.

a) En el diagrama entrpico TS, el ciclo se


representa por un rectngulo, recorrido en el
sentido que se indica en la figura. En realidad, se
trata de un ciclo reversible de Carnot, ya que est
constituido por dos isotermas y dos adiabticas.
De la definicin de entropa, se sigue que el calor
intercambiado es
dQ
dS 
l Q  TdS  T S
T
b) En la expansin 12 el sistema cede energa en
forma de calor:

T
1

300 K

200 K

4
1500 cal/K

3500 cal/K

Q12  T1S12  300q 3500 1500


 300q 2000  6q105 cal

c) En la compresin 34 el sistema absorbe energa en forma de calor:


Q34  T3S34  200q 1500  3500
 200q 2000  4q105 cal

b) En el ciclo completo, la energa interna del sistema permanece constante, por lo que el
Primer Principio de la Termodinmica nos permite calcular fcilmente el trabajo realizado:

U  Q W  0 l W  Q  Q12 Q34  2q105 cal

- 339 -

Fsica Universitaria: Problemas de Fsica

Termologa - T00.34

30. La figura muestra el ciclo terico de una mquina trmica ideal en


el diagrama T-S. El calor absorbido en la fuente caliente es de 924
cal y se cede calor a una fuente trmica a 0 C. Calcular: a) La
temperatura T de la fuente caliente. b) El calor cedido a la fuente
fra. c) El trabajo proporcionado por dicha mquina. d) El
rendimiento de la mquina.

3
5 S(cal/K)

Se trata de un ciclo de Carnot, constituido por dos isotermas y dos adiabticas.


a) En el proceso isotrmico 12 ser
Q
Q
924
S12 
l T

 308 K
T
S12
3
b) Aplicamos la relacin
Q Qa
Ta
273
 0 l Qa   Q  
q924  819 cal
T Ta
T
308
c) A partir del Primer Principio de la Termodinmica,
W  Q Q a  924  819  105 cal  439 J
d) El rendimiento de una mquina est definido como el cociente
entre el beneficio y el coste:
W 105
I 
 0.114  11.4%
Q 924

- 340 -

hogar

T
Q
W

m quina
trmica

Q
T
refrigerante

Fsica Universitaria: Problemas de Fsica

Termologa - T00.35

31. Un acondicionador de aire funciona segn un ciclo reversible de Carnot. Su potencia frigorfica es de
10 kilofrigoras/hora cuando extrae calor de un local a 24 C y lo cede al exterior a 35 C. a) Calcular la
potencia que debe suministrar el motor. b) Con esa misma potencia del motor, calcular la potencia
frigorfica cuando el local se encuentra a 21 C y el exterior a 40 C.

Mcal
Q 2  10
h
a) Aplicamos el Primer y Segundo Principios de la Termodinmica a la mquina reversible:
Q1 Q 2  W

l
Q1 Q 2

0
T1=308 K
T1 T2
Q1

Mcal
W  10.37 10  0.37
W

308
Mcal
T1 

q10  10.37
Q1   Q2  
Q2
297
h
T
2

Datos: 1 kfrigora  1 Mcal l

T2=297 K

De modo que la potencia del motor es:


Mcal 4.18 MJ 1 h
P  W  0.37
 430 W
h 1 Mcal 3600 s

b) De nuevo, aplicamos el Primer y Segundo Principios de la Termodinmica:


Q Q  W
2
1
Q1 Q 2
W


l


Q
Q
1 2  0
T1
T2
T1  T2
T1=313 K
T1 T2

Q1

W
Q2
T2=294 K


313
Mcal
T1
Q1 
W 
0.37)
 6.1

313  294
h
T1  T2


294
Mcal
T2
W 
0.37)
 5.7
Q2 
294
313
h
T
T


2
1

De modo que la potencia frigorfica es:


Mcal
kfrigoras
Q 2  5.7
 5.7
h
h

- 341 -

Fsica Universitaria: Problemas de Fsica

Termologa - T00.36

32. Una mquina trmica reversible funciona intercambiando calor con tres focos trmicos cuyas temperaturas
son: T1 = 500 K, T2 = 400 K y T3 = 300 K. La mquina toma una cantidad de calor Q1 = 700 kcal del primer
foco y realiza un trabajo de 1 kWh. a) Calcular las cantidades de calor intercambiadas con los otros focos.
b) Determinar el rendimiento de la mquina. c) Calcular los cambios de entropa en los distintos niveles
trmicos y el total.

W  1 kWh  1000q3600  3.6q106 J  860 kcal


a) Aplicamos el Primer y el Segundo Principio de la Termodinmica:
Q1 Q2 Q3  W
Q2 Q3  W  Q1

Q Q
l Q2 Q3
Q3
Q
1
2
 0
  1
T1
T1 T2 T3
T2 T3
de modo que disponemos de dos ecuaciones con dos incgnitas (Q2, Q3), que nos conducen a
Q2 Q3  W  Q1  160
Q2 Q3  160

1 1 1 1
1
l Q2 Q3
l % 1 1   
Q2
Q3
700

3 4 12


4
3
 140
400 300
500
3
4
Q2

12

160

140

1
3

2320 kcal

Q3 12

b) Por definicin de rendimiento de una mquina


W
W
860
860
K
28.5%
Qabs Q1  Q2 700  2320 3020
c) Cambios de entropa en cada nivel trmico:

Q
700
kcal

 1.4
S1  1 

T1 500
K

Q2 2320
kcal


 5.8
S 2 
S  0
T2
400
K

Q3 2160
kcal


 7.2
S3 

T3
300
K

- 342 -

160

1
4

140

2160 kcal

T1=500K
Q1=700kcal
Q2
W=860kcal
T2=400K
Q3
T3=300K

Fsica Universitaria: Problemas de Fsica

Termologa - T00.37

33. Dos mquinas trmicas reversibles funcionan acopladas: una como mquina trmica y la otra como mquina
frigorfica. La primera mquina absorbe 30 kcal de un foco a 600 K y cede calor a otro foco a 200 K. El
trabajo producido por la mquina trmica se le suministra a la mquina frigorfica, que intercambia calor
con dos focos a 200 K y 300 K. Determinar todos los intercambios de calor de las mquinas con sus focos
calorficos.

Aplicamos el Primer y Segundo Principios de la Termodinmica a la primera mquina:


Q1 Q2  W
30 Q2  W
W  20 kcal
Q Q
l 30
l
Q
1
2
2

Q2  10 kcal
0

0
600 200
T1 T2
y a la segunda, teniendo en cuenta que W' = -W = -20 kcal, de modo que
Q1a Q2a  W a
Q1a Q2a  20

Q1a  60 kcal

Q a Q a
l Q1a
l

a
Q
1
2
2

0

0

Q2a  40 kcal
T1a T2a

300 200

600 K
Q1

300 K
W

Q1
W

Q2

Q2
200 K

200 K

- 343 -

Fsica Universitaria: Problemas de Fsica

Termologa - T00.38

34. Disponemos de 100 g de nitrgeno (N2) a 25C y 30 atm. Los sometemos a una expansin adiabtica brusca
contra una presin exterior constante de 10 atm, hasta que el gas alcanza esta presin. Admtase que el gas
tiene un comportamiento ideal. a) Determinar la temperatura final del gas. b) Calcular los cambios que
experimentan la energa interna y la entropa del gas en el proceso de expansin.
Datos: R = 0.08205 atm.L/(mol.K) = 1.987 cal/(mol.K ), M(N2)=28 g/mol.

p
30 atm

i
Proceso
irreversible

298 K

10 atm

241 K

Se trata de una expansin adiabtica irreversible, contra


una presin exterior constante, ya que no existe
equilibrio entre la presin del gas y la presin exterior,
de modo que tan slo los estados inicial (i) y final (f) del
sistema son estados de equilibrio..
a) Calcularemos la temperatura final (que ser distinta
de la que se obtendra en un proceso reversible) a partir
del Primer Principio de la Termodinmica, con Q = 0,
por tratarse de un proceso adiabtico:
%U  Q W  0 W  W

Por tratarse de una gas ideal que se expande contra una presin exterior constante, tendremos:
U  nCV (Tf  Ti )

nRTf nRTi

pf

W  pext (Vf Vi )  pf p  p  nR Tf  p Ti


f

i
i

p
CV f R

p
p
pi
nCV (Tf  Ti )  nR Tf  f Ti l (CV R )Tf  CV f RTi l Tf 
Ti

pi
pi
Cp

5 R 10 R
30 q 298  2.83 q 298  241 K  32 C
Tf  2
7 R
3.50
2
b) La variacin de energa interna en el proceso ser:
100 5
q q1.987 q (241 298)   1011 cal   4227 J
U  nCV (Tf  Ti ) 
28 2
Para calcular la variacin de entropa, partimos de la formulacin del Segundo Principio en
funcin de la entalpa:
Q
dT
dp
Q  nC p dT V d p l dS 
 nC p
 nR
T
T
p
de modo que, para un proceso entre los estados inicial (i) y final (f), ser:
'S

S f  Si

nC p ln

Tf
p
 nR ln f
pi
Ti

100
10
7 241
u 1.987 u ln
 ln
30
28
2 298

2.52

cal
J
10.55
K
K

producindose un incremento de entropa, por tratarse de un proceso de expansin espontnea


(irreversible).

- 344 -

Fsica Universitaria: Problemas de Fsica

Termologa - T00.39

35. Dos moles de nitrgeno a 27C estn contenidos en un cilindro cerrado por un mbolo mvil
y aislado trmicamente del exterior. Inicialmente el gas est a una presin de 4 atm debido a
una pesa que hay sobre el mbolo. Cuando se retira la pesa, el gas se expande bruscamente
contra una presin exterior constante de 1 atm. a) Determinar el estado final del gas.
b) Calcular el trabajo realizado por el gas y las variaciones de energa interna, entalpa y
entropa. Datos: R = 0.08205 atmL/molK = 1.987 cal/mol.K.

Se trata de una expansin adiabtica irreversible, contra


una presin exterior constante, ya que no existe
equilibrio entre la presin del gas y la presin exterior,
de modo que tan slo los estados inicial (i) y final (f)
del sistema son estados de equilibrio..
a) Calcularemos la temperatura final a partir del Primer
Principio, con Q = 0 (proceso adiabtico):
U  Q W  0 W  W

p
Proceso
irreversible

300 K

1 atm

Por tratarse de una gas ideal que se expande contra una


presin exterior constante, tendremos:
U  nCV (Tf  Ti )

nRTf nRTi

p
W  pext (Vf  Vi )  pf

 nR Tf  f Ti

pi
pi
pf

p
p
nCV (Tf  Ti )  nR Tf  f Ti l (CV R )Tf  CV f

pi
pi

4 atm

f
12.3 L

RTi

236 K
38.7 L

CV
l Tf 

pf
pi

Cp

R
Ti

5 R 1 R
4 q300  2.75 q300  236 K  37 C
o sea, Tf  2
7 R
3.50
2
nRTf
2q 0.08205q 236

 38.7 L
El volumen final ser: Vf 
pf
1
b) Las variaciones de energa interna y de entalpa en el proceso sern:
5
U  nCV (Tf  Ti )  2 q q1.987 q (236  300)   639 cal   2673 J
2
7
H  nC p (Tf  Ti )  2 q q1.987 q (236  300)   895 cal   3742 J
2

El trabajo realizado durante el proceso de expansin es W  U  2673 J


Para calcular la variacin de entropa, partimos de la formulacin del Segundo Principio en
funcin de la entalpa:
Q
dT
dp
Q  nC p dT  vdp l dS 
 nC p
 nR
l
T
T
p
T
p
7 236
1
cal
J
S  Sf  Si  nC p ln f  nR ln f  2q1.987 q ln
 ln  2.15
 9.01

2 300
4
Ti
pi
K
K
producindose un incremento de entropa por tratarse de un proceso de expansin adiabtica
espontnea (irreversible).

- 345 -

Fsica Universitaria: Problemas de Fsica

Termologa - T00.40

36. Dos moles de un gas biatmico a 200C y 20 atm, se expansionan adiabtica y reversiblemente hasta una
temperatura final de 100C. a) Calcular la presin y el volumen finales. b) Si el proceso hubiese sido no
adiabtico y constituido por una transformacin isobrica seguida de otra isocrica, calcular la cantidad de
calor que intercambiara el sistema hasta alcanzarse el mismo estado final. c) Por ltimo, considrese un
proceso de expansin adiabtico y no reversible contra una presin exterior constante igual a la presin final
del proceso en los apartados anteriores; cul ser la temperatura final alcanzada? Determinar la variacin
de entropa en este proceso.

Gas biatmico:
5
7
7
CV  R
Cp  R
H   1.4
2
2
5
nRT1 2q 0.08205q 473.15
V1 

 3.88 L
20
p1

20 atm

857 K

a) Por tratarse de un proceso adiabtico, ser


8.71 atm
p11HT1H  p21H T2 H , de modo que

473 K
2

3.5
T 1H
473.15
p2  p1 1  20
 8.71 atm

373.15
T

3.88 L

7.03 L

373 K
V

V2 

nRT2 2q 0.08205q373

 7.03 L
p2
8.71

b) Determinamos la temperatura correspondiente al punto 3:

V1 V3

T1 T3

T3 

V3
7.03
473.15  856.62 K
T1 
3.88
V1

Q p  nC p (T3  T1 )  2 q 7 1.987 q (856.62  473.15)  5333.66 cal


2

Q  Q p QV  530.40 cal

QV  nCV (T2  T3 )  2 q 5 1.987 q (373.15  856.62)   4803.26 cal


2

c) Por tratarse de un proceso adiabtico, ser Q = 0; por tratarse de un proceso de expansin
contra una presin externa constante, ser  W = p2 (Vf - V1). Entonces, aplicando el Primer
Principio al proceso irreversible y utilizando la ecuacin de estado en el estado final, se sigue

U  Q W nCV (Tf  T1 )   pext (Vf V1 )

nC T pextV1

Tf  V 1

nRTf


p
V
nRT
V
nC p
ext f
f
f

pext

1.987
2q 52 1.987q 473.15 8.71q3.88q 0.08205
Tf 
 396.85 K
2q 72 1.987
2q 0.08285q396.85
 7.48 L
8.71
El cambio que experimenta la entropa se calcula mediante una expresin general que
relaciona los dos estados, inicial y final, de equilibrio; i.e.,
T
V
396.85
7.48
cal
%S  nCV ln f nR ln f  2q 52 1.987q ln
2q1.987q ln
 0.86
T1
V1
473.15
3.88
K
Vf 

- 346 -

Fsica Universitaria: Problemas de Fsica

Termologa - T00.41

37. En un sistema aislado, un foco a 127 C transfiere una cantidad de calor de 600 J a otro foco a 27 C, sin la
obtencin de trabajo y mantenindose constantes las temperaturas de los focos. a) Cul es la variacin de
entropa del sistema (o del universo) en este proceso?. b) Cunto calor, como mximo, se puede
transformar en trabajo? c) En el caso anterior, cul sera la variacin de entropa del sistema (o del
universo)?

Datos: T1 = 400 K, T2 =300 K, Q = 600 J


a) En la figura se esquematiza el proceso mediante el cual el
foco o subsistema caliente cede calor al foco o subsistema fro,
mantenindose constantes las temperaturas respectivas.
Obviamente, se trata de un proceso irreversible en el que el
foco caliente cede una cantidad de calor Q1 = -600 J y el foco
fro recibe una cantidad de calor Q2 = +600 J. As, la variacin
de entropa durante el proceso ser
Q
J
Q Q
600 600
S  i  1 2 

 1.5 2.0  0.5
400
300
K
T1 T2
i Ti

T1 = 400K

Sistema
aislado

Entorno
del sistema

T2 = 300K

de modo que la entropa del sistema aumenta, como corresponde al proceso irreversible que
tiene lugar en su seno. Puesto que el sistema est aislado, no intercambiando calor ni trabajo
con su entorno, la variacin de entropa del Universo coincide con la del sistema.
b) Para calcular el .mximo rendimiento de la conversin de calor en trabajo, imaginamos una
mquina reversible de Carnot que opere entre esas temperaturas, de modo que aplicando el
Primer y Segundo Principios de la Termodinmica:
T1 = 400K
Q1 Q2  W
W  Q1 Q2  600  450  150 J

l
T2
300
Q1 Q2

0



Q2   T Q1   400 600  450 J


Q1
T
T

2
1
1

Se pueden transformar 150 J por cada 600 J que se toman del foco
caliente, cedindose 450 J al foco fro.
W
c) En las condiciones del apartado anterior, al tratarse de un
Q2
rendimiento mximo en la conversin de calor en trabajo, la
variacin de entropa ser nula, como queda bien patente en la
segunda de las ecuaciones (2 Principio) que hemos utilizado para
T2 = 300K
calcularlo.

- 347 -

Fsica Universitaria: Problemas de Fsica

Termologa - T00.42

38. a) Una mquina frigorfica irreversible, movida por un motor de 1 kW, extrae 50 kcal/min de un recinto
fro. Cul es su rendimiento o eficacia? Qu cantidad de calor recibe el foco caliente? b) Si el rendimiento
de la mquina fuese la mitad del correspondiente a un ciclo de Carnot que operase entre los mismos lmites
de temperatura (-20C y 130C), cul sera el trabajo recibido por minuto por el sistema activo? qu
cantidad de calor se cedera entonces al foco caliente?

Datos: W  1000 W

50 000q 4.18
Q 2 
 3483 W
60

a) Primer Principio:
Q1  Q 2 W  3483 1000  4483 W

Eficacia de la mquina frigorfica:


Q 2
3483
Ff 

 3.483  348%
1000
W
Rendimiento funcionando como mquina trmica:
W
1000
I

 0.22  22 %
4483
Q1
Obsrvese que se trata de una mquina irreversible, de modo que:
Q1 Q 2 4483 3483



 11.12 13.77  2.64 W/K (irreversible)
T1 T2
403
253
Q1 Q 2

b 0 (teorema de Clausius)
T1 T2

T1
Q1
W
Q2
T2
T1=403K
Q1
W
Q2
T2=253K

b) Rendimiento de un ciclo reversible de Carnot que trabaje entre las


mismas temperaturas:
T1
T1  T2
T2
253
I
 1  1
 0.37  37 % l I a   0.19
I
T1
T1
403
2
W
Del Primer Principio de la definicin del rendimiento se sigue:



Q1a  Q2a W a

T2
l (1 I a) Q1a  Q 2a l

W a
I a 
l W a  I a Q1a

Q1a

Q 2a
50 000
kcal
a
Q1  (1 I a)  1 0.19  61.43 min  4280 W

W a  0.19q 4280  796 W

- 348 -

Q1

Q2

Fsica Universitaria: Problemas de Fsica

Campo elctrico. E01.1

1. Una esfera slida, no conductora y de radio R, posee una densidad volumtrica de carga proporcional a la

distancia desde su centro, i.e., U = Ar, siendo A una constante. Determinar el valor del campo elctrico en el
interior y en el exterior de la distribucin de carga y representar su magnitud en funcin de r.

Aplicaremos el teorema de Gauss a una superficie gaussiana de radio r concntrica con la


esfera slida:
q
q
1 q
2
vS E<dS  F0 l vS E dS  E vS dS  ES  E 4Qr  F0 l E  4QF0 r 2

siendo q la carga elctrica contenida en la superficie gaussiana, que calcularemos


descomponiendo la esfera slida cargada en capas esfricas. En efecto la carga que pose una
de tales capas, de radio r y espesor dr, ser
r

dq  S dV  Ar 4Qr 2 dr  4Q Ar 3dr l q  dq  4Q A r 3dr  Q Ar 4


0

a) En el interior de la esfera, para r  R, ser


dS

E

dr
r
dq

1 Q Ar 4
A 2

r
2
4QF0 r
4F0

b) En el exterior de la esfera, para r ! R, la carga que


contribuye al campo es Q  Q AR 4 , con independencia
del valor de r, de modo que ser
E

1 Q AR 4 AR 4 1

4QF0 r 2
4F0 r 2

c) Justamente en la superficie de la esfera,


para r = R, ambas expresiones anteriores
conducen a

E
AR 2
4F0

E

- 349 -

AR 2
4F0

Fsica Universitaria: Problemas de Fsica

Campo elctrico. E01.2

2. Un conductor esfrico hueco tiene un radio interno R1 y un radio externo R2. En el centro de la cavidad
existe otro conductor esfrico macizo, de radio R0, que posee una carga +Q. a) Determinar el campo
elctrico y el potencial en un punto cualquiera P en funcin de la distancia r al centro de las esferas, desde
r = 0 hasta r = . b) Representar grficamente E(r) y V(r) en funcin de r. Nota: No es necesario hacer los
clculos detallados para cada etapa.

Campo elctrico
Como es bien sabido (demostrable mediante el teorema de Gauss), a efecto del clculo del
campo elctrico es como si toda la carga estuviese en el centro del conductor cargado.
Obviamente, el campo es nulo en el interior de cada
conductor.
E
r  R0 E  0

R0

R1

R2

V
4

R0

R1

r  R0

E

1 Q
4QF0 R02

R0  r  R1

E

1 Q
4QF0 r 2

r  R1

E0

R1  r  R2

E0
E

1 Q
4QF0 R22

R2  r  d E 

1 Q
4QF0 r 2

r  R2

R2

R0
O

R1
R2

Potencial elctrico
Para determinar el potencial, con un referencial nulo para una distancia infinita, procederemos
en cada una de las regiones indicadas en la figura.
d

c V (r )  Vrd  E<dr 
r

d V

1 Q
4QF0 R2

dr
Q 1
1 Q
 

2
r
4QF0 r r
4QF0 r

R2 b r  d

R1 b r b R2
R1

e V (r )  VR1 E<dr 
r

1 1
Q 1


4QF0 R2 R1 r
f V (r ) 

Q
4QF0

1 Q
Q

4QF0 R2 4QF0

R1

dr
1 Q
Q


r 2 4QF0 R2 4QF0

R0 b r b R1

1
1
Q 1

4QF0 R2 R1 R0

r b R0

- 350 -

1 1
 
r r
R

Fsica Universitaria: Problemas de Fsica

Campo elctrico. E01.3

3. Una esfera conductora cargada se encuentra a un potencial de 300 V; a 10 cm de su superficie el potencial


elctrico es de 200 V. a) Determinar el radio la esfera conductora b) Cul es su carga? c) Calcular el valor
del campo elctrico en la proximidad de la superficie de la esfera.

Determinamos el potencial en la superficie de la esfera (V0) y a una distancia h de la misma


(V):
300 V

1 Q
V0 
200 V

4F0 R
V
R h
u

l 0

R+h

1
Q
V
R
V 
4F0 R h

a) Determinamos el radio de la esfera:


300 R 10

3R  2 R 20 R  20 cm
200
R
b) Calculamos su carga:
0.20q300
Q  4F0 RV0 
 6.67 q109 C = 6.67 nC
9q109
c) Determinamos el campo elctrico en la superficie de la esfera:
1 Q V0
300
V
E0 
 
 1500
2
4F0 R
R 0.20
m

- 351 -

Fsica Universitaria: Problemas de Fsica

Campo elctrico. E01.4

4. Sobre dos esferas conductoras, de radios 0.10 m y 0.15 m, se depositan cargas elctricas de +100 nC y
+200 nC, respectivamente. Ponemos las esferas en contacto y luego las separamos de nuevo. Calcular la
carga final y el potencial de cada esfera.

Durante este proceso


x La carga neta total permanece constante
x Se transfiere carga hasta que finalmente se igualan los potenciales
Escribimos las ecuaciones correspondientes:
V2
V1
Qn  Q1 Q2  Q1a Q2a
Q1a Q2a  Qn

1 Q1a
1 Q2a l Q1a Q2a
(2 ec. 2 incogn.)
V a  4QF R  4QF R

Q1
0
1
0
2
Q2

R1 R2
1
Resolvemos las dos ecuaciones anteriores
Q1a Q2a
Qn
Antes de ponerlas en contacto


R1 R2 R1 R2
y sustituimos los valores dados en el enunciado, de modo
que

R1
10
Q1a 
Qn  300  120 C

R1 R2
25

R2
15
Qn  300  180 C
Q2a 
R

R
25
1
2

El potencial final, comn a las dos esferas conductoras, es


Va

1 Q1a
120q109
 9q109
 10.8 kV
4QF0 R1
101

Hemos tenido en cuenta la conocida propiedad de la suma de fracciones:

- 352 -

Q 1

Q 2

Despus de haber estado en contacto

a c aoc
 
b d bod

Fsica Universitaria: Problemas de Fsica

Campo elctrico. E01.5

5. Sobre dos esferas conductoras, de radios 10 cm y 15 cm, se depositan cargas elctricas de -100 nC y
+350 nC, respectivamente. a) Calcular el potencial de cada esfera cuando estn muy alejadas entre s.
b) Ponemos las esferas en contacto y luego las separamos de nuevo. Calcular la carga final y el potencial de
cada esfera.

V1

V2

a) Calculamos los potenciales:

V1 

1 Q1
100q109
 9q109
 9 kV
4QF0 R1
101

V2 

1 Q2
350q109
 9q109
 21 kV
4QF0 R2
15q102

Q1

Q2

Antes de ponerlas en contacto

b) Cuando se ponen en contacto:


x La carga neta total permanece constante
x Se transfiere carga hasta que finalmente se igualan los potenciales
Escribimos las ecuaciones correspondientes:
Q1a Q2a  Q1 Q2
Q1 Q2  Q1a Q2a
V
V

1 Q1a
1 Q2a l Q1a R1
V a 


(2 ec. 2 incogn.)
Q2a R2
4QF0 R1 4QF0 R2

Q 1
Q 2
Resolvemos 1 las dos ecuaciones anteriores
Q1a Q2a
Qn


R1 R2 R1 R2
Despus de haber estado en contacto

y sustituimos los valores dados en el enunciado, de modo que

R1
10
Q1a 
Qn  250  100 nC

R1 R2
25

R2
15
Qn  250  150 C
Q2a 
R

R
25
1
2

El potencial final, comn a las dos esferas conductoras, es


Va

1 Q1a
100q109
 9q109
 9 kV
4QF0 R1
101

Hemos tenido en cuenta la conocida propiedad de la suma de fracciones:

- 353 -

a c aoc
 
b d bod

Fsica Universitaria: Problemas de Fsica

Campo elctrico. E01.6

6. Una carga elctrica puntual, +q, est situada a una distancia D del centro de una esfera conductora de radio
R. a) Determinar el potencial elctrico al que se encuentra la esfera. b) Unimos la esfera a tierra mediante un
hilo conductor largo y delgado (de influencia despreciable). Calcular la magnitud de la carga elctrica
inducida sobre la esfera. (Explicar y hacer los esquemas grficos oportunos para cada apartado, indicando la
posicin de las cargas inducidas sobre la esfera.)

a) Todos los puntos de la esfera conductora en equilibrio


estn al mismo potencial, por lo que basta con determinar el
potencial al que se encuentra su centro (O). Este potencial
ser igual al creado en O por la carga puntual +q; i.e.,
1 q
V
[1]
+q
4QF0 D

ms el creado por las cargas inducidas (de uno y otro signo)


sobre la superficie de la esfera conductora, como se ilustra en la figura. Como la carga neta
inducida es nula y todos los elementos de dicha carga se encuentran a la misma distancia del
centro de la esfera conductora, su contribucin al potencial en O ser nula. As el potencial de
la esfera ser el expresado por [1].
b) La esfera conductora, despus de conectarla a tierra, quedar con una carga inducida
negativa. Esto es as porque la toma a tierra suministra carga negativa que neutraliza la
carga positiva inducida en la esfera, como se muestra en la figura. Cuando suprimamos la
toma a tierra, la carga negativa (-Q) quedar atrapada
D
sobre la superficie de la esfera (carga por induccin). El
R
potencial de la esfera ser ahora
1 q
1 Q
1 q Q


V
  0
4QF0 D 4QF0 R
4QF0 D R
puesto que est conectada a tierra (referencial nulo de +q
potenciales). En consecuencia, la carga negativa inducida
ser
q Q
R

l Q q
D R
D
disminuyendo, como era de esperar, cuando aumenta la
distancia D.

- 354 -

-Q
Flujo de
cargas negativas

+Q

Fsica Universitaria: Problemas de Fsica

Campo elctrico. E01.7

7. Disponemos de una esfera conductora,

Q2

R3

Q2

R3

R
R
maciza, de radio R1, que posee una carga
Q
elctrica neta +Q1, y de otra esfera
R
R
conductora, hueca, de radios interior R2 y
exterior R3, que posee una carga elctrica
Figura 1
Figura 2
neta +Q2. Inicialmente, las esferas estn
separadas y muy distantes la una de la otra.
a) Dibujar esquemticamente la distribucin de carga en cada una de las esferas. b) Calcular el potencial al
que se encuentra cada esfera y la d.d.p. entre ellas. c) Ahora, colocamos la esfera maciza en el interior de la
hueca, en posicin concntrica, como se ilustra en la figura 2. Dibujar esquemticamente la distribucin de
carga en cada una de las esferas. d) Determinar el campo elctrico en la regin comprendida entre ambos
conductores concntricos y calcular la d.d.p. entre ellos.
2

a) Tanto en la esfera maciza como en la hueca, la carga elctrica se distribuye uniformemente


sobre la superficie externa de las esferas, no existiendo carga elctrica ni en el interior de los
conductores ni en la superficie interna del conductor hueco.
b) El potencial al que se encuentra la esfera maciza es el
+
Q
+
+
R
de su superficie (el mismo que en su interior) y viene
+ R
+
+Q
+
dado por
+
+
+
+
R
+
+
1 Q1
+
V1 
+
+
4QF0 R1
+
+
2

Del mismo modo el potencial al que se encuentra la esfera


hueca (el mismo que en su interior, incluido el hueco) viene dado por
1 Q2
V2 
R3
4QF0 R3
y la d.d.p. entre ellas es

1 Q1 Q2
V12  V1 V2 

4QF0 R1 R3

+ -

+ R2
-

+
+ Q2*
-Q1
+
+Q1
+
+
+
+ E

R1 +

+
+

c) Aparece una carga elctrica inducida, Q1, sobre la superficie


+ - +
interior del conductor hueco, de modo que el campo elctrico en el
+ +
interior del conductor hueco (al igual que en el interior del
macizo) sea nulo (Teorema de Gauss). La carga elctrica sobre la superficie exterior del
conductor hueco ser ahora Q2* = Q1 + Q2, ya que su carga neta de este conductor debe
permanecer constante (Q2 = -Q1 + Q2*).
d) Como consecuencia del Teorema de Gauss, la intensidad del campo elctrico en un punto
situado a una distancia r del centro comn de las esferas, con R1 < r < R2, tan solo est
determinado por la carga Q1, tiene direccin radial y vale
1 Q1
E
4QF0 r 2

La d.d.p. entre los dos conductores se calcula como la circulacin del campo elctrico entre
los puntos 1 y 2, indicados en la figura, a lo largo de una lnea de campo:
2

V12  E<dr  E dr 
1

Q1
4QF0

R2
R1

Q1 1 2
Q 1
dr
1
  1   0

2

r
4QF0 r R1 4QF0 R1 R2
R

de modo que la esfera maciza siempre est a mayor potencial que la hueca, con independencia
de las cargas de una y otra.
- 355 -

Fsica Universitaria: Problemas de Fsica

Campo elctrico. E01.8

8. Considrese un conductor aislado, como el que se indica en la figura, que posee


una carga elctrica +Q en equilibrio. a) Cunto vale el campo elctrico en el
interior del conductor? Razonar la respuesta. b) Dnde se sita la carga
elctrica? Hacer un esquema de la distribucin de la carga elctrica en el
conductor. En que zonas de la superficie del conductor es mayor la densidad de
carga? Razonar las respuestas. c) Aplicar el Teorema de Gauss para calcular la
intensidad del campo elctrico en las proximidades de la superficie exterior del
conductor En que zonas de la superficie del conductor es ms intenso el campo
elctrico?

conductor
Pie
aislante

a) El campo elctrico en el interior de un conductor en


equilibrio es nulo. Esto es as porque, si el campo no
fuese nulo, las cargas elctricas se moveran bajo la
+
+
+ ++
accin del mismo y el conductor no estara en equilibrio.
+
+
+ ++
+
b) La carga elctrica se distribuye en la superficie del
+
conductor. Este resultado es una consecuencia inmediata
del teorema de Gauss, al aplicarlo a una superficie
Poder de
gaussiana en el interior del conductor.
las puntas
La densidad de carga es mayor en las zonas de mayor
curvatura, tal como se ilustra en la figura. Es decir, tiende
a acumularse en las zonas ms puntiagudas. A este efecto se le conoce como poder de las
puntas.
c) Consideramos una superficie gaussiana de forma cilndrica, con una de sus bases en el
interior de conductor y la otra fuera del mismo. Por ser el campo nulo en el interior del
conducto, no hay flujo a travs de la base interior de la superficie gaussiana. En el exterior del
conductor, cerca de su superficie, el campo es normal a la
 E
superficie por ser sta una superficie equipotencial, por lo que
dS
tampoco existe flujo a travs de la superficie lateral del cilindro
+ +
gaussiano. Siendo la densidad superficial de carga, el clculo
del flujo a travs de la base exterior del cilindro y el teorema de
E=0 +
Gauss nos permiten escribir:
+
T dS
T
d'  E dS 
l E
F0
F0
Como consecuencia, el campo elctrico ser ms intenso donde sea mayor la densidad de
carga, esto es, en las zonas de mayor curvatura.

- 356 -

Fsica Universitaria: Problemas de Fsica

Campo elctrico. E01.9

9. Una esfera dielctrica homogneamente cargada (densidad de carga, U), de radio R y

z
R

centrada en el sistema de coordenadas posee una oquedad tambin esfrica de radio


R/2, dispuesta tal como indica la seccin transversal representada. Determinar: a) El
campo elctrico y el potencial en los puntos P del eje Oy. b) La velocidad mnima
que se debe imprimir a una carga q y masa m, situada en A, para que dicha carga
salga del campo de accin del campo creado por la esfera.

y
A

a) Podemos considerar una esfera de radio R, sin oquedad, con carga positiva Q1, y otra esfera
de radio R/2, correspondiente a la oquedad, con carga negativa Q2, siendo
3
4
4
Q1  S Q R 3
Q2  S Q R
2
3
3
Calculamos el campo y el potencial elctricos creados por las distribuciones esfricas y
homogneas de cargas Q1 y Q2 del mismo que si fuesen cargas puntuales ubicadas en el
centro de las correspondientes esferas.
3

1 Q1
1 S 3 QR
S R3 1
E



1P
2
2
4QF0 y
4QF0
3F0 y 2
y

S 4 Q R3 S R3 1
V  1 Q1  1
3

1P 4QF y
4QF0
3F0 y
y
0

4
R

1
1 S 3 Q 2
1
S R3
Q2

E




2P

2
2

4QF0 y  R
4QF0
3F0 8 y  R 2

yR

2
2
2

4
R

1
1 S 3 Q 2
1
S R3
Q2

V





2P

4QF0 y  R
4QF0
3F0 8 y  R

yR

2
2
2

Aplicamos el Principio de Superposicin:

S R3 1
1
1
S R 3 1






EP E1P E2P
VP  V1P V2P


3F0 y 8 y  R
3F0 y 2 8 y  R 2

b) Aplicando el Principio de conservacin de la energa (potencial electrosttica +cintica), y


teniendo en cuenta que vd  0 y Vd  0 , se sigue

2qVA
1
1 2
qVA mv 2  qVd mvd
 0 l v2 
m
2
2

SR2
SR2 q
S R3 1
1
con VA 
de donde
v


2F0 m
3F0 R 8 R  R 4F0

- 357 -

Fsica Universitaria: Problemas de Fsica

Campo elctrico. E01.10


z

10. Un plano infinito situado en el plano de coordenadas xy posee una densidad de

+
+
+ +
+
x

+
E
dS
+

E2

+
V1
+

+
x


P(x,y,z)

+
+

+
V2

y
+

+
V2

a) Consideramos una superficie gaussiana de forma cilndrica, de


generatriz infinitesimal y perpendicular al plano cargado y con cada
una de sus bases a cada lado del plano, como se ilustra en la figura.
Por ser el campo perpendicular al plano (razones de simetra), no hay
flujo a travs de la superficie lateral del cilindro gaussiano. Siendo
la densidad superficial de carga, el clculo del flujo saliente (+) a
travs de cada una de las bases del cilindro y el teorema de Gauss nos
permiten escribir:
T dS
T
l E
d'  E dS E dS  2 E dS 
F0
2F0

P(x,y,z)

V1

carga superficial uniforme V1 = 354 nC/m2. Un segundo plano tambin infinito est
situado en el plano xz y su densidad de carga es V2 = 177 nC/m2. Determinar el
vector campo elctrico en un punto genrico P de coordenadas (x, y, z), utilizando
el teorema de Gauss.

E1
y

que es la expresin del campo elctrico creado por un plano


infinito cargado con una densidad de carga ; vemos que el
campo es independiente de la distancia al plano.
b) El campo creado en un punto cualquiera del espacio viene
dado por la superposicin de los campos creados en dicho
punto por cada uno de los planos:
T
T
E1  1 j
E2  2 k
2F0
2F0
T1
T2
1
E  E1 E2 
j
k
T1 j T2k

2F0
2F0
2F0

con V1 = 354u10-9 C/m2, V2 = 177u10-9 C/m2 y H0 = 8.85u10-12 C2/N.m2, de modo que


1
kV
354 j 177k
q109  20 j 10k

E
12
2q8.85q10
m
o bien
kV
10
E  22.36
R  arctg  26.6
m
20
en todos los puntos del espacio.

- 358 -

Fsica Universitaria: Problemas de Fsica

Campo elctrico. E01.11

11. Dos hilos conductores rectilneos, indefinidos y paralelos, separados por una distancia de 1 m en un medio
de constante dielctrica 3 ( r=3) se ejercen una fuerza atractiva de 1.08u10-7 N por cada metro de longitud.
Sabiendo que uno de ellos tiene una distribucin uniforme de carga  = 3u10-9 C/m, calcular la carga
existente en un tramo de 5 m del otro hilo.

Mediante el teorema de Gauss determinamos el campo elctrico creado por uno cualquiera de
los hilos, digamos el primero, a una distancia h del mismo:
q
(Teorema de Gauss)
E
v E<d S  F0
h
q
1 M1
E 2Qhl1  1 l E 
F
2QF h
q
con M1  1 y F  Fr F0  3F0
l1
La fuerza que acta sobre un elemento de longitud l2 y carga q2 =2l2 en el segundo hilo es:
1 M1
F
1 M1M2
M2l2 l

F  q2 E 
l2 2QF h
2QF h
i.e., la fuerza por unidad de longitud entre ambos hilos.
E

l2

F/l 2

Despejando 2 de la expresin anterior, ...


M2 

2QFh F 2Q q3q8.85q1012 q1

q 1.08q107
  6q109 C/m =  6 nC/m
M1 l2
3q109

y, en 5 m de hilo, tendremos
q2

6 u 5

- 359 -

30 nC

Fsica Universitaria: Problemas de Fsica

Campo elctrico. E01.12

12. Tres distribuciones lineales de carga, indefinidamente largas, y paralelas al eje


Z, poseen unas densidades lineales de carga uniformes de valores O, O y -O.
Sabiendo que las rectas pasan por los puntos A, B y C respectivamente.
Determinar: a) Campo elctrico en el origen de coordenadas. b) Trabajo
necesario para llevar un electrn desde el origen de coordenadas hasta el
punto de coordenadas (0, 0,1).

O
A(1,1)

-O
x

C(-1,0)

B(1,-1)

a) Mediante el Teorema de Gauss, determinamos el campo elctrico creado por uno


cualquiera de los hilos a una distancia r del mismo:
q
E dS  E
E<dS 
dS 
v
v
v

dS E
F0
q
1 2M
l E
E 2Qrl 
r
F0
4QF0 r

l
EA

La direccin del campo elctrico creado en O por cada una


de las distribuciones de carga se indica en la figura y sus
mdulos son:
1 2O
1 2O
EB
EC
4SH 0 2
4SH 0 1

El campo elctrico resultante (E) en el origen de coordenadas viene dado por la suma
vectorial
E  E A E B EC
tiene la direccin del eje x negativo y su mdulo es

2M 1 2
1 2
2M
M
E  EA cos 45 EB cos 45 EC 

1 
q2 

4QF0
4QF0 2 2
QF0
2 2
? E

O
i
SH 0

b) Puesto que el campo elctrico E es


perpendicular al eje z en todos los puntos de ste
(adems, E es constante), la circulacin de E entre
dos puntos cualquiera del eje z ser nula, lo que
significa que el eje z es equipotencial; i.e.,
V V0  

(0,0,1)

(0,0,0)

-O

A(1,1)

EC
x

C(-1,0)

E<dr  0 l V  cte.

En consecuencia, el trabajo realizado por el


campo sobre un electrn (o cualquier otra carga)
al desplazarse a lo largo de dicho eje ser nulo; i.e.,
W  q(V0 V )  0

- 360 -

EB

EA

B(1,-1)

Fsica Universitaria: Problemas de Fsica

Campo elctrico. E01.13

13. Sea una varilla recta y delgada de longitud l, uniformemente

cargada con una carga elctrica +q. a) Encontrar la expresin


del potencial elctrico en el punto P en funcin de su distancia
x al extremo de la varilla. b) A partir de la expresin anterior,
obtener la expresin del campo elctrico en P en funcin de
dicha distancia x.

Descomponemos la varilla en elementos de longitud d[, que soportan una carga elemental dq,
situados a una distancia [ del extremo izquierdo
[ d[
de la varilla, tal como se indica en la figura.
P
Siendo O la densidad lineal de carga, podemos
escribir
x
l
q
dq O d[ con O
l
a) Clculo del potencial.
El potencial en P se obtiene como suma (escalar) de las contribuciones de todos loe elementos
a lo largo de la varilla. Procedemos por integracin:
V

1
4QF0

dq
M

l  Y x 4QF0

dY

l x  Y   4QF

<ln(l x  Y )>0 

M
l x
ln
4QF0
x

q
l x
V
ln
x
4QF0l

b) Clculo del campo.


Expresamos el campo elctrico como el gradiente del potencial elctrico con signo negativo;
esto es,
dV
ya que V  V ( x)
E  V l E  
dx
Procedemos por derivacin:
dV
q d l x
q 1
1
q l

ln
E 


 

dx
4QF0l dx
4QF0l l x x
4QF0l x(l x)
x
?

1
q
4SH 0 x(l  x)

- 361 -

Fsica Universitaria: Problemas de Fsica

Campo elctrico. E01.14

14. Consideremos tres varillas no conductoras, cada una de ellas de longitud l y cargadas
uniformemente con cargas elctricas +Q, +Q y Q, respectivamente, como se indica en la
figura. Determinar el campo elctrico en el punto P equidistante de las tres varillas.

P
+Q

-Q
+Q

E2

Por razones de simetra, los campos elctricos creados en P


por cada una de las tres varillas cargadas tienen todos la
misma magnitud (E1 = E2 = E3 = E) y las direcciones
indicadas en la figura. As, el campo elctrico resultante
tiene una intensidad:

ET


+Q

E1

E3

ET

-Q

E 22  12

E 5

1
y forma un ngulo R  arctg  26.57 con la horizontal.
2
Para calcular el valor de E, descomponemos la varilla en
elementos de longitud dx, que poseen una carga dq = dx, siendo  =Q/l la densidad lineal de
carga. El campo elemental creado por tal elemento en P ser
1 dq
1 M dx
1 M dx
1 M

2
dE  4QF s 2  4QF s 2  4QF h 2 cos R  4QF h dR
0
0
0
0

dR
con h  s cos R l s  h / cos R ; x  h tg R l dx  h
2
cos
R

+Q

En tanto que calculemos el campo en puntos de la mediatriz de la varilla, las componentes


paralelas a sta se compensarn por simetra, y el campo ser perpendicular a la varilla, de
modo que, al integrar todas las contribuciones elementales, tendremos:
Q
M
M
M 2
2 M 2
Q/4
4
E  dE cos R 
sen R >Q / 4 

<

Q cosR dR 

4QF0 h  4
4QF0 h
4QF0 h 2
2 4QF0 h
con h = l/2 y  =Q/l, de modo que

dE
dE cos

2Q
2SH 0 l 2

ET

E 5

P
x

h=l/2

dx

- 362 -

10Q
2SH 0 l 2

Fsica Universitaria: Problemas de Fsica

Campo elctrico. E01.15

15. Un anillo de alambre fino, de radio R, posee cierta carga elctrica +Q. Cmo se mover una carga puntual,
de masa m y carga elctrica q, que se encuentra inicialmente en reposo en un punto situado sobre el eje del
anillo, a una distancia x << R de su centro?

Por razones de simetra, la fuerza que ejerce la carga +Q


del anillo sobre la carga puntual q es
1 Qq
F 
cos R
4QF0 r 2

-q

donde el signo negativo indica que la fuerza es atractiva,


tal como se muestra en la figura, con cos R  x / r , de
modo que
1 Qq
Qq
x
F 
x 
4QF0 r 3
4QF0 R 2 x 2
3/2

x
R
+Q

Si consideramos tan slo distancias x << R, de modo que


la carga puntual se encuentra siempre cerca del centro del anillo, la expresin anterior se
reduce a
Qq
Qq x
Qq
x  kx con k 
 
F 
3
3

4QF R
4QF R 3
4QF R
0

As pues, se trata de una fuerza regida por la ley de Hooke (fuerza dirigida hacia la posicin
de equilibrio cuyo mdulo es proporcional a la elongacin). Bajo la accin de dicha fuerza, la
ecuacin del movimiento de la carga puntual es
F  mx l  kx  mx l mx kx  0
que es la ecuacin diferencial de un movimiento armnico simple de frecuencia
X

k
Qq

m
4QF0 mR 3

y periodo
T

4QF0 mR 3
2Q
 2Q
X
Qq

Mtodo de la energa potencial del sistema:


1/2

1 Q q

Qq
Qq
x2
E
1 2



1/2
4QF0
r
4QF0 R R
4QF0 R 2 x 2

de modo que F  

x

Qq 1 x 2

1
4QF0 R 2 R 2

dE
Qq x
Qq
x , y sigue como antes
x
2  

dx
4QF0 R R
4QF0 R 3

- 363 -

Fsica Universitaria: Problemas de Fsica

Campo elctrico. E01.16

16. Consideremos un disco de radio R y espesor despreciable con densidad superficial de carga elctrica
uniforme en una de sus caras. a) Determinar el potencial en un punto del eje de simetra de revolucin del
disco que est situado a una distancia z del mismo. b) A partir del resultado del apartado anterior, sin
recurrir a mtodos de integracin, determinar el campo elctrico en los puntos de dicho eje. c) Verificar los
resultados para z  0 y para z  .

a) Descomponemos el disco en coronas circulares concntricas, de radio r y ancho dr, como


se ilustra en la figura. Cada una de esas coronas posee una carga elctrica infinitesimal,
dq  TdS  T 2Qr dr equidistante del punto P donde
calculamos el potencial:
E
1 dq
1 2QT r dr
T r dr
T
P



dV 
ds
4QF0 s
4QF0
s
2F0 s
2F0
s
z
donde hemos tenido en cuenta que

dq= d

s 2  r 2 z 2 l s ds  r dr .
Integrando V 

T
2F0

R2 z 2

ds 

T
2F0

R2 z 2  z

b) Obtenemos el campo a partir del potencial mediante la operacin gradiente, i.e.,


E  V . Por la simetra que presenta el problema, el campo en los puntos del eje z est
dirigido a lo largo de dicho eje, por lo que podemos escribir:
sV
dV

Ez  
dz
sz
de modo que, efectuando la derivada, es
T d
T
z

1
E 
R2 z 2  z 

2
2
2F0 dz
2F0
R z

c)

TR

para z l 0 V  2F

para
z
V
0
l
d


E

T
2F0

E0

- 364 -

(plano cargado infinito)

Fsica Universitaria: Problemas de Fsica

Campo elctrico. E01.17

17. Una barra dielctrica de longitud L est colocada perpendicularmente a


una distribucin lineal de carga uniforme e infinitamente larga, de
densidad lineal de carga O positiva. El extremo ms prximo de la barra a
la carga lineal dista de sta D. La barra posee una carga total Q, tambin
positiva y distribuida uniformemente en toda su longitud. Determinar la
fuerza que se ejerce sobre la barra.

+Q
L

Comenzamos encontrando la expresin de la intensidad del campo


elctrico a una distancia y de una distribucin lineal de carga uniforme
e infinitamente larga. Aplicamos el teorema de Gauss a una superficie
gaussiana de forma cilndrica cuyo eje sea la propia distribucin lineal
de carga: esto es,
Ml
M
vS E<dS  v E dS  E v dS  E 2Q yl  F0 l E  2QF0 y
lateral
lateral

Puesto que el campo elctrico no es uniforme a lo largo de toda la barra dielctrica, para
calcular la fuerza ejercida sobre la barra debemos proceder por integracin,
descomponindola en elementos de longitud dy de carga dq  M ady , con M a  Q / L :
dF  E dq 

MM a dy
2QF0 y

l F

MM a
2QF0

D L

dy
MM a
D L

ln
y
2QF0
D

Q
dy

o sea
F

MM a
D L
MQ
D L
ln

ln
2QF0
D
2QF0 L
D

- 365 -

Fsica Universitaria: Problemas de Fsica

Campo elctrico. E01.18

18. Un plano infinito posee una densidad de carga superficial de +4.5 nC/m2 y coincide con el plano yz en el

origen; un segundo plano infinito posee una densidad de carga superficial de -4.5 nC/m2 y se localiza en un
plano paralelo al plano yz en x = 2 m. Determinar la fuerza elctrica ejercida sobre una carga puntual de
10 nC situada en: (a) x = 1,8 m y (b) x = 5 m.

E=0
A

E=508 V/m

a) El campo elctrico en la zona comprendida entre los dos planos paralelos es uniforme y su
intensidad es
Eint 

T
4.5q109

 508 V/m
F0 8.854q1012

y la fuerza ejercida sobre la partcula cargada es


F  qE  10q109 q 508  5.08q106 N

b) El campo elctrico fuera de la zona comprendida entre los dos planos paralelos es nulo, por
poseer estos densidades de cargas iguales y opuestas.
Obviamente, la fuerza sobre la partcula cargada tambin ser nula.

- 366 -

Fsica Universitaria: Problemas de Fsica

Capacidad elctrica. E02.1

1. Una esfera maciza conductora de radio R, situada en el vaco, est cargada con una carga +Q. a) Determinar
el campo elctrico y el potencial que existen en un punto P, exterior a la esfera, situado a una distancia r del
centro de la misma. b) dem en un punto P interior a la esfera. c) Calcular la capacidad del conductor. d) En
qu cambiaran los resultados anteriores si la esfera conductora fuese hueca?

a) Aplicaremos el teorema de Gauss a una superficie gaussiana de radio r concntrica con la


esfera slida:

v E<dS  F
S

+Q

2
E dS  E
v dS  ES  E 4Qr 
S

E

Q
F0

1 Q
4QF0 r 2

siendo Q la carga elctrica contenida en la superficie gaussiana,


que es la que posee el conductor.
La diferencia de potencial se calcula como la circulacin del
campo:
r
d
d dr
Q
V (r )  V (d)  E<dr  E<dr 

d
r
r
4QF0
r2
= V (r ) 

Q
4QF0

1
1 Q

r d 4QF0 r
r

b) Aplicaremos el teorema de Gauss a una superficie gaussiana de radio r < R concntrica con
la esfera slida:
Q
vS E<dS  F0  0 l = E  0
r
+Q
E=0

por se nula la carga elctrica contenida en la superficie gaussiana, ya


que la carga del conductor se sita en su superficie.
La diferencia de potencial se calcula como la circulacin del campo:
r

V ( r )  V ( R )  E < dr 
R

1 Q
4QF0 R

por lo que el potencial del conductor es el mismo en todos sus puntos, incluida su superficie.
c) Por definicin, la capacidad de un conductor viene dada como el cociente de la carga que
posee y el potencial al que se encuentra (con V = 0); esto es,
Q
Q
C 
 4QF0 R
Q
V
4QF0 R
de modo que, en el caso de una esfera conductora, es directamente proporcional a su radio.
d) Los resultados anteriores no se modifican, ya que la carga Q seguir situndose en la
superficie externa del conductor, presentndose la misma distribucin de carga tanto si el
conductor es hueco como macizo.

- 367 -

Fsica Universitaria: Problemas de Fsica

Capacidad elctrica. E02.2

2. Calcular la energa electrosttica asociada a un conductor esfrico, de radio R, que posee una carga elctrica
+Q.

El potencial al que se encuentra la esfera conductora es


1 Q
V
4QF0 R
La esfera cargada se comporta como un condensador cuya otra armadura se encuentra en el
infinito. La energa almacenada por un condensador cargado viene dada por
1
1
1 Q
1 Q2

U  QV  Q
2
2 4QF0 R 8QF0 R

Otro mtodo
En el interior del conductor, el campo elctrico es nulo, en tanto que en su exterior el campo
elctrico viene dado por la expresin
1 Q
E
4QF0 r 2
r
dr
siendo r > R, la distancia al centro de la esfera.
La densidad de energa en el campo elctrico viene dada por la
expresin
1
1
1 Q2
Q2
u  F0 E 2  F0

2 2
4
2
2 16Q F0 r
32Q 2 F02 r 4
que decrece con la cuarta potencia de la distancia, por lo que para determinar la energa
asociada a todo el espacio exterior al conductor deberemos proceder por integracin.
La energa asociada a una capa esfrica de radio r y espesor dr, tal como se ilustra en la
figura, cuyo volumen es d.  4Qr 2 dr , ser

dU 

Q2
Q2
Q 2 dr
d. 
4Qr 2 dr 
2 2 4
2 2 4
32Q F0 r
32Q F0 r
8QF0 r 2

y la energa asociada a todo el espacio exterior al conductor ser


U

dr
Q2 1
1 Q2




R r 2 8QF0 r R 8QF0 R
q2 6 u 5 30 nC

Q2
8QF0

- 368 -

Fsica Universitaria: Problemas de Fsica

Capacidad elctrica. E02.3

3. Una esfera conductora, de radio R, que posee una carga elctrica +Q, est envuelta por una capa dielctrica
de espesor 3R y permitividad relativa igual a 3. El conjunto se encuentra aislado en el aire, a una distancia
muy grande de cualquier otro conductor. a) Dibujar esquemticamente la distribucin de carga elctrica en
el conductor y la polarizacin en el dielctrico. b) Expresar el campo elctrico en el dielctrico y en el aire
en funcin de la distancia r al centro de la esfera conductora. c) Calcular el potencial al que se encuentra la
esfera conductora. d) Calcular la capacidad elctrica de la esfera conductora.

a) La carga elctrica +Q se sita sobre la superficie de


la esfera conductora, tal como se ilustra en la figura.
Ea
La capa de material dielctrico se polariza, orientndose los dipolos en la direccin radial, dando lugar a
la aparicin de carga de polarizacin, de signos
opuestos, en las superficies interior y exterior de la
4R
Ed
capa dielctrica.
b) Campos elctricos:
1 Q
R  r  4R
x en el dielctrico: Ed 
+
conductor Q +
4QF r 2
R +
+
1 Q
V +
r  4R
x el aire: Ea 
2
4QF0 r

aire

dielctri

c) Calculamos el potencial de la esfera conductora como la circulacin del campo elctrico


desde su superficie hasta el infinito (potencial de referencia nulo), de modo que
R
d
4 R dr
d dr
Q
Q

V  Vd   E<dr  E<dr 
2

R
d
4QF R r
4QF0 4 R r 2
integramos
Q 1
Q 1
Q 1
1
Q 1





4QF r 4 R 4QF0 r d
4QF R 4 R 4QF0 4 R
Q
Q
3
Q 1
1 3



1
4 R
4QFr F0 4 R 4QF0 4 R 4QF0 Fr
R

4R

V

y sustituimos Fr  3
V

1 3 Q
Q
1

 V0
1
4QF0 3 4 R 8QF0 R 2

siendo V0 el potencial que tendra la esfera conductora en el aire (vaco) sin la envuelta
dielctrica.
d) La capacidad de la esfera conductora ser
Q
Q
C 
 8QF0 R  2C0
V Q / 8QF0 R
siendo C0 la capacidad que tendra la esfera conductora en el aire (vaco) sin la envuelta
dielctrica.

- 369 -

Fsica Universitaria: Problemas de Fsica

Capacidad elctrica. E02.4

4. Una barra dielctrica de longitud L est colocada perpendicularmente a


una distribucin lineal de carga uniforme e infinitamente larga, de
densidad lineal de carga O positiva. El extremo ms prximo de la barra a
la carga lineal dista de sta D. La barra posee una carga total Q, tambin
positiva y distribuida uniformemente en toda su longitud. Determinar la
fuerza que se ejerce sobre la barra.

+Q
L

Comenzamos encontrando la expresin de la intensidad del campo


elctrico a una distancia y de una distribucin lineal de carga uniforme
e infinitamente larga. Aplicamos el teorema de Gauss a una superficie
gaussiana de forma cilndrica cuyo eje sea la propia distribucin lineal
de carga: esto es,
Ml
M
vS E<dS  v E dS  E v dS  E 2Q yl  F0 l E  2QF0 y
lateral
lateral

Puesto que el campo elctrico no es uniforme a lo largo de toda la barra dielctrica, para
calcular la fuerza ejercida sobre la barra debemos proceder por integracin,
descomponindola en elementos de longitud dy de carga dq  M ady , con M a  Q / L :
dF  E d q 

MM a dy
2QF0 y

l F

MM a
2QF0

D L

dy
MM a
D L

ln
y
2QF0
D

Q
dy

o sea
F

MM a
D L
MQ
D L
ln

ln
2QF0
D
2QF0 L
D

- 370 -

Fsica Universitaria: Problemas de Fsica

Capacidad elctrica. E02.5

5. Un condensador plano, de dielctrico el vaco, tiene una superficie de armaduras S y una separacin entre
ellas h (muy pequea). Si en sus armaduras se depositan unas cargas elctricas +Q y Q, hllense: a) La
fuerza de atraccin que se ejercen entre s las armaduras. b) El trabajo que hay que realizar para separar las
armaduras al doble (2h). c) Variacin de la energa elctrica del condensador al separarse las armaduras.

+Q

-Q

S
E

La intensidad del campo elctrico entre las armaduras del condensador viene
dada por
T
Q
E 
F0 F0 S
de modo que es independiente de la separacin h entre las armaduras, en
tanto que sta sea suficientemente pequea.
La densidad de energa (u) almacenada en el campo elctrico est dada por la
expresin
u

dU 1
1 Q2
 F0 E 2 
d9
2
2 F0 S 2

1
1 Q2
1 Q2
dU  F0 E 2 d9 
S dh 
dh
2
2
2 F0 S
2 F0 S

c) Variacin de la energa elctrica del condensador al separar las armaduras al doble ('h =
h):
dU 

1 Q2
dh
2 F0 S

U 

1 Q2
h
2 F0 S

b) Trabajo que hay que realizar para separar las armaduras al doble ('h = h):
U  W 

1 Q2
1 Q2h
%h 
2 F0 S
2 F0 S

a) Fuerza de atraccin que se ejercen entre s las armaduras:


F 

dU
1 Q2

dh
2 F0 S

donde el signo negativo indica que la fuerza es atractiva.

- 371 -

Fsica Universitaria: Problemas de Fsica

Capacidad elctrica. E02.6

h/2

6. Calcular la capacidad de un condensador plano con tres dielctricos entre


sus armaduras, distribuidos como se indica en la figura, sabiendo que sus
permitividades relativas son iguales a 1, 2 y 3, respectivamente, que todos
tienen el mismo espesor h/2 y que los dielctricos 1 y 2 tienen el mismo
tamao.

La capacidad de un condensador plano de superficie S y separacin l entre sus armaduras


viene dada por la expresin:
S
S
C  F  Fr F0
l
l
Los dos primeros dielctricos se comportan como una asociacin de dos condensadores en
paralelo, por lo que presentan una capacidad equivalente
S /2
S /2
S
C 12
 C1 C2  F0
2F0
 3F0
h/2
h/2
h
La asociacin en serie de esta capacidad equivalente con la correspondiente al tercer
dielctrico representa una nueva capacidad equivalente dada por
1
1
1
1
1
h
h
h






S
S
3F0 S 6F0 S 2F0 S
Ceq C 12
C3 3F
3F0
0
h
h/2
de modo que la capacidad del condensador plano con tres dielctricos es
S
Ceq  2F0
h

- 372 -

Fsica Universitaria: Problemas de Fsica

Capacidad elctrica. E02.7

7. Disponemos de dos condensadores, de capacidades C1 y C2, que han sido cargados, por separado, a las
tensiones V1 y V2 respectivas, siendo V1 V2. Los conectamos en paralelo, uniendo los bornes de igual
polaridad. a) Demostrar que la energa electrosttica almacenada en este montaje es inferior a la energa
total cuando los condensadores estaban separados. b) Por qu ocurre esto?

a) Energa electrosttica total almacenada en los condensadores


separados:
1
U  C1V12 C2V22

2
Capacidad del condensador equivalente al montaje en paralelo:
Ceq = C1 + C2
La carga neta almacenada en los condensadores permanece constante
al unirlos en paralelo:
Q1 Q2  C1V1 C2V2
C V C2V2
l V 1 1

a
a
Q1 Q2  C1 C2
V
C1 C2
Energa electrosttica total almacenada en los condensadores
montados en paralelo:
1
1 C1V1 C2V2

U a  CeqV 2 
2
2
C1 C2

Q1=C1V1

+
+
Q2=C2V2
Q1=C1V

+
Q2=C2V

Variacin de la energa almacenada


%U  U a U 

+
Q=CeqV

1 C 2V 2 C22V22 2C1C2V1V2 C12V12 C1C2V22 C1C2V12 C22V22


 1 1


2
C1 C2
C1 C2

1 C1C2
1 CC
2V1V2 V12 V22
  2 C 1 C2 V1 V2
2 
2 C1 C2
1
2

1
2
  Ceq V1 V2
 0
2
de modo que la energa electrosttica disminuye 1.
b) La energa electrosttica disminuye debido a que, durante la conexin de los
condensadores, pasa carga de un condensador a otro, establecindose una corriente elctrica a
travs de los hilos de conexin, desprendindose calor en ellos. La cantidad de calor
desprendida no depende de la resistencia de los hilos de conexin; si stos presentan una
resistencia pequea, la intensidad de la corriente ser grande, e inversamente.

Obsrvese que si fuese V1 = V2, sera U = 0; y que si C1 = C2 y V2 = 0, sera U=U/2

- 373 -

Fsica Universitaria: Problemas de Fsica

Capacidad elctrica. E02.8

8. Dos condensadores de placas paralelas, cada uno con una capacidad de C1 = C2 = 2 PF, estn conectados en
paralelo a una batera de 12 V. a) Determinar la carga en cada condensador y la energa total almacenada
por los condensadores. b) A continuacin, los condensadores se desconectan de la batera y entre las placas
del condensador C2 se inserta un dielctrico de constante k = 2.5. En estas condiciones, determinar la
diferencia de potencial entre las placas de cada condensador, la carga depositada sobre cada uno de ellos y
la energa total almacenada por ambos.

a) Ambos soportan la misma tensin (12 V), y como tienen la misma capacidad, ser
Q1 = Q2 = CV = 2 u 12 = 24 PC
C1
V
Qtotal  Q1 Q2  48 C
12 V

2
2
1
1

U1  2 C1V  2 q 2q12  144 J U  U U  288 J

1
2
2
2
1
1

U 2  2 C2V  2 q 2q12  144 J


b) La carga total permanece invariable despus de desconectar la batera.
C1a  C1  2 F
Q1a  C1aV a

con Q1a Q2a  Qtotal


C2a  kC1  5 F
Q2a  C2aV a
C1

= (C1a C2a ) V a  Qtotal

Va

Qtotal
48 C

 6.86 V
C1a C2a
7 F

Ambos condensadores soportan la misma d.d.p.


Q a  C aV a  2q 6.86  13.7 C

1
1

con Q1a Q2a  48 C

Q a  C2aV a  5q 6.86  34.3 C

2
U1a  12 C1aV a 2  12 q 2q 6.862  47 J

U a  U1a U 2a  165 J

2
2
1
1

U 2a  2 C2aV a  2 q 2q 6.86  118 J

- 374 -

2 F

C2
2 F

2 F

C2
5 F

Fsica Universitaria: Problemas de Fsica

Capacidad elctrica. E02.9


C3

9. En la asociacin de condensadores de la figura se aplica una


diferencia de potencial de 50 V entre A y B. Hllese: a) La carga en
cada uno de los condensadores. b) La carga en el condensador
equivalente a la asociacin de todos ellos. c) Energa almacenada en
cada uno de los condensadores. d) Energa almacenada en el
condensador equivalente.

3 PF
B

A
C1

2P F

C2

2P F

Los condensadores estn asociados en serie:


1
1
1
1 1

  1 l Cs  1 F
Cs C1 C2 2 2
El condensador C3 est en paralelo con Cs:
C  C3 Cs  3 1  4 F
a) La d.d.p. entre A y B, VAB, se reparte por igual entre los condensadores C1 y C2, por lo que
las d.d.p. en estos y sus cargas son:
V1  V2  12 VAB  12 50  25 V l q1  q2  C1V1  2q 25  50 C

El condensador C3 soporta la tensin VAB:


V3  VAB  50 V l q3  C3V3  3q50  150 C
b) El condensador equivalente soporta la d.d.p. VAB:
qeq  CeqVAB  4q50  200 C
c) La energa almacenada en un condensador puede calcularse mediante la frmula
1
U  CV 2 , de modo que:
2
1
1
1
1
U1  U 2  C1V12  2q 252  625 J
U 3  C3V32  3q502  3750 J
2
2
2
2
d) La energa almacenada en el condensador equivalente sera:
1
1
U eq  CeqVeq2  4q502  5000 J
2
2

- 375 -

Fsica Universitaria: Problemas de Fsica

Capacidad elctrica. E02.10


2 F/100 V

10. En la asociacin serie-paralelo de condensadores que se muestra en la


figura, determinar la capacidad y la tensin de ruptura del condensador
equivalente entre los bornes A y B.

12 F/400 V
A

2 F/100 V
12 F/400 V
A

6 F/50 V

La asociacin en serie equivale a un condensador de

B capacidad C :
eq

12 F/400 V

Los dos condensadores en


paralelo equivalen a uno de capacidad Cp:
Cp  C1 C2  2 6  8 F

Ceq 

6 F/50 V

Cp C3
Cp C3

8q12 96

 4.8 F
8 12 20

La tensin de ruptura de los dos condensadores en paralelo


B ser igual a la menor de ellas; esto es, 50 V, como se indica
8 F/50 V
en la figura.
En la asociacin en serie, los dos condensadores se reparten
la tensin y tienen la misma carga:

Vp V3  VAB
Vp V3  VAB

4.8 F/83 V

CpVp  C3V3  0

CpVp  C3V3

Y resolviendo ese sistema de dos ecuaciones con dos


Condensador equivalente
incgnitas:

C3
12
3
Vp 
VAB  VAB  VAB  50 V
l VAB  83 V

Cp C3
20
5

Cp
8
2
VAB  VAB  VAB  400 V l VAB  1000 V
V3 
Cp C3
20
5

La tensin de ruptura del condensador equivalente ser la menor de las anteriormente


calculadas; esto es,
Vrup  83 V

Otro mtodo
Cargas mximas (ruptura) para cada uno de los dos condensadores:
Qs  8q50  400 C
Q3  12q 400  4800 C
Los dos condensadores en serie tendrn la misma carga, de modo que la tensin de ruptura
del condensador equivalente se calcula a partir del que admite menor carga de ruptura; i.e.,
Q
400
Vs  VAB  s 
 83 V
CS
4.8

- 376 -

Fsica Universitaria: Problemas de Fsica

Capacidad elctrica. E02.11


2 F/100 V

11. En la asociacin serie-paralelo de condensadores que se muestra en la


figura, determinar la capacidad y la tensin de ruptura del condensador
equivalente entre los bornes A y B.

6 F/50 V

B
3.5 F/400 V

Los dos condensadores en serie equivalen a uno de capacidad Cs:


CC
2q 6 12
  1.5 F
Cs  1 2 
C1 C2 2 6 8
La asociacin en paralelo equivale a un condensador de
capacidad Ceq:
A
Ceq  Cs C3  1.5 3.5  5 F

2 F/100 V

6 F/50 V
B

3.5 F/400 V

La tensin de ruptura ser, obviamente, inferior a la del


condensador situado en la rama inferior; i.e.,
VAB  400 V

5 F/133 V
B
En la rama superior, los dos condensadores en serie se A
reparten la tensin y tienen la misma carga:
Condensador equivalente
V1 V2  VAB
V1 V2  VAB
l

C1V1  C2V2
C1V1  C2V2  0
Y resolviendo ese sistema de dos ecuaciones con dos incgnitas:
C2
6
3

V1  C C VAB  8 VAB  4 VAB  100 V l VAB  133 V


1
2

C1
2
1
VAB  VAB  VAB  50 V l VAB  200 V
V2 
C1 C2
8
4

La tensin de ruptura del condensador equivalente ser la menor de las anteriormente


calculadas; esto es,
Vrup  133 V

Otro mtodo
Cargas mximas (ruptura) para cada uno de los tres condensadores:
Q1  2q100  200 C
Q2  6q50  300 C
Q3  3.5q 400  1400 C
Los dos condensadores en serie tendrn la misma carga, de modo que la tensin de ruptura
del condensador equivalente se calcula a partir del que admite menor carga de ruptura; i.e.,
Q
200
Vs  VAB  1 
 133 V
CS 1.5

- 377 -

Fsica Universitaria: Problemas de Fsica

Capacidad elctrica. E02.12


C

12. Determnese la capacidad equivalente entre los puntos A y B del conjunto de


condensadores de la figura, siendo C la capacidad de cada uno de ellos.

D
B
Puesto que todos los condensadores son idnticos, el condensador que se A
encuentra en la rama CD est puenteando dos ramas idnticas, por lo que los nodos C y D
se encontrarn siempre al mismo potencial y el condensador situado en esa rama no se cargar
nunca; por lo que puede suprimirse.
Nos queda el sistema ilustrado en la segunda figura, compuesto por tres ramas (en paralelo)
entre A y B. Cada una de las dos ramas superiores, constituidas por dos condensadores en
serie, presentan una capacidad:
C
1
1 1
C

l C1 
C1 C C
2

y la capacidad equivalente entre A y B, correspondiente a las tres


ramas en paralelo, ser:
C C
Ceq  C  2C
2 2

- 378 -

D
A

Fsica Universitaria: Problemas de Fsica

Corriente continua. E03.1

RL
RL
la distribucin de una seal elctrica (v.g., televisin
por cable) a una gran cantidad de abonados. Cada
abonado representa una resistencia de carga RA entre
RA
fuente
la lnea de distribucin y tierra, en tanto que la
de seal
resistencia de la lnea de distribucin entre los
puntos de conexin de los diferentes abonados se
considera como una resistencia constate RL.
Demostrar que la resistencia equivalente vista desde la entrada de la seal es

RL

1. En la figura se muestra un esquema de circuito para

Req 

1
RL 4 RA RL RL2
2

RA

RA

Sugerencia: Puesto que hay un gran nmero de abonados, la resistencia equivalente no variar apreciablemente
si un abonado (digamos el primero) cancela su suscripcin. En consecuencia, la resistencia equivalente de la
seccin del circuito situado a la derecha de la primera resistencia de carga es prcticamente igual a Req.

De conformidad con la sugerencia, el circuito se simplifica en


la forma que se indica en la segunda figura, con una resistencia
Req en paralelo con la resistencia de carga del primer abonado,
lo que representa una resistencia Rpar igual a
R R
1
1
1


l Rpar  A eq
Rpar RA Req
RA Req

RL

RA

La resistencia equivalente vista desde la fuente seal es


RA Req
Req  RL Rpar  RL
l
RA Req
Req  RL

R eq

fuente
de seal

RL

RA Req
RA Req

Rpar
fuente
de seal

De modo que operando y despejando Req obtenemos


RA Req Req2  RL RA RL Req RA Req

R  RL Req  RL RA  0
2
eq

y resolviendo esta ecuacin de segundo grado en Req, obtenemos


RL o RL2 4 RL RA
1
 RL RL2 4 RL RA
2
2
habiendo descartado el signo negativo porque conducira a una Req negativa.

Req 

- 379 -

Fsica Universitaria: Problemas de Fsica

Corriente continua. E03.2

2. Disponemos de un generador de 12.0 V de f.e.m. y 1  de resistencia interna.


R1

12.0 V, 1 

Necesitamos suministrar una tensin de 3.0 V a una carga externa, por lo que
recurrimos a un divisor de tensin, tal como se esquematiza en la figura.
a) Determinar el valor de la resistencia R2 para R1 = 9 . b) Determinar el
generador equivalente entre los bornes A y B. c) Calcular la tensin VAB cuando
suministramos corriente a una carga externa de R = 7.5 .

A
R2

R
B

a) Al no existir carga externa, la intensidad que suministra el generador


y la d.d.p. entre los bornes AB son:
12
12

I


Rcirc 1 9 R2 10 R2
12 R2
30
VAB  IR2 
 3 l 12 R2  30 3R2 l R2   3.33 
10 R2
9

12.0 V, 1 

b) La f.e.m. equivalente entre los bornes AB coincide con la d.d.p. entre dichos bornes en
circuito abierto; por consiguiente:
eq  3.0 V
9:
Si cortocircuitamos los bornes AB, la intensidad de cortocircuito
A
ser:
12.0
I corto 
 1.2 A
3.33 :
10

por lo que la resistencia del generador equivalente ser:


eq
3.0
req 

 2.5 
I corto 1.2

3.0 V, 2.5 

Tambin puede calcularse, ms rpidamente, teniendo en cuenta que entre AB tenemos dos
resistencias de 10  y 3.33  en paralelo, por lo que
10q3.33
 2.5 
req 
10 3.33
c) Calculamos la intensidad a partir de las caractersticas del
A
generador equivalente:
3.0
3.0
7.5 

 0.3 A  300 mA
I
2.5 7.5 10
B
y la d.d.p. pedida ser
VAB  IR  0.3q 7.5  2.25 V

- 380 -

Fsica Universitaria: Problemas de Fsica

Corriente continua. E03.3

3. Dos bateras con fuerza electromotriz  = 5 V y resistencia interna r = 2 :, pueden conectarse a una
resistencia R = 6 : en serie o en paralelo. a) Determinar el generador equivalente y la potencia suministrada
a la resistencia R en cada caso. b) Qu mtodo de conexin suministra la mayor potencia a R? Y si R fuese
igual a 1 :?

Bateras en serie:
s  i  5 5  10 V
Generador equivalente:
rs  ri  2 2  4 8


I s  s  10  1 A

Rs,tot 4 6

R  6 8 Ps  s I s  10q1  10 W

Ps,R  I s2 R  12 q 6  6 W

(ms potencia)


10

2A
Is  s 

Rs,tot 4 1

R 18
Ps  s I s  10q 2  20 W

Ps,R  I s2 R  22 q1  4 W

Bateras en paralelo:
p  5 V (idnticas)

Generador equivalente: 1
1 1 1
   1 l rp  1 8
ri 2 2
rp


I  p  5  0.71 A
p Rp,tot 1 6

R  6 8 Pp  p I p  5q 0.71  3.57 W

P  I 2 R  0.712 q 6  3.06 W
p
p,R

Ip  p 
 2.5 A

Rp,tot 1 1

R  1 8 Pp  p I p  5q 2.5  12.5 W

Pp,R  I p2 R  2.52 q1  6.25 W

(ms potencia)

- 381 -

5 V, 2 

5 V, 2 

5 V, 2 
R
5 V, 2 

Fsica Universitaria: Problemas de Fsica

Corriente continua. E03.4

4. Una batera tiene una f.e.m. de 12 V y una resistencia interna de 1.00 . a) Qu resistencia externa
deberemos conectar a la batera para obtener una potencia de 20 W? b) dem una potencia de 40 W?
c) Qu potencia mxima puede transferir la batera a una resistencia externa?

Aplicamos la ecuacin del circuito y sustituimos la intensidad de la corriente en la expresin


de la potencia disipada en la resistencia de carga:
,r

2
+
I
l PR  I 2 R 
R
(r R)2
r R
a
b
Operamos en est ltima expresin para despejar R:
PR r 2 PR R 2 2 PR rR   2 R  0 l

PR R    2 PR r
R PR r  0
2

Resolvemos esta ecuacin de segundo grado con respecto a R:

 2  2 PR r
o  2  2 PR r

R

 4 PR2 r 2

2 PR

 2  2 PR r
o 

 2  4 PR r

2 PR

a) Sustituyendo valores:
R

122  2q 20q1.0
o12

122  4q 20q1.0

2q 20

104 o 96

 5.0
0.2 
40

b) De nuevo sustituimos valores:

122  2q 40q1.0
o12

122  4q 40q1.0

64 o 48i
2q 40
80
de modo que no existen soluciones reales. Ocurre que la batera no puede proporcionar esa
potencia a la resistencia de carga.
c) La potencia mxima corresponde a una carga igual a la resistencia interna de la batera, esto
es R = r, de modo que
R

PR ,mx 

2
2
122
r

 36 W
2
(r r )
4r 4q1.0

PR
36 W
20 W

R1

- 382 -

R2

Fsica Universitaria: Problemas de Fsica

Corriente continua. E03.5

5. Calcular y justificar el valor de R para que el puente de la figura est equilibrado.

El puente est equilibrado cuando no circula corriente por la rama CD (rama puente) lo que
significa que la distribucin de intensidades es la que se indica en la figura. Para que se
C
presente tal circunstancia deber ser nula la diferencia de
potencial entre los puntos o nudos C y D; esto es, los puntos C y
5:
R
D estn al mismo potencial, de modo que
I1 I1
VAC  VAD l 5 I1  3I 2
A
B
I2 I2
V V
l RI  6 I
CB

DB

y dividiendo m.a.m. estas dos ecuaciones tenemos


5 3

l 3q R  5q 6
R 6
que es la conocida regla del producto en cruz, de modo que
5q 6
R
 10 
3

- 383 -

3:

6:

12 V, r = 1 :

Fsica Universitaria: Problemas de Fsica

Corriente continua. E03.6

6. a) Determinar la intensidad de corriente que recorre cada una de las ramas del circuito que se muestra en la
figura. b) dem cuando aadimos una resistencia de 1  colocada entre A y B.

a) Cada una de las dos ramas soporta una tensin de 12 V, de


modo que las intensidades que las recorren son:
12
i1  i2 
2A
2 4
y la intensidad que suministra el generador es:
i  i1 i2  4 A
Otro mtodo
La resistencia equivalente de cada una de las dos ramas es 6  y la resistencia equivalente de
las dos ramas en paralelo es:
RR
6q 6 36
R 1 2 
 3
R1 R2 6 6 12

La intensidad total se reparte por igual entre las dos ramas:


12
i 4
i1  i2  2 A
3
b) El puente no est equilibrado, por lo que aplicamos el mtodo de Maxwell para las
intensidades de malla:


6  2 4
12 6 2 4 I1





I
0
2
7
1
=
2 7 1  132  3

2

0 4 1 7 I 3
4 1 7
I1 

12 2 4
1
12 7 1 576

 4.364 A
0 7 1 
132 0 1 7
132 1 7
132

I2 

6 12 4
12 2 1 216
1
2 0 1 

 1.636 A
132 4 0 7
132 4 7
132

6 2 12
1
12 2 7
360
2 7

 2.727 A
0 


4
1
132 4 1 0 132
132
Las intensidades de rama son:
i1  4.264 A i2  2.728 A i3  1.636 A i4  1.091 A i5  1.637 A i6  2.727 A
I3 

en los sentidos indicados en la figura.

- 384 -

Fsica Universitaria: Problemas de Fsica

Corriente continua. E03.7

7. En el circuito que se esquematiza en la figura, determinar el valor de la

1k

resistencia R desconocida y la intensidad que circula por cada rama.

5mA
A
25 V
1k

1k

Abordamos el clculo de las intensidades de malla por el mtodo


matricial de Maxwell:
2 I  I  I  25

1 1 I1
1 2 3
25 2

0  1 1 R 0 I 2 l I1 1 R
I 2  0

0 1
I
0
2
3
I1 2 I 3  0
Sistema de tres ecuaciones con cuatro incgnitas (las tres intensidades y R) a las que
aadiremos la indicacin del ampermetro para tener una cuarta ecuacin:
I 3  I 2  5 mA
Resolvemos el sistema de las tres primeras ecuaciones:
%  4 1 R
 1 R
 2  1 3R

25 1 1
50 1 R

1
0 1 R 0 
I1 
1 3R
% 0
0
2

1k

I2

15 mA 5 mA

2 25 1
1
50
I 2  1 0 0 
1 3R
% 1 0
2

25 V

I1

10 mA

5 mA
10 mA

I3

1k

2
1 25
25 1 R

1
I 3  1 1 R 0 
1 3R
% 1
0
0
Y considerando la cuarta ecuacin:
I3  I 2 

25 1 R

50
25 R  25
30


 5 mA l R   3 k
1 3R
1 3R
1 3R
10

De modo que:
I1 

200
 20 mA
10

I2 

50
 5 mA
10

- 385 -

I3 

100
 10 mA
10

1k

Fsica Universitaria: Problemas de Fsica

Corriente continua. E03.8

8. Las aristas de un tetraedro estn formadas por resistencias. Las seis aristas tienen la misma resistencia R.
Hllese el valor de la resistencia equivalente entre dos vrtices cualesquiera.
C

Visto desde uno cualquiera de sus vrtices (v.g., D), las resistencias
de las aristas del tetraedro se pueden representar en el plano del
papel, tal cono se indica en el primer esquema.
El primer esquema se transforma obviamente en el segundo, en el
que apreciamos que la resistencia CD est puenteando en un
puente equilibrado ABCD. En consecuencia, no pasa corriente
por la rama CD, de modo que podemos eliminarla (tercer esquema).
El tercer esquema nos muestra tres ramas entre A y B, cuyas
resistencias se indican (cuarto esquema).
Finalmente, en el cuarto esquema tenemos tres resistencias en
paralelo, dos de ellas de valor 2R y otra de valor R; por estar en
paralelo, ser
1
1
1
1 1 1 2 2




Req 2 R 2 R R
R
2R
= Req 

D
B

A
C

R
A

R
2

2R
2R
A

- 386 -

R2

Fsica Universitaria: Problemas de Fsica

Corriente continua. E03.9

9. Dado el circuito de la figura determinar la intensidad de corriente que pasa por la resistencia de 2 : situada
entre A y B y la d.d.p. entre esos puntos.

Aplicamos el mtodo de Maxwell, con las corrientes de malla indicadas en el esquema:


10 7 2 0
7 2 0
I1

20  2 9 4 I 2 l %  2 9 4  242 3


I 3
0 0 4 6
0 4 6
Resolvemos:
I1 

5:

10 2 0
1
140
20 9 4 
 0.5785 A
% 0 4 6 242

10V
7 10 0
1
720
I 2  2 20 4 
 2.975 A
% 0 4 6 242

I1

3:

A
2:

I2
B

La intensidad de la corriente que circula por la


rama AB, en el sentido de AB, es la suma de las
dos corrientes de malla; esto es,
iAB  I1 I 2  0.5785 2.975  3.55 A
La d.d.p. entre A y B ser:
VAB  RABiAB  2q3.55  7.10 V

- 387 -

2:

I3

20V
4:

Fsica Universitaria: Problemas de Fsica

Corriente continua. E03.10


A

10. Determinar la resistencia equivalente entre los puntos A y B del circuito

3:

representado en la figura.
2:

Suministramos corriente al circuito conectando un generador de


f.e.m. entre los nudos A y B y resolvemos el circuito por el mtodo
de las mallas de Maxwell:


 8 2 6 I1
0  2 11 6 I 2 l %  348 3
0 6 6 14 I

3
Tan slo estamos interesados en la intensidad de corriente
suministrada por el generador externo, ya que
 2 6
%
1
I1 
0 11 6  11 
% 0 6 14
%

6:

2:

6:
B

3:

A
I2

6:

2:

2:
I3


I1

6:

nos permite calcular la resistencia equivalente entre A y B:



%
348
348
RAB  


 2.95 
11 6 118
I1 %11
6 14

RAB


I1
B

- 388 -

Fsica Universitaria: Problemas de Fsica

Corriente continua. E03.11

11. En el esquema del circuito de corriente continua de la figura, determnese: a) Las

2:

intensidades que circulan por cada rama. b) La diferencia de potencial entre los
puntos A y B.

1:
A

6V

3:

5:
1:

6V

Resulta ms cmodo volver a dibujar el circuito en la forma que se


indica. Lo resolveremos por el mtodo de Maxwell, considerando las
corrientes de malla que se muestran en la figura:
6 8 2 5 I1


6  2 6 1 I 2 l %  174  3

6 5 1 8 I
3

2:

Resolvemos para las corrientes de malla:

6 5

1
0
I 2  2 6 1 
0A

6 2 5
% 5 6
174

1
348
I1  6 6 1 
 2 A
8 2 6

% 6 1 8 174

I  1 2 6 6  348  2 A

3
% 5 1 6 174

y las intensidades de rama son las que se indican en la


figura (circuito en serie).

2:
3:

b) La diferencia de potencial entre A y B la calculamos


por el camino ACB:
VAB  2q 2  6  2 V

2A
I3

6V

5:

2A
I2
2 A 1:

C
2:

1:
I1

6V

Mtodo rpido (simetras):


Es fcil observar la simetra que presenta el circuito con respecto
al eje que se indica en la figura. Como consecuencia de dicha
simetra, los puntos D y E, por un lado, y los puntos B y C, por
otro, estn al mismo potencial, por lo que no circular intensidad
por las ramas DE y BC y pueden suprimirse. Nos queda un
circuito serie, como se indica en la figura (circuito resaltado), por
el que circula una intensidad
4
6 6
12
i

 2A
4 R 2 1 2 1 6

- 389 -

C
2:

3:

6V

6V
E

2A
A

1:

5:
1:
B

2:

Fsica Universitaria: Problemas de Fsica

Corriente continua. E03.12

que pasan por cada resistencia. b) Fuerza electromotriz y


resistencia interna del generador equivalente entre A y B
en la segunda figura.

6:

6:

12. En el circuito de la figura, determinar: a) Las intensidades


10V

4:

4V 10V

2:

4:
A

3:

a) Aplicamos el mtodo de las mallas con la notacin de Maxwell, siendo I1, I2, y I3 las
corrientes de malla:
10

0

4

I1

I2

I3

10 4 0 I1


4 9 2 I 2 o '


0 2 2 I 3

10 4 0

4 9 2 108

0 2 2

4 0
108
9 2
1A
108

2 2
10 0
0

0 2
0A
108
4 2
10 4 10
1
216
4 9 0
2 A
108
'

0  2 4

10
1
0
'
4
10
1
4
'
0

i6  i4  1 A

6:
1A
4:

10V

2A
4V

2:
1A

2A

3:
i2  2 A

i3  0 A

designando por ix las intensidades de corrientes de rama que circulan por las resistencia Rx.
b) La f.e.m. del generador equivalente es igual a la diferencia de potencial entre los puntos A
y B y la resistencia interna de dicho generador es la resistencia equivalente de las resistencias
de 6 : y 4 : en paralelo.
6: B
10 V
I
Intensidad que circula por el circuito:
1 A
(6 4) 8
10V
Diferencia de potencial entre A y B: VAB  (1 A)q 4 8  4 V
1A
4:
6q 4 24
Resistencia equivalente entre A y B: req 

 2.4 8
6 4 10
A
Por consiguiente:
eq  4 V
req  2.4 8

- 390 -

Fsica Universitaria: Problemas de Fsica

Corriente continua. E03.13

13. En el circuito de la figura, determinar la f.e.m. y la resistencia interna del generador equivalente entre A y B.

Abierto:
Aplicamos el mtodo de Maxwell para determinar las intensidades de malla I1 e I2, que son
tambin las intensidades que circulan por las resistencias de 12 
y 8 , respectivamente.
6V,3:
4V,2:
16 20 5 I1
2

l %  275 
I2
6 5 15

Resolvemos para las corrientes de malla


1 16 5 170

 0.764 A
I1  6 15 
%
275

1 20 16 200
 0.145 A
I 2 

% 5 6 275

I1

10V,5:

12:

I2

8:

Determinamos ahora la intensidad en cortocircuito; esto es, con los bornes AB


cortocircuitados. Resolvemos el nuevo circuito por el mtodo de las mallas de Maxwell.
La f.e.m. equivalente entre AB es igual a la d.d.p. entre AB (en circuito abierto):
eq  VAB  12q 0.764  8q 0.145  8.00 V
En corto:

I
0 20 12 8 c
16  12 20 5 I1a l %  1100  3

6 8 5 15 I 2a

6V,3:

Resolvemos para la intensidad de corto:


0 12 8
1
2200
I c   16 20 5 
2A
 6 5 15 1100

I1

4V,2:

10V,5:

I2

12: Ic 8:

La resistencia interna del generador equivalente entre A y B es


eq 8
req 
 4
Ic
2
Otro mtodo:
Para determinar la resistencia equivalente entre AB, observamos
que la resistencia de 5  est puenteando las dos ramas en
paralelo que forman el circuito. El puente est equilibrado a
efectos de resistencias (aunque circula corriente por l, debido a la
batera de 10 V), ya que 3u8 = 2u12, por lo que podemos ignorar
(suprimir) la resistencia puente y el circuito se reduce al que se
muestra en la figura. La resistencia equivalente entre AB es:
5q 20 100

4
req 
5 20
25

- 391 -

3:

12:
A

2:

8:
B

Fsica Universitaria: Problemas de Fsica

Corriente continua. E03.14

14. Consideremos el circuito de c.c. que se muestra en la figura. a) Determinar la f.e.m. y la resistencia interna
del generador equivalente entre los bornes A y B. b) Calcular la potencia mxima que puede suministrar el
circuito de la figura a una resistencia de carga externa conectada entre A y B. Determinar el valor de dicha
resistencia de carga.

12 V, 18 :

a) La f.e.m. del generador equivalente entre AB es igual


a la diferencia de potencial entre AB sin carga externa.
Para determinarla, resolvemos el c.c. de la figura por el
mtodo de las mallas de Maxwell.


16 30 6 I1 l %  30 6  864  2
I 2
2  6 30
6 30

1 16 6 492 41


 0.569 A
% 2 30 864 72

I1 

I2 

6 V, 9 :
4V
6:

6:

2
15 :

1 30 16 156 13


 0.181 A
864 72
% 6 2

VAB  0.569q 6 0.181q15  3.417 2.708  6.125 V


= eq  6.125 V
Determinamos ahora la intensidad en cortocircuito; esto es, con los bornes AB
cortocircuitados. Resolvemos el nuevo circuito por el mtodo de las mallas de Maxwell.


I c'
0 21 6 15

16  6 30 6 I1
12 V, 18 :
2 15 6 30
6 V, 9 :
I '
2
21 6 15
%  6 30 6  9234 3
15 6 30

0 6 15
1
5292
Ic 
16 30 6 
 0.5731 A
9234 2 6 30 9234

req 

eq

Ic

4V
6:

1
6:

2
15 :

Ic

6.125
 10.69 
0.5731

b) Un generador suministra la mxima potencia a una carga externa cuando el valor de sta
coincide con el de la resistencia interna del generador. Por consiguiente,
Rext  req  10.69 

eq
R eq2
e2q
6.1252
R
 I R 


 0.877 W = 877 mW
2

r R
4R
4 R 4q10.69
2

Pmax

- 392 -

Fsica Universitaria: Problemas de Fsica

Corriente continua. E03.15

15. El circuito que se muestra en la figura est constituido por cinco generadores de f.e.m. asociados en
puente. a) Determinar la f.e.m. y la resistencia del generador equivalente entre A y B. b) Calcular la
intensidad que suministra cada generador cuando cortocircuitamos A y B.

Abierto:
El valor de la f.e.m. equivalente coincide con el de la
diferencia de potencial entre A y B en circuito abierto.
Resolvemos el circuito que se indica en la figura:
A
12 8 4 I1
2

32

%

l

I 2
12 4 6

3 V, 1 

6 V, 2 
12 V, 4 

I1

1 12 4 24

 0.75 A
32 12 6
32
48
1 8
12

 1.50 A
I2 


4
12
32
32
I1 

I2

6 V, 2 

3 V, 1 

eq  VAB  2q (0.75) 1q1.5  (6  3)  1.5 1.5 9  9 V

Ic 

9
 2 1
1
288
12
8 4 
6A
48 12 4 6
48

6 V, 2 

3 V, 1 
3.75 A

I1

B
I2

2.25 A

6 V, 2 

Ahora, podemos determinar fcilmente la resistencia


interna del generador equivalente entre A y B, ya que


9
I c  eq l req  eq   1.5 
req
Ic
6

1.50 A
2.25 A

12 V, 4 

En corto:
Determinaremos la intensidad en cortocircuito entre A
y B utilizando las corrientes de malla que se indican en
la figura
9 3 2 1
I c
A


I1 l %  48 3



12
2
8
4



I 2
12 1 4 6

4.50 A

3 V, 1 

Ic

b) Calculamos las otras dos corrientes de malla con A y B en cortocircuito:

I1 

3
9
1
1
180
2 12 4 
 3.75 A
48 1 12 6
48

I2 

3 2
9
1
72
2 8
12 
 1.50 A
48 1 4 12 48

A partir de las corrientes de malla (Ic, I1 y I2) se obtienen fcilmente las corrientes de rama,
i.e., las corrientes que suministran cada uno de los generadores. As, el generador de 12 V
(puente) suministra 3.75 1.50 = 2.25 A (hacia arriba). Las dems corrientes de rama estn
indicadas en la figura con sus valores y sentidos propios.

- 393 -

Corriente continua. E03.16


6 V, 1

16. En el circuito que se esquematiza en la figura, determinar el generador


equivalente (f.e.m. y resistencia interna) entre los terminales A y B.

3

6 V, 1

Fsica Universitaria: Problemas de Fsica

3

6 V, 1

2A

6 V, 1

A
I1

2A

3

VAB  3q 2 1q 2
 6
 6  2 6  10 V l

eq  10 V

6 V, 1

3
B

6 3 1

1
240
 6 A l I c  eq
5 1 
12

40 12 1 5 40
req

- 394 -

6 V, 1

I1

6 V, 1

En corto:
Para determinar la resistencia interna del
generador equivalente entre A y B cortocircuitamos esos bornes y calculamos la intensidad de
cortocircuito (Ic) que circula por la rama A y B
de resistencia nula. Aplicamos el mtodo
matricial de Maxwell para calcular las
intensidades de malla:
6 4 3 1 I c



12  3 5 1 I1a l %  40  2
12 1 1 5 I a
2

B
I2

2A

1 12 1 48

2A
24 12 5
24
1 5
48
12
I2 

 2 A
24 1 12
24
I1 

Ic 

3

2A

4A

En abierto:
La d.d.p. entre A y B coincide con la f.e.m.
equivalente entre esos puntos. Abordamos el
clculo de las intensidades de malla por el
mtodo matricial:
12 5 1 I1
2


l   24 
I2
12 1 5

6 V, 1

3

I2
6 V, 1

Ic

l req 

eq

Ic

10
 1.67 
6

Fsica Universitaria: Problemas de Fsica

Corriente continua. E03.17


6V

1

2

17. En el circuito que se esquematiza en la figura: a) Calcular la intensidad de la


B

corriente que circula por la resistencia de 4  y la d.d.p. entre N y M;


b) Determinar el generador equivalente (f.e.m. y resistencia interna) entre los
terminales A y B.

4

2

Abordamos el clculo de las intensidades de malla por el mtodo matricial:


6 7 4 I1
2

6 V 1
l %  33 
M
I2
6 4 7
0.55 A

6 4 18
  0.5455 A
6 7 33
7 6 18
  0.5455 A
4 6 33

0.55 A

4

I1

I2

0.55 A

i4  1.091 A l VNM  4 1.091


 4.364 V x 4.4 V

1

2

1.10 A

1
33
1
I1 
33
I1 

6V

0.55 A

2

1

6V

b) En abierto:
La d.d.p. entre A y B coincide con la f.e.m. equivalente entre esos puntos:

VAB  2 0.5455
4 1.091
2 0.5455
 6.546 V l
En corto:
Para determinar la resistencia interna del generador
equivalente entre A y B cortocircuitamos esos bornes y
calculamos la intensidad de cortocircuito (Ic) que circula
por la rama A y B de resistencia nula. Aplicamos el
mtodo matricial de Maxwell para calcular las
intensidades de malla:
6 3 2 1 I c


6  2 7 4 I1a l %  48 3

6 1 4 7 I 2a

Ic 

6 2 1
1
216
 4.5 A
6
7 4 
48 6 4 7
48

l Ic 

eq
req

l req 

eq
Ic

I1a  3.00 A

6.546
 1.46 
4.5

- 395 -

eq  6.546 V x 6.6 V

6V

2

1

4
B

A
I1

I2

2

6V
Ic

I 2a  1.50 A

1

Fsica Universitaria: Problemas de Fsica

Corriente continua. E03.18

18. Determinar la f.e.m. y la resistencia interna del generador equivalente al circuito de se representa en la
figura, entre los bornes A y B.

Mtodo general:
Abierto:
0 10 6 I 2
I 2  0

0  6 8
I 3 l I  0
3
VAB  0
 12  6
 18 V l  = 18 V

2

12 V

B
2
8V

En corto:

6
4V
10V

2

18 3 2 1 I1a

a
0  2 10 6 I 2 %  66  l

0 1 6 8 I 3a
I corto  I1a 

1

6V

3
1

18 2 1
1
792
 12 A
0 10 6 
66 0 6 8
66

18 V 1.5 

I 2a  I 3a  6 A


18
I corto  eq l req  eq   1.5 
req
I corto 12

12 V

2

6V

1
B

6

4V
8V

2

10 V

1

12 V

2

6V

1
B

8V

2

10 V 1 

18 V

3

18 V

3

Otro mtodo:
En virtud del resultado obtenido en el primer apartado del
mtodo general, llegamos a la conclusin de que el puente
est equilibrado en f.e.m.s, ya que no circula corriente por
la rama de puente, y en resistencias por lo que podemos
suprimirlo. Entonces, el circuito se reduce a asociaciones
serie y paralelo de generadores de f.e.m.
1. Asociacin de generadores en serie. Se suman las f.e.m y
se suman las resistencias internas.
2. Asociacin de generadores en paralelo. Como todos tienen
la misma f.em., polaridad resistencia interna, la f.e.m.
equivalente es la de cualquiera de ellos y la resistencia
equivalente corresponde a la de una asociacin de resistencias
en paralelo; esto es,
rr
r 3
1
1 1

l req  1 2    1.5 
req r1 r2
r1 r2 2 2

- 396 -

18 V 1.5 

Fsica Universitaria: Problemas de Fsica

Corriente continua. E03.19


1

6V

1

19. En el circuito que se esquematiza en la figura: a) Determinar el


generador equivalente (f.e.m. y resistencia interna) entre los terminales
A y B b) Calcular la d.d.p. entre CD en circuito abierto y
cortocircuitado entre AB

6V
1

1

6V

1

6A

En abierto:
La d.d.p. entre A y B coincide con la f.e.m. equivalente entre esos puntos. Abordamos el
1
clculo de las intensidades de malla por el mtodo
6 V 1
C
matricial:
3A
3A



12  3 1 I1 l %  8  2
6V
B
12 1 3 I 2
A
I1
I2
1 12 1 24
I1 

3A
1
8 12 3
8
3A
3A
24
1 3
12
D
1

 3 A
I2 
6V 1
8 1 12
8

VAB  1 3
1 3
 6
 6 V l

eq  6 V

6V

1

VCD  1 6
 6
 0 V

En corto:
Para determinar la resistencia interna del generador
equivalente entre A y B cortocircuitamos esos bornes
y calculamos la intensidad de cortocircuito (Ic) que
circula por la rama A y B de resistencia nula.
Aplicamos el mtodo matricial de Maxwell para
calcular las intensidades de malla:
6 2 1 1 I c



12  1 3 1 I1a l %  8  2

12 1 1 3 I a
2
Ic 

6 1 1

1
4
12
3 1   6 A l I c  eq
req
8 12 1 3
8

I1a 

2
6 1
1
4
1 12 1   6 A
8 1 12 3
8

I 2a 

6V

l req 

1

I1

I2
1

1

eq

Ic

D
Ic

6V

1

6
1 
6

2 1 6
1
0
1 3
12   0 A
8 1 1 12 8

VCD  1 6
 6
 0 V

Otro mtodo:
El clculo de la resistencia equivalente puede abordarse ms fcilmente teniendo en cuenta
que el puente de 5 resistencias (prescindiendo de las f.e.m.s) est equilibrado:
1

1

1
B

1
B

2

1
1

A
1

1

1

- 397 -

1

2

Fsica Universitaria: Problemas de Fsica

Corriente continua. E03.20

20. En el circuito que se esquematiza en la figura, determinar el generador

6 V, 1

3

equivalente (f.e.m. y resistencia interna) entre los terminales A y B.

B
3
6 V, 1

3

En abierto:
La d.d.p. entre A y B coincide con la f.e.m. equivalente entre esos puntos. Abordamos el
clculo de las intensidades de malla por el
3
6 V, 1
mtodo matricial:
0.6 A
060A


6  7 3 I1 l %  40  2
3

I 2
6 3 7
B
A
I1

1 6 3 24

 0.60 A
7
40 6
40
1 7 6 24

 0.60 A
I2 
40 3 6
40

I2

1.20 A

I1 

0.6 A

0.60 A

6 V, 1

3

eq  VAB  3q 0.60


1q 0.60
 6
 1.8  0.60 6  7.2 V
 3q 0.60
3q 1.20
3q 0.60
 1.8 3.60 61.8  7.2 V
En corto:
Para determinar la resistencia interna del generador
equivalente entre A y B cortocircuitamos esos bornes y
calculamos la intensidad de cortocircuito (Ic) que circula
por la rama A y B de resistencia nula. Aplicamos el
mtodo matricial de Maxwell para calcular las
intensidades de malla:

I c
6 4 1 3

6  1 7 3 I1a l %  72  3
6 3 3 7 I a
2
Ic 

3

6 V, 1

3

I1

I2

6 V, 1

3

Ic

6 1 3

1
7.2
288
 4 A l req  eq 
 1.8 
6 7 3 

Ic
72 6 3 7 72
4

Otro mtodo para la resistencia equivalente entre AB:


Consideramos tan solo las resistencias de todos los elemento (incluidas las resistencias
internas de los generadores) y aadimos un generador entre AB, para disponer de intensidad
en el circuito. Entonces, resolvemos para calcular la intensidad que suministra el generador:
 4 1 3
%  72 3
I1

1
3
0  1 7 3 I 2 l
%11  40 3
0 3 3 7 I
B
A

I
3
1
2

%
72


I1  %11 l Req  

 1.8 
%
I1 %11 40

3
1

3

I1

- 398 -

Fsica Universitaria: Problemas de Fsica

Corriente continua. E03.21

21. Consideremos el circuito que se representa en la figura una vez que los

1F/100V

2
2F/300V

100 V

condensadores se han cargado completamente. a) Calcular la carga y la


tensin que soporta cada condensador. b) Determinar el valor mximo
aplicable de la f.e.m. del generador si las tensiones de ruptura de los
condensadores son 100 V, 200 V y 300 V, respectivamente, tal como se
indica en la figura.

2
1F/200V

A
1F/100V

2
100 V

2F/300V

2
1F/200V

En rgimen estacionario, los condensadores estn


completamente cargados y no conducen corriente
elctrica.
 100
I 
 25 A
R
4
a) Dada la simetra que presenta el circuito (puente
equilibrado), ser
VBD  0
VAD  VDB  12 VAB  50 V

El condensador de 2F no soporta tensin alguna y, por tanto, no adquiere carga elctrica.
Cada uno de los dos condensadores de 1F soporta una tensin de 50 V y adquieren una
misma carga
Q  CV  1 Fq50 V  50 C
b) El condensador de 2F no soporta tensin alguna y, por tanto, no nos preocupa al efecto
f.e.m. mxima aplicable.

La f.e.m. mxima aplicable ser de 200 V, ya que, al repartirse por igual entre los dos
condensadores de 1F, se alcanza la tensin de ruptura del que la tiene ms baja (100 V).

- 399 -

Fsica Universitaria: Problemas de Fsica

Corriente continua. E03.22

22. Consideremos el circuito representado en la figura. a) Calcular las intensidades de corriente que circulan por
cada rama, indicando su sentido en un esquema. b) Asignando el potencial nulo al punto A, determinar los
potenciales en los puntos B, C, D y E.
1
2 D
C
B

0.25 V

I1

I3

0.50 V

a) Mtodo de la mallas de Maxwell:


0.25 5 4 0
I1


0  4 7 3 I 2 l %  50

0.50 0 3 5
I3

I2
3

4
A

I1 

0.25 4 0
1
0.5
0
7 3 
 0.010 A  10 mA
50 0.50 3 5
50

I2 

5
0.25
0
1
2.5
0
4
3 
 0.130 A  50 mA
50 0 0.50 5
50

I3 

5 4 0.25
1
6.5
0 
4 7
 0.050 A  130 mA
50 0 3 0.50
50

Las corrientes de malla (corregidas) y las corrientes de rama se indican en la segunda figura.
b) Clculo de d.d.p. o de potenciales referidos a
10 mA
130 mA
VA=0:
D
C
B
Vab
 iR   l
JJG
0.25 V

10

1

2

50

4
60 mA

3
80 mA

A
50 mA

- 400 -

130

0.50 V

VBA  VB  (0.25)  0.25 V  250 mV

VCA  VC  4q 60  240 mV

VDA  VD  (0.50)  0.50 V  500 mV

VEA  VE  0

Fsica Universitaria: Problemas de Fsica

Corriente continua. E03.23


2:

23. En el circuito esquematizado en la figura, calcular: a) La fuerza


electromotriz y resistencia interna del generador equivalente entre A y B.
b) La intensidad que pasara por una resistencia de 1 : colocada entre A y
B.

4:

12 V
4:

2:

a) Por cada una de las ramas circula una intensidad de corriente:


 12
I   2A
R
6
La d.d.p. entre A y B coincide con la f.e.m. equivalente entre esos puntos, de modo que
VAB  2(2) 4(2)  4 8  4 V

con el borne A positivo.


Para determinar la resistencia interna del generador equivalente entre A y B cortocircuitamos
esos bornes y calculamos la intensidad de cortocircuito (icorto) que circula por la rama A y B
de resistencia nula. Aplicamos el mtodo matricial de Maxwell para calcular las intensidades
de malla:


I
12 6 2 4 1
2:

4:
0 I %=96 3
0  2 6
2

0 4 0
6 I
3
2A
B
A
2A
12 V

12 2 4
1
12 6 0
0 6
0 
 4.5 A
I 
1 96
96 0 6
0 0
6

6 2 12
1
12 2 6
2 6 0 
 3.0 A
I 
3 96
96 4 0
4 0 0
i  I  I  1.5 A (A l B)
c
3 2
4
8

r  
  2.67 
eq i
1.5 3
c
4
12
  1.09 A
b) La intensidad pedida es: I 
8 / 3 1 11

2:

2:

4:
I2

I1
A
12 V

6 12 4
1
12 2 0
2 0 0 
 1.5 A
I 
2 96
96 4 6
4 0 6

4:

B
I3

4:

2:

1:
I

4 V, 8/3:

- 401 -

Fsica Universitaria: Problemas de Fsica

Corriente continua. E03.24

24. Con un hilo homogneo de seccin constante construimos los lados y las diagonales,

que se unen en el centro, de un hexgono regular. Sea R la resistencia de cada uno de


los tramos de hilo. a) Determinar la resistencia equivalente entre dos vrtices
contiguos del hexgono. b) Se conectan esos dos vrtices contiguos a una batera de
que suministra una intensidad de corriente I. Calcular la intensidad de la corriente en
cada uno de los conductores.

Todos los puntos situados sobre la lnea de simetra (discontinua) estn al mismo potencial y,
por tanto, podemos fusionarlos en O. Reduciendo sucesivamente en serie y en paralelo, se
11
obtiene finalmente que la resistencia equivalente entre A y B es Req 
R
20
Las intensidades se calculan a partir del ltimo esquema, al que asignamos arbitrariamente
una corriente de 1 A. Retrocediendo hacia los esquemas anteriores, teniendo en cuenta que en
dos ramas en paralelo, de resistencias R1 y R2, la intensidad se reparte de modo que

R2

I1 

I
I
I
R


R
R R2
1
2
1 R2

l I  I1 1 I1  I1 1
l

R1
R2
R2
V  I1 R1  I 2 R2

I2 

R1 R2


40 A
1

22

33/40

7/40

11

11

1
1

1/3 1

1 1/3

1 3/40 O

1 1/3

1/3 1/3

4/7

11/7

11/20

7/40

4/7

- 402 -

33/40

11/7

1/3 1

7/40

O 11/40

11/40

33/40

7/40

4/3

4/3

3/40

1/3 1

4/40

1/3 1

1/3

7/40

1 1/3

O
3

B
33/40

4/40

Fsica Universitaria: Problemas de Fsica

Corriente continua. E03.25


A

25. Cada uno de los vrtices del hexgono est unido a cada uno de los dems
vrtices por ramas de resistencia elctrica R. a) Calcular la resistencia elctrica
equivalente entre los nodos A y B. b) dem eliminando la rama AB.

C
a) Al estar cada vrtice unido a cada uno de los restantes, no hay E
diferencia alguna, a efectos de resistencia elctrica, entre dos vrtices
D
consecutivos o entre dos que no lo sean. As, nos resultar ms fcil
aplicar simetras cuando tomamos dos vrtices opuestos, A y D en la figura, ya que los puntos
o nudos B  F y E  F se encuentran obviamente al mismo potencial, lo que nos permite
reducir el circuito del modo que sigue en las figuras.
En el paso del segundo esquema al tercero, hemos suprimido la rama A-BF-CE-D porque
corresponde a un puente equilibrado, de modo que por la resistencia puente , R/4, no circula
corriente y podemos suprimirla.

R/2

R/2

B=F

R/3

C=E

B=F
R/4
C=E

R/2

R/2

D
D
R
En definitiva, RAB 
3
b) Si quitamos una de las resistencias de valor R entre A y B quedar
R
a 
RAB
2

R/2

- 403 -

Fsica Universitaria: Problemas de Fsica

Corriente continua. E03.26


1:

26. En el circuito de la figura: a) Determinar la


resistencia equivalente entre los bornes A y B.
b) Calcular la intensidad de la corriente en
cada una de las ramas cuando se conecta el
circuito al de la batera, tal como se indica en
la figura.

1:

C 1:

4:

6V

1:

1:

1:

4:

1:

G 1:

4:

2:

1:

1:

a) Reducimos el circuito comenzando por la cola, mediante agrupaciones serie y paralelo.


1:

4:

1:

1:

1:

2:

4:

1:

1:

1:

2:

4:

1:

Descubrimos fcilmente in proceso reiterativo que nos lleva finalmente a la reduccin


deseada entre los bornes A y B.
Por consiguiente, la resistencia equivalente entre A y
B es Req  4 

b) La intensidad que proporciona la batera es



6
I
 1 A
1:
Rcircuito 6

6V

1:

4:

1:

2:

4:

En el primer y segundo esquema,


B
D
B
1:
podemos observar fcilmente que
la intensidad que llega a cada uno de los nudos C, E, G, se divide a
partes iguales entre la rama vertical y la horizontal; esto es, se divide por
dos, de modo que las intensidades en las ramas sucesivas son: 1 A,
500 mA, 250 mA, 125 mA, ...

1: A

1:

C 1:
0.5A

1A
4:

6V

1:

1:

1:

G 1:

0.25A

4:
0.5A
D

1:

4:

2:

0.25A
F

- 404 -

1:

1:

Corriente continua. E03.27

27. En una lnea de transporte de corriente continua, que tiene


125 V

400 m
F

50 A

Rf

R-R1
45 A

B
106.5 V

125 V

lo que nos permite calcular la resistencia


del tramo de longitud x:

tierra

R1

A x

VAB  50 R1 45 R  R1
 5 R1 45 R

fuga

5A

Expresamos la d.d.p. entre los puntos


extremos de la lnea de transporte (A y B),
siendo R la resistencia total de la misma:

400 m

400 m de longitud y 0.4  de resistencia, se ha producido una


derivacin a tierra por un mal aislamiento. La corriente de
entrada en la lnea es de 50 A a 125 V y la de salida de 45 A a
106.5 V. Determinar el punto de la lnea en el que se ha
producido la derivacin y la resistencia elctrica de la misma
(i.e., la resistencia de fuga).

106.5 V

Fsica Universitaria: Problemas de Fsica

V  45 R 125 106.5
 45q 0.4
R1  AB

 0.1 
5
5
de modo que ser:
x
400

l x  100 m
0.1 0.4
Expresamos ahora la d.d.p. entre A y C, a lo largo del camino AFGC:
VAC  50 R1 5Rf
y de aqu calculamos la resistencia de fuga:
V  50 R1 125  50q 0.1
Rf  AC

 24 
5
5

50 A

Rf

R-R1
45 A

Operando, obtenemos dos ecuaciones con dos incgnitas (R1,Rf):


50 R1 Rf
 45Rf  125
50 R1 5 Rf  125
l

50 Rf 45 R  R1 Rf
 106.5

45 R1  5 Rf  106.5  45 R  124.5

Sumando m.a.m. tenemos: 5R1  0.5 l


Y de la primera ecuacin: Rf 

125  50 R1
5

R1  0.1 
l

- 405 -

Rf  24 

106.5 V

400 m

R1
5A

x
125 V

Otro planteamiento (mtodo de Maxwell):


Consideramos las dos mallas que se indican
en la figura:
50
125 R1 Rf
Rf



106.5 Rf
R  R1 Rf 45

Fsica Universitaria: Problemas de Fsica

Corriente continua. E03.28

28. Una subestacin de corriente continua desarrolla una potencia de 400 kW cuando suministra energa a una
instalacin situada a 100 m de distancia, a travs de una lnea elctrica de 3.5  de resistencia (cada cable
conductor) constituida por conductores de cobre (resistividad,  =1.75 10-8 m. a) Calcular la potencia
que se disipa en la lnea elctrica cuando la tensin que suministra la subestacin es de 5 kV. Qu
porcentaje de prdida de potencia se produce en la lnea elctrica? b) dem si la tensin fuese de 15 kV.
c) Calcular la seccin de los cables. d) Determinar la densidad de corriente elctrica en los cables en ambos
casos, as como la intensidad del campo elctrico en los mismos.

a) Intensidad que circula por la lnea: I 

400 kW
 80 A
5 kV

Potencia disipada en la lnea:


Plinea 44.8

 0.112  11.2%
P
400

400 kW
 26.6 A
b) Intensidad que circula por la lnea: I 
15 kV
Plinea  I 2 Rlinea  802 q 7  44.8 kW

Potencia disipada en la lnea:



Plinea  I 2 Rlinea  26.6 2 q 7  4.98 kW

Plinea 4.98

 0.0124  1.24%
P
400

c) Seccin de los cables:


RS

l
S

l S S

l 1.75q108 q100

 5q107 m 2  0.5 mm 2
R
3.5

S
0.5
QD2
lD2
2
 0.80 mm
4
Q
Q
d) Recordamos la definicin de densidad de corriente y la expresin de la ley de Ohm
(microscpica):
A
A
A
A
I
l j1  160q106 2  160
j
j2  53.3q106 2  53.3
m
mm 2
m
mm 2
S
V
V
E  S j l E1  2.80
E2  0.93
m
m
S

3.5 :
R


I

3.5 :

- 406 -

Fsica Universitaria: Problemas de Fsica

Corriente continua. E03.29

29. Una lnea de tranva, de 10 km de longitud,

x
10 km, 0.1 /km
est alimentada por dos generadores de corriente continua, de (1100 V, 10 ) y (1000 V,
10 
10 
10 ) respectivamente, conectados cada uno
en un extremo de la lnea, como se muestra en
1000 V
1100 V
la figura. La resistencia elctrica del cable y de
las vas son de 0.1 /km. El tranva requiere
10 km, 0.1 /km
una intensidad de corriente de 100 A para su
funcionamiento. Para una posicin genrica, x
(km), del tranva, determinar las intensidades y potencias que suministran al tranva cada uno de los dos
generadores y la tensin de alimentacin del mismo.

0.1x

Resolvemos el circuito por el mtodo de


las mallas de Maxwell, siendo R11 y R22
las resistencias de las mallas:
R11  10 2q 0.1x  10 0.2 x

0.1(L-x)

A
Rt=0

10
1

t

I1

10
2

I2

R22  10 2q 0.1( L  x) 
 10 0.2(10  x)  12  0.2 x
R11 R22  22 


1   t  R11

2   t 0

I t  I1 I 2 

0.1x

I  1
1 R11 R22

0 I1
l

R22 I 2
1
I 2 
R11 R22

1   t
2   t

0
R22

0.1(L-x)

1   t
R11

R11 1   t
 t
 2
0 2   t
R22

1
1   t 2   t




1
R R22
 t  1 2  11

 1 2 
t

R11
R22
R11 R22 R11 R22
R11 R22
R11 R22

Despejamos la f.c.e.m. del tranva:


1

2  It
R  R112  R11 R22 I t 11200  60 x 4 x 2
R
R22
 t  11
 22 1

R11 R22
R11 R22
22
R11 R22
Calculamos la tensin de alimentacin VAB del tranva: VAB  0q I  t
 t
Calculamos la intensidad de corriente
suministrada por cada generador:

   t 1300  20 x
I1  1


R11
22

2   t 900 20 x

I 2 
R22
22

I t  I1 I 2  100 A
As como las potencias:

V
520
515

Tensin de alimentacin del


tranva en funcin de la
distancia x.

510
505
500
495
490
2

- 407 -

10 x

Fsica Universitaria: Problemas de Fsica

Corriente continua. E03.30

1
P1  VAB I1 
11200  60 x 4 x 2
1300  20 x

484

1
11200  60 x 4 x 2
900 20 x

P2  VAB I 2 
484

Pt  P1 P2 

50
11200  60 x 4 x 2

11

0.1x

Otro mtodo:
R1  10 2q 0.1x  10 0.2 x

10

Rt=0

I1

1

R2  10 2q 0.1( L  x) 

0.1(L-x)

0.1x

2

t

It

10 0.2(10  x)  12  0.2 x
R1 R2  22 8

10

I2

0.1(L-x)

Resolvemos el circuito por el mtodo de Kirchof:


R I  R2 I 2  1  2  
1 malla 4IR  4
l 1 1

1 nudo (A) 4I  0
I1 I 2  I t
R R2
%  1
 R1 R2
1
1

 R2  R2 I 100 12  0.2 x


100 1300  20 x

1



I1 
1
R1 R2 I
R1 R2
22
22

R1 
R1 I   100 10 0.2 x
100 900 20 x
I  1

1 R R 1 I  R R 
22
22
1
2
1
2

Calculamos la tensin de alimentacin del tranva y su f.c.e.m.:


VAB  0q I  t
 t
t  VAB  I1 R1  1
 1  I1 R1  1100 

P1  VAB I1 

 1 11200  60 x 4 x 2
1300  20 x

484

P2  VAB I 2 

2
 484 11200  60 x 4 x
900 20 x

50
Pt  P1 P2  11200  60 x 4 x 2

11

1300  20 x
11200  60 x 4 x 2
10 0.2 x

22
22
V
520
515

Tensin de alimentacin del


tranva en funcin de la
distancia x.

510
505
500
495
490
2

- 408 -

10 x

Fsica Universitaria: Problemas de Fsica

Corriente continua. E03.31


200 :

30. En el circuito de la figura, sabiendo que el condensador est inicialmente


descargado. a) Determinar la corriente inicial que suministra la batera,
inmediatamente despus de cerrar el interruptor S. b) Calcular la intensidad
de la corriente estacionaria a travs de la batera despus de transcurrir un
largo perodo de tiempo. c) Determinar la carga del condensador en las
circunstancias del apartado anterior.

5 PF

50 V

600 :
300 :
S

a) En el instante inicial, t = 0, el condensador conduce y el circuito equivalente, en ese


instante, es el que se ilustra en la figura. Las dos resistencias en paralelo equivalen a una
resistencia de
200 :
300q 600
 200 8
Rp 
300 600
5 PF
50 V
de modo que
600 :
300 :
50
50
I0 

 0.125 A = 125 mA
S
200 200 400
b) Transcurrido un tiempo suficientemente grande, desaparece
el estado transitorio de carga y el condensador corta la corriente en la rama AB en la que se
encuentra, por lo que el circuito equivalente es el que se indica en la figura. La intensidad de
corriente que suministra la batera es
50
50
Id 

 0.0625 A = 62.5 mA
200 600 800
200 :
A
c) La tensin entre los bornes del condensador es VAB (ya que
5 PF
por la resistencia de 300 : no circula corriente) de modo que
50 V
600 :
VAB  I d R600  0.0625q 600  37.5 V
300 :

S
B

y la carga del condensador ser


Q  CVAB  5q37.5  187.5 C

- 409 -

Fsica Universitaria: Problemas de Fsica

Corriente continua. E03.32

31. Un condensador de 1 PF tiene deteriorado el dielctrico, dando fugas de corriente, comportndose como si
estuviera en paralelo con una resistencia de 95 :. Se pretende cargar el condensador con una fuente de
24 V de f.e.m. y 1 : de resistencia interna. a) Determinar la mxima diferencia de potencial entre las
armaduras del condensador que podemos alcanzar. Dibujar el esquema correspondiente al proceso de carga.
b) Si se interrumpe el proceso de carga, calcular el tiempo necesario para descargarse en la relacin 1/e;
i.e., determinar el valor de la constante de tiempo del condensador. Dibujar el esquema correspondiente al
proceso de descarga.

Comenzamos determinando el generador de f.e.m. equivalente


entre A y B:

24
I

 0.25 A
Rf r 95 1
eq  VAB  IRf  0.25q95  23.75 V

Req 

Rf

Rf r
95q1 95


 0.99 
Rf r 95 1 96

a) La mxima diferencia de potencial entre las armaduras del


condensador que podemos alcanzar ser de 23.75 V.
b) La constante de tiempo en los procesos de carga y de
descarga de un condensador de capacidad C a travs de una
resistencia R viene dada por la expresin U  RC .
Carga:
6

U  Req C  0.99q1q10

6

 0.99q10

eq
A

Req

s  0.99 s

Descarga:
U  Rf C  95q1q106  95q106 s  95 s
q i
Qf
I0

q i
Q0

q (t )  Qf 1  et / RC

0.63Qf

Proceso de descarga

Proceso de carga

0.37I0

q (t )  Q0et / RC

0.37Q0

i (t )  I 0et / RC

t
-0.37I0

-I0

- 410 -

t / RC

i (t )  I 0e

Fsica Universitaria: Problemas de Fsica

Corriente continua. E03.33

32. En el circuito representado en la figura, el condensador est inicialmente


descargado. Pasamos el conmutador S a la posicin A y lo mantenemos en
ella durante 10 s, al cabo de los cuales lo pasamos a la posicin B.
a) Determinar la mxima intensidad de corriente que ha suministrado la
batera. b) Calcular la carga y la tensin que adquiri el condensador durante
el proceso de carga. c) Calcular la intensidad y la potencia mximas que
proporciona el condensador a la resistencia de 10 : durante el proceso de
descarga del condensador. d) Comparar las intensidades mximas de carga y
de descarga, as como la rapidez de cada uno de esos procesos, justificando y
argumentado los resultados.

10 k:

A
i
1 mF

W
DESCARGA
10 :

i
+
1 mF
Q
q
t
i

A
B
S

10:

1 mF

b) La carga final del condensador, si tuviera tiempo de cargarse


completamente, sera
Qf  1 m F q12 V = 12 mC
(t l d)

I0
i

10 k:

Proceso de carga del condensador: Constante de tiempo


RC = 10 k: u 1 mF = 10 s.
a) La intensidad mxima se presenta en el instante en que comienza
la carga del condensador:
12 V
I0 
(t  0)
 1.2 mA
10 k8

CARGA
12V

12V

pero, como interrumpimos la carga al cabo de 10 s, el condensador


tan solo adquiere
Q  Qf (1 et / RC )  12 mC q (1 e1 )  0.636 q12 mC = 7.59 mC
y la tensin del condensador en ese instante es
Q 7.59 mC
V 
 7.59 V
C
1 mF
Proceso de descarga del condensador: Constante de tiempo
RC = 10 : u 1 mF = 10 ms.
c) La intensidad mxima se presenta en el instante en que comienza
la descarga del condensador a travs de la resistencia de 10 ::
7.59 V
I0 
(t  0)
 0.759 A = 759 mA
10 8

y la potencia mxima que proporciona el condensador se presenta


en ese mismo instante:
P  V0 I 0  7.59 q 0.759  5.75 W
I0

d) La intensidad y la rapidez de descarga son mucho mayores que la de carga por tener lugar
sta a travs de una resistencia mucho mayor (1000:1), lo que repercute en las
correspondientes constantes de tiempo (RC).

- 411 -

Fsica Universitaria: Problemas de Fsica

Corriente continua. E03.34

33. En el circuito esquematizado en la figura, cerramos el interruptor en el


instante t = 0, inicindose el proceso de carga del condensador. Supongamos
que sea despreciable la resistencia interna de la batera y sean R = 1 M:,
C = 1 mF y  = 100 V. a) Expresar en funcin del tiempo la intensidad de la
corriente y la carga del condensador. b) Al cabo de 10 s abrimos el
interruptor. Calcular el valor de la carga que retiene el condensador y la
energa almacenada en el mismo.

C
b

a) En un instante genrico, la suma de ls tensiones soportadas por la resistencia y por el


condensador es igual a la tensin entre los bornes del generador de f.e.m., de modo que:
q
  Vad  iR
[1]
C
Derivando esta expresin con respecto al tiempo, conseguimos que desaparezca la variable q
que representa la carga del condensador en el instante t. Luego, integrando

di 1
di dt
i0 l

0 l
dt C
i
RC
i  I 0 e-t / RC

I0

t
di
1
i
t

dt l ln  

i
RC 0
I0
RC

l i  I 0 e-t / U

[2]

donde  = RC es la constante de tiempo del circuito RC. A partir de las expresiones [1] y [2],
se sigue fcilmente que

t  0 l q  0 l i  I 0 
para
R

t  d l i  0 l q  Qf  C
Determinamos la expresin de la carga del condensador en funcin del tiempo a partir de la
expresin de la intensidad, i(t); por nueva integracin, tenemos
q
t
dq
 I 0 e-t / U l dq  I 0 e-t / U dt l
0
0
dt
t

q  I 0 ( RC ) e-t / U  RC 1 et / U
 Qf 1 et / U
[3]
0
R
b) Sustituyendo los valores dados en el enunciado, tenemos
U  RC  1 M q1 mF = 1000 s
Qf  C  100 V q1 mF  100 mC
de modo que
q10  100 1 e-10/1000
 100 1 e-0.1
 0.995 mC  995 C
6
1 q102
1 995q10


 4.95q104 J = 495 J
2 C
2
103
2

U10 

- 412 -

Fsica Universitaria: Problemas de Fsica

Corriente continua. E03.35

R=1000 :

34. Expresar en funcin del tiempo la carga acumulada y la d.d.p. en


bornes de cada uno de los condensadores C1, C2 y C3 de la figura
considerando t = 0 en el instante en que se cierra el interruptor y
que los condensadores estn inicialmente descargados.

C 1=3P F
C2=6P F

C3=4P F

12V

En el proceso de carga de un condensador a travs de una resistencia, la carga que adquiere el


mismo y la d.d.p. entre bornes son funciones del tiempo y vienen dadas por las expresiones:
q (t )   C (1 e-t / RC )

v(t )  q / C   (1 e-t / RC )

Determinamos la capacidad del condensador equivalente entre A y B :


CC
3q 6
R
C12  1 2 
 2 F
Ceq  2 4  6F
A
C1 C2 3 6
de modo que

Ceq
B

C  12 V q 6 F = 72 NC

RC  1000  q 6 F= 6 ms = 0.006 s

La carga del condensador equivalente y la diferencia de potencial


entre los puntos A y B vienen dada por
t
t

RC
RC
q   1 e eq  72 1 et /0.006
C
vAB   1 e eq  12 1 et /0.006
V

Los condensadores C12 (C1 +C2 en serie) y C3 soportan la misma tensin:

q  48 1 et /0.006
C

v  12 1 et /0.006
V
q
q
q
q

3
3
12  3 l 12  3 l

t /0.006

C
C
2
4
q  24 1 e
v  12 1 et /0.006
V

C

12
3
12
12

Los condensadores C1 y C2 en serie adquieren la mima carga:

1  8 1 et /0.006
V


v

1 C

1
q  q  q  24 1 et /0.006
C l

12
1
2

v  2  4 1 et /0.006
V

2 C

- 413 -

Fsica Universitaria: Problemas de Fsica

Corriente continua. E03.36

35. En el circuito de la figura, y una vez estabilizadas las corrientes (pasados los efectos
transitorios, i.e., transcurridos un tiempo muy grande) se pide calcular: a) La intensidad que circula por cada rama. b) La carga del condensador.
El circuito que alimenta al condensador, entre A y B, es equivalente a un generador de
corriente continua, determinar: c) La fuerza electromotriz y resistencia interna del
generador equivalente. d) La constante de tiempo de carga y descarga del condensador.

4:

10PF

16:

8:

24V

12:

a) Cuando desaparecen los efectos transitorios, el condensador


estar completamente cargado y no dejar pasar la corriente, por lo
que podemos suprimir la rama en la que se encuentra el
condensador. La resistencia equivalente externa y la resistencia total
de este circuito se determina fcilmente:
Rserie  28 16  24 

Rpar 

24
2

 12 

Rtot  12 12  24 

0.5 A

y la carga del condensador:


Q  CVAB  10 Fq12 V=120 C

24:

B
0.5 A

1A

16:

8:

La intensidad que suministra el generador y las intensidades en cada


rama son:
24
i

i

1 A
iramas   0.5 A
Rtot 24
2
b) Calculamos la d.d.p. entre los bornes del condensador:
VAB  VAC VCB  0 0.5q 24  12 V

24V

12:

A
4:
24:
C

B
16:

8:

c) La f.e.m. equivalente entre A y B viene representada por la d.d.p.


entre A y B sin carga. Entonces, del apartado anterior, resulta obvio
que eq  VAB  12 V

24:

12:

24V

La resistencia equivalente entre A y B se calcula fcilmente a partir del esquema adjunto:


1
1
1
1
1
10 
 
l Rpar  6 
= req  4 6  10 
12 V
Rpar 24 24 12 6
por lo que el circuito equivalente al dado es el que se muestra en la
figura.
d) La constante de tiempo del sistema RC (serie) ser:
U  RC  10  q10 F  100 s  0.1 ms

- 414 -

10 F

Fsica Universitaria: Problemas de Fsica

Corriente continua. E03.37

36. Un galvanmetro tiene una resistencia interna de 10 . Cuando pasa una corriente de 15 mA a travs del
galvanmetro, la aguja avanza una divisin de la escala. a) Qu resistencia debemos poner en serie del
galvanmetro para que el conjunto pueda utilizarse como voltmetro, en el que una divisin represente a
10 V? b) Qu resistencia debemos poner en paralelo al galvanmetro para que el conjunto pueda utilizarse
como ampermetro, en el que una divisin represente 0.1 A?

a) Sea RS la resistencia serie que debemos colocar. La d.d.p. de 10 V aplicada entre los bornes
A y B del voltmetro resultante deber proporcionar una intensidad de corriente de 15 mA a
travs del galvanmetro. Aplicando la ley de Ohm entre
IG = 15 mA
RG = 10 
A y B tenemos:
G
V
VAB  I G ( RG RS ) l RS  AB  RG
IG
RS

Con los datos del problema ser:


10
RS 
10  667 10  657 8
10 V
A
B
0.015
b) Sea RSh la resistencia de derivacin (shunt) que debemos colocar en paralelo con el
galvanmetro a fin de que parte de la intensidad se desve por ella. Por otra parte, la d.d.p.
entre A y B ser la misma a travs de la rama del galvanmetro que a travs de la rama del
shunt, de modo que tenemos
I Sh  I  I G

IG = 15 mA
I  I G I Sh

G
l
IG

RG I G  RSh I Sh

RSh 
RG
RG = 10 

I
=
100
mA
I

Sh

RS
Con los datos del problema ser:
A
B
RSh
I Sh  100 15  85 mA

ISh = 85 mA

RSh  15 10  1.8 8

85

- 415 -

Fsica Universitaria: Problemas de Fsica

Campo magntico. E04.1

1. Una partcula cargada, de masa m y carga elctrica q, se mueve con una


velocidad v en el vaco. En estas condiciones, la partcula penetra en una
zona, de anchura h, en la que existe un campo magntico uniforme B, en
direccin perpendicular a dicho campo. a) Determinar el valor mnimo de
B para que la partcula no pueda atravesar la zona. b) Qu desviacin
experimentar la partcula, tras atravesar la zona, si el campo magntico
tiene una intensidad que es la mitad de la calculada en el apartado anterior?

x
x
x
x
x

x
x
x
x
x

x
x
x
x
x

x
x
x
x
x Bx
x
x
x
x

x
x
x
x
x

v
q

a) La fuerza que acta sobre la partcula cargada viene dada por la frmula de Lorentz:
F  q vqB
de modo que, al ser perpendicular a la velocidad, tan solo modifica la direccin de sta, dando
lugar a una trayectoria circular de radio R, tal que
F  qvB  man  m

v2
R

l R

mv
qB

Puesto que el radio de la trayectoria es inversamente


proporcional a la intensidad del campo magntico, y
se requiere que R  h para que la partcula no
atraviese la zona, deber ser:
mv
mv
Bb
l Bmn 
qh
qh
b) Si se reduce la intensidad del campo magntico,
aumenta el radio de la trayectoria y, si R  h, la
partcula atraviesa la zona y sale desviada un cierto
ngulo .
Para
1
mv
B '  Bmn 
2
2qh
el radio del arco de trayectoria circular ser
mv
mv
R'

 2h
qB ' q mv

x
x
x
x

vx

x
F
x
x

x
x
R
x
x

x
x
x Bx
x
x
x
x

x
x
x
x

x
x
x B x
x
x
x  x

x
x
x
x

 v
x
x
x
x

x
x
x
x

x
R
x
x
x
C

2 qh

El ngulo de desviacin que experimenta la partcula tras atravesar la zona se determina


fcilmente a partir de la figura:
h
h
sen R  
 0.5 l R  30
R ' 2h

- 416 -

Fsica Universitaria: Problemas de Fsica

Campo magntico. E04.2

2. Un varilla conductora y homognea est suspendida mediante dos hilos verticales


conductores, tal como se muestra en la figura. La varilla posee una masa de
0.040 kg/m y se encuentra en un campo magntico uniforme, perpendicular al
plano de la figura y dirigido hacia adentro, de 3.6 T. Calcular la intensidad de la
corriente que debe circular por la varilla para que sea nula la tensin mecnica en
los hilos que la soportan. Cul deber ser el sentido de la corriente?

En las condiciones descritas en el enunciado del problema, la varilla debe estar soportada
exclusivamente por la fuerza que ejerce el campo magntico sobre ella
(ausencia de tensin mecnica en los hilos).
El peso de la varilla deber ser igual y de sentido opuesto a la fuerza que
ejerce el campo magntico uniforme B sobre un conductor rectilneo de
longitud l por el que circula una corriente i: esto es,
F  i l q B

donde la direccin de l es la del sentido convencional de la corriente en el conductor.


Puesto que nos dan la densidad lineal de masa (), la masa
total de la varilla es m = l, de modo que

P  mg  Ml
g

l Mlg  ilB

F  ilB
M g 0.040q9.8

 0.109  109 mA
B
3.6
y su sentido es el indicado en la figura.

i

- 417 -

Fsica Universitaria: Problemas de Fsica

Campo magntico. E04.3

3. Dos conductores rectilneos indefinidos estn contenidos en el plano xy,


paralelos al eje x como se indica en la figura (uno en y = - 6 cm y el otro
en y = 6 cm). Por cada uno de ellos circula una intensidad de 20 A.
Determinar el campo magntico de induccin B en los puntos del eje x
cuando: a) Las dos corrientes circulan en el sentido negativo del eje de
las x. b) La corriente en el conductor situado en y = - 6 cm circula en el
sentido positivo del eje de las x y la del situado en y = 6 cm en sentido
contrario.

y = - 6 cm
y
x

y = 6 cm

a) En el mismo sentido
El campo magntico resultante en todos los
puntos del eje x es nulo, ya que los
B2
conductores contribuyen con campos iguales
y opuestos en dichos puntos. Esto es as, por
y
c
d
B1
ser idnticas las intensidades de corriente y
las distancias de los conductores al eje x.
b) En sentidos opuestos
El campo magntico creado por una corriente
rectilnea indefinida a una distancia h del
conductor que la transporta viene dada por la
B1 B2
expresin,
N I
y
Bi  0
c
d
2Q h
como se deduce fcilmente a partir del
Teorema de Ampre, y su sentido es el
indicado en la figura, para cada uno de los conductores. En todos los puntos del eje x, ambos
conductores contribuyen por igual y en la misma direccin al campo magntico en dicho eje,
por lo que ser;
N I
B  B1 B 2  0 k
Q h
Sustituyendo los valores dados en el enunciado:
N I
20
B  0 k  4q107 q
 1333q107  133 T
Q h
0.06

- 418 -

Fsica Universitaria: Problemas de Fsica

Campo magntico. E04.4

4. Tres conductores rectilneos largos y paralelos pasan a travs de los


vrtices de un tringulo equiltero de lado 10 cm como se indica en la
figura, donde el punto indica que la corriente sale del papel hacia el
lector y la cruz que entra en el papel. Si la intensidad de cada corriente es
de 10 A, hallar: a) El campo magntico en el conductor superior debido a
los otros dos conductores inferiores. b) La fuerza por unidad de longitud
sobre el conductor superior.

10 cm

10 cm

10 cm

Utilizando el teorema de Ampre, calculamos el campo magntico B creado por un conductor


rectilneo indefinido:

v B dr  N I
0

Por ser B & dr , en todo el camino de integracin, ser


Bdr = B dl. Adems, el mdulo de B es el mismo en todos los
puntos de la lnea de circulacin, por lo que resulta:
N 2I
B
v dl  B 2Qr  N0 I l B  4Q0 r

I
B

r
dr

a) Los mdulos de los campos magnticos pedidos son B1  B2 

N0 2 I
en las direcciones
4Q r

que se indican en la figura. Y el campo magntico resultante es


N 2 I
N 2 I 3 N0 2 3I
cos 30  2 0
B  B1 cos 30 B2 cos 30  2 0

4Q r
4Q r 2
4Q r

y sustituyendo valores
N0 2 3I
2 3 q10
 107 q
 3.47q105 T
4Q r
0.10
b) La fuerza que acta sobre un conductor rectilneo que
transporta una corriente viene dada por F  I l q B , y por
ser l ? B ser
FIlB
F
1
 IB  10q3.47q105  3.47q104 N/m
l

B2

B

- 419 -

60

30
30
B1
60

Fsica Universitaria: Problemas de Fsica

Campo magntico. E04.5

5. a) Enuncie la ley de Ampre. Es vlida para toda trayectoria que rodea a un


conductor? Indique para que y en que casos resulta til. b) Cuatro conductores
rectilneos, indefinidos y paralelos en el vaco, transportan corrientes de igual
magnitud I = 4 A, tal como se indica en la figura. Determinar el campo magntico
en el punto P situado en el centro del cuadrado, determinado por los cuatro
conductores cuyo lado mide 0.2 m.

0.2 m

Teorema de Ampre:
La circulacin del campo de induccin magntica (B) a lo largo de una trayectoria cerrada (C)
es igual al producto de P0 por la intensidad neta (I) que fluye a travs de cualquier superficie
que tenga a la trayectoria de circulacin (C) como contorno. Esto es,

v B<dl = N I
0

Este teorema es vlido para cualquier trayectoria cerrada en un campo magntico.


Se utiliza para determinar el campo magntico de induccin (B), resultando especialmente til
cuando calculamos la circulacin de B a lo largo de una lnea de campo magntico y es
constante el mdulo de B (esto es, B), a lo largo de dicha lnea.
b) Determinamos el campo magntico B creado por un largo conductor rectilneo, que
transporta una corriente I, a una distancia r del mismo. Aplicando el teorema de Ampre,
siendo C una trayectoria circular de radio r que coincide con una lnea de campo:
NI
N 2I
v B<dl = v B d l  B v d l  B 2Qr  N0 I l B  2Q0 r  4Q0 r
C
C
C
y aplicando esta expresin a uno cualquiera de los cuatro conductores, teniendo en cuenta que
r  l 2 / 2 , siendo l el lado del cuadrado, tenemos
N
N 2 2I
2I
2 2 q4
 0
 107
 5.7 T
B 0
4Q l 2 / 2 4 Q l
0.2
N 2 2I
N 8I
Btotal  4 B cos 45  2 2 B  0
(2 2)  0
4Q l
4Q l
7 8 q 4
= Btotal  10
 16 T
0.2
en la direccin indicada en la figura.

- 420 -

Btotal

Fsica Universitaria: Problemas de Fsica

Campo magntico. E04.6

6. Determinar, razonada y detalladamente, la expresin de la fuerza por unidad de longitud con que interactan
dos conductores indefinidos y paralelos que conducen intensidades i1 e i2.

Comenzamos determinando, mediante el teorema de Ampre, el


campo magntico a una distancia r de un largo conductor rectilneo
I
que transporta una intensidad de corriente I. Para ello calculamos la
circulacin de campo B a lo largo de una trayectoria circular (lnea de
campo) situada en un plano perpendicular al conductor:
r
N 2I
B
v B<dl  v Bdl  B v dl  B 2Qr  N0 I l B  4Q0 r
As, los campos magnticos creados por cada una de las corrientes
rectilneas indefinidas en las posiciones ocupadas por el otro conductor, situado a una
distancia h, tienen las direcciones indicadas en la figura y sus mdulos son:
N 2i
N 2i
B1  0 1
B2  0 2
4Q h
4Q h
y las fuerzas de interaccin mutua, que vienen dadas por,
F  I (l q B) , tienen las direcciones indicadas y sus mdulos
I1
B2
son
N 2i i
F12 N0 2i1i2
F12

F12  i1 l q B 2  i1lB2  0 1 2 l l
l
4Q h
4Q h
N0 2i1i2
F21 N0 2i1i2

l l
F21  i2 l q B1  i2lB1 
l
4Q h
4Q h
F21
de modo que la fuerzas de interaccin mutua es atractiva si
B1
I2
ambas corriente tienen la misma direccin (como se indica en
la figura) o repulsiva si tienen direcciones opuesta, siendo la
fuerza por unidad de longitud igual a
F N0 2i1i2

l
4Q h

- 421 -

Fsica Universitaria: Problemas de Fsica

Campo magntico. E04.7

7. Por un conductor cilndrico, con 2 cm de dimetro, circula una corriente continua de 1 A, uniformemente
distribuida en la seccin del conductor. Determinar el valor del campo magntico en el interior del
conductor en funcin de la distancia r al eje del mismo.

Seccin recta del conductor: S  Q R 2  Q q 0.012  3.14q104 m 2


Densidad de corriente: j 

I
1

 3.18q103 A/m 2
S 3.14q104

B
r

Teorema de Ampre:

Qr 2
v B dl  Ni l B 2Qr  N Q R 2 I
Sustituyendo valores: B 

NI
l B
r
2Q R 2

4Q q107 q1
r  2q103 r
2Q q 0.012

- 422 -

(S.I.)

i:

I
B

Fsica Universitaria: Problemas de Fsica

Campo magntico. E04.8

8. Sobre una espira cuadrada de lado a, por la que circula una intensidad i, acta un campo magntico B
uniforme y constante. Calcular el momento de las fuerzas que actan sobre la espira, direccin y sentido en
los siguientes casos: a) El campo magntico B es paralelo a una diagonal de la espira. b) El campo
magntico B es paralelo a uno de los lados de la espira. c) El campo magntico es perpendicular al plano de
la espira. d) El campo magntico forma 30 con la normal al plano de la espira.

El momento dipolar magntico (m) de la espira y el momento dinmico (M) que ejerce el
campo magntico uniforme (B) sobre la espira vienen dados por las expresiones siguientes:
m  IS  ia 2k

l M  iS q B
= M  iSB sen R  ia 2 B sen R

M

m
q
B

a) M  ia 2 B sen 90  ia 2 B
2 / 2
2 / 2
0

2
2

M  m q B  ia 0q B  2 / 2  ia B 2 / 2
0
0
1

m
y

i
B

M
a

i
a

b) M  ia 2 B sen 90  ia 2 B
0



1
0
M  m q B  ia 2 0q B 0  ia 2 B 1
1
0
0

m
M

B
a

z
i

m
y

a
a

z
B

m
y

i
a
x

c) M  ia 2 B sen 0  0
0
0
0

2
2

M  m q B  ia 0q B 0  ia B 0  0

1
1
0

M
a

1
d) M  ia 2 B sen 30  ia 2 B
2
0

M  m q B  ia 0q B  sen 30 

1
cos 30
sen 30
2 1

ia B
 ia 2 B 0 
0
2 0
0

- 423 -

Fsica Universitaria: Problemas de Fsica

Campo magntico. E04.9


z

9. Una espira cuadrada de lado a est inclinada ngulo D respecto al plano


horizontal, tal como se muestra en la figura, pudiendo girar alrededor de
su eje de simetra paralelo al eje x. Si existe un campo magntico
uniforme B = B k y la espira est recorrida por una intensidad de corriente
I, determinar el peso que habra que suspender de uno de los lados de la
espira (cul?) para que no gire.

B
D

D
y

I
C

Sobre cada uno de los cuatro lados de la espira acta


una
fuerza
que
viene
dada
por
la
B
expresin
F

I
(
l
q
B
)
,
donde
I
es
la
intensidad
de
F
B
corriente, l es la longitud del lado (en la direccin de la
corriente) y B es el campo magntico.
A
D
D
I
y
Sobre los lados AC y BD de la espira actan unas
fuerzas en las direcciones indicadas en la figura,
D
iguales y opuestas, que por ser paralelas al eje de
C
x
F
rotacin de la espira no producen momento con
respecto al mismo.
Sobre los AB y CD de la espira actan tambin fuerzas iguales y opuestas, en las direcciones
que se indican en la figura, de mdulo
F  IaB
A
B
F=IaB
que constituyen un par de fuerzas, cuyo
momento es
M
z

a senD

M par  F a sen B  Ia 2 B sen B

a
Mpeso

D
a cosD

F=IaB
P=mg

en la direccin que se indica en la figura.


As, deberemos compensar dicho momento
suspendiendo una pesa en el lado CD de la
espira, de modo que proporcione un
momento con respecto al eje de rotacin

igual y opuesto al del par; esto es,


M par  M peso

a
l Ia 2 B sen B  P cos B l P  2 IaB tg B
2

Otro modo de calcular Mpar El momento dipolar


magntico (m) de la espira y el momento dinmico
(Mpar) que ejerce el campo magntico externo (B) sobre
la espira son
m  IS

l M par  I S q B
M par  m q B
= M par  ISB sen B  Ia 2 B sen B
que es el mismo resultado obtenido anterior.

- 424 -

B
D

M
a
Mpeso

C
P=mg

Fsica Universitaria: Problemas de Fsica

Campo magntico. E04.10

10. Con un conductor de longitud 8a se forman dos espiras cuadradas de lado a


exponindose a la accin de un campo magntico uniforme y constante como se
indica en la figura. Si por el conductor se hace circular una corriente de
intensidad i, determinar: a) Las fuerzas que se ejercen sobre los lados 1-2 y 3-4.
b) El momento resultante sobre las espiras.

a
2
3

a) Aplicando la expresin de la fuerza que acta sobre un conductor


rectilneo que transporta una corriente,

1
con B  B 0
F  i l q B

0
1
resulta
B

2
3
i

z
a

2 / 2
0
1

2
Lado 1-2: F12  iaB 2 / 2q 0  iaB
0
2 1
0 0

0
0 1


Lado 3-4: F34  iaB 1q 0  iaB 0

0 0
1

b) Calculamos los momentos (M) dinmicos a partir


de los correspondientes momentos magnticos (m = iS) de las espiras.
Momento sobre la espira superior:
0


0 1
M1  m1 q B  i S1 q B  ia 2 B 0q 0  ia 2 B 1
0
1 0
Momento sobre la espira inferior:
0


0 1
2

M 2  m 2 q B  i S 2 q B  ia B 0 q 0  ia B 1

1 0
0
2

Momento total: M  M1 M 2  0

- 425 -

Fsica Universitaria: Problemas de Fsica

Campo magntico. E04.11


z

11. Una espira con forma de tringulo rectngulo, de catetos b y c, representada,


forma un ngulo de 45 respecto al plano xz. Por la espira circula una
intensidad de corriente I en el sentido indicado y puede girar libremente
entorno al eje z. Si la espira est dentro de un campo magntico uniforme
B = B i, determinar: a) El momento magntico de la espira. b) La fuerza
magntica sobre cada lado de la espira. c) El momento de las fuerzas
magnticas sobre la espira.

I
b
y
B
45

a) El momento magntico de la espira est definido como m  IS , donde I es la intensidad


que la recorre y S la superficie de la espira, de modo que
1
m  IS  bcI l
2
cos 45
1

1
2
2
1
bcI 1
m  m  cos 45  bcI
1 
2

2
4

0
0

0
b) La fuerza que ejerce el campo magntico sobre un conductor rectilneo de longitud l que
transporta una corriente I viene dada por F  I l q B
, de modo que:
0



1
0

Fb  I b q B
 Ib 0 q B 0  IbB 1  IbB j

1
0
0
Fc  I cq B
 Ic

1

0
1

2
2
2
IcB 0  
IcB k
1q B 0 

2 0
2
2
0

1

Fa Fb Fc  0 l Fa  Fb  Fc  IbB j

c) El momento de las fuerzas magnticas sobre


la espira o momento dinmico es:

M  mqB 

2
IcB k
2

1 1
0


2
2
bcIB 1q0 
bcIB 0

4
4
0 0
1

b
m

45

B 45 c
x

- 426 -

Fsica Universitaria: Problemas de Fsica

Campo magntico. E04.12

12. Un conductor muy largo que transporta una corriente I se dobla en la forma indicada en

la figura. Determinar el campo magntico en el punto P.


P
a
a

Comenzamos estableciendo la expresin del valor del campo magntico B a


2a
una distancia a de un conductor rectilneo (no infinito) que transporta una
intensidad de corriente I. Para ello, recurrimos a la ley de Biot-Savart, con la notacin que se
indica en la figura:

a  s sen R l s 

N0 d l qe s
N0
dx sen R

sen R
I
I
dB 
con
B

a
r2
s2
4Q
4Q

dR
x  a cotg R l dx 

sen 2 R

P
s
es

dB
a

T
-x

dl

eje x

de modo que
R2

B

N0
N I
a
sen 2 R N0 I
R
I
d
sen
sen R dR  0 < cos R >R2
R
R

2
2

1
a
4Q
sen R
4Q a R
4Q a
1

B

N0 I
(cos R1  cos R2 )
4Q a

Para los conductores verticales (semi-infinitos) ser:


Bvert 

N0 I
N I
2

(cos 0  cos135 )  0 1
4Q a
4Q a
2

Para el conductor horizontal ser:


Bhorz 

N0 I
N
(cos 45  cos135 )  0
4Q a
4Q

2 N0 I
I 2

2

2 4Q a
a 2

y el campo magntico en el punto P, habida cuenta de que los tres conductores contribuyen en
la misma direccin (perpendicular al plano del dibujo y saliente), ser:

N I
N 2I
2
2  0
(1 2)
B  2 Bvert Bhorz  0 2 1

4Q a
4Q a
2

- 427 -

Fsica Universitaria: Problemas de Fsica

Campo magntico. E04.13

13. Hallar la corriente que debe pasar por una bobina estrecha, de 15 espiras, y 10 cm de radio, para que en su
centro se produzca un campo magntico igual al campo magntico terrestre en el ecuador, que vale 70 PT.

Cada elemento infinitesimal de la espira contribuye al campo magntico en el centro de la


espira de acuerdo con la expresin
N d l qe r
(ley de Biot-Savart)
dB  0 I
r2
4Q
dl
que, aplicada a una espira de radio R, queda en la forma
N dl
er
dB 0 I 2
4Q R
I
Sumamos los mdulos de todas las contribuciones de los
dB
elementos dl para obtener el campo magntico en el centro de la
B
espira:
Bespira  d B 

N0 I
4Q R 2

2QR

N0 I

d l  4Q R

y para una bobina de N espiras apretadas ser:


N N0 I
B
2R
de modo que
I

2 RB 2q 0.1q 70q106

 0.743 A = 743 mA
N0 N
4Q q107 q15

- 428 -

2Q R 

N0 I
2R

Fsica Universitaria: Problemas de Fsica

Campo magntico. E04.14

14. Un disco aislante de radio R est cargado elctricamente con una carga +Q repartida uniformemente por su
superficie. Calclese el campo magntico B que se origina en su centro al girar alrededor de su eje de
simetra con velocidad angular Z.

Q
QR2
Z
Consideramos una corona circular r y espesor dr, cuya
superficie diferencial es d S  2Qr d r y que posee una
drdr
B
carga elctrica infinitesimal
Q
2Q
dq TdS 
2Q r d r  2 r d r
r
2
QR
R
er r
Esta carga se mueve con una velocidad v, por lo que crea
v=Zu r
un campo magntico dado por la ley de Biot-Savart:
N
v qe
dB  0 dq 2 r
4Q
r
con v   qr   q re r l v qe r  ( q re r )qe r  re r q ( qe r )  r
La densidad de carga superficial es T 

por lo que el campo elemental creado por la rotacin Z de la corona es:


N
NQ
r N 2Q
r
dB  0 dq 2  0 2 r dr 2  0 2 dr
4Q
4Q R
2Q R
r
r
Integrando la expresin anterior, dado que todas las coronas circulares contribuyen en la
misma direccin, obtenemos
R
NQ
N QR
NQ
B  0 2  d r  0 2   0 
0
2Q R
2Q R
2Q R
As, el campo magntico en el centro del disco tiene la mima direccin que Z y viene dado
por la expresin:
NQ
B 0 
2Q R

- 429 -

Fsica Universitaria: Problemas de Fsica

Campo magntico. E04.15

15. Una semicircunferencia, de radio r, de material dielctrico est uniformemente

cargada con una densidad lineal de carga O. Cuando la hacemos girar con
velocidad angular Z constante alrededor de su dimetro, calcular el campo
magntico B que se producir en el punto O.

Descomponemos la semicircunferencia en elementos de longitud dl = r d, con una carga


dq = dl = r d.
Campo magntico creado por una carga (dq) en movimiento (ley de Biot):
dB 
 cos R

con er   sen R
0

N0
v qe
dq 2 r
4Q
r
x

r  y

x  r cos R

y  r sen R

X x 0

La velocidad del elemento de carga dq genrico es: v   qr  0 q y  0

0 0 X y

sen 2 RdR
0  cos R X y sen R

De modo que v qer  0 q sen R  X y cos R  X r  sen R cos RdR

0
0
X y 0

Y sustituyendo en la ley de Biot, obtenemos:


sen 2 RdR
sen 2 RdR

N0 dq
N0 MrdR
N0

dB 
X r  sen R cos RdR 
MX  sen R cos RdR
v q er 
2
2

4Q r
4Q r
4
Q

0
0

Integramos para todo el arco semicircular, entre 0 y :


Q

Q sen 2 RdR  R  sen 2R  Q


0

2
4 0 2

Q
Q
2
sen R cos RdR  sen R  0
2
0

y
dq
r

er

Y el campo resultante en O es
Q / 2

N0
N
MX 0  0 MX i
B
4Q 0 8

dB

dq

er



O
dB

N0M

8
de modo que su direccin es la de , i.e., la del dimetro.

= B

- 430 -

Fsica Universitaria: Problemas de Fsica

Campo magntico. E04.16

16. Por dos conductores rectilneos e indefinidos, situados

perpendicularmente entre s, en un plano, circulan intensidades


constantes I1 e I2, como se indica en la figura. a) Determinar el
vector campo magntico creado por dichas corrientes en un punto
genrico P del plano Oxy. b) En qu punto del plano Oxy el
campo es nulo?

P(x,y)

I2
O

I1

a) Comenzamos determinando, mediante el teorema de


Ampre, el campo magntico a una distancia r de un largo
conductor rectilneo que transporta una intensidad de corriente I. Para ello calculamos la
circulacin de campo B a lo largo de una trayectoria circular (lnea de campo) situada en un
plano perpendicular al conductor:
N I
I
v B dl  v B dl  B v dl  B 2Qr  N0 I l B  2Q0 r
El campo magntico en P(x,y) se obtiene como la superposicin de
los campos magnticos creados por cada uno de los conductores en
r
dicho punto; esto es,
B
N I
N I
N I
I
B  B1 B 2  0 1 k  0 2 k  0 1  2 k
2Q y
2Q x
2Q y x
b) La condicin de que el campo magntico sea nulo nos lleva a escribir
N I
I
I
I
I
B  0 1  2  0 l 1  2  0 l y  1 x

y x
I
2Q y x
2

que es la ecuacin de una recta que pasa por el origen de coordenadas y cuya pendiente es
I1/I2.

- 431 -

Fsica Universitaria: Problemas de Fsica

Campo magntico. E04.17

17. Determinar el flujo magntico a travs de una espira cuadrada de lado a, situada a una
distancia b de un hilo conductor, contenido en su mismo plano, que transporta una
corriente constante I.

I
a
b

Comenzamos determinando, mediante el teorema de Ampre, el campo


magntico a una distancia r de un largo conductor rectilneo que transporta una intensidad de
corriente I. Para ello calculamos la circulacin de campo B a lo
largo
de una trayectoria circular (lnea de campo) situada en un
I
plano perpendicular al conductor:
N 2I
v B dl  v B dl  B v dl  B 2Qr  N0 I l B  4Q0 r

r
As pues, el campo magntico es perpendicular al plano de la espira,
B
su sentido es hacia adentro y su magnitud decrece con la distancia al
hilo rectilneo conductor.
Calculamos el flujo a travs de una franja estrecha, de espesor dr, de
superficie dS = adr, situada a una distancia r del hilo.
a
I
N 2I
N
dr
d' =B dS  B dS  0
a dr  0 2 Ia
r
4Q r
4Q
r
a
e integramos sobre toda la superficie de la espira,
b

dr

b a

'

N0
b a
dr N0
2 Ia

2 Ia ln
Q
r
b
4Q
4
b

- 432 -

Fsica Universitaria: Problemas de Fsica

Induccin magntica. E05.1

1. En una zona del espacio existe un campo magntico uniforme B = -Bk. Una varilla delgada, conductora, de
longitud L est situada paralelamente al eje Oy. Determinar la fuerza electromotriz inducida en la varilla
cuando sta se mueve con velocidad constante v e indicar el extremo de la varilla que estara a mayor
potencial en los siguientes casos: a) v = vi, b) v = vj, c) v = vk.

La f.e.m. inducida en un conductor rectilneo, de longitud l, que se mueve con velocidad v en


un campo magntico uniforme B viene dada por el producto mixto de esas tres magnitudes;
i.e.,
y
+ +
  l < v q B

A + + +
de donde se siguen fcilmente los resultados.
0 l
0
a)   l < v q B
 v 0 0  lvB ,
0 0 B

+
C
+

+ x

o sea, dirigida desde C hacia A (en la direccin de L), por lo que el extremo A estar a mayor
potencial que el C.
0 l
0
b)   l < v q B
 0 v 0  0
0 0 B

por lo que todos los puntos de la varilla se encuentran al mismo potencial.


0 l
0
a)   l < v q B
 0 0 v  0
0 0 B

por lo que todos los puntos de la varilla se encuentran al mismo potencial.

- 433 -

Fsica Universitaria: Problemas de Fsica

Induccin magntica. E05.2

2. Por un conductor rectilneo e indefinido circula una intensidad I. Un segundo conductor de longitud l, est
situado perpendicularmente al primer conductor y se desplaza en la misma direccin y sentido que la
intensidad con una velocidad v. Determinar la diferencia de potencial inducida que se origina entre los
extremos del segundo conductor.

Comenzamos determinando, mediante el teorema de Ampre, el campo


magntico a una distancia r de un largo conductor rectilneo que
I
transporta una intensidad de corriente I. Para ello calculamos la
circulacin de campo B a lo largo de una trayectoria circular (lnea de
r
campo) situada en un plano perpendicular al conductor:
B
NI
v B<dl  v Bdl  B v dl  B 2Qr  N0 I l B  2Q0 r
Como el conductor MN se mueve en el
campo magntico creado por el otro conductor, se induce en el
u
u
u
u
u
una f.e.m.. Como el campo magntico B no es constante en todo
el conductor MN, consideramos un elemento de longitud dr, en
v
I u uB u
u
u
el que la f.e.m. inducida ser
d  dl <( v qB)  v B d r
ru
u M
u dr u
u
N

ua u

ul

u

en la direccin del producto vectorial de vuB; esto es, de N


hacia M, como se ilustra en la figura.
La f.e.m. inducida en todo el conductor se obtiene por
integracin:
  vB dr  B

N0 Iv
2Q

a l

d r N0 Iv a l
ln

r
a
2Q

La diferencia de potencial entre los extremos del conductor en movimiento coincide con el
valor de la f.e.m. anteriormente calculada, estando el extremo M a mayor potencial que el N.

- 434 -

Fsica Universitaria: Problemas de Fsica

Induccin magntica. E05.3

3. Una varilla conductora, de longitud L, est girando con velocidad angular constante

(Z) alrededor de un eje fijo perpendicular a ella y que pasa por su extremo. En el
mismo plano de rotacin de la varilla, un conductor rectilneo indefinido transporta
una corriente elctrica constante (I) y pasa por el extremo de la varilla. Determinar la
f.e.m. inducida en la varilla en funcin del tiempo.

I
L

De conformidad con la notacin que se indica en la figura, el campo magntico creado por
una corriente rectilnea indefinida en un elemento del conductor viene dado por
N I N
I
B 0  0
2Q x 2Q r cos Xt
La velocidad de dicho elemento es v = r, en la direccin que se indica en la figura. Como
consecuencia de su movimiento, en el elemento del conductor
se induce una f.e.m. dada por la expresin
B
N
I
d  dr ( v q B)  v B dr  X r 0
dr

v
2Q r cos Xt
dr
N IX
x
= d   0
dr
2Q cos R
d
r
e integrando a lo largo de toda la varilla, para tener en cuenta
I
las contribuciones de todos los elementos, obtenemos la f.e.m.
Zt
inducida en la varilla:
L
N IX
N IXL
  0
dr   0

2Q cos R 0
2Q cos R
en la direccin que se indica en la figura.

- 435 -

Fsica Universitaria: Problemas de Fsica

Induccin magntica. E05.4

4. Una varilla conductora, de longitud L, est girando con velocidad angular


constante () alrededor de un eje fijo perpendicular a ella y que pasa por su
centro. En el mismo plano de rotacin de la varilla, un conductor rectilneo
indefinido transporta una corriente elctrica constante (I). Determinar la f.e.m.
inducida en la varilla en el instante que se indica en la figura.

I
a

El campo magntico creado por un conductor rectilneo indefinido, a una distancia h del
N 2I
y
y, en la
mismo viene dado por la expresin B  0
Q h
4
b
v
figura adjunta, es perpendicular al plano del dibujo,
B
h
entrante a la derecha del hilo conductor. Por consiguiente,
en un elemento infinitesimal de la varilla, de longitud dx,


x
situado a una distancia x de su centro, la magnitud del
a
L
N 2I
. Puesto que dicho
campo magntico es B  0
4
Q b x
I
elemento se est moviendo con una velocidad v = x,
generar una f.e.m. inducida elemental dada por
N 2I X
N X I x dx
d  dl ( v q B)  dx vB   0
x dx   0
4Q b x
4Q b x
en la direccin indicada en la figura. Integrando la expresin anterior, para sumar las
contribuciones de todos los elementos de la varilla, tenemos
 

N0 X I
4Q

L /2

N XI
x dx
b x   40 Q
L / 2

L/ 2

N XI
h b
h dh   40 Q
 L /2

a L

1 h dh  
a

N0 X I
a L

L  b ln

4Q
a

habiendo utilizado el siguiente cambio da variable h  b x l dh  dx . El resultado anterior


lo rescribimos en la forma:

N XI
a L
1
  0 (a L) ln
 L

4Q
a
2
Otro mtodo: Ley de Faraday
N0 I
r dr dR
2Q b r
N
N X I r dr
d'
dR
I
d  
 0
 0
r dr
dt
2Q b r
dt
2Q b r

dS  (rdR ) dr  r dr dR


d
r

dS
dr

 

N0 X I
2Q

L /2

d'  B dS 

N XI
r dr
 ...  0

4Q
b
r
L / 2

- 436 -

(a 1 L) ln a L  L

2
a

Fsica Universitaria: Problemas de Fsica

Induccin magntica. E05.5


z

5. La espira rectangular de base b = 0.2 m y altura h = 0.5 m de la figura se traslada


con velocidad constante v = 3 j (m/s) en una regin del espacio donde existe un
campo magntico no homogneo dado por B = y i (T). Determinar la fuerza
electromotriz inducida en la espira, indicando en un esquema el sentido de la
corriente que se origina.

h
b

y
x

La f.e.m. inducida sobre un conductor rectilneo, de longitud l, que se mueve se mueve con
una velocidad v en un campo magntico B, viene dada por la expresin:
  l ( v q B)
y su sentido es el del producto vectorial vB.
En el caso en que los tres vectores sean ortogonales entre s, la expresin anterior se reduce a
 l v B
Las f.e.m. inducidas sobre los lados superior e inferior de la espira son
nulos; ya que el producto mixto es nulo por ser l v.
Las f.e.m. inducidas sobre los lados laterales de la espira tienen el
sentido que se indica en la figura, siendo sus magnitudes:
1  hv B1  hv y
2  hv B2  hv ( y b)

v
1

2

lo que da como resultado una f.e.m. neta en la espira en el sentido


horario, tal como se ilustra en la figura, siendo su magnitud:
  2  1  hv ( y b)  hv y  hb v  Sv
donde S es la superficie de la espira.
La espira estar recorrida por una corriente elctrica inducida en el sentido horario, cuya
intensidad ser
 Sv
I 
R
R
siendo R la resistencia de la espira.

- 437 -

Fsica Universitaria: Problemas de Fsica

Induccin magntica. E05.6

6. Una bobina rectangular de 80 vueltas, 20 cm de anchura y 30 cm de


longitud est situada en un campo magntico B = 0.8 T perpendicular al
plano de la bobina y dirigido hacia dentro de la pgina. Como se indica en
la figura, tan slo la mitad de la bobina se encuentra en la regin del campo
magntico. La resistencia de la bobina es de 30 :. Determinar la magnitud
y direccin de la corriente inducida al desplazarse la bobina con una
velocidad de 2 m/s en los siguientes casos: a) hacia la derecha, b) hacia
arriba y c) hacia abajo.

+
+
+
+
+

+
+
+
+
+

+
+
+
+
+

+
+
+
+
+

+
+
+
+
+

80 vueltas

+
+
+
+
+

30cm
20cm

d'
dt
a) En este caso el flujo a travs de la bobina permanece constante, por lo que ser:
 0
I 0
Ley de la induccin de Faraday:   N

h
b) El flujo '  BS  Blx  Bl ( vt ) vara (aumenta) con el tiempo,
2
de modo que
d'
 NBlv  80q 0.8q 0.20q 2  25.6 V
dt
 25.6
I 
 0.853 A { (ley de Lenz)
R
30
h
c) El flujo '  BS  Blx  Bl (  vt ) vara (disminuye) con el tiempo,
2
de modo que
N

d'
 NBlv  80q 0.8q 0.20q 2  25.6 V
dt
 25.6
I 
 0.853 A z (ley de Lenz)
R
30

N

+
+
+
+
+

+
+
+
+
+

+
+
+
+
+

+ +
+ v+
l+ +
+ +
I+ +

+
+
+
+
+

+
+
+
+
+

+
+
+
+
+

+
+
+
+
+

+ +
+v+
l+ +
+ +
I+ +

+
+
+
+
+

Otro mtodo:
A partir de la expresin de la f.e.m. inducida sobre un conductor rectilneo en movimiento en
un campo magntico,   l ( v q B) (producto mixto), se siguen fcilmente los siguientes
resultados:
a)   0 , por ser l & v (el producto mixto es nulo)
b)
  NlvB  80q 0.20q 2q 0.8  25.6 V
 25.6
 0.853 A
I 
R
30
c)
   NlvB  80q 0.20q 2q 0.8   25.6 V
 25.6
  0.853 A
I 
R
30

- 438 -

v
l

-I
+I

-v

Fsica Universitaria: Problemas de Fsica

Induccin magntica. E05.7

7. Un conductor rectilneo indefinido y una espira cuadrada estn situados en un


mismo plano. La espira, de lado a, tiene el lado ms cercano al conductor paralelo al
mismo y a una distancia igual a su lado. Determnese la fuerza electromotriz
inducida en la espira cuando por el conductor rectilneo circula una corriente

i  I sen Xt .

r
B

u
u
u
u
u
u
u
u
a

El teorema de Ampre nos permite determinar la intensidad del campo


magntico a una distancia r de un largo conductor rectilneo que
transporta una intensidad de corriente i. Para ello, calculamos la
circulacin de campo B a lo largo de una trayectoria circular (lnea de
campo) situada en un plano perpendicular al conductor:
N i
v B<dl  v Bdl  B v dl  B 2Qr  N0i l B  2Q0 r
El campo magntico es perpendicular al plano de la espira, su sentido es
hacia adentro y su magnitud decrece con la distancia
al hilo rectilneo conductor.
u u au u
u
Calculamos el flujo a travs de una franja estrecha, de
u u u u
u
espesor dr, de superficie dS = adr, situada a una
u u u u
u
distancia r del hilo.
u u u u
u
a
N i
N
dr
u u u u
u
d' =B dS  B dS  0 a dr  0 ia
2Q r
2Q r
u u u u
u
u
u u u u
e integramos sobre toda la superficie de la espira,

u
u
u
ru
u
u
u
u u dr
u

'

2a
N

N0
dr N0
ia

ia ln 2  0 Ia ln 2 sen Xt
2Q

2Q a r
2Q

De acuerdo con la ley de Faraday, la f.e.m. inducida en la espira es


N
d
N

d'
 
  0 Ia ln 2 sen Xt   0 I X a ln 2 cos Xt

2Q
dt
2Q

dt

- 439 -

Fsica Universitaria: Problemas de Fsica

Induccin magntica. E05.8

8. Una espira cuadrada de lado a, tiene una resistencia R, y penetra perpendicularmente


en una franja donde existe un campo magntico B, con una velocidad v que forma un
ngulo de 45 con el lmite de la franja. Determnese la fuerza que hay que realizar
cuando penetre en dicha franja, y la fuerza al salir de la misma. Indquese el sentido
de la intensidad inducida y la fuerza en ambos casos.

v
u u u u u u
u u u u u u

Fuerza electromotriz inducida por un conductor rectilneo de longitud l que se mueve con una
velocidad v en un campo magntico B:
ind  l ( v q B)
Fuerza que ejerce un campo magntico B sobre un conductor rectilneo, de longitud l, que
transporta una corriente I:
F  I (l q B )
En ambos casos, tan solo la componente vertical de la velocidad, i.e.
la corriente inducida.
Entrando

2v / 2 , contribuye a

2
2
vB 
avB (de izquierda a derecha)
2
2

2 avB
I ind  ind 
(sentido antihorario)
R
2 R
2 a 2 vB 2
(sentido hacia arriba)
F  I ind aB 
R
2
Fap  F (hacia abajo, se opone a entrar)

ind  a

I
u
u
u
u
u

u
u
u
u
u

u
u
u
u
u

u
u
u
u
uF

uF u
u u
u u
u u
u u

ap

u u
u u
uu
u u
u2 u
v
2

u
u
u
u
u

u
u
u
u
u

u
u
u
u
u

Saliendo

2
vB 
2

I ind  ind 
R

2
avB (de izquierda a derecha)
2
2 avB
(sentido horario)
2 R
2 a 2 vB 2
F  I ind aB 
(sentido hacia arriba)
R
2
Fap  F (hacia abajo, se opone a salir)

ind  a
u
u
u
u
u

u
u
u
u
u

u
u
u
u
u

u
u
u
u
u

uF u u
u u u
u u u
u u u
u u u

u
u
u
u
u

u
u
u
u
u

u
u
u
u
u
I

u
u
u
u
u

Nota: Las fuerzas sobre los lados laterales son iguales y


opuestas, por lo que tienen una resultante nula.

I
Fap

- 440 -

Fsica Universitaria: Problemas de Fsica

Induccin magntica. E05.9


z

9. En una regin del espacio existe un campo magntico giratorio cuya


expresin en funcin del tiempo viene dada por

B  B0 cos Xt j B0 sen Xt k

Una espira cuadrada de lado a gira alrededor del eje Ox con igual velocidad
angular Z. Determinar la f.e.m. inducida si: a) el sentido de giro de la espira
coincide con el del campo magntico, b) el sentido es opuesto.

Expresamos vectorialmente la superficie contorneada por la espira; esto es


S  S cos(oXt ) j S sen (oXt ) k  S cos Xt j o S sen Xt k
donde el doble signo de la velocidad corresponde a las dos posibilidades previstas en el
enunciado del problema.
El flujo a travs de la espira es


0
0

B0 S  cte


'  B S  B0 cos Xt < S cos Xt  B0 S (cos 2 Xt o sen 2 Xt ) 
B0 S cos 2Xt
B sen Xt o S sen Xt
0
Y la f.e.m. inducida viene dada por
d'
d
ind = 
  B0 S (cos 2 Xt o sen 2 Xt )   X B0 S (2 cos Xt senXt o 2senXt cos Xt )
dt
dt

0
= ind  2X B0 S (cos Xt senXt B senXt cos Xt ) 

2X B0 Ssen2Xt
a) El sentido de giro de la espira coincide con el del campo magntico:
ind  2X B0 S (cos Xt senXt  senXt cos Xt )  0

lo que resulta obvio, ya que al girar la espira con la misma velocidad angular y en el mismo
sentido con que lo hace el vector campo magntico, el flujo a travs de ella permanece
constante.
b) El sentido de giro de la espira es opuesto al del campo magntico:
ind  2X B0 S (cos Xt senXt senXt cos Xt )  2X B0 S (2sen Xt cos Xt ) 
 2X B0 a 2 sen(2Xt )

lo que tambin resulta obvio, ya que el resultado anterior se puede escribir en la


forma ind  2X B0 a 2 sen(2Xt )  8 B0 a 2 sen 8t , ya que al girar la espira con la misma
velocidad angular, pero en sentido opuesto al de rotacin del vector campo magntico,
equivale a una rotacin de la espira con una velocidad angular :  = 2Z en un campo
magntico estacionario.

- 441 -

Fsica Universitaria: Problemas de Fsica

Induccin magntica. E05.10

10. La espira de forma cuadrada, de lado a y resistencia elctrica R, que se muestra


en la figura, gira alrededor del eje x con velocidad angular constante . En el
instante t = 0, su posicin es la representada en la figura. Supongamos que
existe un campo magntico no uniforme de direccin constante y paralela a la
del eje z, tal que su mdulo venga dado por B = Ky, donde K es una constante
positiva. a) Determinar la intensidad que circula por la espira en un instante
genrico t; i.e., i(t). b) Representar el sentido de la intensidad cuando la espira se
encuentra en cada uno de los cuadrantes que recorre en su movimiento.

a) La f.e.m. inducida sobre un conductor rectilneo, de longitud l, que se mueve con una
velocidad v en un campo magntico uniforme B viene dada por
  l < ( v q B)

Puesto que la f.e.m. tiene el sentido definido por el producto


vectorial ( v q B) , tan solo se induce f.e.m. a lo largo del lado
de la espira opuesto al eje de rotacin. Adems, el campo
magntico, aunque no es uniforme, presenta un valor constante
a lo largo de ese lado (conductor), por lo que podemos aplicar
la expresin anterior; esto es,
  avB sen Xt  a aX
Ka cos Xt
sen Xt  Ka 3X sen Xt cos Xt 

1 3
Ka X sen 2Xt

Otro mtodo
Como la espira se encuentra en movimiento (rotacin), el flujo que la atraviesa
vara en el transcurso del tiempo; esta variacin del flujo produce una f.e.m.
inducida en la espira.
En un instante genrico t, la posicin de la espira estar determinada por el
ngulo t. Para calcular el flujo que la atraviesa en ese instante, observamos que
el campo magntico no tiene la misma intensidad en todos los puntos de la
superficie de la espira. En consecuencia, procedemos por integracin,
descomponiendo la superficie de la espira en bandas estrechas de espesor
infinitesimal d, tal como se ilustra en la figura, y superficie dS = ad, en las que
el campo magntico tiene un valor constante B = Ky = Kcost. As, obtenemos
a

'  B<dS  BdS cos Xt  K I cos Xt


adI
cos Xt  Ka cos 2 Xt IdI 
0

1 3
Ka cos 2 Xt
2

Aplicamos la ley de Faraday para determinar la f.e.m. inducida en la bobina en un instante genrico,

( ind  

d' 1 3
1
 Ka X 2 cos Xt sen Xt  Ka 3X sen 2Xt

dt
2
2

La intensidad de la corriente que circula por la espira en un


instante genrico es
ind 1 Ka 3X
sen 2Xt


R
2 R
b) De acuerdo con la regla de Lenz y con la del producto
( v q B) , el sentido de la corriente inducida es la indicada para
cada uno de los cuadrantes.
iind 

- 442 -

Fsica Universitaria: Problemas de Fsica

Induccin magntica. E05.11

11. Una bobina rectangular, de 80 vueltas y de dimensiones 20 cm u 30 cm, est


situada en un campo magntico B = 0.8 T dirigido hacia dentro de la pgina.
Como indica la figura, slo la mitad de la bobina se encuentra en la regin del
campo magntico. La resistencia elctrica de la bobina es de 30 :. Determinar la
magnitud y direccin de la corriente inducida al desplazarse la bobina con una
velocidad de 2 m/s (a) hacia la derecha, (b) hacia arriba y (c) hacia abajo.

u
u
u
u

La f.e.m. inducida en un conductor rectilneo en movimiento en un


campo magntico externo viene dada por el producto mixto   l < ( v q B) .

u
u
u
u

u
u
u
u

u
u
u
u
b =30cm

a =20cm

En los tres supuestos considerados, tan solo contribuyen a la f.e.m. inducida neta los
conductores situados en el lado superior de la bobina, en los que
a


0

l  0
B  0
u u u u

0
B

u u lu u
u u u u
u u u u
+I
y

v

a) En este caso ser v  0 , por lo que el producto mixto ser
0
nulo ( l & v ). As,

 0 l I 0

0

b) En este caso ser v  v , por lo que
0

a =20cm

a 0 0
a vB


NavB

  N 0<v q 0  N 0< 0  NavB l I 

R
0 0 B
0 0

  NavB  80q 0.20q 2q 0.8  25.6 V l I 

25.6
 0.85 A
30

en el sentido antihorario.
0

c) En este caso ser v  v , por lo que
0
a 0 0
a vB


NavB
  N 0<vq 0  N 0< 0  NavB l I 

R
0 0 B
0 0

  NavB  80q 0.20q 2q 0.8  25.6 V l I 

en el sentido horario.

- 443 -

25.6
 0.85 A
30

Fsica Universitaria: Problemas de Fsica

Induccin magntica. E05.12

12. Una espira conductora circular elstica se expansiona a una velocidad constante, de modo que su radio viene
dado por R = R0 + vt. La espira se encuentra en una regin de campo magntico constante perpendicular a la
misma. Determinar la fuerza electromotriz generada en la espira. Despreciar los efectos posibles de
autoinduccin.

La fuerza electromotriz inducida en la espira viene dada por la expresin:


d'
(ley de induccin de Faraday)
 
dt
con
'  B S  BQ R0 vt

de modo que
 

d
d
2
2
BQ R0 vt
 Q B R0 vt
 2Q Bv R0 vt

dt
dt

Esto es,
  2Q Bv R0 vt

en el sentido antihorario (ley de Lenz).

- 444 -

uuuuuuu
B
uuuuuu
uuuuuuu
R
uuuuuu
uuuuuuu
uuuuuu

Fsica Universitaria: Problemas de Fsica

Induccin magntica. E05.13

13. Un anillo delgado y conductor, de radio r, se encuentra en un campo magntico uniforme de direccin
perpendicular al plano del anillo y que vara con el tiempo segn la ley B = kt, donde k es una constante
positiva. Determinar la intensidad del campo elctrico en el anillo (su mdulo y su sentido). Explicar la
naturaleza de dicho campo elctrico.

Calculamos el flujo magntico a travs del anillo

'  BS  Qr 2 kt
Puesto que el flujo vara con el tiempo, se induce en el anillo una f.e.m. que viene dada por la
ley de Faraday:
d'
 
 Qr 2 k
dt
u u u u u u u u u u u
u u u u u u u u u u u
y tiene el sentido que se indica en la figura (antihorario).
u u u u u u u u u u u
La f.e.m. se define como la circulacin del campo u u u u u u u u u u u
E
elctrico (no-electrosttico) a lo largo de todo el anillo. u u u u u u u u une u u
Calculamos dicha circulacin a lo largo de una lnea de u u u u u u u u u u u
u u u u u Bu u u u u u
campo:

v Ene <dl  v Enedl  Ene v dl  2QrEne

de modo que

Qr 2 k
kr


2Qr
2Qr
2
y tiene el mismo sentido que la f.e.m.

Ene 

- 445 -

u
u
u
u
u
u
u

u
u
u
u
u
u
u

u
u
u
u
u
u
u

u
u
u
u
u
u
u

u
u
u
u
u
u
u

u
u
u
u
u
u
u

u
u
u
u
u
u
u

u u
u u
fem
u u
u u
u u
u u
u u

u
u
u
u
u
u
u

u
u
u
u
u
u
u

Fsica Universitaria: Problemas de Fsica

Induccin magntica. E05.14

14. Una varilla metlica gira con velocidad angular constante alrededor de un eje per- .
pendicular a ella y que pasa por uno de sus extremos, deslizando sobre un anillo .
conductor de radio l, como se esquematiza en la figura. El eje de la varilla est .
conectado al borne positivo de un generador de f.e.m., cuyo borne negativo est .
conectado al anillo. Si existe un campo magntico uniforme perpendicular al plano
del anillo, determinar la velocidad angular que adquirir la varilla y el sentido de .

.
.
.B
.
.

. .
. .
. .

. .
. .

.
.
l
.
.
.

.
.
.
.
.

sta.

El generador produce una corriente elctrica, en el sentido indicado en la figura, que


tomaremos como sentido positivo.
La varilla se encuentra en un campo magntico externo (B) y est recorrida por la intensidad
de corriente I. En consecuencia, la varilla est sometida a una fuerza F  I l q B
,
uniformemente distribuida que da lugar a un momento resultante que la hace girar en el
sentido horario con velocidad angular creciente (movimiento acelerado de rotacin).
Como consecuencia del movimiento de la varilla conductora en un campo magntico externo
(B), se induce en ella una cierta f.e.m.. El campo no-electrosttico en la varilla, dado por,
Ene  v qB , est dirigido a lo largo de la varilla, hacia el extremo fijo de la misma (sentido
negativo). Cada elemento de la varilla en rotacin tiene una velocidad distinta, esto es, v = rZ,
siendo r la distancia del elemento al extremo fijo de la varilla. Calculamos la f.e.m. inducida
mediante integracin:
l
1
ind  Ene <dl  v qB
<dl   vB dr  X B r dr   X Bl 2
cond
cond
cond
0
2
La varilla quedar libre de la accin de fuerzas y momentos cuando alcance una cierta
velocidad angular tal que la f.e.m. inducida sea igual (y opuesta) a la del generador, ya que
entonces la intensidad de la corriente ser nula. Esto es,
 ind  IR  0 l

  ind

 ind  IR  0 l

  ind

de modo que

.
.
.
.
.

. .
. .
B
. .
. .
. .

.
.l
.

.
.

. .
. .
.v F.
. .
. .

ind

ind


I

- 446 -

Fsica Universitaria: Problemas de Fsica

Induccin magntica. E05.15

15. Una bobina muy larga tiene 1000 espiras/m de longitud y est recorrida por la una intensidad i = 3 cos 100 t
(en unidades del S.I.) En su interior y sobre su mismo eje, colocamos una pequea bobina de 1 cm de radio
y 50 espiras independientes de las de la bobina larga. a) Calcular la intensidad del campo magntico en el
interior de la bobina larga. b) Determinar el valor del flujo magntico a travs de la bobina pequea.
c) Calcular la f.e.m. inducida en la bobina pequea.

a) Determinamos el campo magntico B en el interior de


la bobina o solenoide largo a partir del Teorema de
Ampre, calculando la circulacin de B a lo largo de una
lnea de campo:

B
N

S
l

v B<dr  v B dl  B v dl  Bl  N Ni

B dbil

de donde
N0 Ni
 N0 ni  N0 nI cos Xt 
l
 4Q q107 q1000q3cos100t  0.0012Q cos100t

B

= B  3.77 q103 cos100t (S.I.)


que puede considerarse uniforme en el interior de la bobina.
b) Flujo ligado a travs de la bobina pequea:
'  NBS  50q 3.77 q103 cos100t
Q 0.01
 5.92q105 cos100t (S.I.)
2

c) Determinamos la f.e.m. a partir de la Ley de la Induccin de Faraday:


d'
d
 
  5.92q105 cos100t
 5.92q103 sen100t (S.I.)
dt
dt
de modo que mx  5.92 mV .

- 447 -

Fsica Universitaria: Problemas de Fsica

Induccin magntica. E05.16

16. Determinar el coeficiente de autoinduccin de un solenoide trico constituido por N espiras. El radio medio
del toro es R y su seccin cuadrada de lado a.

Determinamos el campo magntico B en el interior del solenoide a


partir del Teorema de Ampre, calculando la circulacin de B a lo
largo de la lnea de campo circular de radio R:
R

v B<dr  v B dl  B v dl  B 2Q R  N NI
0

de donde
N0 NI N0 NI

2Q R
l
El flujo ligado en el solenoide ser

B

N0 N 2 S
N N 2S
I 0
I
2Q R
l
El coeficiente de autoinduccin se define como el flujo ligado por unidad de intensidad de
corriente:

N '  NSB 

L

( N ' ) N0 N 2 S N0 N 2 S


I
l
2Q R

o bien, con S = a2, tenemos

L

( N ' ) N0 N 2 a 2 N0 N 2 a 2


I
l
2Q R

a
2

a
2

a
Eje del toroide

Clculo ms detallado.
La intensidad del campo magntico B no es constante en
toda la seccin del solenoide toroidal, sino que decrece
con la distancia r al eje de simetra de rotacin del
mismo, ya que
N NI N NI
B 0  0
2Qr
l
por lo que el clculo correcto del flujo a travs de una
espira se debe hacer mediante integracin; i.e.,

dS
R

r
espira

dr
a

R
N NIa 2 R a
dr N NIa
'
a r  02Q ln 2a  02Q ln 2R  a
S
R
R
R
2
2
2
de modo que el flujo ligado y la autoinduccin vienen dados por

N NI
N NIa
B<dS  0 adr  0
2
2Q
Q
r
a

N' 

N0 N 2 Ia 2 R a
( N ' ) N0 N 2 a 2 R a
ln
ln
l L

2Q
2R  a
2Q
2R  a
I

- 448 -

Fsica Universitaria: Problemas de Fsica

Induccin magntica. E05.17


12V 12:

17. En el esquema de la figura, determinar: a) La intensidad que pasa por cada rama en el
instante inicial (t = 0), cuando se conecta la batera. b) La intensidad que pasa por cada
rama cuando alcanza el estado estacionario (t = f).

4:
10PF

q
0.
C
Al ser VAB  0 por la resistencia de 4 : no pasar corriente.

10PH

a) Inicialmente el condensador est descargado por lo que VAB 


B

12V 12:

di
 0 , luego en ese instante i no
dt
vara; como antes era nula, ahora seguir sindolo.
En definitiva, la corriente circula tal como se indica en la figura, a travs de
la resistencia de 12 : y del condensador (que deja pasar), siendo su
intensidad
En la rama de la bobina ser VAB  L

4:
10PF
10PH

 12
 1 A
R 12
siendo nula en los otros elementos.
4:
b) Una vez haya transcurrido un tiempo suficientemente largo (t o f), se
alcanza el estado estacionario y todas esas magnitudes permanecern
10PF
constantes.
di
di
10PH
En la rama de la bobina ser
 0 l VAB  L  0 y la bobina deja
dt
dt
pasar la corriente.
Por ser VAB  0 , la intensidad por la rama de la resistencia de 4 : ser nula. Por la misma
razn ser nula la intensidad en la rama del condensador, ya que habrn pasado todos los
efectos transitorios de carga del condensador.
En definitiva, la corriente circula tal como se indica en la figura, a travs de la resistencia de
12 : y de la bobina (que deja pasar), siendo su intensidad
 12
I   1 A
R 12
siendo nula en los otros elementos.
B

12V 12:

I

- 449 -

Fsica Universitaria: Problemas de Fsica

Induccin magntica. E05.18

18. Consideremos un circuito RL en serie conectado a un generador de f.e.m. en


corriente continua. En el instante t = 0 se cierra el interruptor S y se inicia la
corriente de cierre en el circuito. a) Expresar la intensidad de la corriente de cierre
en funcin del tiempo. b) Definir la constante de tiempo de circuito RL y explicar
su significado. c) Explicar el comportamiento del circuito RL a partir de los
elementos resistencia y autoinduccin.

a) En el instante inicial el interruptor (S) est abierto y la


S
autoinduccin est descargada. Al conectar el generador
(t = 0), comienza a circular la corriente.
Calculamos la d.d.p. entre a y b a travs de los elementos pasivos
y a travs de generador (de resistencia interna nula):
i
di
Vab  vR vL  Ri L  
R
dt
Separamos las variables i y t:
a
di
di
dt
L    iR l

  iR L
dt
E integramos
u   / R
 i
i
di
R t
0  / R
 i  L 0 dt utilizando el cambio de variable: du  di


ln

i
L
c

ind

i
 / R
 i R
du
  ln u   ln  / R
 i 0   ln
 t
u
L
 / R

R
R
 t
 t
 / R
 i
R
i
i
  t l 1
 e L l 1 e L 
L
 / R

 / R

 / R


t  0 i  0

i  I f 1 e1
 0.63 I f
t  U


t  d i  I f 
R

b) Constante de tiempo (  ): tiempo que deber transcurrir en el proceso de cierre hasta que la
intensidad de la corriente alcanza el 63% de su valor final.
c) Al aumentar la intensidad de la corriente,
i
se induce una f.e.m. en la L tal que se opone
If
al aumento de la intensidad, por lo que
i (t )  I f 1  et / RC
retrasa el aumento de est; la L se comporta 0.63If
como un elemento de inercia que almacena
energa en el campo magntico que se va
creando en la autoinduccin a medida que
pasa la corriente.


t / U
i  1 e

R
=

U  L / R

- 450 -

Fsica Universitaria: Problemas de Fsica

Induccin magntica. E05.19

19. Un circuito serie, alimentado con tensin alterna de 125 V y 50 Hz de frecuencia, est formado por una
resistencia de 8 :, una autoinduccin 12 : de reactancia y un condensador de 6 : de reactancia. a) Qu
intensidad circular por el circuito? b) Si la corriente alterna vara su frecuencia a 25 Hz, sin variar su
tensin, qu intensidad circular por el circuito? c) Determinar elemento en paralelo necesario para corregir
completamente el factor de potencia, en cada caso.

a)
Tensin alterna de 125 V y 50 Hz de frecuencia
X  2QO  100Q rad/s

'  8 12 j - 6j = 8 + 6j = 10 36.9 8 (inductivo)


=

# 125 0

 12.5 36.9  10  7.5 j A
' 10 36.9

8j :

-6j :

125 V 50 Hz

b) Tensin alterna de 125 V y 25 Hz de frecuencia


X a  2QO  50Q rad/s
X La X a L X a 1


 
= X La  6 8

2
XL
X
X La

X Ca  XC  X  2 = X a  12 8

X Ca X aC X a
' a  8 6 j -12 j  8 - 6 j  10 -36.9 8 (capacitativo)
a 

12 :

125 0
#a

 12.5 36.9  10 7.5 j A
' a 10 36.9

12 :

6j :

-12j :

125 V 25 Hz

c1) Como el circuito es inductivo, hay que colocar un condensador en paralelo.


V
I react  I sen G 
 X CV
XC
I sen G
7.5

 191q106 F = 191 F
C
XV
100Q q125
c2) Como el circuito es capacitativo, hay que colocar una autoinduccin en paralelo.
V
V

I react  I sen G 
X L XL
V
125

 106q103 H = 106 mH
L
X I sen G 50Q q 7.5

- 451 -

Fsica Universitaria: Problemas de Fsica

Corriente alterna. E06.1

1. Para determinar la resistencia Rb y la autoinduccin Lb de una bobina se la coloca en serie con una resistencia pura y calibrada de 1 k y se miden las cadas de tensin en esta resistencia, en la bobina y en el
circuito serie completo, obtenindose los siguientes resultados a 50 Hz: 180 V, 50 V y 220 V, respectivamente. a) Dibujar el diagrama fasorial de tensiones. b) Determinar la resistencia Rb y la autoinduccin Lb de
la bobina.

a) La intensidad de la corriente (I) es la misma en la


resistencia y en la bobina.
En la resistencia, la intensidad y la tensin estn en
fase.
En la bobina (impedancia inductiva), la intensidad
est retrasada respecto de la tensin; o lo que lo
mismo, la tensin est adelantada respecto de la
intensidad.
En consecuencia, el diagrama fasorial de tensiones es
el que mostramos en la figura.
b) Puesto que #  #R #b (suma fasorial), ser
V 2  VR2 Vb2 2VRVb cos G l cos G 

Z
bobina

VR

Vb
V
V=220 V

I
VR=180V

Vb=50V
I

V 2 VR2 Vb2 2202 1802  502



 0.75
2VRVb
2q180q50

de modo que la tensin en la bobina est adelantada G  41.4 respecto de la intensidad.


La intensidad de la corriente es

I

VR
180

 180 mA
R 1000

La impedancia de la bobina es
'b 

50 41.4
#b

 278 41.4  208 184 j 
i
0.180 0

de modo que su resistencia y autoinduccin valen:


Rb  208 
Lb 

Xb
184

 585 mH
X
2Q q50

- 452 -

Fsica Universitaria: Problemas de Fsica

Corriente alterna. E06.2

2. En una red de 220 V, 50 Hz, se desea instalar una lmpara incandescente especificada para consumir una
potencia de 60 W a una tensin mxima de 120 V. Para conectar esta lmpara a la red antes indicada se
pretende instalar un condensador en serie con la misma. a) Qu capacidad deber tener dicho condensador?
b) Si se variara la frecuencia a 60 Hz, qu potencia consumira la lmpara antes y despus de colocar el
mismo condensador?

a) Determinamos la intensidad de trabajo de la lmpara y su resistencia elctrica a partir de


los valores nominales de su potencia y tensin de trabajo:
Pnom 60
P VI o I nom
0.5 A
Vnom 120

V
R

o R

Vnom
I nom

120
0.5

240 :

Si conectsemos la lmpara directamente a la red de 220 V, la


potencia que consumira sera
2

I = 0.5 A

R = 240 :

120 V,
50 Hz

V
220

 202 W
R
240
muy superior a la potencia nominal (60 W) y la lmpara se fundira.
Para evitarlo, colocamos una reactancia en serie con la lmpara para aumentar la impedancia
de la rama y disminuir la intensidad de la corriente a travs de ella.
Determinamos el valor de la reactancia apropiada para limitar el paso de intensidad por la
lmpara:
V
V
220
I nom 
l Z

 440 
Z
I nom
0.5
P

Z 2  R2 X 2

X  Z 2  R 2  4402  2402  369 

Conocida la reactancia, determinamos la capacidad del


condensador:
1
1
1
X
l C

 8.63q106 F  8.63 F
XC
X X 100Q q369

I = 0.5 A

R = 240 :

220 V,
50 Hz

b) Si variamos la frecuencia, manteniendo el mismo condensador, variar la reactancia y la


intensidad que circula por la resistencia.
1
100Q q369

 308  l Z a  R 2 X a 2  2402 3082  369  l
Xa
X aC
120Q
220
V

 0.60 A l P a  I a 2 R  0.602 q 240  86 W
Ia 
Z a 369
lo que resulta excesivo por superar en un 44% a la potencia nominal de la lmpara (60 W) por
lo que sta se fundir a corto plazo.

- 453 -

Fsica Universitaria: Problemas de Fsica

Corriente alterna. E06.3

3. Con corriente alterna monofsica de 220 V y 50 Hz se alimenta una


carga. Al conectarle en paralelo un condensador de 891 F se
corrige el factor de potencia desde 0.4 a 0.8. a) Determinar la
intensidad que consume dicha carga. b) Calcular la capacidad del
condensador que debiera sustituir al anterior para corregir totalmente
el factor de potencia.

220 V
50 Hz

IC

Clculos previos:
X  2Q q50  100Q rad/s
C  0.891 mF  891 F
cos G  0.4 l G  66.4

XC  100Q q 0.891q103  0.28  -1


cos G  0.8 l G  36.9

a) Deduccin de la expresin para correccin parcial del factor de


potencia a partir del diagrama fasorial:
V
 XCV
I act (tg G  tg G a)  I corr l I act (tg G  tg G a) 
XC

XCV
0.28q 220

 40 A
(tg G  tg G a) tg 66.4  tg 36.9
I
40
I  act 
 100 A l   100 66.4 A
cos G cos 66.4
Ia
40
 50 A l  a 50 36.9 A
I a  act 
cos G a cos 36.9

Iact

= I act 

b) para corregir totalmente el factor de potencia deber ser I = 0,


de modo que:
V
 XCV
I act tg G  I corr l I act tg G 
XC

I act tg G 40q tg 66.4



 1.32q103  1.32 mF
120Q q 220
XV
En estas condiciones, ser  aa 40 0 A .
= C

- 454 -

IC =
Icorr

Fsica Universitaria: Problemas de Fsica

Corriente alterna. E06.4

4. A una red de c.a. de 125 V y 5 Hz se conectan las cargas que se indican en la


figura:

X C = 2.5 ;

(capacidad pura)

Z1  28 ;

f.p.  0.8

(inductivo)

Z 2  28 ; f.p.  0.6

(inductivo)

a) Calcular la intensidad que circula por cada carga. b) Calcular la potencia consumida por las cargas.
c) Determinar la capacidad del condensador que hay que colocar en paralelo con la carga total para corregir
totalmente el factor de potencia.

Clculos de impedancias:
% C 2.5 j 2.5 90 
%1 2837

22.4  16.8j 

% 2 2853

16.8  22.4j 

' par

'1 ' 2
'1 + ' 2

2837 2853
39.2  39.2j

784 90

39.4 2

10 2 45

10  10 j 14.14 2 45 

45

' % C  ' par 2.5 j  10  10 j 10  7.5 j 12.5 37 


a) Determinacin de intensidades:
# 125 0
10 37 A 8  6 j A

' 12.5 37
#C
#par
1
2

% C 10 37 u 2.5 90

25 127 V

 ' par 10 37 u14.14 45


#par

141.4 8

'1

28 37

#par

141.4 8

'2

2853

141.4 8 V

5.05 29 A
5.05 45 A

b) Factor de potencia y potencia consumida:


I 37 o f.p. cos 37 0.8 (inductivo)

P VI cos I

125 u 10 u 0.8 1000 W 1 kW

c) Correccin total del factor de potencia con condensador:

V
 XCV
1/ XC
I sen G 10qsen 37
C

 153 F
100Q q125
XV
I react  I sen G 

- 455 -

Fsica Universitaria: Problemas de Fsica

Corriente alterna. E06.5

I1

5. Un circuito de corriente

alterna de 220 V est


XL=10:
compuesto de seis elementos
(resistencias, capa- A
B
C
D I2
E
cidades y autoinducciones
R=10:
XL=10:
XC=10:
XC=10:
puras) colocados como se
I3
indica en la figura, en la
I
que se indican los valores
R=1 :
de sus resistencias o de sus
~
220
V
reactancias capacitativas o
inductivas. Determnense: a) La diferencia de potencial entre los bornes de cada uno de los elementos (con
sus defases). b) Las intensidades que circulan por cada uno de los seis elementos (con sus defases). c) La
potencia consumida por cada uno de los elementos.

3 impedancias en serie: 's  10 10 j 10 j  10 


3 impedancias en paralelo:

1
1 1
1
1



l 'p  1 
' p 1 10 j 10 j 1

Impedancia total (A-E): ' AE  11 


Intensidad total y en los elementos en serie:  
a) Tensiones parciales:

#AB  20 0 q10 0  200 0 V


#BC  20 0 q10 90  200 90 V
#CD  20 0 q10 90  200 90 V
#DE  20 0 q1 0  20 0 V

#AE 220 0

 20 0 A
' AE
11 0

b) Intensidades parciales:
1 

20 0
#DE

 2 90 A
10 90
'1

2 

20 0
#DE

 2 90 A
10 90
'2

3 

#DE 20 0

 20 0 A
10
'3

c) Tan slo se consume potencia en los elementos resistivos:

P1  I 2 R  202 q10  4000 W

P  4 400 W

2
2
P2  I R  20 q1  400 W

- 456 -

Fsica Universitaria: Problemas de Fsica

Corriente alterna. E06.6

22.28 mH

6. En el circuito de la figura, hllense: a) La intensidad total y en


220 V
50 Hz

8:

Clculo de impedancias: X  2QO  100Q rad/s


X L  X L  100Q q 22.28q103  7  l % L  7 j  7 90 
1
1
1


 8  l % C  8 j  8 90 
6
0.125
XC 100Q q398q10
11 1
1

 0.125 0.125 j
 0.177 45 l ' p  5.66 45  4  4 j

' p 8 8 j
XC 

'  7 j 4  4 j
 4 3 j
 5 37 

(inductivo)

a) y b) Clculo de intensidades y de tensiones:

# 220 0

 
 44 37  35.2 26.4 j
A

' 5 36.87

#  %  44 7  308 V
L
53
37 90
L
#p  ' p  44 37 5.66 45  249 82 V l

249 82
#

  p 
 3182 A
R
8
R

249 82
#p


 31 8 A
 C 
%
8 90

c) Clculo de potencias:
PR  VR I R  7 744 W

PL  VL I L cos 90  0 (obvio)

PC  VC I C cos 90  0 (obvio)
Tan solo se consume potencia en el elemento resistivo.
d) Correccin total del factor de potencia:
El factor de potencia inductivo se corrige con un condensador en paralelo con la carga.
atrasada:
I react  I sen G
V
V
adelantada: I cond 

 XCV
Iact=I cosI V
X C 1/ XC
I
I
I
I senG
I sen G  XCV l C  react 
X
XV
V
V
I
Sustituyendo valores:
C
26.4
C
 382q106 F  382 F
100Q q 220

f.p.  cos G  cos 37  0.8 (inductivo)

P  VI cos G  220q 44q cos 37  7 744 W

Icond

Ireact=I senI

retrasada

adelantada

- 457 -

398 F

cada uno de los elementos. b) La diferencia de tensin en


bornes de cada elemento. c) La potencia total y la consumida
por cada elemento. d) Caractersticas del elemento que
tendramos que conectar en paralelo para corregir totalmente el
factor de potencia.

Fsica Universitaria: Problemas de Fsica

Corriente alterna. E06.7

7. En el circuito de la figura, determnense: a) La intensidad total e intensidades que circulan por cada rama.
b) Diferencias de tensin en bornes de cada elemento. c) Potencia disipada en el circuito y factor de
potencia. d) Diagrama fasorial de tensiones e intensidades.
e) Capacidad del condensador a colocar en paralelo con el circuito para
R
corregir el factor de potencia a 0.95.
220 V
Datos: R  8 

L1  L2 

0.12
H
Q

C

2.5
mF
Q

L1

50 Hz

L2

Clculo de impedancias:
rad
s
0.12
 12 8 l % L1  % L2  12 j  12 90 8
X L1  X L2  X L  100Q q
Q
1
Q q103

 4 8 l % C   4 j  4 90 8
XC 
XC 100Q q 2.5
12 90 4 90 48 0
'A'B
' par 


 6 90 8
'  8  6 j 12 j  8 6 j  10 37
'A 'B
12 j  4 j
8 90
X  2QO  2Q q50  100Q

(inductivo)

a) Clculo de intensidades y tensiones:


#R ' R 22 37 8 0 176 37 V
22 37 A = (17.6 - 13.2j) A o
10 37
#L2 ' L2 22 3712 90 264 53 V
#par 132 127

11 217 A
 L1
12 90
' L1

#par ' par 22 37 u 6 90 132 127 V o


#par 132 127

33 37 A
C '
4 90
C

c) Potencia y factor de potencia:


P  VI cos G  220q 22q cos 37  3872 W
f.p. = cos G = cos 37  0.8
(inductivo)

#
'

220 0

Iact tgI

Icond

I senI

- 458 -

Iact tgI

I senI

Puesto que la instalacin resulta inductiva, deberemos colocar un condensador en paralelo que
compense la corriente reactiva retrasada de la instalacin.
e) Correccin parcial:
Iact=I cosI
V
I act tg G  I act tg G a  I cond
I
V
I
 XCV
I act tg G  tg G a

1/ XC
I
I act tg G  tg G a

C
I
XV
17.6q tg 36.87  tg18.19

C
 107 F
100Q q 220

Fsica Universitaria: Problemas de Fsica

Corriente alterna. E06.8

8. a) En el circuito de C.A. que se representa en la figura, calcular la


diferencia de potencial (mdulo y fase) entre los puntos A y B
Cunto marca el voltmetro? b) Dibujar el diagrama fasorial de
intensidades. c) Explicar si el circuito es capacitativo o inductivo.
Calcular la impedancia del circuito.

A
1.06 mF

Calculamos la frecuencia angular y las reactancias de los elementos reactivos:


X  2O  100 rad/s
X L  X L  100Q q12.73q103  4 
% L  4 j  4 90 

l
1
1
X C 


3
% C  3j  3 90 

XC 100Q q1.06q103
a) Las impedancias de cada una de las ramas son:
I
4:

'  4  3 j  5 37 

I1 A
220 V

'  3 4 j  5 53 

2
50 Hz
-3j :
Las intensidades en cada rama son:

# 220 0

1 

 44 37 A  35.2 26.4 j
A

'
5 37

I1

220 0
#

j

44
A
26.4
35.2
A






53
2

'2
5 53

37
  1 1  61.6  8.8 j
 62.2 8 A

53

La d.d.p. entre los puntos A y B viene dada por


#AB  11  2  2  1% L   2 % C

3:

4:
220 V
50 Hz

V
12.73 mH

3:
I2

4j :

V
8

I
I2

esto es,
#AB   35.2 26.4 j
4 26.4  35.2 j
3  61.6  211.2 j
 220 106 V
#AB  35.2 26.4 j
3 j
 26.4  35.2 j
4 j
 61.6  211.2 j
 220 106 V
b) En la figura mostramos el diagrama fasorial de intensidades.
c) Sin necesidad de calcular, vemos que el circuito es inductivo, ya que la intensidad est
retrasada con respecto a la tensin aplicada.
En efecto, si calculamos la impedancia de las dos ramas en paralelo, tenemos:
5 37 5 53
25 16
25 16
''
' 1 1 


 3.54 8 
'1 '1 4  3 j
3 4 j
7 1j
50 8

O bien
'

220 0
#

 3.54 8 
62.2 8


- 459 -

Fsica Universitaria: Problemas de Fsica

Corriente alterna. E06.9

9. a) En el circuito de C.A. que se representa en la figura, calcular la


diferencia de potencial (mdulo y fase) entre los puntos A y B Cunto
marca el voltmetro? b) Dibujar los diagramas fasoriales de impedancias e
intensidades. c) Con base en los diagramas fasoriales (sin calcular),
explicar si el circuito es capacitativo o inductivo.

4:
220 V
50 Hz

796 F
A

3:

9.55 mH

Calculamos la frecuencia angular y las reactancias de los elementos reactivos:


X  2O  100 rad/s
X L  X L  100Q q9.55q103  3 
% L  3j  3 90 

l
1
1
X C 


4
% C  4 j  4 90 

XC 100Q q 796q106
a) Las impedancias de cada una de las ramas son:
'1  4 3j  5 37 
I

4:
4:

' 2  3  4 j  5 53 
I1 A
220 V

I2
B
50 Hz
Las intensidades en cada rama son:
3:
3:

220 0
#
 

 44 37 A  35.2  26.4 j
A
1 '1
5 37

220 0
#
I2
 2 

 44 53 A  26.4 35.2 j
A
5 53
'2

53
Z1
I
La d.d.p. entre los puntos A y B viene dada por
37
8
Z
#AB  11  2 % C  1% L   2  2
V
53
esto es,
#AB  44 37 4 0 44 53 4 90  176 37 176 37  0

37

Z2

I1

#AB  44 37 3 90 44 53 3 0  132 53 132 53  0

de modo que los punto A y B estn al mismo potencial y el voltmetro marca cero.
b) En la figura mostramos los diagramas fasoriales de impedancias e intensidades.
c) Sin necesidad de calcular, vemos que el circuito es capacitativo, ya que la intensidad est
adelantada con respecto a la tensin aplicada.
En efecto, si calculamos la intensidad total y la impedancia de las dos ramas en paralelo,
tenemos:
  1 1  35.2  26.4 j
26.4 35.2 j
 61.6 8.8 j
 62.2 8 A
'

5 37 5 53
25 16
25 16
'1'1



 3.54 8 
'1 '1 4 3 j
3  4 j
7 1j
50 8



220 0
#

 62.2 8 A
' 3.54 8

- 460 -

Fsica Universitaria: Problemas de Fsica

Corriente alterna. E06.10

10. En el circuito de c.a. representado en la figura, determinar el elemento (bobina o condensador) que hay que
colocar entre A y B para corregir completamente el factor de potencia.
Datos: R = 4 :, C = 1.061 mF, L = 9.549 mH.

Calculamos las reactancias de la bobina y del condensador:

X L  X L  100Q q9.549q103  3
1
1

3
XC 100Q q1.061q103
Las impedancias de las ramas en paralelo son:
'1  R jX L  4 3 j  5 37 
XC 

' 2  R  jX C  4  3 j  5 37 

A
220 V
50 Hz

R
R

La impedancia de estas dos ramas en conjunto es


5 37 5 37
25 0
''

 3.13 0 
'12  1 2 
'1 ' 2 4 3 j
4  3 j
8 0
La impedancia de la carga completa es
3.13 0 4 0
' '
 1.75 0 
'  12 R 
3.13 4
'12 ' R
Por consiguiente, la carga es puramente resistiva, su factor de potencia es cos 0 = 1, y no
necesita correccin alguna.

- 461 -

Fsica Universitaria: Problemas de Fsica

Corriente alterna. E06.11

11. a) En el circuito de C.A. que se representa en la figura, calcular la


diferencia de potencial (mdulo y fase) entre los puntos A y B Cunto
marca el voltmetro? b) Dibujar los diagramas fasoriales de impedancias e
intensidades. c) Explicar si el circuito es capacitativo o inductivo.

4:
220 V
50 Hz

3:
A

9.55 mH

Calculamos la frecuencia angular y las reactancias de los elementos reactivos:


X  2O  100 rad/s
X L  X L  100Q q9.55q103  3 
% L  3j  3 90 

l
1
1
X C 
% C  4 j  4 90 

4

XC 100Q q 796q106
a) Las impedancias de cada una de las ramas son:
I
'1  4 3j  5 37 
4:
3:

' 2  3  4 j  5 53 
I1 A
220 V
I2

50 Hz
Las intensidades en cada rama son:
-4j :
3j :

220
0
  # 
 44 37 A  35.2  26.4 j
A
1 '1
5 37
I2

220 0
53
#
Z1
I
 2 

 44 53 A  26.4 35.2 j
A
'2
5 53

37
8
Z
V
53
  1 1  61.6 8.8 j
 62.2 8 A
37
La d.d.p. entre los puntos A y B viene dada por
#AB  11  2  2  1% L   2 % C

Z2

V
796 F

I1

esto es,

#AB   35.2  26.4 j


4 26.4 35.2 j
3  61.6 211.2 j
 220 106 V
#AB  35.2  26.4 j
3 j  26.4 35.2 j
4 j
 61.6 211.2 j
 220 106 V
de modo que los punto A y B estn al mismo potencial y el voltmetro marca cero.
b) En la figura mostramos los diagramas fasoriales de impedancias e intensidades.
c) Sin necesidad de calcular, vemos que el circuito es capacitativo, ya que la intensidad est
adelantada con respecto a la tensin aplicada.
En efecto, si calculamos la impedancia de las dos ramas en paralelo, tenemos:
5 37 5 53
25 16
25 16
''


 3.54 8 
' 1 1 
'1 '1 4 3j
3  4 j
7 1j
50 8
o bien
'

# 220 0

 3.54 8 
62.2 8


- 462 -

Fsica Universitaria: Problemas de Fsica

Corriente alterna. E06.12

12. Al circuito de la figura se le aplica una tensin alterna de 250 V a 50 Hz.


a) Determinar las intensidades que circulan por cada rama y la intensidad
total. Dibujar el diagrama fasorial de intensidades. b) Calcular la diferencia
de potencial entre los puntos A y B y su desfase con respecto a la tensin de
alimentacin.

a) Calculamos la frecuencia angular y las reactancias de


los elementos reactivos:
X  2QO  100Q rad/s
3

X L  X L  100Q q127.3q10  40  l

1
106

 40 
XC 100Q q 79.6
% C  40 j  40 90 
XC 

250 V
50 Hz

30:

30:

79.6 F

40j :

30 :

I1

30 :

-40j 

Las impedancias de cada una de las ramas y las intensidades que las recorren son:

250 0
  # 
 5 53  3  4 j
A
1

'1 50 53
'1  30 40 j  50 53 

' 2  30  40 j  50 53 

250 0
#

 2 

 5 53  3 4 j
A
' 2 50 53

Impedancia total:
50 5350 53 2500
''

 41.67 0 
'T  1 2 
'1 ' 2
60
60
I2
 T  1  2  3  4 j
3 4 j
 6 0 A
53

I
Intensidad total:
250 0
#
 T 

 60 A
53

' T 41.67 0

I1
b) La d.d.p. entre los puntos A y B viene dada por
#AB  11   2 % C , de modo que operando
En forma mdulo-argumento:
#AB  5 53 30 0  5 53 40 90  150 53  200 37 
 90 120 j
 160 120
j  70 0 j
 70 180 V
En forma binmica:

#AB  3  4 j
30  3 4 j
40 j

 90 160
120 120
j  70 0 j
 70 180 V
Esto es, 70 V en contrafase con la tensin aplicada.

- 463 -

250 V
50 Hz

% L  40 j  40 90 

127.3 mH

200
VC
37
VAB 70

V
VR

53 37
150

I2

Fsica Universitaria: Problemas de Fsica

Corriente alterna. E06.13

13. En el circuito de la figura, existe una tensin de 220 V a 50 Hz entre los


puntos A y B. Hllese la diferencia de tensin alterna entre los puntos C y
D, y su desfase con respecto a la tensin de alimentacin.

5:

2
mF
C Q

50
mH
Q

5:
D

Calculamos la frecuencia angular y las reactancias de los


A
elementos reactivos:
X  2QO  100Q rad/s
1
1000 Q 1000


 5  l % C  5 j  5 90 
XC 
XC 100Q 2
200
50
X L  X L  100Q 103  5  l % L  5 j  5 90 
Q
Las impedancias de cada una de las ramas y las intensidades que
las recorren son:
A

220 0
  # 
31.11
A

45
1 '1
'1  5  5 j  50 45 
50 45

  #  220 0  31.11
' 2  5 5 j  50 45 
45 A
2 '
50 45
2

220V
50Hz

5:

+5j :

La d.d.p. entre los puntos C y D viene dada por


#CD  #CA #AD  #AC #AD  11  2 % L
de modo que

#CD  31.11 45 5 0 31.1145 5 90  156 45 156 45  0


resultando que los puntos C y D estn al mismo potencial.

- 464 -

-5j :

5:
D
220V
50Hz

Fsica Universitaria: Problemas de Fsica

Corriente alterna. E06.14

14. En el circuito de corriente alterna de la figura se desea conocer: a) La


intensidad total y por rama. b) La diferencia de potencial en bornes de cada
elemento. c) Diagrama fasorial de dichas intensidades y diferencia de
potencial.
220V
50Hz
1
0.03
F, L 
H.
Datos: R  4, C 

300Q

I1

I3

I2

R
R

Clculo de las reactancias y de las impedancias de rama:

0.03
'1  4 3 j  5 37 8

' L  3 90  3 j 8
X L  X L  100Q q Q  3 8

l
l ' 2  4  3 j  5 37 8

X  1  300Q  3 8

' C  3 90  3 j 8
C

' 3  4  4 0 8
XC 100Q

a) Impedancia total e intensidad total:


1
1
1
1


 0.2 37 0.2 37 0.25 0  0.2 cos 37 0.2 cos 37 0.25  0.57 0 81
' T '1 '1 '1
= ' T  1.75 0 8 l  T 

220 0
#

 124.4 0 A
' T 1.75 0

b) Intensidades de rama:

# 220 0

 44 37 A
1 
5 37
'

220 0
#


 44 37 A
 2 

5 37
'2

220 0
#

 55 0 A
 3 
4 0
'3

c) Tensiones que soportan cada uno de los elementos:


#R1  1 ' R1  44 37 4 0  176 37 V

#   '  44 3  132 V
1 L
37 90
53
L
#   '  44 4  176
R2
2
37 0
37 V
R 2

#C   2 ' C  44 37 3 90  132 53 V

\#  220 V
0
R 3

(en fase con la tensin)

I2
IT
I3
I1

VL
V
VR1

VR2
V
VC
VR3 = V

- 465 -

Fsica Universitaria: Problemas de Fsica

Corriente alterna. E06.15

15. En una lnea de corriente alterna de 220 V y 50 Hz se conectan en paralelo las tres cargas siguientes:
Z1 = 30 :

con

cos I1 = 0.8

inductivo

Z2 = 20 :

con

cos I2 = 0.85

inductivo

Z3 = 40 : con cos I3 = 0.9


inductivo
a) Calcular la intensidad que circula por cada una de las cargas, y la intensidad total. b) Determinar la potencia
total consumida y el factor de potencia del conjunto. c) Evaluar la capacidad del condensador que hay que
colocar para corregir totalmente el factor de potencia.

Determinamos las impedancias de cada carga:


cos G1  0.8 l G1  36.9 l '1  30 36.9  24.0 18.0 j
8
cos G2  0.85 l G2  31.8 l ' 2  20 31.8  17.0 10.5 j
8
cos G3  0.9

l G3  25.8 l '3  40 25.8  36.0 17.4 j


8

a) Las intensidades en cada una de las carga son:


220 0
#
 7.3 36.9  5.9  4.4 j
A
1  1 
'1 30 36.9
2 

220 0
#2

 11.0 31.8  9.4  5.8 j
A
' 2 20 31.8

3 

220 0
#3

 5.5 25.8  5.0  2.4 j
A
'3 40 25.8

I
I1 1 I 2 2 I3 3
V

y la intensidad total es
  1  2  3  20.3 12.6 j
 23.9 31.8 A

La impedancia total es:


'

220 0
#

 9.2 31.8 8
23.9 31.8


b) El factor de potencia y la potencia consumida son


f.p.  cos 31.8  0.85
P  VI cos G  220q 23.9q0.85  4467 W  4.47 kW
c) Deberemos colocar un condensador en paralelo que compense la corriente reactiva
retrasada de la instalacin:

V
1/ XC

C

I react
XV

de modo que

Iact

Ireact

I react  I cond 

12.6
C
 1.82q104  182 F
100Q q 220

- 466 -

Fsica Universitaria: Problemas de Fsica

Corriente alterna. E06.16

16. En el circuito de la figura, la tensin entre bornes de la bobina es


el doble que la tensin que soporta el condensador C1. a) Calcular la d.d.p. entre los bornes del generador y la intensidad que
ste proporciona al circuito. b) Determinar los valores de L, C1 y
C2. c) Evaluar la impedancia total del circuito y su factor de
potencia.

50Hz

C2

C1
A

9A

8:
A

18 A

12 A

rama 3: #AB   3'3  12 08 0  96 0 V

rama 2:
'2 

96 0
#AB
1

 5.33 90 8 l Z 2 
2
XC 2
18 90

rama 1:
V
VAB
I1  AB 
1
Z1
XL 
XC1
X L I1  2
Z3 
L

I1
XC1

l Z1  X L 

l XL 

2
XC1

l C2 

1
1

 597 F
X Z 2 100Q q5.33

V
1
96
 AB 
 10.67 8 l '1  10.67 90 8
9
XC1
I1

l L

2
X 2C1

2
1
1
1


 10.67 l C1 
 298 F
XC1 XC1 XC1
100Q q10.67
2

100Q
298q106
2

 68 mH

Asociacin de impedancias en paralelo:


j
j
1
1
1
1
1
1
1
1






 0.125 0.094 j  0.153 37 8 1
' '1 ' 2 '3 10.67 j 5.33j 8
10.67 5.33 8
de modo que '  6.4 37 8 y cos G  cos 37  0.8 (capacitativo)
Intensidad suministrada por el generador:
96 0
#
 
 15 37 A (adelantada)
' 6.4 37

- 467 -

Fsica Universitaria: Problemas de Fsica

Corriente alterna. E06.17

17. Al circuito de la figura se le aplica una tensin alterna de 220 V a 50 Hz.


a) Determinar las intensidades que circulan por cada rama y la intensidad
total. Dibujar el diagrama fasorial de intensidades. b) Calcular la diferencia
de potencial entre los puntos A y B y su desfase con respecto a la tensin de
alimentacin.

11:
220 V
50 Hz

127.3 mH

11:

176.8 F

a) Calculamos la frecuencia angular y las reactancias de los elementos reactivos:


X  2QO  100Q rad/s

11:

220 V
50 Hz

I1

40j 

11:

X L  X L  100Q q127.3q103  40 

I2

% L  40 j  40 90 

1
106

 18 
XC 100Q q176.8
% C  18 j  18 90 

XC 

-18j 

Las impedancias de cada una de las ramas y las intensidades que las recorren son:

220 0
  # 
 10 0  10 A
1 '1
'1  11 11  22 
22 0

' 2  40 j 18 j  22 j=22 90 

220 0
#
 2 

 10 90  10 j A
'2
22 90

22 0 22 90 22 0 22 90 22 90
22 90
''
Impedancia total: ' T  1 2 



 15.6 45 
22 22 j 22 1 1j
1 1j
'1 ' 2
2 45
 T  1  2  10 10 j  1445 A

220 0
Intensidad total:
#

 T 

 14 45 A
' T 15.6 45

b) Calculamos la d.d.p. entre los puntos A y B por el


Camino superior:
#AB  11  2 % L   10
11 10 j
40 j
  110
400
 290 V

Camino inferior:
#AB  1 2   2 % C  10
11 10 j
18 j
 110
 180
 290 V

Esto es, 290 V en fase con la tensin aplicada.


45

I1=10A V

110 V

400 V
290 V

I=14A
110 V

180 V
290 V

I2=10A

- 468 -

Fsica Universitaria: Problemas de Fsica

Corriente alterna. E06.18

8:

18. En el circuito de la figura, hllense: a) La intensidad total y en cada


uno de los elementos. b) La diferencia de tensin en bornes de
cada elemento. c) Potencia y factor de potencia. d) Capacidad del
condensador que hay que colocar en paralelo para corregir 220 V
totalmente el factor de potencia.
50 Hz

Clculos preliminares:

X L  X L  100Q q 6.37 q103  2  l % L  2 j 

1
1

XC 

 4  l % C  4 j 

XC 100Q q 796q106

(2 j)(4 j)
8 j2

'


 4 j  4 90 

par

(2 j) (4 j) 2 j

'  8 4 j 2 j  8 6 j  10 36.9 
a) Determinacin de intensidades:
220 0
#
 
 22 36.9 A  (17.6 13.2 j) A
' 10 36.9

6.37mH

6.37mH

796F

IC

#par  ' par  22 36.9 q 4 90  88 53.1 V

L 

37

#L

 44 36.9 A
2 90
%L
88 53.1

I
IL

88 53.1
#
C  C 
 44 143.1 A
2 90
%C
VL

b) Tensin que soporta cada elemento:


#R    22 36.9 q8 0  176 36.9 V

Vpar

#R  88 53.1 V

37

#L  % L  22 36.9 q 2 90  44 53.1 V

c) Factor de potencia y potencia


G  36.9
cos G  cos 36.9  0.8

VR
(inductivo)

P  VI cos G  220q 22q 0.8  3872 W


d) Correccin total del factor de potencia con condensador:
V
220 V
 XCV
I react  I sen G 
1/ XC
50 Hz
I sen G 22qsen 36.9

 191 F
C
XV
100Q q 220

- 469 -

191F

Fsica Universitaria: Problemas de Fsica

Corriente alterna. E06.19


15:

19. En el esquema de la figura determnense: a) La intensidad que


circula por cada rama. b) La diferencia de potencial entre A y B.
c) La potencia y el factor de potencia. d) Valor de la capacidad que
220 V
hay que colocar en paralelo para corregir el factor de potencia.
50 Hz

1
F
700S
1:

0, 07
A

B
1:

a) Calculamos las impedancias de cada elemento y de las asociaciones en paralelo y total:

1
700Q
]  1 7 j  50 82  7.07
X C  XC  100Q  7 
82 

0.07
] L  1 7 j  50 82  7.07 82 
7
X L  X L  100Q
Q

50 82 50 82
] C] L
50

  25 0 8
]p 
]  40 0 8
2
] C ] L (1 7 j) (1 7 j)
# 220 0
 
 5.5 0 A (en fase)
#p  ' p  5.5 0 q 25 0  137.5 0 V
40 0
]
C 

137.5 0
#

 19.45 82 A
]C
50 82

L

137.5 0
#

 19.45 82 A
]L
50 82

b) Clculo de la d.d.p. entre A y B:


#AB   C q1  L q1   C  L  (2.75 19.25 j)  (2.75 19.25 j)  38.5 j  38.5 90 V

c) Factor de potencia y potencia:


G  0 l f.p.  cos G  1 l P  VI cos G  220q5.5  1210 W
d) No hay nada que corregir

- 470 -

Fsica Universitaria: Problemas de Fsica

Corriente alterna. E06.20

20. En el circuito de corriente alterna de la figura se desea conocer: a) La


intensidad total y por rama. b) La diferencia de potencial en bornes de
cada elemento. b) Diagrama vectorial de dichas intensidades y 220 V
diferencia de potencial.
50 Hz
Datos: R  4 C  1061 F L  9.55 mH

R
C

'1  R  4 0 
' 2  X Lj  9.55q103 q100Q j  3j  3 90 
'3  R  4 0 
A

1
1

 3 j  3 90 
XCj 1061q106 q100Q j
'5  4 0 
'4 

I1

a) Calculamos las intensidades a partir de la ley de Ohm


para C.A.:
B
220
220
#
0
0
1 


 44 37 A  35.2  26.4 j A
5 37
'1 ' 2 4 3 j
2 

220 0
220 0
#


 44 37 A  35.2 26.4 j A
'3 ' 4 4  3 j 5 37

3 

220 0
#

 55 0 A  55 A
40
'5

  1  2  3  125.4 0 A

b) Calculamos las tensiones:


#1  1'1  44 37 q 4 0  176 37 V
#2  1' 2  44 37 q3 90  132 53 V
#3   2 '3  44 37 q4 0  176 37 V
#4   2 ' 4  44 37q 3 90  132 53 V
#5  #  220 0 V

c) Diagrama fasorial:

IT
A

B
I2
C
- 471 -

C
Z2

(en fase con la tension)

I1

Z3

Z1

I3

I2

D
Z4

I3

Z5

Fsica Universitaria: Problemas de Fsica

Corriente alterna. E06.21

5:

21. En el circuito representado en la figura, determinar: a) Intensidad


en cada rama. b) Diferencia potencial entre A y B. c) Potencia y
factor de potencia. d) Dibujar el diagrama fasorial.

F
220 V
700S
50 Hz
0.4

Clculo de impedancias:
X  2QO  2Q q50  10Q

1:

IA
A
1:

B
0.07

rad
s

1
700Q

 7  l %C   7 j 
XC 100Q
0.07
 7  l % LB  7 j 
X LB  X LB  100Q q
Q
0.4
 40  l % L  40 j 
X L  X L  100Q q
Q
' A  1 7 j = 50 82
' b  1 7 j = 50 82
XC 

' par 

50 0
'A 'B

 25 0 
2 0
'A 'B

'  30 40 j = 50 53

(inductivo)

a) Clculo de intensidades:

# 220 0
4.4 53 A = (2.64 - 3.52j) A

' 50 53
#
par ' par 4.4 53 u 25 0 110 53 V

(retrasada)

#par 110 53

15.56 29 (13.64  7.48 j) A


 A
'A
50 82

  A  B

#par 110 53



15.56 135 (11.00  11.00 j) A
B '
50 82
B

b) Diferencia potencial entre A y B:


VAB I A RA  I B X LB 15.56 291 0  15.56 135 7 90 15.56 29  108.89 45
(13.64  7.48 j)  (77.00-77.00j)

(63.36  84.48 j) 105.6 127

c) Potencia y factor de potencia:


P VI cos I 220 u 4.4 u cos 53 580.80 W
# 220 0 4.4 53

f.p. = cos I = cos 53 0.6


P 580.80 W
(580.80  774.40 j) W

Q 774.40 VA

968 53

A
B

I


I
#

- 472 -

(inductivo)

IB

Fsica Universitaria: Problemas de Fsica

Corriente alterna. E06.22

22. En el circuito que se muestra en la figura, el alternador suministra una


tensin alterna de 50 Hz y en la resistencia de 4  se disipa una potencia
de 16 W. a) Calcular la intensidad en cada rama y la tensin entre bornes
del generador, as como la intensidad de la corriente suministrada por
ste. b) Determinar el factor de potencia de toda la carga. c) Evaluar la
potencia consumida y la potencia reactiva de la carga y de cada uno de los
elementos.

5

10 

1

4

a) Calculamos las impedancias de cada rama y la de la carga total.


'1  1 5 j  26 79  5.10 79 
' 2  4 10 j  116 68  10.77 68 
'

5.10 7910.77 68
54.92 11 54.92 11
'1' 2



 7.77 56 
'1 ' 2 1 5 j
4 10 j

5 5j
7.07 45

Calculamos la intensidad en la segunda rama a partir de


la potencia disipada en la resistencia:

P  I 2 R l I  P / R  16 / 4  2 A

capacitativo

10 

5

La tensin entre A y B (suministrada por el generador)


ser:
#   2 ' 2  2 6810.77 68  21.54 0 V

I2

I1

4

1

Las intensidades en cada rama y la suministrada por el


generador son:
21.54 0
#

 4.22 79 = 0.81 4.14j
A
1 
'1 5.10 79

Z2

I1

 2  2 68  0.75 1.85 j


A


# 21.54 0

 2.77 56  1.56 2.29 j
A
' 7.77 56

b) La impedancia de la carga total es capacitativa, con un


argumento (ngulo)
G  56 l f.p.  cos G  0.57 (capacitativo)
c) Clculo de las potencias:
En la resistencia de la rama 1 se consume:

-56

I2

P  I R  4.22 q1  18 W
2
1

56

Z1

Las potencias reactivas en el condensador y en la bobina son:


Pcond  4.222 q5  89 W

P  I 2 X l
2

Pbob  2.00 q10  40 W


La potencia total:

P  34 W
 #  21.54 0 2.77 56  60 56  34 49 j
W l

Q  49 W capacitativa

- 473 -

Fsica Universitaria: Problemas de Fsica

Corriente alterna. E06.23

23. Cuando se conecta un circuito RLC serie a una lnea de 120 V


eficaces y 60 Hz de frecuencia, circula una corriente eficaz de 11 A,
adelantada 45 respecto de la tensin. a) Hallar la potencia
suministrada al circuito y la resistencia hmica del mismo. b) Si la
autoinduccin es L = 0.05 H, hallar la capacidad C presente en el
circuito. c) Qu capacidad o autoinduccin debern aadirse para
que el factor de potencia sea 1?

A
R
120 V
60 Hz

Lcorr
L

X  2QO  2Q q 60  120Q rad/s


'

# 120 0

 10.9 45  =(7.71-7.71j) 
 11 45

de modo que el circuito RCL es capacitativo.


a) La resistencia hmica es la parte real de la impedancia:
R  7.71 
X  7.71  (capacitativo)
G = -45
La potencia suministrada al circuito es P  V I cos G  120q11qcos 45  933.4 W
b) Calculamos la capacidad pedida:
X L  X L  120Q q 0.05  18.85 
X  X L  XC
XC 

1
XC

X C  X L  X  18.85  (7.71)  26.56 

l C

I
Ireact

Icorr

1
1

 99.9 F
X X C 120Q q 26.56

c) Para corregir completamente el factor de potencia


necesitamos colocar una autoinduccin en paralelo con la carga
total:
V
l
I react  I sen G 
X Lcorr
Lcorr 

45
Iact

V
120

 0.041 H  41 mH
X I sen G 120Q q11qsen 45

- 474 -

Fsica Universitaria: Problemas de Fsica

Corriente alterna. E06.24

24. A una red de corriente alterna de 220 V/50 Hz se conectan (en paralelo) un motor que consume 3 kW con un
factor de potencia 0.8 inductivo y 50 lmparas fluorescentes de 40 W (cada una) y factor de potencia 0.4
inductivo. a) Determinar la intensidad total que consume esta instalacin. b) Calcular el factor de potencia
del consumo. c) Dibujar el diagrama fasorial de tensiones e intensidades. d) Encontrar la capacidad del
condensador que hay que colocar en paralelo para corregir totalmente el factor de potencia.

Con los datos del enunciado, completamos la tabla que se adjunta, sirvindonos de las
expresiones que se indican en los encabezamientos de las columnas de la Tabla:.
Generador: 220 V / 50 Hz;

intensidades en amperios (A)

P (W)

Elementos

cos 

I act  P V

I react  I act tg G

I  I act cosG

1 motor

3000

0.8 ind

36.9

13.6

10.2

17.0

50 tubos u40

2000

0.4 ind

66.4

9.1

20.8

22.7

Totales

5000

0.59 ind

53.9

22.7

31.1

38.5

ab) Calculamos la intensidad que consume la instalacin, el desfase y el factor de potencia del
consumo:
V
I  I 2 I 2  22.7 2 31.12  38.5 A
act

react

G = arctg

I react
31.1
 arctg
 53.9
I act
22.7

I 1  17 37

cos G  cos 53.9  0.59 (inductivo)


c) En la figura adjunta presentamos los diagramas fasoriales de las
intensidades y de la tensin (la misma en todos los elementos en
paralelo).

I 2  22.7 66

I  38.5 54

Icond

Ireact =I senI

retrasada

adelantada

d) Puesto que la instalacin es inductiva, deberemos colocar un elemento capacitativo


(condensador) en paralelo con la carga para corregir el factor de potencia:
V
V
I react 

 XCV l
Iact =I cosI V
X C 1/ XC
I
I
I
31.1
 450 F
C  react 
100Q q 220
XV
V
I
C

- 475 -

Fsica Universitaria: Problemas de Fsica

Corriente alterna. E06.25

25. Una instalacin elctrica (220 V, 50 Hz) se compone de los siguientes consumos:
50 lmparas fluorescentes de 40 W cada una, y factor de potencia 0.40 (inductivo)
1 motor de 2 kW de consumo y factor de potencia 0.80 (inductivo)
2 calefactores elctricos (resistivos) de 0.5 kW cada uno
a) Calcular las intensidades que producen cada uno de esos tres grupos de consumo. b) Calcular el factor de
potencia del conjunto. c) Determinar el elemento (indicando condensador o bobina, y su valor) que hay que
aadir en paralelo para corregir totalmente el factor de potencia.

Iact

Ireact

Utilizando las relaciones que se indican, completamos la tabla:

P  VI cos G  VI act l I act 

V
l I  act

cosG

I  I sen G  I act tg G l I react  I act tg G

react
I
G  arctg react
I act

I
I

I1 1 I2 2 I3 3
V

Grupo

Elementos

P (W)

f.p.=cos I

Iact (A)

Ireact (A)

I (A)

50 tubos 40 W

2000

0.40 (i)

66.42

9.09

20.83

22.73

1 motor

2000

0.80 (i)

36.87

9.09

6.82

11.36

2 calef 500 W

1000

1.00

0.00

4.55

0.00

4.55

Totales

5000

0.64 (i)

50.58

22.73

27.65

35.79

a) Intensidades en cada uno de los tres grupos son:


1  22.73 66.4 A (retrasada)
Lmparas:
Motor:

 2  11.36 36.9 A (retrasada)

Calefactores:

 3  4.55 0 A

(en fase)

Icond

Ireact=I senI

retrasada

adelantada

b) El factor de potencia del conjunto vale 0.64 (inductivo)


c) Puesto que la instalacin resulta inductiva, deberemos colocar un condensador en paralelo
que compense la corriente reactiva retrasada
Iact=I cosI V de la instalacin:
I
I
I
V
l C  react
I react  I cond l I react 
1/ XC
XV
V
I
C
de modo que
27.65
C
 4q104  400 F
100Q q 220

- 476 -

Fsica Universitaria: Problemas de Fsica

Corriente alterna. E06.26

26. A una red alterna de 220 V / 50 Hz se conectan (en paralelo) un motor que consume 1.5 kW con factor de
potencia inductivo 0.8 y 50 lmparas fluorescentes de 40 W (cada una) y factor de potencia 0.4 tambin
inductivo. a) Calcular la intensidad total que consume esta instalacin. b) Hallar el factor de potencia del
conjunto. c) Determinar a capacidad del condensador que hay que colocar en paralelo para corregir
totalmente el factor de potencia.

Iact

Ireact

Utilizando las relaciones que se indican, completamos la tabla:

P  VI cos G  VI act l
I act 

V
l I  act

cosG

I  I sen G  I act tg G l I react  I act tg G

react
I
G  arctg react
I act

I
I
I1 1

I2 2

Grupo

Elementos

P (W)

f.p.=cos I

Iact (A)

Ireact (A)

I (A)

1 motor

1500

0.80 (i)

36.9

6.82

5.11

8.52

50 tubos 40 W

2000

0.40 (i)

66.4

9.09

20.83

22.73

Totales

3500

0.52 (i)

58.5

15.91

25.94

30.43

a) Intensidad total que consume la instalacin:   30.43 58.5 A

Icond

Ireact=I senI

retrasada

adelantada

b) El factor de potencia del conjunto vale 0.52 (inductivo)


c) El factor de potencia inductivo se corrige con un condensador en paralelo con la carga.
atrasada:
I react  I sen G
V
V
adelantada: I cond 

 XCV
Iact=I cosI V
X C 1/ XC
I
I
I
I senG
I react  XCV l C  react 
V
X
XV
V
I
Sustituyendo valores:
C
25.94
C
 3.75q104  375 F
100Q q 220

- 477 -

Fsica Universitaria: Problemas de Fsica

Corriente alterna. E06.27

27. En una estacin de bombeo (220 V 50 Hz) tenemos instalados 3 motores y un grupo de resistencias con las
siguientes caractersticas:
Motor 1:

2 CV,

cos M = 0.85

Motor 2:

5 CV,

cos M = 0.7

Motor 3:
3 CV, cos M = 0.6
Resistencias:
3 CV = 2208 W
a) Calcular las intensidades absorbidas por cada motor o grupo. b) Qu condensador habr que poner para
llevar la instalacin completa a un cos I = 0.95? Datos: 1 CV = 736 W.

Utilizando las relaciones que se indican, completamos la tabla:

I
l

I act
cosG

G  arctg

I react
I act

Iact

Ireact

P
P  VI cos G  VI act l I act 
V

I react  I sen G  I act tg G l I react  I act tg G

Elementos

P (W)

f.p.=cos I

Iact (A)

Ireact (A)

I (A)

Motor1

1472

0.85 (i)

31.79

6.69

4.15

7.87

Motor 2

3680

0.70 (i)

45.57

16.73

17.07

23.90

Motor 3

2208

0.60 (i)

53.13

10.04

13.39

16.73

Resistencias

2208

1.00

0.00

10.04

0.00

10.40

9568

0.78 (i)

38.51

43.49

34.61

55.76

 3  16.73 53.13 A (retrasada)


 4  10.04 0 A

(en fase)

I
I

Icond

Resistencias:

Iact tgI

Motor 3:

(retrasada)

Iact tgI

 2  23.90 45.57 A

I senI

Motor 2:

Iact=I cosI

I senI

a) Intensidades en cada uno de los cuatro grupos son:


1  7.87 31.79 A (retrasada)
Motor 1:

b) El factor de potencia del conjunto vale 0.78 (inductivo)


Puesto que la instalacin resulta inductiva, deberemos colocar un condensador en paralelo que
compense la corriente reactiva retrasada de la instalacin:
Correccin parcial:
V
 XCV
I act tg G  I act tg G a  I cond l I act tg G  tg G a

1/ XC
I act tg G  tg G a
43.49q tg 38.51  tg18.19


 294 F
C
XV
100Q q 220

- 478 -

Fsica Universitaria: Problemas de Fsica

Corriente alterna. E06.28

28. Se realiza la iluminacin de una sala de trabajo con 200 lmparas fluorescentes de 220 V/40 W y un factor
de potencia de 0.4 (inductivo). a) Determinar la intensidad total que las alimenta. b) La capacidad de
condensador que deber colocarse en paralelo a la entrada de la instalacin para corregir totalmente el factor
de potencia. c) La capacidad de los condensadores que deberan colocarse en paralelo con cada una de las
lmparas fluorescentes para corregir totalmente el factor de potencia. Nota: Esta es una alternativa a la
solucin del apartado anterior.

Cada tubo consume 40 W / 220 V


con cos=0.4 inductivo ( = 66.4).
P  VI cos G l
P
40

 0.455 A
I
V cos G 220q 0.4

220 V
50 Hz

200 tubos
40 W

#  220 0 V

  0.455 66.4 A  0.182  0.417 j A

a) Intensidad total:  total  n  200q 0.455 66.4  90.9 66.4 A  36.4  83.3 j A
Correccin total del factor de potencia:
El factor de potencia inductivo se corrige con
un condensador en paralelo con la carga.
I
I
atrasada:
I react  I sen G
V
V
V

 XCV
adelantada: I cond 
I
X C 1/ XC
C
I
I senG
I sen G  XCV l C  react 
XV
XV
b) Con condensador nico para toda la instalacin:
83.3
Ctotal 
 1.2q103 F  1.2 mF
100Q q 220
c) Con un condensador para cada uno de los tubos:
0.417
Ctubo 
 6q1036F  6 NF
100Q q 220
Obsrvese que estos 200 condensadores en paralelo equivalen a
Ceq  200Ctubo  200q 6 F  1200 F  1.2 mF

Iact=I cosI

Icond

Ireact=I senI

retrasada

adelantada

- 479 -

Fsica Universitaria: Problemas de Fsica

Corriente alterna. E06.29

29. Un generador de corriente alterna (200 V) suministra una intensidad de 100 A a una instalacin que tiene un
factor de potencia inductivo de 0.6. Cuando ponemos un condensador en paralelo con dicha instalacin, el
factor de potencia pasa a valer 0.9, tambin inductivo. a) Hallar la intensidad que pasa por el mencionado
condensador. b) Qu intensidad suministra el generador despus de la correccin del factor de potencia?

Puesto que el factor de potencia es inductivo, la intensidad de


corriente est atrasada con respecto al voltaje un ngulo:
G  arccos 0.6  53.13

La intensidad ser:   100 53.130  60  80 j A


La impedancia total de la instalacin ser:
200 0
#
#

l ' 
 2 53.13 
'
 100 53.13
Cuando ponemos el condensador en paralelo, el retraso de la
intensidad se reduce hasta un valor:
G a  arccos 0.9  25.84

Icond

200V

Icarga
Z

Iact=I cosI

I
I

Iact tgI

Icond

Iact tgI

I senI

 60 tg 53.13  tg 25.84
 51 A

I senI

a) Por el condensador circula una corriente adelantada


90 respecto de la tensin, cuyo valor se determina
fcilmente a partir del diagrama de intensidades:
I cond  I act tg G  I act tg G a  I act tg G  tg G a


100 A

200V

 cond  51 90 A

b) Despus de colocar el condensador, el generador suministra una corriente I que se


determina a partir del diagrama de intensidades:
 a  I act  j I act tg G a
 60  60 tg 25.84 j  60  29 j
A  91 -25.84 A

- 480 -

También podría gustarte